You are on page 1of 448

Editorial Board

Sh. K.B.Gupta
Content Writers
Dr. Alok Kumar, Dr. Rakesh Kumar Gupta
Revised by
Dr. Sumita Jain
Content Reviewer from the DDCE/COL/SOL
Dr. Neha Nainwal and Dr. Promila Bharadwaj
Academic Coordinator
Mr. Deekshant Awasthi

© Department of Distance and Continuing Education


ISBN: 978-81-19417-08-7
1st Edition: 2023
E-mail: ddceprinting@col.du.ac.in
commerce@col.du.ac.in

Published by:
Department of Distance and Continuing Education
Campus of Open Learning/School of Open Learning,
University of Delhi, Delhi-110007

Printed by:
School of Open Learning, University of Delhi
DISCLAIMER

Disclaimer

u Unit I-V are edited versions of study material prepared for the courses
under Annual & CBCS Mode.
u Corrections/Modifications/Suggestions proposed by Statutory Body, DU/
Stakeholder/s in the Self Learning Material (SLM) will be incorporated in
the next edition. However, these corrections/modifications/suggestions will
be uploaded on the website https://sol.du.ac.in. Any feedback or suggestions
can be sent to the email- feedbackslm@col.du.ac.in

Printed at: Taxmann Publications Pvt. Ltd., 21/35, West Punjabi Bagh,
New Delhi - 110026 (18000 Copies, 2023)

© Department of Distance & Continuing Education, Campus of Open Learning,


School of Open Learning, University of Delhi
Contents

PAGE

UNIT 1
Lesson 1 : Preparation of Frequency Distribution and their Graphical Presentation
1.1 Learning Objectives 3
1.2 What is Frequency Distribution 3
1.3 Types of Frequency Distribution 4
1.4 Principles of Frequency Distribution 8
1.5 Graphs 13
1.6 Summary 25
1.7 Self-Assessment Questions 26

Lesson 2 : Measures of Central Tendency - Mathematical and Positional Averages


2.1 Learning Objectives 31
2.2 What is Central Tendency? 32
2.3 Objectives of Central Tendency 32
2.4 Characteristics 33
2.5 Types of Averages 33
2.6 Mean 34
2.7 Geometric Mean 46
2.8 Harmonic Mean 51
2.9 Median 55
2.10 Other Positional Averages 59
2.11 Calculation of Missing Frequencies 61
2.12 Mode 63
2.13 Summary 73
2.14 Self-Assessment Questions 74

PAGE i
© Department of Distance & Continuing Education, Campus of Open Learning,
School of Open Learning, University of Delhi
B.COM. (PROGRAMME)

PAGE

Lesson 3 : Measures of Variation – Absolute and Relative


3.1 Learning Objectives 81
3.2 Need and Importance 81
3.3 What is Variation? 83
3.4 Requisites of a Good Measure of Variation 83
3.5 Types of Variation 83
3.6 Methods Computing Variation 84
3.7 Revisionary Problems 115
3.8 Summary 122
3.9 Self-Assessment Questions 123

Lesson 4 : Skewness and Kurtosis


4.1 Learning Objectives 131
4.2 Tests of Skewness 131
4.3 Nature of Skewness 133
4.4 Characteristics of Skewness 133
4.5 Methods of Skewness 133
4.6 Measures of Kurtosis 146
4.7 Comparison among Variation, Skewness, Kurtosis 147
4.8 Summary 148
4.9 Self Assessment Questions 148

Lesson 5 : Moments
5.1 Learning Objectives 153
5.2 Concept of Central Moments 153
5.3 Sheppard’s Method 163
5.4 &RHI¿FLHQWV RI 0RPHQWV 
5.5 Summary 167
5.6 Self-Assessment Questions 168

ii PAGE
© Department of Distance & Continuing Education, Campus of Open Learning,
School of Open Learning, University of Delhi
CONTENTS

PAGE

UNIT 2
Lesson 1 : Theory of Probability
1.1 Learning Objectives 177
1.2 Probability Distribution 177
1.3 Basic Terminology in Probability 180
1.4 Methods of Assigning Probability 185
1.5 Computation of Probability 189
1.6 Laws of Probability 194
1.7 Bayes’ Theorem 202
1.8 Expected Value 206
1.9 Summary 208
1.10 Self-Assessment Questions 210

Lesson 2 : Probability Distributions


2.1 Learning Objectives 217
2.2 Probability Distribution 217
2.3 Binomial Distribution 219
2.4 Poisson Distribution 225
2.5 Normal Distribution 230
2.6 Summary 242
2.7 Self-Assessment Exercise 243

Lesson 3 : Statistical Decision Theory


3.1 Learning Objectives 248
3.2 Probability in Decision Making 248
3.3 Decision Making Process 251
3.4 Decision Under Uncertainty 254
3.5 Decision Under Risk 256
3.6 Expected Value of Perfect Information (EVPI) 258

PAGE iii
© Department of Distance & Continuing Education, Campus of Open Learning,
School of Open Learning, University of Delhi
B.COM. (PROGRAMME)

PAGE
3.7 Decision Tree 263
3.8 Summary 268
3.9 Self-Assessment Questions 269

UNIT 3
Lesson 1 : Simple Correlation
1.1 Learning Objectives 279
1.2 Introduction 279
1.3 Utility of Correlation 280
1.4 Difference between Correlation and Causation 281
1.5 Types of Correlation 282
1.6 Methods of Studying Correlation 283
1.7 Summary 301
1.8 Self-Assessment Questions 302

Lesson 2 : Regression Analysis


2.1 Learning Objectives 307
2.2 Introduction 307
2.3 Difference between Correlation and Regression 309
2.4 Principle of Least Squares 310
2.5 Methods of Regression Analysis 310
2.6 3URSHUWLHV RI 5HJUHVVLRQ &RHI¿FLHQWV 
2.7 Standard Error of an Estimate 324
2.8 Summary 326
2.9 Self-Assessment Questions 326

UNIT 4
Lesson 1 : Index Numbers
1.1 Learning Objectives 335
1.2 Introduction 336

iv PAGE
© Department of Distance & Continuing Education, Campus of Open Learning,
School of Open Learning, University of Delhi
CONTENTS

PAGE
1.3 Features of Index Numbers 337
1.4 Problems of Index Numbers 338
1.5 Methods of Constructing Index Numbers 341
1.6 Tests of Adequacy or Consistency 350
1.7 Chain Base Index 353
1.8 Splicing 356
1.9 Consumer Price Index 358
1.10 Index Number of Industrial Production 360
1.11 Limitations of Index Numbers 361
1.12 Construction of BSE Sensex and NSE Nifty 361
1.13 Summary 370
1.14 Self-Assessment Questions 371

UNIT 5
Lesson 1 : Time Series Analysis
1.1 Learning Objectives 379
1.2 Introduction 379
1.3 Components of Time Series 380
1.4 Models of Time Series 383
1.5 Methods of Measuring Trend 384
1.6 Second Degree Parabola 396
1.7 Exponential Trend 398
1.8 Shifting the Trend Origin 400
1.9 Conversion of Annual Trend to Monthly Trend 401
1.10 Measurement of Seasonal Variations 403
1.11 Summary 418
1.12 Self-Assessment Questions 419
Glossary 431

PAGE v
© Department of Distance & Continuing Education, Campus of Open Learning,
School of Open Learning, University of Delhi
UNIT-1

PAGE 1
© Department of Distance & Continuing Education, Campus of Open Learning,
School of Open Learning, University of Delhi
L E S S O N

1
Preparation of Frequency
Distribution and their
Graphical Presentation
STRUCTURE
1.1 Learning Objectives
1.2 What is Frequency Distribution
1.3 Types of Frequency Distribution
1.4 Principles of Frequency Distribution
1.5 Graphs
1.6 Summary
1.7 Self-Assessment Questions

1.1 Learning Objectives


After reading this lesson, you should be able to:
‹ Learn a frequency distribution and types of distributions.
‹ Learn the principles and procedure of preparing a frequency distribution.
‹ Learn the graphical presentation of distribution with the help of histogram, frequency
polygon, smoothed frequency curves and gives.

1.2 What is Frequency Distribution


Collected and classified data are presented in a form of frequency distribution. Frequency
distribution is simply a table in which the data are grouped into classes on the basis of
common characteristics and the number of cases which fall in each class are recorded. It
shows the frequency of occurrence of different values of a single variable. A frequency
distribution is constructed for satisfying three objectives :

PAGE 3
© Department of Distance & Continuing Education, Campus of Open Learning,
School of Open Learning, University of Delhi
B.COM. (PROGRAMME)

Notes (i) to facilitate the analysis of data.


(ii) to estimate frequencies of the unknown population distribution from
the distribution of sample data and
(iii) to facilitate the computation of various statistical measures.

1.3 Types of Frequency Distribution


1. Univariate Frequency Distribution.
2. Bivariate Frequency Distribution.
This chapter consists Univariate frequency distribution. Univariate
distribution incorporates different values of one variable only whereas the
Bivariate frequency distribution incorporates the values of two variables.
The Univariate frequency distribution is classified further into three
categories :
(i) Series of Individual observations,
(ii) Discrete frequency distribution, and
(iii) Continuous frequency distribution.
Series of individual observations, is a simple listing of items of each
observation. If marks of 20 students in statistics of a class are given
individually, it will form a series of Individual observations.
Marks obtained in Statistics:
Roll Nos. 1 2 3 4 5 6 7 8 9 10 11 12 13 14 15 16 17 18 19 20
Marks 60 71 80 41 94 33 81 41 78 66 85 35 61 55 98 52 50 91 30 88

Marks in Ascending Order Marks in Descending Order


30 98
33 94
35 91
41 88
41 85
50 81
52 80
55 78
60 71
61 66

4 PAGE
© Department of Distance & Continuing Education, Campus of Open Learning,
School of Open Learning, University of Delhi
BUSINESS STATISTICS

Marks in Ascending Order Marks in Descending Order Notes


66 61
71 60
78 55
80 52
81 50
85 41
88 41
91 35
94 33
98 30
Discrete Frequency Distribution : In a discrete series, the data are
presented in such a way that exact measurements of units are indicated.
The observations are arranged into group by using the method of tally bars.
In a discrete frequency distribution. With the help of tally bars frequency
can be count the number of times each observation in the given data.
The column shows all values of the variable. In the second column, a
vertical bar called tally bar against the variable, we write a particular
value has occurred four times, for the fifth occurrence, a cross tally
mark (/) on the four tally bars to make a block of 5. The technique of
putting cross tally bars at every fifth repetition facilitates the counting
of the number of occurrences of the value. After putting tally bars for
all the values in the data; we count the number of times each value is
repeated and write it against the corresponding value of the variable in
the third column entitled frequency. This type of representation of the
data is called discrete frequency distribution.
We are given marks of 50 students :
70 55 51 42 57 40 26 43 46 41 46 48 33 40 26 40
40 41 43 53 45 60 47 63 53 33 50 40 33 40 26 53
59 33 65 78 39 55 48 15 26 43 59 51 39 15 45 26
60 15
We can construct a discrete frequency distribution from the above given
marks.

PAGE 5
© Department of Distance & Continuing Education, Campus of Open Learning,
School of Open Learning, University of Delhi
B.COM. (PROGRAMME)

Notes Marks of 50 Students


Marks Tally Bars Frequency
15 ||| 3
26 |||| 5
33 |||| 4
39 || 2
40 |||| 5
41 || 2
42 | 1
43 ||| 3
45 | 2
46 || 2
47 | 1
48 || 2
50 | 1
51 || 2
53 ||| 3
55 ||| 3
57 | 1
59 || 2
60 | 1
61 | 1
63 | 1
65 | 1
70 | 1
78 | 1
Total 50
The presentation of the data in the form of a discrete frequency distribution
is better than arranging but it does not condense the data as needed and
is quite difficult to understand and comprehend. This distribution is quite
simple in case the values of the variable are repeated otherwise there
will be hardly any condensation.
Continuous Frequency Distribution : If the identity of the units about
a particular information is collected, is not relevant nor is the order in
which the observations occur, then the first step of condensation is to

6 PAGE
© Department of Distance & Continuing Education, Campus of Open Learning,
School of Open Learning, University of Delhi
BUSINESS STATISTICS

classify the data into different classes by dividing the entire group of Notes
values of the variable into a suitable number of groups and then recording
the number of observations in each group. Thus, if we divide the total
range of values of the variable (marks of 50 students) i.e. 78 – 15 =
63 into groups of 10 each, then we shall get (63/10) 6 groups and the
distribution of marks is displayed by the following frequency distribution :

Marks of 50 students
Marks (×) Tally Bars Number of Students
(f)
15–25 ||| 3
25–34 |||| |||| 9
35–45 |||| |||| ||| 13
45–55 |||| |||| ||| 13
55–65 |||| |||| 9
65–75 || 2
75–85 | 1
Total 50
The various groups into which the values of the variable are classified
are known as classes, the length of the class interval (10) is called the
width of magnitude of the class. Two values, specifying the class, are
called the class limits. The presentation of the data into continuous classes
with the corresponding frequencies is known as continuous frequency
distribution. There are two methods of classifying the data according to
class intervals :
(i) Exclusive method
(ii) Inclusive method
In an exclusive method, the class intervals are fixed in such a manner
that upper limit of one class becomes the lower limit of the following
class. Moreover, an item equal to the upper limit of a class would be
excluded from that class and included subsequently in the next class. The
following data are classified on this basis.
Income (Rs.) No. of Persons
200–250 50
250–300 100

PAGE 7
© Department of Distance & Continuing Education, Campus of Open Learning,
School of Open Learning, University of Delhi
B.COM. (PROGRAMME)

Notes Income (Rs.) No. of Persons


300–350 70
350–400 130
400–450 50
450–500 100
Total 500
It is clear from the example that the exclusive method ensures continuity
of the data in as much as the upper limit of one class is the lower limit
of he next class. Therefore, 50 persons have their incomes between 200
to 249.99 and a person whose income is 250 shall be included in the
next class of 250–300.
According to the inclusive method, an item equal to upper limit of a
class is included in that class itself. The following table demonstrates
this method.
Income (Rs.) No. of Persons
200–249 50
250–299 100
300–349 70
350–399 130
400–449 50
450–499 100
Total 500
Hence in the class 200–249, we include persons whose income is between
Rs. 200 and Rs. 249.

1.4 Principles of Frequency Distribution


Though the great importance of classification in statistical analysis, no
hard and fast rules be laid down for it. A statistician uses his discretion
for classifying a frequency distribution and sound experience, wisdom,
skill and suitability for an appropriate classification of the data. However,
the following guidelines must be considered to construct a frequency
distribution:
1. Types of Classes : The classes should be clearly defined and should
not lead to any ambiguity. They should be exhaustive and mutually
exclusive so that any value of variable corresponds to only class.

8 PAGE
© Department of Distance & Continuing Education, Campus of Open Learning,
School of Open Learning, University of Delhi
BUSINESS STATISTICS

2. Number of Classes : The choice about the number of classes into Notes
which a given frequency distribution should be divided depends
upon:
(i) The total frequency which means the total number of observations
in the distribution.
(ii) The nature of the data which means the size or magnitude of
the values of the variable.
(iii) The desired accuracy.
(iv) The convenience regarding computation of the various descriptive
measures of the frequency distribution such as means, variance
etc.
The number of classes should neither be too small nor too large. In case
the classes are few, the classification becomes very broad and rough
which might obscure some important features and characteristics of the
data. The accuracy of the results decreases as the number of classes
becomes smaller. On the other hand, too many classes will result in
very few frequencies in each class. This will give an irregular pattern
of frequencies in different classes thus makes the frequency distribution
irregular. Moreover a large number of classes will render the distribution
too unwieldy to handle. The computational work for further processing
of the data will become quite tedious and time consuming without any
proportionate gain in the accuracy of the results. Hence a balance should
be maintained between the loss of information in the first case and
irregularity of frequency distribution in the second case, to arrive at a
pleasing compromise giving the optimum number of classes. Normally,
the number of classes should not be less than 5 and more than 20. Prof.
Sturges has given a formula :
k = 1 + 3.322 log n
where k refers to the number of classes and n is the total frequency or
number of observations. The value of k is rounded to the next higher
integer :
If n = 100 k = 1 + 3.322 log 100 = 1 + 6.6448 = 8
If n = 10,000 k = 1 + 3.322 log 10,000 = 1 + 13.288 = 14

PAGE 9
© Department of Distance & Continuing Education, Campus of Open Learning,
School of Open Learning, University of Delhi
B.COM. (PROGRAMME)

Notes However, this rule should be applied only when the number of observations
are not very small.
Moreover, the number or class intervals should be such that they give
uniform and unimodal distribution which means that the frequencies in
the given classes increase and decrease steadily and there are no sudden
jumps. The number of classes should be an integer preferably 5 or
some multiples of 5, 10, 15, 20, 25 etc. which are quite convenient for
numerical computations.
3. Size of class intervals : Because the size of the class interval is
inversely proportional to the number of classes in a given distribution,
the choice about the size of the class interval will also depend upon the
sound subjective judgment of the statistician. An approximate value of
the magnitude of the class interval say i can be calculated with the help
of Sturge’s Rule :
Range
i=
1 + 3.322 log n
where i stands for class magnitude or interval, Range is calculated by
taking the difference between largest and smallest value of the distribution,
and n refers to total number of observations.
If we are given the following information; n = 400, Largest item = 1300
and Smallest item = 340.
1300 − 340 960 960
then, i = = = = 99.54(100 approx)
1 + 3.322log 400 1 + 3.322 × 2.6021 9.644
Another rule of thumb for determining the size of the class interval is
that the length of the class interval should not be greater than 1 th of the
4
estimated population standard deviation. If 6 is the estimate of population
standard deviation then the length of class interval is given by: i • 
The size of class intervals should be taken as 5 or multiples of 5,10,15 or
20 for easy computations of various statistical measures of the frequency
distribution, class intervals should be so fixed that each class has a
convenient mid-point around which all the observations in that class cluster.
It means that the entire frequency of the class is concentrated at the mid
value of the class. This assumption will be true only if the frequencies
of the different classes are uniformly distributed in the respective class

10 PAGE
© Department of Distance & Continuing Education, Campus of Open Learning,
School of Open Learning, University of Delhi
BUSINESS STATISTICS

intervals. It is always desirable to take the class intervals of equal or Notes


uniform magnitude throughout the frequency distribution.
4. Class boundaries : If in a grouped frequency distribution there are gaps
between the upper limit of any class and lower limit of the succeeding
class (as in case of inclusive type of classification), there is a need to
convert the data into a continuous distribution by applying a correction
factor for continuity for determining new classes of exclusive type. The
lower and upper class limits of new exclusive type classes are called
class boundaries.
If d is the gap between the upper limit of any class and lower
limit of succeeding class, the class boundaries for any class are given
by :
1 ⎫
Upper class boundary = Upper class limit + d ⎪
2 ⎪

1 ⎪
Lower class boundary = Lower class limit − d
2 ⎪⎭
d/2 is called the correction factor.
Let us consider the following example to understand:
Marks Class Boundaries
20 – 24 (20 – 0.5, 24+ 0.5) i.e., 19.5 – 24.5
25 – 29 (25 – 0.5, 29 + 0.5) i.e., 24.5 – 29.5
30 – 34 (30 – 0.5, 34 + 0.5) i.e., 29.5 – 34.5
35 – 39 (35 – 0.5, 39 + 0.5) i.e., 34.5 – 39.5
40 – 44 (40 – 0.5, 44 + 0.5) i.e., 39.5 – 44.5
d 35 − 34 1
Correction factor = = = = 0.5
2 2 2
5. Mid-value or class mark: Mid value or class mark is the value of a
variable which lies exactly at the middle of a class. Mid-value of any class
is obtained on dividing the sum of the upper and lower class limits by 2.
Mid value of a class = 1 [Lower class limit + Upper class limit]
2
The class limits should be selected in such a manner that the observations
in any class are evenly distributed throughout the class interval so that
the actual average of the observations in any class is very close to the
mid-value of the class.

PAGE 11
© Department of Distance & Continuing Education, Campus of Open Learning,
School of Open Learning, University of Delhi
B.COM. (PROGRAMME)

Notes 6. Open end classes : The classification is termed as open end classification
if the lower limit of the first class or the upper limit of the last class
or both are not specified and such classes in which one of the limits is
missing are called open end classes. For example, the classes like the
marks less than 20 or age below 60 years. As far as possible open end
classes should be avoided because in such classes the mid-value cannot
be accurately obtained. But if the open end classes are inevitable then it
is customary to estimate the class mark or mid-value for the first class
with reference to the succeeding class. In other words, we assume that
the magnitude of the first class is same as that of the second class.
Example 1 : Construct a frequency distribution from the following data
by inclusive method taking 4 as the class interval :
10 17 15 22 11 16 19 24 29 18
25 26 32 14 17 20 23 27 30 12
15 18 24 36 18 15 21 28 33 38
34 13 10 16 20 22 29 19 23 31
Solution : Because the minimum value of the variable is 10 which is
a very convenient figure for taking the lower limit of the first class
and the magnitude of the class interval is given to be 4, the classes for
preparing frequency distribution by the Inclusive Method will be 10-13,
14-17, 18-21, 22-25,............38-41.
Frequency Distribution
Class Interval Tally Bars Frequency (f)
10 – 13 |||| 5
14 – 17 |||| ||| 8
18 – 21 |||| ||| 8
22 – 25 |||| || 7
26 – 29 |||| 5
30 – 33 |||| 4
34 – 37 || 2
38 – 41 | 1

12 PAGE
© Department of Distance & Continuing Education, Campus of Open Learning,
School of Open Learning, University of Delhi
BUSINESS STATISTICS

Example 2 : Prepare a statistical table from the following : Notes


Weekly wages (Rs.) of 100 workers of Factory A
88 23 27 28 86 96 94 93 86 99
82 24 24 55 88 99 55 86 82 36
96 39 26 54 87 100 56 84 83 46
102 48 27 26 29 100 59 83 84 48
104 46 30 29 40 101 60 89 46 49
106 33 36 30 40 103 70 90 49 50
104 36 37 40 40 106 72 94 50 60
24 39 49 46 66 107 76 96 46 67
26 78 50 44 43 46 79 99 36 68
29 67 56 99 93 48 80 102 32 51
Solution : The lowest value is 23 and the highest 106. The difference
in the lowest and highest value is 83. If we take a class interval of 10,
nine classes would be made. The first class should be taken as 20–30
instead of 23–33 as per the guidelines of classification.
Frequency Distribution of the Wages of 100 Workers
Wages (Rs.) Tally Bars Frequency (f)
20 – 30 |||| |||| ||| 13
30 – 40 |||| |||| | 11
40 – 50 |||| |||| |||| ||| 18
50 – 60 |||| |||| 10
60 – 70 |||| | 6
70 – 80 |||| 5
80 – 90 |||| |||| |||| 14
90 – 100 |||| |||| || 12
100 – 110 |||| |||| | 11
Total 100

1.5 Graphs
The guiding principles for the graphic representation of the frequency
distributions are precisely the same as for the diagrammatic and graphic
representation of other types of data. The information contained in a
frequency distribution can be shown in graphs which reveals the important

PAGE 13
© Department of Distance & Continuing Education, Campus of Open Learning,
School of Open Learning, University of Delhi
B.COM. (PROGRAMME)

Notes characteristics and relationships that are not easily discernible on a simple
examination of the frequency tables. The most commonly used graphs
for charting a frequency distribution for the general understanding of the
details of the data are :
1. Histogram
2. Frequency polygon
3. Smoothed frequency curves/Frequency Curves
4. Ogives or cumulative frequency curves.

1.5.1 Histogram
The term ‘histogram’ must not be confused with the term ‘historigram’
which relates to time charts. Histogram is the best way of presenting
graphically a simple frequency distribution. The statistical meaning of
histogram is that it is a graph that represents the class frequencies in a
frequency distribution by vertical adjacent rectangles.
While constructing histogram the variable is always taken on the X-axis
and the corresponding frequencies on the Y-axis. Each class is then
represented by a distance on the scale that is proportional to its class-
interval. The distance for each rectangle on the X-axis shall remain
the same in case the class-intervals are uniform throughout; if they are
different the width of the rectangles shall also change proportionately.
The Y-axis represents the frequencies of each class which constitute the
height of its rectangle. We get a series of rectangles each having a class
interval distance as its width and the frequency distance as its height.
The area of the histogram represents the total frequency.
The histogram should be clearly distinguished from a bar diagram. A bar
diagram is one-dimensional i.e., only the length of the bar is important
and not the width, a histogram is two-dimensional, that is, in a histogram
both the length and the width are important. However, a histogram can
be misleading if the distribution has unequal class-intervals and suitable
adjustments in frequencies are not made.
The technique of constructing histogram is explained for :
(i) distributions having equal class-intervals and
(ii) distributions having unequal class-intervals.

14 PAGE
© Department of Distance & Continuing Education, Campus of Open Learning,
School of Open Learning, University of Delhi
BUSINESS STATISTICS

When class-intervals are equal, take frequency on the Y-axis, the variable Notes
on the X-axis and construct rectangles. In such a case the heights of the
rectangles will be proportional to the frequencies.
Example 3 : Draw a histogram from the following data :
Classes Frequency
0 – 10 5
10 – 20 11
20 – 30 19
30 – 40 21
40 – 50 16
50 – 60 10
60 – 70 8
70 – 80 6
80 – 90 3
90 – 100 1
Solution:

When class-intervals are unequal the frequencies must be adjusted before


constructing a histogram. We take that class which has the lowest class-
interval and adjust the frequencies of other classes accordingly. If one
class-interval is twice as wide as the one having the lowest class-interval
we divide the height of its rectangle by two, if it is three times more
we divide it by three etc., the heights will be proportional to the ratios
of the frequencies to the width of the classes.

PAGE 15
© Department of Distance & Continuing Education, Campus of Open Learning,
School of Open Learning, University of Delhi
B.COM. (PROGRAMME)

Notes Example 4 : Represent the following data on a histogram.


Average monthly income of 1035 employees in a construction industry
is given below:
Monthly Income (Rs.) No. of Workers
600 – 700 25
700 – 800 100
800 – 900 150
900 – 1000 200
1000 – 1200 240
1200 – 1400 160
1400 – 1500 50
1500 – 1800 90
1800 or more 20
Solution : Histogram showing monthly incomes of workers
Y
200
NUMBER OF WORKERS

150

100

50

X
600 700 800 900 1000 1100 1200 1300 1400 1500 1800
MONTHLY INCOME

When mid point are given, first we ascertain the upper and lower limits
of each class and then construct the histogram in the same manner.
Example 5 : Draw a histogram of the following distribution :
Life of Electric Lamps Firm A Firm B
in hours
1010 10 287
1030 130 105
1050 482 26
1070 360 230
1090 18 352

16 PAGE
© Department of Distance & Continuing Education, Campus of Open Learning,
School of Open Learning, University of Delhi
BUSINESS STATISTICS

Solution : Since we are given the mid points, we should ascertain the Notes
class limits. To calculate the class limits of various classes, take difference
of two consecutive mid-points and divide the difference by 2, then add
and subtract the value obtained from each mid-point to calculate lower
and higher class-limits.
Life of Electric Frequency Frequency
Lamps Firm A Firm B
1000–1020 10 287
1020–1040 130 105
1040–1060 482 76
1060–1080 360 230
1080–1100 18 352

HISTOGRAM (FIRM A) HISTOGRAM (FIRM A)


500 500

400 400
FREQUENCY

FREQUENCY

300 300

200 200

100 100

1000 1020 1040 1060 1080 1100 1000 1020 1040 1060 1080 1100
LIFE OF LAMPS LIFE OF LAMPS

1.5.2 Frequency Polygon


This is a graph of frequency distribution which has more than four
sides. It is particularly effective in comparing two or more frequency
distributions. There are two ways of constructing a frequency polygon.
(i) We may draw a histogram of the given data and then join by straight
line the mid-points of the upper horizontal side of each rectangle
with the adjacent ones. The figure so formed shall be frequency
polygon. Both the ends of the polygon should be extended to the
base line in order to make the area under frequency polygons equal
to the area under Histogram.

PAGE 17
© Department of Distance & Continuing Education, Campus of Open Learning,
School of Open Learning, University of Delhi
B.COM. (PROGRAMME)

Notes Y
400

NUMBER OF STUDENTS (FREQUENCY)


300

200

100

CLASS MARK

(ii) Another method of constructing frequency polygon is to take the


mid-points of the various class-intervals and then plot the frequency
corresponding to each point and join all these points by straight
lines.
The figure obtained by both the methods would be equal.
Y
400

1 2
NUMBER OF STUDENTS (FREQUENCY)

5
4 5
300 3
5

200 7

3
2

100 r

2 8

1 9
X
0

CLASS MARK

Frequency polygon has an advantage over the histogram. The frequency


polygons of several distributions can be drawn on the same axis, which
makes comparisons possible whereas histogram cannot be usefully

18 PAGE
© Department of Distance & Continuing Education, Campus of Open Learning,
School of Open Learning, University of Delhi
BUSINESS STATISTICS

employed in the same way. To compare histograms we draw them on Notes


separate graphs.

1.5.3 Smoothed Frequency Curve/Frequency Curves


A smoothed frequency curve is popularly known as Frequency Curve.
A smoothed frequency curve can be drawn through the various points
of the polygon. The curve is drawn by free hand in such a manner that
the area included under the curve is approximately the same as that of
the polygon. The object of drawing a smoothed curve is to eliminate as
far as possible all accidental variations which exists in the original data,
while smoothening, the top of the curve would overtop the highest point
of polygon particularly when the magnitude of the class interval is large.
The curve should look as regular as possible and all sudden turns should
be avoided. The extent of smoothening would depend upon the nature
of the data. For drawing smoothed frequency curve it is necessary to
first draw the polygon and then smoothen it. We must keep in mind the
following points to smoothen a frequency graph :
(i) Only frequency distribution based on samples should be smoothened.
(ii) Only continuous series should be smoothened.
(iii) The total area under the curve should be equal to the area under
the histogram or polygon.
The diagram given below will illustrate the point:
HISTOGRAM FREQUENCY POLYGON AND CURVE

50 HISTOGRAM

40
FREQUENCY
CURVE
30
NO. OF LEAVES

20

FREQUENCY
10
POLYGON
6.5
7.5
8.5
9.5
10.5
11.5
12.5
13.5
14.5

LENGTH OF LEAVES (cm)

PAGE 19
© Department of Distance & Continuing Education, Campus of Open Learning,
School of Open Learning, University of Delhi
B.COM. (PROGRAMME)

Notes 1.5.4 Cumulative Frequency Curves or Ogives


We have discussed the charting of simple distributions where each frequency
refers to the measurement of the class-interval against which it is placed.
Sometimes it becomes necessary to know the number of items whose
values are greater or less than a certain amount. We may, for example, be
interested in knowing the number of students whose weight is less than 65
lbs. or more than say 15.5 lbs. To get this information, it is necessary to
change the form of frequency distribution from a simple to a cumulative
distribution. In a cumulative frequency distribution of the frequency of
each class is made to include the frequencies of all the lower or all the
upper classes depending upon the manner in which cumulation is done.
The graph of such a distribution is called a cumulative frequency curve
or an Ogive. There are two method of constructing ogives, namely :
(i) Less than method, and
(ii) More than method.
In the less than method, we start with the upper limit of each class and
go on adding the frequencies. When these frequencies are plotted we get
a rising curve.
In the more than method, we start with the lower limit of each class and
we subtract the frequency of each class from total frequencies. When
these frequencies are plotted, we get a declining curve.
This example would illustrate both types of ogives.
Example 6 : Draw ogives by both the methods from the following data.
Distribution of weight of the students of a college (lbs.)
Weights No. of Students
90.5–100.5 5
100.5–110.5 34
110.5–120.5 139
120.5–130.5 300
130.5–140.5 367
140.5–150.5 319
150.5–160.5 205
160.5–170.5 76
170.5–180.5 43

20 PAGE
© Department of Distance & Continuing Education, Campus of Open Learning,
School of Open Learning, University of Delhi
BUSINESS STATISTICS

Weights No. of Students Notes


180.5–190.5 16
190.5–200.5 3
200.5–210.5 4
210.5–220.5 3
220.5–230.5 1
Solution : First of all we shall find out the cumulative frequencies of
the given data by less than method.
Less than (Weights) Cumulative frequency
100.5 5
110.5 39
120.5 178
130.5 478
140.5 845
150.5 1164
160.5 1369
170.5 1445
180.5 1488
190.5 1504
200.5 1507
210.5 1511
220.5 1514
230.5 1515
Plot these frequencies and weights on a graph paper. The curve formed
is called an Ogive.

PAGE 21
© Department of Distance & Continuing Education, Campus of Open Learning,
School of Open Learning, University of Delhi
B.COM. (PROGRAMME)

Notes
1500

1250

1000

CUMULATIVE FREQUENCY
750

500

250

X
0
90.5
100.5
110.5
120.5
130.5
140.5
150.5
160.5
170.5
180.5
190.5
200.5
210.5
220.5
230.5
SIZES

Now we calculate the cumulative frequencies of the given data by more


than method.
More than (Weights) Cumulative frequencies
90.5 1515
100.5 1510
110.5 1476
120.5 1337
130.5 1037
140.5 670
150.5 351
160.5 146
170.5 70
180.5 27
190.5 11
200.5 8
210.5 4
220.5 1
By plotting these frequencies on a graph paper, we will get a declining
curve which will be our cumulative frequency curve or Ogive by More
than method.

22 PAGE
© Department of Distance & Continuing Education, Campus of Open Learning,
School of Open Learning, University of Delhi
BUSINESS STATISTICS

Y
Notes
1500

1250

1000
CUMULATIVE FREQUENCY

750

500

250

X
0
90.5
100.5
110.5
120.5
130.5
140.5
150.5
160.5
170.5
180.5
190.5
200.5
210.5
220.5
230.5

SIZES

Although the graphs are a powerful and effective media of presenting


statistical data, they are not under all circumstances and for all purposes
complete substitutes for tabular and other forms of presentation. The
specialist in this field is one who recognizes not only the advantages
but also the limitations of these techniques. He knows when to use and
when not to use these methods and from his experience and expertise is
able to select the most appropriate method for every purpose.
Example 7 : Draw an ogive by less than method and determine the number
of companies getting profits between Rs. 45 crores and Rs. 75 crores :
Profits (Rs. crores) No. of Companies
10–20 8
20–30 12
30–40 20
40–50 24
50–60 15
60–70 10
70–80 7
80–90 3
90–100 1

PAGE 23
© Department of Distance & Continuing Education, Campus of Open Learning,
School of Open Learning, University of Delhi
B.COM. (PROGRAMME)

Notes Solution :
Profit (Rs. No. of OGIVE BY LESS THAN METHOD

Crores) Companies
Less than 20 8 100
92
Less than 30 20 80

NO. OF COMPANIES
92–51 = 41
Less than 40 40
60
Less than 50 64 51
Less than 60 79 40

Less than 70 89 20
Less than 80 96
Less than 90 99 20 30 40 45 50 60 70 75 80 85

Less than 100 100 PROFIT RS. IN CRORES

It is clear from the graph that the number of companies getting profits
less than Rs. 75 crores is 92 and the number of companies getting profits
less than Rs. 45 crores is 51. Hence the number of companies getting
profits between Rs. 45 crores and Rs. 75 crores is 92–51 = 41.
Example 8 : The following distribution is with regard to weight in grams
of mangoes of a given variety. If mangoes of weight less than 443 grams
be considered unsuitable for foreign market, what is the percentage of
total yield suitable for it? Assume the given frequency distribution to
be typical of the variety:
Weight in gms. No. of Mangoes
410–419 10
420–429 20
430–439 42
440–449 54
450–459 45
460–469 18
470–479 7
Draw an ogive of ‘more than’ type of the above data and deduce how
many mangoes will be more than 443 grams.
Solution : Mangoes weighing more than 443 gms. are suitable for foreign
market. Number of mangoes weighing more than 443 gms lies in the last
four classes. Number of mangoes weighing between 444 and 449 grams
would be:

24 PAGE
© Department of Distance & Continuing Education, Campus of Open Learning,
School of Open Learning, University of Delhi
BUSINESS STATISTICS

6 324 Notes
× 54 = = 32.4
10 10
Total number of mangoes weighing more than 443 gms. = 32.4 +
45 + 18 + 7 = 102.4
102.4
Percentage of mangoes = = × 100 = 52.25
196
Therefore, the percentage of the total mangoes suitable for foreign
market is 52.25.
OGIVE BY MORE THAN METHOD
Weight more No. of Mangoes
than (gms.)
410 196 OGIVE BY MORE THAN METHOD
200
420 186 180

430 166 160


No. of mangoes

140
440 124 120

450 70 100
80
460 25 60

470 7 40
20
410 420 430 440 450 460 470
Weight in grams

From the graph it can be seen that there are 103 mangoes whose weight
will be more than 443 gms and are suitable for foreign market.

1.6 Summary
A frequency distribution aims to reduce the size of the given set of data
for a better comprehension. An array, which is an arrangement of data
in an ascending or descending order of magnitude, is a useful step in
preparing a frequency distribution. To prepare a frequency distribution, we
have to decide about the class intervals to be taken. The width of class
intervals depends on the number of classes. The number of classes should
not be very small or very large. Given values are considered one by one
and placed in appropriate class intervals. The number of observations in
each class is called the class frequency.
The class intervals may be overlapping Like 10–20, 20–30, etc. or
inclusive like 10–19, 20–29, etc. Inclusive class intervals should be

PAGE 25
© Department of Distance & Continuing Education, Campus of Open Learning,
School of Open Learning, University of Delhi
B.COM. (PROGRAMME)

Notes transformed into exclusive classes, depending on the way the given data
are recorded. Class mid-points are the points that lie halfway between the
two class limits. The frequencies of a distribution can also be cumulated
in ascending or descending order. They are known as ACF and DCF.
respectively. The ACF are ‘less than’ cumulative frequencies while the
DCF are ‘more than’ cumulative frequencies. Absolute class frequencies
may also be expressed as relative frequencies, either as proportions or
percentages. A frequency distribution may have class intervals with equal
or unequal width.
A frequency distribution may be shown graphically by a histogram and
frequency polygon. A histogram consists of bars drawn over class limits
with heights of bars such that the areas of the bars are proportional
to the frequencies of various class intervals. A frequency polygon is a
line chart and is drawn by joining points given by the class mid-points
and class frequencies. Cumulative frequencies arc shown graphically by
means of gives.

1.7 Self-Assessment Questions


Exercise 1 : True or False Statements
(i) Before constructing a frequency distribution, it is necessary that the
data be arranged as an array.
(ii) If the class intervals are given in the exclusive form as 10–20, 20–30,
etc. then a value exactly equal to 20 may be included in either of
these classes.
(iii) In the case of inclusive class intervals, the class mid-points are
determined only after converting them into exclusive form.
(iv) The number and width of class intervals are determined independently
of each other.
(v) A frequency distribution must have all class intervals of equal width.
(vi) A distribution can have both ends open.
(vii) A bivariate frequency distribution can be prepared only when both
the variables involved are discrete or are continuous.
(viii) Relative frequencies are obtained by dividing the frequencies of
various classes by the width of the respective classes.

26 PAGE
© Department of Distance & Continuing Education, Campus of Open Learning,
School of Open Learning, University of Delhi
BUSINESS STATISTICS

(ix) Frequency density is another name for relative frequency. Notes


(x) The proportionate frequencies facilitate comparison between distributions
better than absolute frequencies.
(xi) It is never possible to calculate absolute frequencies from the
proportionate frequencies for a distribution.
(xii) In presenting a distribution graphically, the variable is shown
horizontally while the frequencies are shown vertically.
(xiii) It is necessary that the widths of bars representing various class
intervals of a frequency distribution be always equal.
(xiv) The areas covered by a histogram and a frequency polygon are
equal.
(xv) Strictly speaking, a histogram cannot be drawn for an open-ended
distribution.
Ans.
(i) F (ii) F (iii) T (iv) F (v) F (vi) T
(vii) F (viii) F (ix) F (x) T (xi) F (xii) T
(xiii) F (xiv) T (xv) T
Exercise 2 : Questions and Answers
(i) What is a frequency distribution? Explain the process of preparing
a univariate frequency distribution.
(ii) Explain the following:
(a) Grouping error
(b) Cumulative frequencies
(c) Relative frequencies
(d) Frequency density
(iii) What is a bivariate frequency distribution? How is it constructed?
Can we prepare a bivariate frequency distribution if one of the
variables is discrete and the other is continuous?
(iv) Explain the drawing of histogram when class intervals are equal and
when they are not equal.
(v) What are ogives? How are they constructed and what information
do they provide?

PAGE 27
© Department of Distance & Continuing Education, Campus of Open Learning,
School of Open Learning, University of Delhi
B.COM. (PROGRAMME)

Notes (vi) From the time cards of a factory, the following information has been
obtained about the number of days each one of the 48 workers has
reported late for the work during the last month:
3 0 5 0 6 2 1 0 4 6 5 2 1 1 1 3 4 2 2 5 6 3 0 2
2 3 2 5 4 2 4 3 5 2 2 2 4 6 4 0 3 1 1 4 5 2 1 1

Prepare a frequency distribution using this information. Also, indicate


percentage frequencies.
(vii) XYZ Company collected data regarding the number of interviews
required for each of its 40 sales persons to make their most recent
sale. Following are those numbers:
102 95 90 90 101 60 80 113 102 110 126 66 121 116
139 72 101 93 114 99 112 105 97 100 99 115 129 111
119 81 91 93 119 113 128 110 75 87 107 108
(a) Construct a frequency distribution with six class intervals.
(b) Construct a histogram from the data.
(viii) If the class mid-points in a frequency distribution of weights of a
group of students are 125, 132, 139, 146, 153, 160, 167, 174 and
181 lbs. find:
(a) Size of the class interval.
(b) Class limits assuming weights have been measured to the nearest
pound.
(ix) Convert the following class intervals into exclusive form:
(a) (b) (c)
Diameters (in cm) Age in years Height in inches
0.5–0.9 5–9 60–64
1.0–1.4 10–14 65–69
1.5–1.9 15–24 70–74
2.0–2.4 25–39 75–79
2.5–2.9 40–59 80–84
3.0–3.4 60–79 85–89
(x) The monthly profits earned by 100 companies during the last
financial year are given below:

28 PAGE
© Department of Distance & Continuing Education, Campus of Open Learning,
School of Open Learning, University of Delhi
BUSINESS STATISTICS

Monthly Profit No. of Monthly Profit No. of Notes


(Rs. lakhs) Companies (Rs. lakhs) Companies
20–30 4 60–70 15
30–40 8 70–80 10
40–50 18 80–90 8
50–60 30 90–100 7
(a) Draw an ogive by ‘less than’ method and ‘more than’ method.
(b) Obtain the limits of monthly profits of central 50 per cent of
the companies and check these values against the formula
calculated values.
(xi) The salary distribution of employees of a company is given below
Salary (in ’000 Rs.) No. of Employees
8–10 18
10–12 32
12–14 70
14–16 88
16 –18 64
18–20 44
20–22 24
22–24 10
(a) Show these data by means of a histogram and frequency polygon
on the same graph.
(b) Draw a more-than ogive and using it estimate (i) the number of
employees earning more than Rs. 16,500; and (ii) the number
of employees earning less than Rs. 13,000.
(xii) The following table gives the distribution of weekly income of 160
families:
Weekly Income (Rs.) No. of Families
2,000–4,000 20
4,000–6,000 40
6,000–8,000 50
8,000–12,000 32
12,000–16,000 16
16,000–20,000 2

PAGE 29
© Department of Distance & Continuing Education, Campus of Open Learning,
School of Open Learning, University of Delhi
B.COM. (PROGRAMME)

Notes Draw a ‘less than’ ogive and answer the following from it:
(a) What are the limits within which incomes of the middle 50 per cent
of the families lie?
(b) It is decided that 80 per cent of the families should pay income tax.
What is the minimum taxable income?
(c) What is the minimum income of the richest 30 per cent of the
families?
Ans.
(x) (b) = 47 and 70 (xi) (b) = 126 and 86
(xii) (a) = 5000 – 9250
(b) = 4600
(c) = 8250

30 PAGE
© Department of Distance & Continuing Education, Campus of Open Learning,
School of Open Learning, University of Delhi
L E S S O N

2
Measures of Central
Tendency–Mathematical
and Positional Averages
STRUCTURE
2.1 Learning Objectives
2.2 What is Central Tendency?
2.3 Objectives of Central Tendency
2.4 Characteristics
2.5 Types of Averages
2.6 Mean
2.7 Geometric Mean
2.8 Harmonic Mean
2.9 Median
2.10 Other Positional Averages
2.11 Calculation of Missing Frequencies
2.12 Mode
2.13 Summary
2.14 Self-Assessment Questions

2.1 Learning Objectives


After reading this lesson, you should be able to:
‹ Learn the meaning of central tendency and other averages.
‹ Learn the process of computing arithmetic mean, weighted Mean, Harmonic mean,
Geometric mean, Median, Deciles, Quartiles, Percentiles and Mode under different
situations.

PAGE 31
© Department of Distance & Continuing Education, Campus of Open Learning,
School of Open Learning, University of Delhi
B.COM. (PROGRAMME)

Notes ‹ Comprehend mathematical properties of Arithmetic average.


‹ Learn specific uses of different averages.

2.2 What is Central Tendency?


One of the important objectives of statistical is to find out various
numerical values which explains the inherent characteristics of a frequency
distribution. The first of such measures are averages. The averages are
the measures which condense a huge unwieldy set of numerical data
into single numerical values which represent the entire distribution. The
inherent inability of the human mind to a large body of numerical data
compels us to few constants that will describe the data. Averages provide
us the gist and give a bird’s eye view of the huge mass of unwieldy
numerical data. Averages are the typical values around which other items
of the distribution congregate. This value lie between the two extreme
observation of the distribution and give us an idea about the concentration
of the values in the central part of the distribution. They are called the
measures of central tendency.
Averages are also called measures of location since they enable us to
locate the position or place of the distribution in question. Averages are
statistical constants which enables us to comprehend in a single value
the significance of the whole. According to Croxton and Cowden, an
average value is a single value within the range of the data that is used
to represent all the values in that series. Since an average is somewhere
within the range of the data, it is sometimes called a measure of central
value. An average, known as the measure of central tendency, is the
most typical representative item of the group to which it belongs and
which is capable of revealing all important characteristics of that group
or distribution.

2.3 Objectives of Central Tendency


The most important object of calculating an average or measuring central
tendency is to determine a single figure which may be used to represent
a whole series involving magnitudes of the same variable.

32 PAGE
© Department of Distance & Continuing Education, Campus of Open Learning,
School of Open Learning, University of Delhi
BUSINESS STATISTICS

Second object is that an average represents the entire data, it facilitates Notes
comparison within one group or between group of data. Thus, the
performance of the members of a group can be compared with the average
performance of different group.
Third object is that an average helps in computing various other statistical
measures such as dispersion, skewness. kurtosis etc.

2.4 Characteristics
An average represents the statistical data and it is used for purposes of
comparison, it must possess the following properties.
1. It must be rigidly defined and not left to the mere estimation of
the observer. If the definition is rigid, the computed value of the
average obtained by different persons shall be similar.
2. The average must be based upon all values given in the distribution.
If the item is not based on all values it might not be representative
of the entire group of data.
3. It should be easily understood. The average should possess simple
and obvious properties. It should be too abstract for the common
people.
4. It should be capable of being calculated with reasonable care and
rapidity.
5. It should be stable and unaffected by sampling fluctuations.
6. It should be capable of further algebraic manipulation.

2.5 Types of Averages


Different methods of measuring “Central Tendency” provide us with
different kinds of averages. The following are the main types of averages
that are commonly used :
(A) Mean
(i) Arithmetic mean
(ii) Weighted mean
(iii) Geometric mean

PAGE 33
© Department of Distance & Continuing Education, Campus of Open Learning,
School of Open Learning, University of Delhi
B.COM. (PROGRAMME)

Notes (iv) Harmonic mean


(B) Median
(C) Mode

2.6 Mean

2.6.1 Arithmetic Mean


The arithmetic mean of a series is the quotient obtained by dividing the
sum of the values by the number of items. In algebraic language, if X1,
X2, X3,.........Xn are the n values of a variate X, then the Arithmetic Mean
(X) is defined by the following formula:
1
X = (X1 + X 2 + X3 + ............. + X n )
n
1⎛ n ⎞ ∑X
= ⎜ ∑ Xi ⎟ =
n ⎝ i=1 ⎠ N
Example 1 : The following are the monthly salaries (Rs.) of ten employees
in an office. Calculate the mean salary of the employees: 250, 275, 265,
280, 400, 490, 670, 890, 1100, 1250.
Solution :

∑X
X =
N
250 + 275 + 265 + 280 + 400 + 490 + 670 + 890 + 1100 + 1250 5870
X= = = Rs. 587
10 10
Short-cut Method : Direct method is suitable where the number of items
is moderate and the figures are small sizes and integers. But if the number
of items is large and/or the values of the variate are big, then the process
of adding together all the values may be a lengthy process. To overcome
this difficulty of computations, a short-cut method may be used. Short
cut method of computation is based on an important characteristic of the
arithmetic mean, that is, the algebraic sum of the deviations of a series
of individual observations from their mean is always equal to zero. Thus
deviations of the various values of the variate from an assumed mean

34 PAGE
© Department of Distance & Continuing Education, Campus of Open Learning,
School of Open Learning, University of Delhi
BUSINESS STATISTICS

computed and the sum is divided by the number of items. The quotient Notes
obtained is added to the assumed mean to find the arithmetic mean.
Σdx
Symbolically, X = A + , where A is assumed mean and deviations
N
or dx = (X – A).
We can solve the previous example by short-cut method.
Computation of Arithmetic Mean
Serial Salary (Rupees) Deviations from assumed mean
Number X where dx = (X – A), A = 400
1. 250 – 150
2. 275 – 125
3. 265 – 135
4. 280 – 120
5. 400 0
6. 490 + 90
7. 670 + 270
8. 890 + 490
9. 1100 + 700
10. 1250 + 850
N = 10 Σdx = 1870

Σdx
X=A+
N
By substituting the values in the formula, we get
1870
X = 400 + = Rs. 587
10
Computation of Arithmetic Mean in Discrete Series. In discrete series,
arithmetic mean may be computed by both direct and short cut methods.
The formula according to direct method is:
1 Σ( fX )
X= ( f1 X 1 + f 2 X 2 + ........... + f n X n ) =
n N
where the variable values X1, X2, ........Xn have frequencies f1, f2 ,........fn
DQG 1  Ȉf.

PAGE 35
© Department of Distance & Continuing Education, Campus of Open Learning,
School of Open Learning, University of Delhi
B.COM. (PROGRAMME)

Notes Example 2 : The following table gives the distribution of 100 accidents
during area days of the week in a given month. During a particular
month there were 5 Fridays and Saturdays and only four each of other
days. Calculate the average number of accidents per day.
Days : Sun Mon Tue Wed Thur Fri Sat Total
Number of
accidents : 20 22 10 9 11 8 20 = 100
Solution :
Calculation of Number of Accidents per Day
Day No. of No. of days in Total accidents
Accidents month
X f fX
Sunday 20 4 80
Monday 22 4 88
Tuesday 10 4 40
Wednesday 9 4 36
Thursday 11 4 44
Friday 8 5 40
Saturday 20 5 100
100 N = 30 Σf X = 428

ΣfX 428
X= = = 14.27 = 14 accidents per day
N 30
The formula for computation of arithmetic mean according to short cut
method is
Σfdx
X=A+ where A is Assumed mean, dx = (X – A) and
N
We can solve the previous example by short-cut method as given below :
Calculation of Average Accidents per day
Day X dx = X–A (where f fdx
A = 10)
Sunday 20 + 10 4 + 40
Monday 22 + 12 4 + 48
Tuesday 10 + 0 4 + 0

36 PAGE
© Department of Distance & Continuing Education, Campus of Open Learning,
School of Open Learning, University of Delhi
BUSINESS STATISTICS

Day X dx = X–A (where f fdx Notes


A = 10)
Wednesday 9 – 1 4 – 4
Thursday 11 + 1 4 + 4
Friday 8 – 2 5 – 10
Saturday 20 + 10 5 + 50
30 128
Σfdx 128
X=A+ = 10 + = 14.27 = 14 accidents per day
N 30
Calculation of Arithmetic Mean for Continuous Series : The arithmetic
mean can be computed both by direct and short-cut method. In addition, a
coding method or step deviation method is also applied for simplification
of calculations. In any case, it is necessary to find out the mid-values of
the various classes in the frequency distribution before arithmetic mean
of the frequency distribution can be computed. Once the mid-points of
various classes are found out, then the process of the calculation of
arithmetic mean is same as in the case of discrete series. In case of direct
method, the formula to be used :
Σfm
X= , when m= mid points of various classes and N = the total
N
frequency
In the short-cut method, the following formula is applied:
Σfdx
X=A+ where dx = (m – A) and N =
N
The short-cut method can further be simplified in practice and is named
coding method. The deviations from the assumed mean are divided by
a common factor to reduce their size. The sum of the products of the
deviations and frequencies is multiplied by this common factor and then
it is divided by the total frequency and added to the assumed mean.
Symbolically
Σfd ' x m− A
X=A+ × i , where d ' x = and i = common factor
N i
Example 3 : Following is the frequency distribution of marks obtained
by 50 students in a test of Statistics :

PAGE 37
© Department of Distance & Continuing Education, Campus of Open Learning,
School of Open Learning, University of Delhi
B.COM. (PROGRAMME)

Notes Marks Number of Students


0–10 4
10–20 6
20–30 20
30–40 10
40–50 7
50–60 3
Calculate arithmetic mean by:
(i) Direct method,
(ii) Short-cut method, and
(iii) Coding method.
Solution :
Calculation of Arithmetic Mean
X f m fm dx = m – A m–A fdx fdƍx
d'x =
i
(where A = 25) (where i = 10)
0–10 4 5 20 – 20 – 2 – 80 – 8
10–20 6 15 90 – 10 – 1 – 60 – 6
20–30 20 25 500 0 0 0 0
30–40 10 35 350 + 10 + 1 100 + 10
40–50 7 45 315 + 20 + 2 140 + 14
50–60 3 55 165 + 30 + 3 90 + 9
N = 50 Ȉfm = 1440 Ȉfdx = 190 Ȉfd 'x = +19
Direct Method :
Σfm 1440
X= = = 28.8 marks.
N 50
Short-cut Method:
Σfdx 190
X=A+ = 25 + = 28.8 marks.
N 50
Coding Method:
Σfd ' x 19
X=A+ × i = 25 + × 10 = 25 + 3.8 = 28.8 marks.
N 50

38 PAGE
© Department of Distance & Continuing Education, Campus of Open Learning,
School of Open Learning, University of Delhi
BUSINESS STATISTICS

We can observe that answer of average marks i.e. 28.8 is identical by Notes
all methods.

2.6.2 Mathematical Properties of Arithmetic Mean


(i) The sum of the deviation of a given set of individual observations
from the arithmetic mean is always zero.
,WPHDQVWKDW™ ;;ࡃ  IRULQGLYLGXDOREVHUYDWLRQVDQG™¦ ;;ࡃ )
= 0 for a frequency distribution.
This property can be easily understood with the given example.
Example:- X : 1 2 3 4 5

Algebraic Sum of Deviation from Mean


X X-Xࡃ (Whereas Xࡃ = 3)
1 -2
2 -1
3 0
4 1
5 2
™X =15 ™ ;;ࡃ ) = 0
™X 15
Here, Xࡃ = = = 3
n 5
Symbolically, It is due to this property that the arithmetic mean
is characterised as the centre of gravity i.e., the sum of positive
deviations from the mean is equal to the sum of negative deviations.
(ii) The sum of squares of deviations of a set of observations is the
minimum when deviations are taken from the arithmetic average.
6\PEROLFDOO\ VPDOOHU WKDQ Ȉ ; ± DQ\ RWKHU YDOXH 2.
We can verify the above properties with the help of the following
data :

PAGE 39
© Department of Distance & Continuing Education, Campus of Open Learning,
School of Open Learning, University of Delhi
B.COM. (PROGRAMME)

Notes Values Deviations from Deviations from


assumed mean
X (X – X ) ( X – X )2 (X – A) ( X – A) 2
3 – 6 36 – 7 49
5 – 4 16 – 5 25
10 1 1 0 0
12 3 9 2 4
15 6 36 5 25
Total = 45 0 98 – 5 103
∑ X 45
X= = = 9 , where A (assumed mean) = 10
n 5
(iii) If each value of a variable X is increased or decreased or multiplied
by a constant k, the arithmetic mean also increases or decreases or
multiplies by the same constant k.
Like, Z = X + k (whereas X and Z are variables),
Then zࡃ = Xࡃ + k (whereas Xࡃ and zࡃ are Arithmetic Mean)
Example:-
Algebraic Sum of Deviation from Mean
X Z = X+k (let k is 5)
10 15
20 25
30 35
40 45
50 55
™X=150 ™Z = 175
™X 150 ™Z 150
Here, Xࡃ = = = 30 Then, zࡃ = = = 35
n 5 n 5
It is clearly from the above example that if each observation of
variable X is increased or decreased or multiplied by a constant
k, the arithmetic mean also increases or decreases or multiplies by
the same constant k.

40 PAGE
© Department of Distance & Continuing Education, Campus of Open Learning,
School of Open Learning, University of Delhi
BUSINESS STATISTICS

(iv) If we are given the arithmetic mean and number of items of two Notes
or more groups, we can compute the combined average of these
groups by applying the following formula :
N1X1 + N 2 X 2
X12 =
N1 + N 2

where X12 refers to combined average of two groups,


X1 refers to arithmetic mean of first group,

X 2 refers to arithmetic mean of second group,


N1 refers to number of items of first group, and
N2 refers to number of items of second group
We can understand the property with the help of the following examples.
Example 4 : The average marks of 25 male students in a section is 61
and average marks of 35 female students in the same section is 58. Find
combined average marks of 60 students.
Solution : We are given the following information,
X1 = 61, N1 = 25, X 2 = 58, N2 = 35

N1X1 + N 2 X 2 (25 × 61) + (35 × 58)


Apply X12 = = = 59.25 marks.
N1 + N 2 25 + 35
Example 5 : The mean wage of 100 workers in a factor, running two
shifts of 60 and 40 workers respectively is Rs. 38. The mean wage of 60
workers in morning shift is Rs. 40. Find the mean wage of 40 workers
working in the evening shift.
Solution : We are given the following information,
X1 = 40, N1= 60, X 2 = ?, N2 = 40, X12 = 38, and N = 100

N1X1 +N 2 X 2
Apply X12 =
N1 + N 2

(60 × 40) + (40 × X 2 )


38 = or 3800 = 2400 + 40 X 2
60 + 40
3800 − 2400
X2 = = 35.
40

PAGE 41
© Department of Distance & Continuing Education, Campus of Open Learning,
School of Open Learning, University of Delhi
B.COM. (PROGRAMME)

Notes Example 6 : The mean age of a combined group of men and women is
30 years. If the mean age of the group of men is 32 and that of women
group is 27. find out the percentage of men and women in the group.
Solution : Let us take group of men as first group and women as second
group. Therefore, X1 = 32 years, X 2 = 27 years, and X12 = 30 years. In
the problem, we are not given the number of men and women. We can
assume N1 + N2 = 100 and therefore, N1 = 100 – N2
N1X1 + N 2 X 2
Apply X12 =
N1 + N 2
32N1 + 27N 2
30 = (Substitute N1 = 100 – N2)
100
30 ×100 = 32 (100 – N 2 ) + 27 N 2 or 5N 2 = 200
N 2 = 200 / 5 = 40%
N1 = (100 – N 2 ) = (100 – 40) = 60%

Therefore, the percentage of men in the group is 60 and that of


women is 40.
(v) This is the last mathematical property of arithmetic mean is to find
correct mean when there are one and more than one values were
wrongly written in a given data. Because
∑X
X=
N

Σf = N.X
if we replace each item in the series by the mean, the sum of these
substitutions will be equal to the sum of the individual items. This
property is used to find out the aggregate values and corrected
averages. We can understand the property with the help of an
example.
Example 7 : Mean of 100 observations is found to be 44. If at the time
of computation two items are wrongly taken as 30 and 27 inplace of 3
and 72. Find the corrected average.

42 PAGE
© Department of Distance & Continuing Education, Campus of Open Learning,
School of Open Learning, University of Delhi
BUSINESS STATISTICS

ΣX Notes
Solution : X =
N

∴ ∑ X = N.X = 100× 44 = 4400


Corrected ∑ X = ∑ X + correct items – wrong items = 4400 + 3 +
72 – 30 – 27 = 4418
Corrected ∑ X 4418
Corrected average = = = 44.18
N 100
Calculation of Arithmetic mean in Case of Open-End Classes :
Open-end classes are those in which lower limit of the first class and the
upper limit of the last class are not defined. In these series, we cannot
calculate mean unless we make an assumption about the unknown limits.
The assumption depends upon the class-interval following the first class
and preceding the last class, For example :
Marks No. of students
Below 15 4
15–30 6
30–45 12
45–60 8
Above 60 7
In this example, because all defined class-intervals are same, the assumption
would be that the first and last class shall have same class-interval of
15 and hence the lower limit of the first class shall be zero and upper
limit of last class shall be 75. Hence first class would be 0–15 and the
last class 60–75.
What happens in this case ?
Marks No. of students
Below 10 4
10–30 7
30–60 10
60–100 8
Above 100 4
In this problem because the class interval is 20 in the second class, 30 in
the third, 40 in the fourth class and so on. The class interval is increasing

PAGE 43
© Department of Distance & Continuing Education, Campus of Open Learning,
School of Open Learning, University of Delhi
B.COM. (PROGRAMME)

Notes by 10. Therefore the appropriate assumption in this case would be that
the lower limit of the first class is zero and the upper limit of the last
class is 150. In case of other open-end class distributions the first class
limit should be fixed on the basis of succeeding class interval and the
last class limit should be fixed on the basis of preceding class interval.
If the class intervals are of varying width, an effort should be made to
avoid calculating mean and mode. It is advisable to calculate median.

2.6.3 Weighted Mean


In the computation of arithmetic mean, we give equal importance to each
item in the series.
Raja Toy Shop sell, Toy Cars at Rs. 3 each, Toy Locomotives at Rs. 5
each, Toy Aeroplane at Rs. 7 each and Toy Double Decker at Rs. 9 each.
What shall be the average price of the toys sold ? If the shop sells 4
toys one of each kind.
∑ X 24
(Mean Price) = = = Rs. 6.
N 4
In this case the importance of each toy is equal as one toy of each variety
has been sold. While computing the arithmetic mean this fact has been
taken care of including the price of each toy once only.
But if the shop sells 100 toys, 50 cars, 25 locomotives, 15 aeroplanes and
10 double deckers, the importance of the four toys to the dealer is not
equal as a source of earning revenue. In fact their respective importance
is equal to the number of units of each toy sold, i.e. the importance of
Toy car is 50; the importance of Locomotive is 25; the importance of
Aeroplane is 15; and the importance of Double Decker is 10.
It may be noted that 50, 25, 15, 10 are the quantities of the various
classes of toys sold. These quantities are called as ‘weights’ in statistical
ODQJXDJH :HLJKW LV UHSUHVHQWHG E\ V\PERO : DQG Ȉ: UHSUHVHQWV WKH
sum of weights.
While determining the average price of toy sold these weights are of
great importance and are taken into account to compute weighted mean.

44 PAGE
© Department of Distance & Continuing Education, Campus of Open Learning,
School of Open Learning, University of Delhi
BUSINESS STATISTICS

∑[(W1X1 ) + (W2 X 2 ) + (W3X3 ) + (W4 X 4 )] ∑ WX Notes


Xw = =
W1 + W2 + W3 + W4 ∑W

where W1, W2, W3, W4 are weights and X1, X2, X3, X4 represents the
price of 4 varieties of toy.
Hence by substituting the values of W1, W2, W3, W4 and X1, X2, X3, X4,
we get
(50 × 3) + (25 × 5) + (15 × 7) + (10 × 9)
Xw =
50 + 25 + 15 + 10
150 + 125 + 105 + 90 470
Xw = = = Rs. 4.70
100 100
The table given below demonstrates the procedure of computing the
weighted Mean.
Weighted Arithmetic mean of Toys by the Raja Shop.
Toy Price per toy (Rs.) Number sold Price × weight
X W WX
Car 3 50 150
Locomotive 5 25 125
Aeroplane 7 15 105
Double Decker 9 10 90
∑ W = 100 ∑ WX = 470

∑ WX 470
? Xw = = = Rs. 4.70
∑X 100
Example 8 : The table below shows the number of skilled and unskilled
workers in two localities along with their average hourly wages.
Ram Nagar Shyam Nagar
Worker Number Wages (per hour) Number Wages (per hour)
Category
Skilled 150 1.80 350 1.75
Unskilled 850 1.30 650 1.25
Determine the average hourly wage in each locality. Also give reasons
why the results show that the average hourly wage in Shyam Nagar
exceed the average hourly wage in Ram Nagar, even though in Shyam

PAGE 45
© Department of Distance & Continuing Education, Campus of Open Learning,
School of Open Learning, University of Delhi
B.COM. (PROGRAMME)

Notes Nagar the average hourly wages of both categories of workers is lower.
It is required to compute weighted mean.
Solution :
Ram Nagar Shyam Nagar
Worker X W WX X W WX
Category
Skilled 1.80 150 270 1.75 350 612.50
Unskilled 1.30 850 1105 1.25 650 812.50
Total 1000 1375 1000 1425
1375 1425
Xw = = Rs.1.375 Xw = = Rs. 1.425
1000 1000

It may be noted that weights are more evenly assigned to the different
categories of workers in Shyam Nagar than in Ram Nagar.

2.7 Geometric Mean


In general, if we have n numbers (none of them being zero), then the
G.M. is defined as
G.M. = x1 , x2 ,...........xn = ( x1 , x2 ...........xn )1/ n

In the case of a discrete series, if x1, x2,...........xn occur f1, f2,.......fn times
respectively and N is the total frequency (i.e. N = f1+, f2+,.........fn), then
G.M. = n x1 f1 , x2 f 2 ,...........xn f n

For convenience, use of logarithms is made extensively to calculate the


nth root. In terms of logarithms
⎛ log x1 + log x2 + ........... + log xn ⎞
G.M. = AL ⎜ ⎟
⎝ n ⎠

⎛ ∑ log x ⎞
= AL ⎜ ⎟ , where AL stands for anti log.
⎝ N ⎠
∑ f log x
In discrete series, G.M. = AL
N
∑ f log m
and in case of continuous series, G.M. = AL
N

46 PAGE
© Department of Distance & Continuing Education, Campus of Open Learning,
School of Open Learning, University of Delhi
BUSINESS STATISTICS

Example 9 : Calculate G.M. of the following data : Notes


2, 4, 8
Solution : G.M. = 3
2 × 4 × 8 = 3 64 = 4
In terms of logarithms, the question can be solved as follows :
log 2 = 0.3010, log 4 = 0.6021, and log 8 = 9.9031
Apply the formula:
∑ log x 1.8062
G.M. = AL = AL = AL 0.60206 = 4
N 3
Example 10 : Calculate geometric mean of the following data :
x 5 6 7 8 9 10 11
f 2 4 7 10 9 6 2
Solution :
Calculation of G.M.
x log.x f f log x
5 0.6990 2 1.3980
6 0.7782 4 3.1128
7 0.8451 7 5.9157
8 0.9031 10 9.0310
9 0.9542 9 8.5878
10 1.0000 6 6.0000
11 1.0414 2 2.0828
N = 40 Ȉf = 36.1281

⎛ Σf log x ⎞ ⎛ 36.1281 ⎞
G.M. = AL ⎜ ⎟ = AL ⎜ ⎟ = AL (0.9032) = 8.002
⎝ N ⎠ ⎝ 40 ⎠
Example 11 : Calculate G.M. from the following data :
X f
9.5–14.5 10
14.5–19.5 15
19.5–24.5 17
24.5–29.5 25
29.5–34.5 18
34.5–39.5 12
39.5–44.5 8

PAGE 47
© Department of Distance & Continuing Education, Campus of Open Learning,
School of Open Learning, University of Delhi
B.COM. (PROGRAMME)

Notes Solution :
Calculation of G.M.
X m log m f f log m
9.5–14.5 12 1.0792 10 10.7920
14.5–19.5 17 1.2304 15 18.4560
19.5–24.5 22 1.3424 17 22.8208
24.5–29.5 27 1.4314 25 35.7850
29.5–34.5 32 1.5051 18 27.0918
34.5–39.5 37 1.5682 12 18.8184
39.5–44.5 42 1.6232 8 12.9856
N = 105 Σf log m = 146.7496

⎛ 146.7496 ⎞
G.M. = AL ⎜ ⎟ = AL (1.3976) = 24.98
⎝ 105 ⎠

2.7.1 Uses of Geometric Mean


The Geometric Mean has certain specific uses, some of them are :
(i) It is used in the construction of index numbers,
(ii) It is also helpful in finding out the compound rates of change such
as the rate of growth of population in a country.
(iii) It is suitable where the data are expressed in terms of rates, ratios
and percentage.
(iv) It is quite useful in computing the average rates of depreciation or
appreciation.
(v) It is most suitable when large weights are to be assigned to small
items and small weights to large items.
Example 12: The gross national product of a country was Rs. 1,000
crores 10 years earlier. It is Rs. 2,000 crores now. Calculate the rate of
growth in G.N.P.
Solution : In this case compound interest formula will be used for
computing the average annual per cent increase of growth.

48 PAGE
© Department of Distance & Continuing Education, Campus of Open Learning,
School of Open Learning, University of Delhi
BUSINESS STATISTICS

Pn = Po(1 + r)n Notes


where Pn = prinicipal sum (or any other variate) at the end of the period.
Po = prinicipal sum in the beginning of the period.
r = rate of increase or decrease.
n = number of years.
It may he noted that the above formula can also be written in the
following form :
Pn
r = n −1
Po

Substituting the values given in the formula, we have

2000
r = 10 − 1 = 10 2 − 1
1000
⎡ log 2 ⎤ ⎡ 0.30103 ⎤
= AL ⎢ ⎥ − 1 = AL ⎢ − 1 = 1.0718 − 1 = 0.0718 = 7.18%
⎣ 10 ⎦ ⎣ 10 ⎥⎦
Hence, the rate of growth in GNP is 7.18%.
Example 13 : The price of commodity increased by 5 per cent from
1998 to 1999, 8 per cent from 1999 to 2000 and 77 per cent from 2000
to 2001. The average increase from 1998 to 2001 is quoted at 26 per
cent and not 30 per cent. Explain this statement and verify your result.
Solution : Taking Pn as the price at the end of the period. Po as the
price in the beginning, we can substitute the values of Pn and Po in the
compound interest formula. Taking Po = 100; Pn = 200.72
Pn = Po(1 + r)n
200.72 = 100(1 + r)3
200.72 200.72
or (1 + r)3 = or 1 + r = 3
100 100

200.72
r = 3 − 1 = 1.260 − 1 = 0.260 = 26%
100
Thus increase is not average of (5 + 8 + 77)/3 = 30 per cent. It is 26%
as found out by G.M.

PAGE 49
© Department of Distance & Continuing Education, Campus of Open Learning,
School of Open Learning, University of Delhi
B.COM. (PROGRAMME)

Notes 2.7.2 Weighted G.M.


The weighted G.M. is calculated with the help of the following formula :
G.M. = x1w1.x2 w2 .........xn wn

log x1 × w1 + log x2 × w2 + ...log xn × wn


=
w1 + w2 ...wn

⎡ Σ(log x × w) ⎤
= AL ⎢ ⎥⎦
⎣ Σw
Example 14 : Find out weighted G.M. from the following data :
Group Index number Weights
Food 352 48
Fuel 220 10
Cloth 230 8
House Rent 160 12
Misc. 190 15
Solution :
Calculation of Weighted G.M.
Group Index Weights (w) log x w log x
Number(x)
Food 352 48 2.5465 122.2320
Fuel 220 10 2.3424 23.4240
Cloth 230 8 2.3617 18.8936
House Rent 160 12 2.2041 26.4492
Misc. 190 15 2.2788 34.1820
93 225.1808
⎡ Σw log x ⎤ 225.1808
G.M. (weighted) = AL ⎢ ⎥ = AL = 263.8
⎣ Σw ⎦ 93
Example 15 : A machine depreciates at the rate of 35.5% per annum in
the first year, at the rate of 22.5% per annum in the second year, and
at the rate of 9.5% per annum in the third year, each percentage being
computed on the actual value. What is the average rate of depreciation?

50 PAGE
© Department of Distance & Continuing Education, Campus of Open Learning,
School of Open Learning, University of Delhi
BUSINESS STATISTICS

Solution : Average rate of depreciation can be calculated by taking G.M. Notes


Year X (values taking 100 as base) log X
I 100 – 35.5 = 64.5 1.8096
II 100 – 22.5 = 77.5 1.8893
III 100 – 9.5 = 90.5 1.9566
ȈORJ ;  

⎡ Σ log x ⎤ 5.6555
Apply G.M.= AL ⎢ = = AL 1.8851 = 76.77
⎣ w ⎥⎦ 3
? Average rate of depreciation = 100 – 76.77 = 23.33%.
Example 16 : The arithmetic mean and geometric mean of two values
are 10 and 8 respectively. Find the values.
Solution : If two values are taken as a and b, then
a+b
= 10, and ab = 8
2
Or a + b = 20, ab = 64

then a – b = (a + b) 2 − 4ab = (20) 2 − 4 × 64 = 400 − 256 = 144 = 12


Now, we have a + b = 20, a – b = 12
Solving for a and b, we get a = 4 and b = 16.

2.8 Harmonic Mean


The harmonic mean is defined as the reciprocals of the average of
reciprocals of all items in a series. Symbolically,
N N
H.M. = =
⎛1 1 1 1 ⎞ ⎛1⎞
⎜ x + x + x + ....... x ⎟ Σ ⎜⎝ x ⎟⎠
⎝ 1 2 3 n ⎠

In case of a discrete series,


N

{ }
H.M. =
1
Σ f×
x
and in case of a continuous series,

PAGE 51
© Department of Distance & Continuing Education, Campus of Open Learning,
School of Open Learning, University of Delhi
B.COM. (PROGRAMME)

Notes N

{ }
H.M. =
1
Σ f×
m
It may be noted that none of the values of the variable should be zero.
Example 17 : Calculate harmonic mean from the following data : 5, 15,
25, 35 and 45
Solution :
X 1
X
5 0.20
15 0.067
25 0.040
35 0.029
45 0.022
⎛1⎞
N = 5 Σ ⎜ ⎟ = 0.358
⎝X⎠

N 5
H.M. = = = 14 approx.
⎛ 1 ⎞ 0.358
Σ⎜ ⎟
⎝x⎠
Example 18 : From the following data compute the value of the harmonic
mean :
x : 5 15 25 35 45
f : 5 15 10 15 5
Solution :
Calculation of Harmonic Mean
x f
1
f
x
5 5 0.200 1.000
15 15 0.067 1.005
25 10 0.040 0.400
35 15 0.29 0.435

52 PAGE
© Department of Distance & Continuing Education, Campus of Open Learning,
School of Open Learning, University of Delhi
BUSINESS STATISTICS

x f Notes
1
f
x
45 5 0.022 0.110
⎛ 1⎞
Ȉf = 50 Σ ⎜ f ⎟ = 2.950
⎝ x⎠

N 50
H.M. = = = 17 approx.
⎧ 1⎫ 2.95
Σ⎨ f × ⎬
⎩ x⎭
Example 19 : Calculate harmonic mean from the following distribution:
x f
0–10 5
10–20 15
20–30 10
30–40 15
40–50 5
Solution : First of all, we shall find out mid points of the various
classes. They are 5, 15, 25, 35 and 45. Then we will calculate the H.M.
by applying the following formula :
N

{ }
H.M. =
1
Σ f×
m
Calculation of Harmonic Mean
x (mid points) f 1 1
f
x x
5 5 0.200 1.000
15 15 0.067 1.005
25 10 0.040 0.400
35 15 0.29 0.435
45 5 0.022 0.110
⎛ 1⎞
Ȉf = 50 Σ ⎜ f ⎟ = 2.950
⎝ x⎠

PAGE 53
© Department of Distance & Continuing Education, Campus of Open Learning,
School of Open Learning, University of Delhi
B.COM. (PROGRAMME)

Notes N 50
H.M. = = = 17 approximately
⎛ 1⎞ 2.950
Σ⎜ f × ⎟
⎝ m⎠

2.8.1 Application of Harmonic Mean


Like Geometric means, the harmonic mean is also applicable to certain
special types of problems. Some of them are:
(i) If, in averaging time rates, distance is constant, then H.M. is to be
calculated.
Example 20 : A man travels 480 km. a day. On the first day he travels
for 12 hours @ 40 km. per hour and second day for 10 hours @ 48 km.
per hour. On the third day he travels for 15 hours @ 32 km. per hour.
Find his average speed.
Solution : We shall use the harmonic mean,
N 3 3
H.M. = = = = 39 km. per hour (approx.)
⎛1⎞ 1
+
1
+
1 37 480
Σ⎜ ⎟
⎝ X ⎠ 40 48 32
⎛ 48 + 40 + 32 ⎞
The arithmetic mean would be ⎜ ⎟ = 40 km. per hour
⎝ 3 ⎠
(ii) If, in averaging the price data, the prices are expressed as “quantity
per rupee”. Then harmonic mean should be applied.
Example 21 : A man purchased one kilo of cabbage form each of four places
at the rate of 20 kg., 16 kg., 12 kg. and 10 kg. per rupees respectively.
On the average how many kilos of cabbages he has purchased per rupee.
N 4 4 4 × 240
Solution : H.M. = = = = = 13.5 kg. per rupee.
⎛ 1 ⎞ 1 + 1 + 1 + 1 71 250 71
Σ⎜ ⎟
⎝ x ⎠ 20 16 12 10

Example 22 : Find two numbers whose geometric mean is 18 and


arithmetic mean is 19.5. Also, calculate their harmonic mean.
Solution : Let us assume two numbers are x and y. Now with the help
of given information,

54 PAGE
© Department of Distance & Continuing Education, Campus of Open Learning,
School of Open Learning, University of Delhi
BUSINESS STATISTICS

x+ y Notes
X = = 19.5
2
or x + y = 39 ...(i)

Also, G.M. = xy = 18
or xy = 324 ...(ii)

Now, (x – y)2 = ( x + y ) 2 − 4 xy

= 392 − 4 × 324 = 225


? x – y = ± 15 ...(iii)
With the help of equations (i) and (iii),
we get x = 27 and y = 12
Therefore the two numbers are 12 and 27.

Now, G.M. = AM × HM
G.M.2 = AM × HM
or H.M. = GM2/AM
H.M. = 182/19.5 = 16.62

2.9 Median
The median is that value of the variable which divides the group in two
equal parts. One part comprising the values greater than and the other all
values less than median. Median of a distribution may be defined as that
value of the variable which exceeds and is exceeded by the same number
of observation. It is the value such that the number of observations above
it is equal to the number of observations below it. Thus we know that
the arithmetic mean is based on all items of the distribution, the median
is positional average, that is, it depends upon the position occupied by
a value in the frequency distribution.
When the items of a series are arranged in ascending or descending
order of magnitude the value of the middle item in the series in known
as median in the case of individual observations. Symbolically,

PAGE 55
© Department of Distance & Continuing Education, Campus of Open Learning,
School of Open Learning, University of Delhi
B.COM. (PROGRAMME)

Notes ⎛ N +1⎞
Median = size of ⎜ ⎟ th item
⎝ 2 ⎠
If the number of items is even, then there is no value exactly in the
middle of the series. In such a situation the median is arbitrarily taken
to be halfway between the two middle items. Symbolically,
N ⎛ N +1 ⎞
size of th item + size of ⎜ ⎟ th item
Median = 2 ⎝ 2 ⎠
2
Example 23: Find the median of the following series:
(i) 8, 4, 8, 3, 4, 8, 6, 5, 10.
(ii) 15, 12, 5, 7, 9, 5, 11, 28.
Solution :
Computation of Median
(i) (ii)
Serial No. X Serial No. X
1 3 1 5
2 4 2 5
3 4 3 7
4 5 4 9
5 6 5 11
6 8 6 12
7 8 7 15
8 8 8 28
9 10
N = 9 N = 8

⎛ N +1 ⎞ 9 +1
For (i) series Median = size of the ⎜ ⎟ th item = size of the th
⎝ 2 ⎠ 2
item = size of 5th item = 6
⎛ N +1 ⎞
For (ii) series Median = size of the ⎜ ⎟ th item
⎝ 2 ⎠
⎛ 8 + 1⎞
= size of the ⎜⎝ ⎟ th item
2 ⎠

56 PAGE
© Department of Distance & Continuing Education, Campus of Open Learning,
School of Open Learning, University of Delhi
BUSINESS STATISTICS

size of 4th + size of 5th item 9 + 11 Notes


= = = 10
2 2
Location of Median in Discrete series: In a discrete series, medium is
computed in the following manner:
(i) Arrange the given variable data in ascending or descending order.
(ii) Find cumulative frequencies.
( N + 1)
(iii) Apply Median = size of th item
2
(iv) Locate median according to the size i.e., variable corresponding to
the size or for next cumulative frequency.
Example 24 : Following are the number of rooms in the houses of a
particular locality. Find median from the following given data :
No. of rooms : 3 4 5 6 7 8
No. of houses : 38 654 311 42 12 2
Solution :
Computation of Median
No. of rooms No. of houses Cumulative frequency
X f Cf
3 38 38
4 654 692
5 311 1003
6 42 1045
7 12 1057
8 2 1059

⎛ N +1 ⎞ 1059 + 1
Median = size of ⎜ ⎟ th item = size of th item = 530th item.
⎝ 2 ⎠ 2
Median lies in the cumulative frequency of 692 and the value corresponding
to this is 4.
Therefore, Median = 4 rooms
In a continuous series, median is computed in the following manner :
(i) Arrange the given variable data in ascending or descending order.

PAGE 57
© Department of Distance & Continuing Education, Campus of Open Learning,
School of Open Learning, University of Delhi
B.COM. (PROGRAMME)

Notes (ii) If inclusive series is given, it must be converted into exclusive series
to find real class intervals.
(iii) Find cumulative frequencies.
N
(iv) Apply Median = size of th item to ascertain median class.
2
(v) Apply formula of interpolation to ascertain the value of median.
N N
− cf 0 − cf 0
Median = l1 + 2 × (l2 − l1 ) or Median = l2 − 2 × (l2 − l1 )
f f
where, l1 refers to lower limit of median class
l2 refers to higher limit of median class
cf0 refers cumulative frequency of previous class
f refers to frequency of median class.
Example 25 : The following table gives you the distribution of marks
secured by some students in an examination
Marks No. of Students
0–20 42
21–30 48
31–40 120
41–50 84
51–60 48
61–70 36
71–80 31
Find the median marks.
Solution :
Calculation of Median Marks
Marks No. of students cf
(x) (f)
0–20 42 42
21–30 38 80
31–40 120 200
41–50 84 284

58 PAGE
© Department of Distance & Continuing Education, Campus of Open Learning,
School of Open Learning, University of Delhi
BUSINESS STATISTICS

Marks No. of students cf Notes


(x) (f)
51–60 48 332
61–70 36 368
71–80 31 399
N 399
Median = size of th item = size of th = 199.5th item
2 2
which lies in (31–40) group, therefore the median class is 30.5–40.5
Applying the formula of interpolation.
N
− cf 0
Median = 2
l1 + × (l2 − l1 )
f
199.5 − 80 119.5
= 30.5 + × (10) = 30.5 + = 40.46 marks.
120 12

2.10 Other Positional Averages


The median divides the series into two equal parts. Similarly there are
certain other measures which divide the series into certain equal parts.
There are first quartile, third quartile, deciles percentiles etc. If the items
are arranged in ascending or descending order of magnitude, Q1 is that
value which covers l/4th of the total number of items. Similarly, if the
total number of items are divided into ten equal parts, then, there shall
be nine deciles.
Symbolically,
⎛ N +1⎞
First quartile (Q1) = size of ⎜ ⎟ th item
⎝ 4 ⎠

3( N + 1)
Third quartile (Q3) = size of th item
4
⎛ N +1 ⎞
First decile (D1) = size of ⎜ ⎟ th item
⎝ 10 ⎠
6( N + 1)
Sixth decile (D6) = size of th item
10

PAGE 59
© Department of Distance & Continuing Education, Campus of Open Learning,
School of Open Learning, University of Delhi
B.COM. (PROGRAMME)

Notes ⎛ N +1 ⎞
First percentile (P1) = size of ⎜ ⎟ th item
⎝ 100 ⎠
Once values of the items are found out, then formulae of interpolation
are applied for ascertaining the value of Q1, Q3, D1, D4, P40 etc.
Example 26: Calculate Q1, Q3, D2 and P5 from the following data :
Marks: Below 10 10–20 20–40 40–60 60–80 Above 80
No. of Students: 8 10 22 25 10 5
Solution:
Calculation of Positional values
Marks No. of Students (f) c.f.
Below 10 8 8
10–20 10 18
20–40 22 40
40–60 25 65
60–80 10 75
Above 80 5 80
N = 80
N 80
Q1= size of th item = = 20th item
4 4
Hence Q1 lies in the class 20–40, apply
N
− Cf 0 N
Q1= l1 + 4 × i where l1 = 20, = 20, Cf0 = 18, f = 22 and i= = 20
f 4
By substituting the values, we get
(20 − 18)
Q1 = 20 + × 20 = 20 + 1.8 = 21.8
22
Similarly, we can calculate
3N 3 × 80
Q3 = size of th item = th item = 60th item.
4 4
Hence Q3 lies in the class 40–60

60 PAGE
© Department of Distance & Continuing Education, Campus of Open Learning,
School of Open Learning, University of Delhi
BUSINESS STATISTICS

3N Notes
− Cf 0 3N
Q3 = l1 + 4 × i where l1 = 40, = 60, Cf0= 40, f = 25, i = 20.
f 4

(60 − 40)
? Q3 = 40 + × 20 = 40 + 16 = 56
25
2N
D2= size of th item = 16th item. Hence D2 lies in the class 10–20.
10
2N
− Cf 0
D2= l1 + 10 × i where l1 = 10, = 16, Cf0= 8, f = 10, i = 10.
f
(16 − 8)
D2 = 10+ ×10 = 10 + 8 = 18
10
5N 5 × 80
P5 = size of th item = th item = 4th item. Hence P5 lies in the
100 100
class 0–10
5N
− Cf 0
100 5N
P5 = l1 + × i where l1 = 0, = 4, Cf0 = 0, f = 8, i = 10
f 100
4−0
P5 = 0 + ×10 = 0 + 5 = 5.
8

2.11 Calculation of Missing Frequencies


Example 27: In the frequency distribution of 100 families given below;
the number of families corresponding to expenditure groups 20–40 and
60–80 are missing from the table. However the median is equals to 50.
Find out the missing frequencies.
Expenditure: 0–20 20–40 40–60 60–80 80–100
No. of families: 14 ? 27 ? 15
Solution : We shall assume the missing frequencies for the classes 20–40
to be x and 60–80 to y

PAGE 61
© Department of Distance & Continuing Education, Campus of Open Learning,
School of Open Learning, University of Delhi
B.COM. (PROGRAMME)

Notes Expenditure (Rs.) No. of families C.f


0–20 14 14
20–40 x 14 + x
40–60 27 14 + 27+ x
60–80 y 41 + x + y
80–100 15 41 + 15 + x+ y
N = 100 = 56 + x + y
)URP WKH WDEOH ZH KDYH 1  ȈF = 56 + x + y = 100.
? x + y = 100 – 56 + 44
Median is given as 50 which lies in the class 40–60, which becomes the
median class.
By using the median formula we get:
N
− Cf 0
Median = l1 + 2 ×i
f

50 − (14 + x) 50 − (14 + x)
? 50 = 40 + × (60 − 40) or 50 = 40 + × 20
27 27
36 − x 20
or 50 − 40 = × 20 or 50 − 40 = (36 − x) ×
27 27
or 10 × 27 = 720 − 20x or 270 = 720 – 20x
? 20x = 720 – 270
450
x = = 22.5
20
By substituting the value of x in the equation,
x + y = 44
we get, 22.5 + y = 44
y = 44 – 22.5 = 21.5.
Hence frequency for the class 20–40 is 22.5 and 60–80 is 21.5.
Merits of Median:-
‹ It is very easy to understand.
‹ It can be easily calculated when open ended class distribution is
given because Median is the mid-value of a distribution.

62 PAGE
© Department of Distance & Continuing Education, Campus of Open Learning,
School of Open Learning, University of Delhi
BUSINESS STATISTICS

‹ It is rigidly defined. Notes


‹ It is also not affected by extreme values because Median is positional.
Demerits of Median:-
‹ It is compulsory to arrange observations in order of magnitude
otherwise Median cannot be measured.
‹ Because Median is a positional average then it is not based upon
all values in the distribution which means it is not based on each
and every value in the distribution.

2.12 Mode
Mode is the value of a repeated variable at maximum number of times
or with the highest frequency in a data series. The mode is the most
“fashionable” size in the sense that it is the most common and typical and
is defined by Zizek as “the value occurring most frequently in series of
items and around which the other items are distributed most densely.” In the
words of Croxton and Cowden, the mode of a distribution is the value at
the point where the items tend to be most heavily concentrated. According
to A.M. Tuttle, Mode is the value which has the greater frequency density
in its immediate neighbourhood. In the case of individual observations,
the mode is that value which is repeated the maximum number of times
in the series. The value of mode can be denoted by the alphabet z also.
Example 28 : Calculate mode from the following data:
Sr. Number : 1 2 3 4 5 6 7 8 9 10
Marks obtained : 10 27 24 12 27 27 20 18 15 30
Solution :
Marks No. of Students
10 1
12 1
15 1
18 1
20 1
24 1
27 3
30 1

PAGE 63
© Department of Distance & Continuing Education, Campus of Open Learning,
School of Open Learning, University of Delhi
B.COM. (PROGRAMME)

Notes It is clearly shown in the above example that Mode is 27 marks because
this has the highest frequency in the which means 3 students get the 27
marks in the class.
Calculation of Mode in Discrete series. In discrete series, it is quite
often determined by inspection. We can understand with the help of an
example :
X 1 2 3 4 5 6 7
f 4 5 13 6 12 8 6
By inspection, the modal size is 3 as it has the maximum frequency. But
this test of greatest frequency is not fool proof as it is not the frequency
of a single class, but also the frequencies of the neighbour classes that
decide the mode. In such cases, we shall be using the method of Grouping
and Analysis table.
Size of shoe 1 2 3 4 5 6 7
frequency 4 5 13 6 12 8 6
Solution : By inspection, the mode is 3, but the size of mode may be
5. This is so because the neighbouring frequencies of size 5 are greater
than the neighbouring frequencies of size 3. This effect of neighbouring
frequencies is seen with the help of grouping and analysis table technique.
Grouping table
Size of Shoe Frequency
1 2 3 4 5 6
1 4
9
2 5 18 22
3 13 19
24 31
4 6 18
5 12 26
20 26
6 8
7 6 14

When there exist two groups of frequencies in equal magnitude, then we


should consider either both or omit both while analysing the sizes of items.

64 PAGE
© Department of Distance & Continuing Education, Campus of Open Learning,
School of Open Learning, University of Delhi
BUSINESS STATISTICS

Analysis Table Notes


Column Size of items with maximum frequency
1 3
2 5, 6
3 1, 2, 3, 4, 5
4 4, 5, 6
5 5, 6, 7
6 3, 4, 5
Item 5 occurs maximum number of times, therefore, mode is 5. We can
note that by inspection we had determined 3 to be the mode.
Determination of mode in continuous series : In the continuous series,
the determination of mode requires one additional step. Once the modal
class is determined by inspection or with the help of grouping technique,
then the following formula of interpolation is applied:
f1 − f 0 f1 − f 0
Mode = l1 + (l2 − l1 ) or Mode = l2 − (l2 − l1 )
2 f1 − f 0 − f 2 2 f1 − f 0 − f 2

l1 = lower limit of the class, where mode lies.


l2 = upper limit of the class, where mode lies.
f0 = frequency of the class proceeding the modal class.
f1 = frequency of the class, where mode lies.
f2 = frequency of the class succeeding the modal class.
Example 29 : Calculate mode of the following frequency distribution :
Variable Frequency
0–10 5
10–20 10
20–30 15
30-40 14
40–50 10
50–60 5
60–70 3

PAGE 65
© Department of Distance & Continuing Education, Campus of Open Learning,
School of Open Learning, University of Delhi
B.COM. (PROGRAMME)

Notes Solution :
Grouping Table
X 1 2 3 4 5 6
0–10 5
15
10–20 10 30
25
20–30 15
29 39
30–40 14 39
40–50 10 24 29
15
50–60 5 18
8
60–70 3
Analysis Table
Column Size of item with maximum frequency
1 20–30
2 20–30, 30–40
3 10–20, 20–30
4 0–10, 10–20, 20–30
5 10–20, 20–30, 30–40
6 20–30, 30–40, 40–50
Modal group is 20–30 because it has occurred 6 times. Applying the
formula of interpolation.
f1 − f 0
Mode = l1 + (l2 − l1 )
2 f1 − f 0 − f 2

15 − 10 5
= 20 + (30 − 20) = 20 + (10) = 28.3
30 − 10 − 14 6
Calculation of mode where it is ill defined. The above formula is not
applied where there are many modal values in a series or a distribution.
For instance there may be two or more than two items having the
maximum frequency. In these cases, the series will be known as bimodal

66 PAGE
© Department of Distance & Continuing Education, Campus of Open Learning,
School of Open Learning, University of Delhi
BUSINESS STATISTICS

or multimodal series. The mode is said to be ill-defined and in such cases Notes
the following formula is applied.
Mode = 3 Median – 2 Mean.
Example 30: Calculate mode of the following frequency data :
Variate value Frequency
10–20 5
20–30 9
30–40 13
40–50 21
50–60 20
60–70 15
70–80 8
80–90 3
Solution : First of all, ascertain the modal group with the help of process
of grouping.
Grouping Table
X 1 2 3 4 5 6
10–20 5
14
20–30 9 27
22
30–40 13 43
34
40–50 21 54
41
50–60 20 56
35
60–70 15 43
23
70–80 8 26
11
80–90 3

PAGE 67
© Department of Distance & Continuing Education, Campus of Open Learning,
School of Open Learning, University of Delhi
B.COM. (PROGRAMME)

Notes Analysis Table


Column Size of item with maximum frequency
1 40–50
2 50–60, 60–70
3 40–50, 50–60
4 40–50, 50–60, 60–70
5 20–30, 30–40, 40–50, 50–60, 60–70, 70–80
6 30–40, 40–50, 50–60
There are two groups which occur equal number of items. They are 40–50
and 50–60. Therefore, we will apply the following formula:
Mode = 3 median – 2 mean and for this purpose the values of mean and
median are required to be computed.
Calculation of Mean and Median
Variate frequency mid ⎛ m – 45 ⎞
values ⎜ ⎟
⎝ 10 ⎠
X f m dƍx fdƍx cf
10–20 5 15 – 3 – 15 5
20–30 9 25 – 2 – 18 14
30–40 13 35 – 1 – 13 27
40–50 21 45 0 0 48 Median is the
N
50–60 20 55 + 1 + 20 68 value of th
2
60–70 15 65 + 2 + 30 83 item which lies
70–80 8 75 + 3 + 24 91 in (40–50) group
80–90 3 85 + 4 + 12 94
N = 94 Ȉfdƍ  
N
Σfd ′x − cf 0
X = A+ × i. Med.= l1 + 2 × i.
N f

40 47 − 27 200
= 45 + (10) = 45 + 4.2 = 49.2 = 40 + (10) = 40 + = 49.5
94 21 21
Mode = 3 median – 2 mean
= 3 (49.5) –2 (49.2) = 148.5 – 98.4 = 50.1

68 PAGE
© Department of Distance & Continuing Education, Campus of Open Learning,
School of Open Learning, University of Delhi
BUSINESS STATISTICS

2.12.1 Mode by Graph Notes

Mode can also be computed by curve fitting. The following steps are to
be taken;
(i) Draw a histogram of the data.
(ii) Draw the lines diagonally inside the modal class rectangle, starting
from each upper corner of the rectangle to the upper corner of the
adjacent rectangle.
(iii) Draw a perpendicular line from the intersection of the two diagonal lines
to the X-axis. The abscissa of the point at which the perpendicular
line meets is the value of the mode.
Example 31 : Construct a histogram for the following distribution and,
determine the mode graphically :
X : 0–10 10–20 20–30 30–40 40–50
f : 5 8 15 12 7
Verify the result with the help of interpolation.
Solution :

16-

12-

8-

6-

3-

0 10 20 27 30 40 50
Mode

f1 − f 0
Mode = l1 + (l2 − l1 )
2 f1 − f 0 − f 2
15 − 8 7
= 20 + (30 − 20) = 20 + (10) = 27
30 − 8 − 12 10

PAGE 69
© Department of Distance & Continuing Education, Campus of Open Learning,
School of Open Learning, University of Delhi
B.COM. (PROGRAMME)

Notes Example 32 : Calculate mode from the following data :


Marks No. of Students
Below 10 4
ƍƍ  6
ƍƍ  24
ƍƍ  46
ƍƍ  67
ƍƍ  86
ƍƍ  96
ƍƍ  99
ƍƍ  100
Solution : Since we are given the cumulative frequency distribution of
marks, first we shall convert it into the normal frequency distribution:
Marks Frequencies
0–10 4
10–20 6 – 4 = 2
20–30 24 – 6 = 18
30–40 46 – 24 = 22
40–50 67 – 46 = 21
50–60 86 – 67 = 19
60–70 96 – 86 = 10
70–80 99 – 96 = 3
80–90 100 – 99 = 1
It is evident from the table that the distribution is irregular and maximum
chances are that the distribution would be having more than one mode.
You can verify by applying the grouping and analysing table.
The formula calculate the value of mode in cases of bio-modal
distributions is :
Mode = 3 median – 2 mean.
Computation of Mean and Median

70 PAGE
© Department of Distance & Continuing Education, Campus of Open Learning,
School of Open Learning, University of Delhi
BUSINESS STATISTICS

Marks Mid- Frequency Cumulative ⎛ X – 45 ⎞ Notes


value Frequency ⎜⎝ 10 ⎟⎠

(x) (f) cf (dx) fdx


0–10 5 4 4 – 4 – 16
10–20 15 2 6 – 3 – 6
20–30 25 18 24 – 2 – 36
30–40 35 22 46 – 1 – 22
40–50 45 21 67 0 0
50–60 55 19 86 1 19
60–70 65 10 96 2 20
70–80 75 3 99 3 9
80–90 85 1 100 4 4
Ȉf = 100 Ȉfdx = – 28
Σfdx −28
Mean = A + × i = 45 + ×10 = 42.2
N 100
N 100
Median = size of th item = = 50th item.
2 2
Because 50 is smaller to 67 in cf column. Median class is 40–50
N
− Cf 0
Median = l1 + 2 ×i
f
50 − 46 4
Median = 40 + ×10 = 40 + ×10 = 41.9
21 21
Apply, Mode = 3 median –2 mean
Mode = 3 × 41.9 –2 × 42.2 = 125.7 – 84.3 = 41.3.
Example 33 : Median and mode of the wage distribution are known to
be Rs. 33.5 and 34 respectively. Find the missing values.
Wages (Rs.) No. of workers
0–10 4
10–20 16
20–30 ?
30–40 ?

PAGE 71
© Department of Distance & Continuing Education, Campus of Open Learning,
School of Open Learning, University of Delhi
B.COM. (PROGRAMME)

Notes Wages (Rs.) No. of workers


40–50 ?
50–60 6
60–70 4
Total = 230
Solution : We assume the missing frequencies as 20–30 as x, 30–40 as
y, and 40–50 as 230 – (4 + 16 + x + y + 6 + 4) = 200 – x – y.
We now proceed further to compute missing frequencies :
Wages (Rs.) No. of workers Cumulative frequencies
X f cf
0–10 4 4
10–20 16 20
20–30 x 20 + x
30–40 y 20 + x + y
40–50 200 – x – y 220
50–60 6 226
60–70 4 230
N = 230
N
− cf 0
Apply, Median = l1 + 2 × (l2 − l1 )
f
115 − (20 + x)
= 30 + × (40 − 30)
y
y (33.5 – 30) = (115 – 20 – x)10
3.5 y = 1150 – 200 – 10x
10x + 3.5y = 950 ...................(i)
f1 − f 0
Apply, Mode = l1 + (l2 − l1 )
2 f1 − f 0 − f 2
15 − 8
= 20 + (30 − 20)
30 − 8 − 12
4(3y – 200) = 10 (y – x)
10x + 2y = 800 ....................(ii)

72 PAGE
© Department of Distance & Continuing Education, Campus of Open Learning,
School of Open Learning, University of Delhi
BUSINESS STATISTICS

Subtract equation (ii) from equation (i), Notes


150
1.5y = 150, y = = 100
1.5
Substitute the value of y = 100 in equation (i), we get
10x + 3.5 (100) = 950
10x = 950 – 350
x = 600/10 = 60.
Third missing frequency = 200 –x – y = 200 – 60 – 100 = 40.
Merits of Mode:-
‹ It can be easily calculated and understandable like it can simply find
out merely by inspection.
‹ Like Median, it is also not affected by extreme observations.
‹ It is very easy to calculate Mode from open ended class distribution.
Demerits of Mode:-
‹ It is not based upon all values in the distribution.
‹ It is not rigidly defined.
‹ If bi-modal distribution is given then it is difficult to calculate Mode.

2.13 Summary
Central tendency indicates the location of the centre of a set of data.
It is the average value an average is a typical value which is used to
represent the entire set of values and is used as a benchmark to make
comparisons, a good average is expected to be based on all values; not
affected unduly by the presence of extremely large or small values in
the data, amenable to further algebraic treatment and having sampling
stability. Averages are distinguished as mathematical and positional.
Arithmetic mean is a mathematical average which is most commonly used
and understood and also very extensively used in statistical work. Obtained
by dividing the sum of values by their number, it is easy to calculate. It
enjoys well defined algebraic properties like zero-sum deviations, least
squares, and combined mean. It meets most of the requisites of a good
average. Geometric mean and harmonic mean are other mathematical

PAGE 73
© Department of Distance & Continuing Education, Campus of Open Learning,
School of Open Learning, University of Delhi
B.COM. (PROGRAMME)

Notes averages but they have limited and specific uses. Their calculation is
restricted only to positive values. It is possible to calculate combined
average for two or more sets of data for each of these. Geometric mean,
which is nth root of the product of n values, is basically applied to obtain
average growth rates, price changes and depreciation rates Harmonic mean
is equal to the reciprocal of the arithmetic mean of reciprocals. It is used
to average rates. Harmonic mean is used when the weights are in terms
of the numerator factor of the given rates. Arithmetic mean is correct to
use when weights are in terms of the denominator factor.
Mathematical averages can be simple or weighted and used accordingly
as all values enjoy an equal or unequal weightage. Their values can be
calculated only by using well-defined formulae and cannot be obtained
graphically. Being based on all values, they are affected in a larger
measure by the presence of extreme values. The positional averages
include median and mode. While median refers to the central value in a
set of arrayed values, the mode is that value in a series which appears
the maximum number of times. A given set of individual values or a
frequency distribution may have one or more modal values. If values in
a given set of data are all unique, there is no mode. Mode suffers from
these drawbacks. The positional averages do not possess any mathematical
properties, except that the sum of absolute deviations from median is
the least.
In addition to median, there are a number of partition values that divide
given distribution into a certain number of parts. They include quartiles,
deciles and percentiles. The partition values are not averages but they
are discussed here for the reason that their calculation proceeds in the
same manner as that of median. They are used to locate relative position
of different values clearly (like use of percentiles in the CAT entrance
examinations) and also to calculate measures of variation, skewness etc.

2.14 Self-Assessment Questions

Exercise 1 : True OR False Statements


(i) An average serves as a benchmark for comparisons.
(ii) Mean, median and mode are called positional averages while geometric
mean and harmonic mean are designated as mathematical averages.
74 PAGE
© Department of Distance & Continuing Education, Campus of Open Learning,
School of Open Learning, University of Delhi
BUSINESS STATISTICS

(iii) In the deviation method of calculating arithmetic mean, the mean Notes
is obtained by adding the mean of the deviations to the assumed
mean value.
(iv) Arithmetic mean is not suitable for open-ended frequency distributions.
(v) All averages can be distinguished as being simple and weighted.
(vi) In the weighted arithmetic mean calculation, it is immaterial whether
the weights are expressed as, say, 20% and 80% or as 4 and 16.
(vii) The sum of squares of deviations as well as the sum of deviations
from mean is equal to zero.
(viii) For calculating different measures of central tendency, it is necessary
that all class intervals have equal width.
(ix) Median cannot be calculated in open-ended class frequency distributions.
(x) In an array of 41 items, median is equal to (41 + 1 )/2 = 21.
(xi) Two sets of values. A and B. are identical except that their respective
largest values are 80 and 8,000. The median of both the distributions
shall be same.
(xii) The sum of absolute deviations from median is equal to zero.
(xiii) The quartiles divide a distribution into four equal parts.
(xiv) A distribution has 10 deciles and 100 percentiles.
(xv) In a distribution of wages of the workers of a factory, the 95th
percentile indicates the maximum wage earned by the top 95 per
cent of the workers.
(xvi) The lower quartile in a distribution with a total frequency of 800
is equal to n/4 = 200.
(xvii) The median, quartiles and percentiles can be determined graphically
only by means of a “less than” ogive.
(xviii) The lower and the upper quartiles mark off the limits within which
the middle 50 per cent of the cases fall.
(xix) It is possible to have more than one median in a given distribution.
(xx) For every frequency distribution, the upper and lower quartiles are
located at equal distance from median.

PAGE 75
© Department of Distance & Continuing Education, Campus of Open Learning,
School of Open Learning, University of Delhi
B.COM. (PROGRAMME)

Notes (xxi) A distribution can have more than two modes.


(xxii) In an array of marks (out of 100) scored by students of a class,
the mark 47 appears 48 times (total number of students is 90).
The mode of the set of values is 48.
(xxiii) It is possible to estimate mode from median and mean of a
distribution.
(xxiv) For every distribution, X >Median > Mode.
(xxv) Geometric mean is the nth root of the sum of n values.
(xxvi) Geometric mean is the appropriate measure for averaging ratios
and percentages.
(xxvii) The geometric mean of two unequal values, x and v. is equal to
the geometric mean of their arithmetic mean and harmonic mean
values.
(xxviii) The positional averages are not amenable to algebraic manipulations.
(xxix) Only mathematical averages are distinguishable as being simple
or weighted.
(xxx) Mathematical averages cannot be determined graphically.
Ans.
(i) T (ii) F (iii) T (iv) T (v) F (vi) T
(vii) F (viii) F (ix) F (x) F (xi) T (xii) F
(xiii) T (xiv) F (xv) F (xvi) F (xvii) F (xviii) T
(xix) F (xx) F (xxi) T (xxii) F (xxiii) T (xxiv) F
(xxv) F (xxvi) T (xxvii) T (xxviii) T (xxix) T (xxx) T

Exercise 2 : Questions and Answers


(i) What do you understand by an average? Discuss the desirable
properties of a good measure of central tendency.
(ii) “An average is a number indicating the central value of a group of
observations.” How far is it true for mean, median and mode? Give
illustrations.
(iii) State and explain the properties of arithmetic mean, geometric mean
and harmonic mean.
(iv) Write a note on how you would decide whether arithmetic mean
or harmonic mean should be used to calculate average in a given
case.

76 PAGE
© Department of Distance & Continuing Education, Campus of Open Learning,
School of Open Learning, University of Delhi
BUSINESS STATISTICS

(v) Define median, quartiles, deciles and percentiles. State the property Notes
of median. Does it have any practical application?
(vi) Write a detailed note on the choice of an average. Which average
would be more suitable in the following cases:
(a) Average size of ready-made garments sold by a store.
(b) Average intelligence level of students of a class.
(c) Average rate of growth of population per decade.
(vii) A taxi ride in New Delhi costs Rs. 20 for the first kilometer and Rs.
11 per kilometer thereafter. Assume that the cost of each kilometer
is incurred at the beginning of the kilometer. The waiting charges
are Rs. 30 per hour or a part thereof, subject to a minimum of 15
minutes stay. Calculate the effective average cost per kilometer
to a customer who rides a taxi from the Railway Station for her
home 21.7 kilometers away and chooses to stay for a coffee for 25
minutes on the way.
(viii) Find the arithmetic mean of the first 100 natural numbers.
(ix) The arithmetic mean of a distribution is known to be 55.45. It is
written below with the variate given in codified values. You are
required to determine the class intervals.
dƍ –3 –2 –1 0 1 2 3
f 10 28 30 42 65 15 10
It is known that various dƍYDOXHVKDYHEHHQFDOFXODWHGDV X – A)/10.
(x) In a hotel, a total of 500 bulbs were installed simultaneously and
their failure over time was observed as detailed below.
End of week : 1 2 3 4 5 6 7
No. of failures : 12 40 108 242 346 428 500
You are required to calculate the mean life of the bulbs.
(xi) A factory employs 100 workers. The mean daily wages of 99 of
these workers is Rs. 85 while the wages of the 100th worker are
Rs. 99 more than the mean wages of all the workers. Obtain mean
wages of the workers of the factory.
(xii) The following data gives the distribution of accidents in a large
city over weekdays of the last month :

PAGE 77
© Department of Distance & Continuing Education, Campus of Open Learning,
School of Open Learning, University of Delhi
B.COM. (PROGRAMME)

Notes Day: Sun. Mon. Tue. Wed. Thu. Fri. Sat.


Average number 26 16 12 10 8 10 8
of accidents:
Over the particular month, there were 5 Mondays and 5 Tuesdays.
Calculate the mean number of accidents per day.
(xiii) The table below shows the number of skilled and unskilled workers
in two factories and their average daily wages.
Worker Factory A Factory B
category Number Wages per Number Wages per
day (Rs.) day (Rs.)
Skilled 150 180 350 175
Unskilled 850 130 650 125
Determine the average daily wages for each factory. Also, give reasons
why the results show that the average daily wages in Factory B
are higher than the average daily wages in Factory A, even though
in Factory B the average wages for both categories of workers are
lower.
(xiv) The average sales made by 18 salesmen in a company were reported
to be Rs. 73,560 during the last month. It was discovered later that
the sales made by one of the salesmen was recorded as Rs. 92,280
instead of the actual Rs. 22,280 and that of another salesman at
Rs. 16,630 instead of the actual Rs. 76,630. Compute the actual
average sales made.
(xv) The arithmetic mean of daily wages of 300 weavers and 250 spinning
machine workers are Rs. 198 and Rs. 179, respectively. It is given
that the arithmetic mean of all the workers in the factory is Rs.
187. Find the total number of workers in the factory if it is also
given that the arithmetic mean of the daily wages of the remaining
workers is Rs. 180.5.
(xvi) Of a batch of 20 students, four students failed while 7 students passed
with distinction. The remaining students, who passed ordinarily,
scored the following marks:
47, 49, 52, 52, 57, 58, 59, 64, 66

78 PAGE
© Department of Distance & Continuing Education, Campus of Open Learning,
School of Open Learning, University of Delhi
BUSINESS STATISTICS

The students who failed secured 23 marks on an average while Notes


the students who passed with distinction fetched 76 marks on an
average. With this information, you are required to obtain the mean
and median marks.
(xvii) A survey of 350 families in a town yielded the following information:

No. of children: 0 1 2 3 4 or more


No. of families: 13 94 146 67 30
Find the median number of children in the families.
(xviii) The following table gives the distribution of monthly income of
600 families in a certain city:
Monthly income (in ’000 Rs.) No. of families
10 – 20 60
20 – 30 170
30 – 40 200
40 – 50 60
50 – 60 50
60 – 70 40
70 – 80 20
Draw a ‘less than’ ogive and a ‘more than’ ogive for these data on
a graph. Read the median income and the limits within which the
middle 50 per cent of families have their income.
(xix) Find the missing frequencies in the following distribution if N =
100 and median = 30.
Marks No. of students
0 – 10 10
10 – 20 ?
20 – 30 25
30 – 40 30
40 – 50 ?
50 – 60 10

PAGE 79
© Department of Distance & Continuing Education, Campus of Open Learning,
School of Open Learning, University of Delhi
B.COM. (PROGRAMME)

Notes (xx) The number of sales made by a shoe store in the City Mall during
the past 20 days is as follows:
7 6 13 16 8 5 9 9 10 19
16 8 11 13 7 24 22 15 21 21
Find the 50th, 75th and 88th percentiles.
Ans.
(vii) Rs. 12.14/km (viii) 50.5 (ix) 20–30, 30–40, etc.
(x) 4.15 Weeks (xi) Rs. 86 (xii) 14.27
(xiii) A Rs. 138 B (xiv) Rs. 73.004 (xv) 750
Rs. 143
(xvi) Mean = 56.4, (xvii) 2 (xviii) Med = 33.5, 25–43
Median = 58.5 app.
(xix) 15 and 10 (xx) 12, 18.25 and
21.48

80 PAGE
© Department of Distance & Continuing Education, Campus of Open Learning,
School of Open Learning, University of Delhi
L E S S O N

3
Measures of Variation –
Absolute and Relative
STRUCTURE
3.1 Learning Objectives
3.2 Need and Importance
3.3 What is Variation?
3.4 Requisites of a Good Measure of Variation
3.5 Types of Variation
3.6 Methods Computing Variation
3.7 Revisionary Problems
3.8 Summary
3.9 Self-Assessment Questions

3.1 Learning Objectives


After reading this lesson, you should be able to:
‹ Understand the meaning, need and requisites of a good measure of variation.
‹ Differentiate between absolute and relative measure of variation.
‹ Identify and compute different types of variation such as range, quartile deviation,
average deviation, standard deviation and variance.
‹ Comprehend merits and demerits of different measures and properties of standard
deviation.
‹ Comment upon the variability of problems with the help of coefficient of variation.

3.2 Need and Importance


Measures of central tendency, Mean, Median, Mode, etc., indicate the central position
of a series. They indicate the general magnitude of the data but fail to reveal all the

PAGE 81
© Department of Distance & Continuing Education, Campus of Open Learning,
School of Open Learning, University of Delhi
B.COM. (PROGRAMME)

Notes peculiarities and characteristics of the series. In other words, they fail
to reveal the degree of the spread out or the extent of the variability in
individual items of the distribution. This can be known by certain other
measures, known as ‘Measures of Dispersion’ or Variation.
We can understand variation with the help of the following example:
Series I Series II Series III
10 2 10
10 8 12
10 20 8
ȈX = 30 30 30
ΣX 30 30 30
X= = = 10 X= = 10 X= = 10
N 3 3 3
In all three series, the value of arithmetic mean is 10. On the basis of
this average, we can say that the series are alike. If we carefully examine
the composition of three series, we find the following differences:
(i) In case of 1st series, the value are equal; but in 2nd and 3rd series,
the values are unequal and do not follow any specific order.
(ii) The magnitude of deviation, item-wise, is specific different for the
1st, 2nd and 3rd series. But all these deviations cannot be ascertained
if the value of ‘simple mean’ is taken into consideration.
(iii) In these three series, it is quite possible that the value of arithmetic
mean is 10; but the value of median may differ from each other.
This can be understood as follows:
I II III
10 2 8
10 Median 8 Median 10 Median
10 20 12
The value of ‘Median’ in 1st series is 10, in 2nd series = 8 and in
3rd series = 10. Therefore, the value of Mean and Median are not
identical.
(iv) Even though the average remains the same, the nature and extent of
the distribution of the size of the items may vary. In other words,
the structure of the frequency distributions may differ even though
their means are identical.

82 PAGE
© Department of Distance & Continuing Education, Campus of Open Learning,
School of Open Learning, University of Delhi
BUSINESS STATISTICS

3.3 What is Variation? Notes

Simplest meaning that can be attached to the word ‘dispersion’ is a


lack of uniformity in the sizes or quantities of the items of a group or
series. According to Reiglemen, “Dispersion is the extent to which the
magnitudes or qualities of the items differ, the degree of diversity.” The
word dispersion may also be used to indicate the spread of the data.
In all these definitions, we can find the basic property of dispersion as
a value that indicates the extent to which all other values are dispersed
about the central value in a particular distribution.

3.4 Requisites of a Good Measure of Variation


There are Certain Pre-requisites for a good Measure of Dispersion:
1. It should be simple to understand.
2. It should be easy to compute.
3. It should be rigidly defined.
4. It should be based on each individual item of the distribution.
5. It should be capable of further algebraic treatment.
6. It should have sampling stability.
7. It should not be unduly affected by the extreme items.

3.5 Types of Variation


The measures of dispersion can be either ‘absolute’ or ‘relative’. Absolute
measures of dispersion are expressed in the same units in which the
original data are expressed. For example, if the series is expressed as
Marks of the students in a particular subject; the absolute dispersion, will
provide the value in Marks. The only difficulty is that if two or more
series are expressed in different units, the series cannot be compared on
the basis of dispersion.
‘Relative’ or ‘Coefficient’ of dispersion is the ratio or the percentage of
a measure of absolute dispersion to an appropriate average. The basic
advantage of this measure is that two or more series can be compared
with each other, despite the fact they are expressed in different units.

PAGE 83
© Department of Distance & Continuing Education, Campus of Open Learning,
School of Open Learning, University of Delhi
B.COM. (PROGRAMME)

Notes Theoretically, ‘Absolute measure’ of dispersion is better. But from a


practical point of view, relative or coefficient of dispersion is considered
better as it is used to make comparison between series.

3.6 Methods computing variation


Methods of studying dispersion are divided into two types:
(i) Mathematical Methods : We can study the ‘degree’ and ‘extent’
of variation by these methods. In this category, commonly used
measures of dispersion are :
(a) Range
(b) Quartile Deviation
(c) Average Deviation
(d) Standard deviation and coefficient of variation.
(ii) Graphic Methods : Where we want to study only the extent of
variation, whether it is higher or lesser a Lorenz-curve is used.

3.6.1 Range
It is the simplest method of studying dispersion. Range is the difference
between the smallest value and the largest value of a series. While
computing range, we do not take into account frequencies of different
groups.
Formula: Absolute Range = L– S
L −S
Coefficient of Range =
L+S
where, L represents largest value in a distribution.
S represents smallest value in a distribution.
We can understand the computation of range with the help of examples
of different series.

84 PAGE
© Department of Distance & Continuing Education, Campus of Open Learning,
School of Open Learning, University of Delhi
BUSINESS STATISTICS

(i) Raw Data Notes


Example 1 : Marks out of 50 in a subject of 12 students, in a class are
given as follows :
12, 18, 20, 12, 16, 14, 30, 32, 28, 12, 12 and 35.
In the example, the maximum or the highest marks obtained by a candidate
is ‘35’ and the lowest marks obtained by a candidate is ‘12’. Therefore,
we can calculate range;
L = 35 and S = 12
Absolute Range = L – S = 35 – 12 = 23 marks
L − S 35 − 12 23
Coefficient of Range = = = 0.49 = approx.
L + S 35 + 12 47
(ii) Discrete Series
Example 2 :
Marks of the Students in No. of students
Accounts (out of 50)
(X) (f)
Smallest 10 4
12 10
18 16
Largest 20 15
Total 45
Absolute Range = 20 – 10 = 10 marks
20 − 10 10
Coefficient of Range = = = 0.34 approx.
20 + 10 30
(iii) Continuous Series
Example 3:
X frequencies
10 – 15 4
S = 10 15 – 20 10
L = 30 20 – 25 26
25 – 30 8
Absolute Range = L – S = 30 –10 = 20 marks

PAGE 85
© Department of Distance & Continuing Education, Campus of Open Learning,
School of Open Learning, University of Delhi
B.COM. (PROGRAMME)

Notes L − S 35 − 12 20
Coefficient of Range = = = = 0.5 approx.
L + S 35 + 12 40
Range is a simplest method of studying dispersion. It takes lesser time
to compute the ‘absolute’ and ‘relative’ range. Range does not take into
account all the values of a series, i.e.it considers only the extreme items
and middle items are not given any importance. Therefore, Range cannot
tell us anything about the character of the distribution. Range cannot
be computed in the case of ‘open ends’ distribution i.e., a distribution
where the lower limit of the first group and upper limit of the higher
group is not given.
The concept of range is useful in the field of quality control and, to
study the variations in the prices of the shares etc.

3.6.2 Quartile Deviation


The concept of ‘Quartile Deviation’ does take into account only the
values of the ‘Upper quartile’ (Q3) and the ‘Lower quartile’ (Q1). Quartile
Deviation is also called ‘inter-quartile range’. It is a comparatively better
method when we are interested in knowing the range within which certain
proportion of the items fall.
‘Quartile Deviation’ can be obtained as :
(i) Inter-quartile range = Q3 – Q1
Q3 − Q1
(ii) Semi-quartile range =
2
Q3 − Q1
(iii) Coefficient of Quartile Deviation =
2
Calculation of Inter-quartile Range, semi-quartile Range and Coefficient
of Quartile Deviation in case of Raw Data
Example 4 : Suppose the values of X are : 20, 12, 18, 25, 32, 10
In case of quartile-deviation, it is necessary to calculate the values of
Q1 and Q3 by arranging the given data in ascending or descending order.
Therefore, the arranged data are : (in ascending order)

86 PAGE
© Department of Distance & Continuing Education, Campus of Open Learning,
School of Open Learning, University of Delhi
BUSINESS STATISTICS

X = 10, 12, 18, 20, 25, 32 Notes


No. of items = 6
⎛ N +1⎞ ⎛ 6 +1⎞
Q1 = the value of ⎜ ⎟ th item = ⎜ ⎟ = 1.75th item
⎝ 4 ⎠ ⎝ 4 ⎠
= the value of 1st item + 0.75 (value of 2nd item – value of 1st
item)
= 10 + 0.75 (12 – 10) = 10 + .75(2) 10 + 1.50 = 11.50
⎛ N +1⎞ ⎛ 6 +1⎞
Q3 = the value of 3 ⎜ ⎟ th item = 3 ⎜ ⎟
⎝ 4 ⎠ ⎝ 4 ⎠
= the value of 3(7/4)th item = the value of 5.25th item
= the value of 5th item + 0.25 (the value of 6th item minus the value
of 5th item)
= 25 + 0.25 (32 – 25) = 25 + 0.25(7) = 26.75.
Therefore,
(i) Inter-quartile range = Q3 – Q1 = 26.75 – 11.50 = 15.25
Q3 − Q1 15.25
(ii) Semi-quartile range = = = 7.625
2 2
Q3 − Q1 26.75 − 11.50 15.25
(iii) Coefficient of Quartile Deviation = = = = 0.39
Q3 + Q1 26.75 + 11.50 38.25
approx.
Calculation of Inter-quartile Range, semi-quartile Range and Coefficient
of Quartile Deviation in discrete series
Example 5 : Suppose a series consists of the salaries (Rs.) and number
of the Workers in a factory:
Salaries (Rs.) No. of workers
60 4
100 20
120 21
140 16
160 9

PAGE 87
© Department of Distance & Continuing Education, Campus of Open Learning,
School of Open Learning, University of Delhi
B.COM. (PROGRAMME)

Notes Solution : In the problem, we will first, compute the values of Q3 and Q1.
Salaries (Rs.) No. of workers Cumulative frequencies
(x) (f) (c.f.)
60 4 4
100 20 24 – Q lies in this cumulative
1

120 21 45 frequency
140 16 61
160 9 70
1  Ȉf = 70
Calculation of Q1 : Calculation of Q3 :
⎛ N +1⎞ ⎛ N +1⎞
Q1 = size of ⎜ ⎟ th item Q3 = size of 3 ⎜ ⎟ th item
⎝ 4 ⎠ ⎝ 4 ⎠

⎛ 70 + 1 ⎞ ⎛ 70 + 1 ⎞
= size of ⎜ ⎟ th item = 17.75th item = size of 3 ⎜⎝ 4 ⎟⎠ th item = 53.25th
⎝ 4 ⎠ item

17.75 lies in the cumulative frequency 24, 53.25 lies in the cumulative frequency 61
which is corresponding to the value which is corresponding to Rs. 140
Rs. 100
? Q1 = Rs. 100 ? Q3 = Rs. 140
(i) Inter-quartile range = Q3 – Q1= Rs. 140 – Rs. 100 = Rs. 40
Q3 − Q1 ⎛ 140 − 100 ⎞
(ii) Semi-quartile range = =⎜ ⎟ = Rs. 20
2 ⎝ 2 ⎠
Q3 − Q1 140 − 100 40
(iii) Coefficient of Quartile Deviation = = = = 0.17
Q3 + Q1 140 + 100 240
approx.
Calculation of Inter-quartile range, semi-quartile range and Coefficient
of Quartile Deviation in the case of continuous series
Example 6 : We are given the following data :
Salaries (Rs.) No. of workers
10–20 4
20–30 6

88 PAGE
© Department of Distance & Continuing Education, Campus of Open Learning,
School of Open Learning, University of Delhi
BUSINESS STATISTICS

Salaries (Rs.) No. of workers Notes


30–40 10
40–50 5
Total 25
In this example, the values of Q3 and Q1 are obtained as follows :
Salaries (Rs.) No. of workers Cumulative frequencies
(x) (f) (c.f.)
10–20 4 4
20–30 6 10
30–40 10 20
40–50 5 25
N = 25
N
− cf 0 N
Q1 = l1 + 4 ×i is used to find out Q1 group
f 4
where l1 = lower limit of Q1 group
f = frequency of Q1 group
i = magnitude of Q1 group (l2 – l1)
cf0 = cumulative frequency of the group preceeding Q1group.
N 25
Therefore, or or 6.25. It lies in the cumulative frequency 10,
4 4
which is corresponding to class 20–30.
Therefore, Q1 group is 20–30.
6.25 − 4
Q1 = 20 + ×10 = 20 + 3.75 = 23.75
6
N
where, l1 = 20, f = 6, i = 10, = 6.25, cf0 = 4
4
3N
− Cf 0
Q3 = l1 + 4 ×i
f

PAGE 89
© Department of Distance & Continuing Education, Campus of Open Learning,
School of Open Learning, University of Delhi
B.COM. (PROGRAMME)

Notes 3N 3 × 25 75
= = = 18.75 which lies in the cumulative frequency 20, which
4 4 4
is corresponding to class 30–40.
Therefore Q3 group is 30–40.
3N
where, l1 = 30, i = 10, = 18.75, cf0 = 10, f = 10
4
18.75 − 10
Q3 = 30 + ×10 = Rs. 38.75
10
Therefore :
(i) Inter-quartile range = Q3 – Q1 = Rs. 38.75 – Rs. 23.75 = Rs. 15.00
Q3 − Q1 15.00
(ii) Semi-quartile range = = = 7.50
2 2
(iii) Coefficient of Quartile Deviation =
Q3 − Q1 Rs. 38.75 – Rs. 23.75 15
= = = 0.24.
Q3 + Q1 Rs. 38.75 + Rs. 23.75 62.50

Advantages of Quartile Deviation


Some of the important advantages of this measure of dispersion are :
(i) It is easy to calculate. We are required simply to find the values of
Q1 and Q3 and then apply the formula of absolute and coefficient
of quartile deviation.
(ii) It has better results than range method. While calculating range, we
take only the extreme values that make dispersion erratic. In the
case of quartile deviation, we take into account middle 50% items.
(iii) The quartile deviation is not affected by the extreme items.
Disadvantages
(i) It is completely dependent on the central items. If these values are
irregular and abnormal the result is bound to be affected.
(ii) All the items of the frequency distribution are not given equal
importance in finding the values of Q1 and Q3.
(iii) Because it does not take into account all the items of the series,
considered to be inaccurate of dispersion.

90 PAGE
© Department of Distance & Continuing Education, Campus of Open Learning,
School of Open Learning, University of Delhi
BUSINESS STATISTICS

Similarly, sometimes we calculate percentile range, say, 90th and 10th Notes
percentile as it gives slightly better measure of dispersion, in certain
cases. If we consider the calculations, then
(i) Absolute percentile range = P90 – P10
P90 − P10
(ii) Coefficient of percentile range =
P90 + P10

This method of calculating dispersion can be applied generally in the


case of open end series where the importance of extreme values are not
considered.

3.6.3 Average Deviation


Average deviation’ is defined as a value, which is obtained by taking the
average of the deviations of various items, from a measure of central
tendency, Mean or Median or Mode, after ignoring negative signs.
Generally, the measure of central-tendency, from which the deviations
are taken, is specified in the problem. If nothing is mentioned regarding
the measure of central tendency specified then deviations are taken from
median because the sum of the deviations (after ignoring negative signs)
is minimum.
Computation in case of raw data
Σ|d |
(i) Absolute Average Deviation about Mean or Median or Mode =
N
where: N = Number of observations,
|d| = deviations taken from Mean or Median or Mode ignoring signs.
Average Deviation about Mean or Median or Mode
(ii) Coefficient of A.D. =
Mean or Median or Mode
Steps to Compute Average Deviation:
(i) Calculate the value of Mean or Median or Mode.
(ii) Take deviations from the given measure of central-tendency and
they are shown as d.
(iii) Ignore the negative signs of the deviation that can be shown as |d|
DQG DGG WKHP WR ILQG Ȉ _d|.

PAGE 91
© Department of Distance & Continuing Education, Campus of Open Learning,
School of Open Learning, University of Delhi
B.COM. (PROGRAMME)

Notes (iv) Apply the formula to get Average Deviation about Mean or Median
or Mode.
Example 7 : Suppose the values are 5, 5, 10, 15, 20. We want to calculate
Average Deviation and Coefficient of Average Deviation about Mean or
Median or Mode.
Solution :
Average Deviation about mean (Absolute and Coefficient).
Deviation from mean Deviations after ignoring signs
(X) d |d|
ΣX
5 – 6 6 X = N

5 – 6 6 ZKHUH 1  Ȉ;  


55
10 + 1 1 X = 5 = 11
15 + 4 4
20 + 9 9
ȈX = 55 Ȉ _d| = 26
Σ | d | 26
Average Deviation about Mean = = = 5.2.
N 5
Coefficient of Average Deviation about Mean =
Mean Deviation about Mean 5.2
= = 0.47.
Mean 11
Average Deviation (Absolute and Coefficient) about Median
X Deviation from Deviations after ignoring
median d negative signs |d|
5 – 5 5
5 – 5 5
Median 10 0 0
15 + 5 5
20 + 10 10
N = 5 Ȉ|d| = 25

92 PAGE
© Department of Distance & Continuing Education, Campus of Open Learning,
School of Open Learning, University of Delhi
BUSINESS STATISTICS

Σ | d | 25 Notes
Average deviation about Median = = = 5.2 .
N 5
Coefficient of Average Deviation about median
A.D. about Median 5
= = = 0.5
Median 10
Average Deviation (Absolute and Coefficient) about Mode
X Deviation from mode d | d |
5 0 0
Mode 5 0 0
10 + 5 5
15 + 10 10
20 + 15 15
N= 5 Ȉ _d|= 30
Σ | d | 30
Average deviation about Mode = = = 6.
N 5
A.D. about Mode 6
Coefficient of Average Deviation about mode = = = 1.2.
Mode 5
Average deviation in case of discrete and continuous series
Σf | d |
Average Deviation about Mean or Median or Mode =
N
where N = No. of items
| d | = deviations from Mean or Median or Mode, after ignoring
negative signs.
Coefficient of A.D. about Mean or Median or Mode
A.D. about Mean or Median or Mode
=
Value of Mean or Median or Mode
Example 8 : Suppose we want to calculate coefficient of Average Deviation
about Mean from the following discrete series:
X Frequency
10 5
15 10

PAGE 93
© Department of Distance & Continuing Education, Campus of Open Learning,
School of Open Learning, University of Delhi
B.COM. (PROGRAMME)

Notes X Frequency
20 15
25 10
30 5
Solution : First of all, we shall calculate the value of arithmetic Mean,
Calculation of Simple Mean
X f fX
10 5 50
15 10 150
20 15 300
25 10 250
30 5 150
N = 45 ȈfX = 900
Calculation of Coefficient of Average Deviation about Mean
Deviation Deviations after ignoring Ȉf|d|
from mean negative signs | d |
X f d
10 5 – 10 10 50
15 10 – 5 5 50
20 15 0 0 0
25 10 + 5 5 50
30 5 + 10 10 50
N= 45 Ȉf |d| = 200
A.D. about Mean 4.4
Coefficient of Average Deviation about Mean = = = 0.22
Mean 20
Σ | d | 200
Average Deviation about Mean = = = 4.44 approx.
N 45
In case we want to calculate coefficient of Average Deviation about
Median from the following data:
Class Interval Frequency
10–14 5
15–19 10
20–24 15

94 PAGE
© Department of Distance & Continuing Education, Campus of Open Learning,
School of Open Learning, University of Delhi
BUSINESS STATISTICS

Class Interval Frequency Notes


25–29 10
30–34 5
N = 45
First of all we shall calculate the value of Median but it is necessary
to find the ‘real limits’ of the given class-intervals. This is possible by
subtracting 0.5 from the lower-limits and added to the upper limits of
the given classes. Hence, the real limits shall be : 9.5–14.5, 14.5–19.5,
19.5–24.5, 24.5–29.5 and 29.5–34.5.
Calculation of Median
Class Intervals f c.f.
9.5–14.5 5 5
14.5–19.5 10 15
19.5–24.5 15 30
24.5–29.5 10 40
29.5–34.5 5 45
N =45
N
− Cf 0
Median = l1 + 2 ×i
f

where l1 = lower limit of median group


i = magnitude of median group
f = frequency of median group
Cf0 = cumulative frequency of the group preceding median group
n
= size of median group
2

N 45
? Median size = th item i.e. = 22.5
2 2
It lies in the cumulative frequency 30, which is corresponding to class
interval 19.5–24.5.
Median group is 19.5–24.5

PAGE 95
© Department of Distance & Continuing Education, Campus of Open Learning,
School of Open Learning, University of Delhi
B.COM. (PROGRAMME)

Notes 22.5 − 15 7.5


Median = 19.5 + × 5 = 19.5 + × 5 = 19.5 + 2.5 = 19.5 + 2.5 = 22
15 15
Calculation of Coefficient of Average Deviation about Median
Class Frequency Mid points Deviation from Deviations after ignoring
Intervals
f x median (22) negative signs |d| f|d|
9.5–14.5 5 12 – 10 10 50
14.5–19.5 10 17 – 5 5 50
19.5–24.5 15 22 0 0 0
24.5–29.5 10 27 + 5 5 50
29.5–34.5 5 32 + 10 10 50
N = 45 Ȉf |d| = 200
A.D. about Median
Coefficient of Average Deviation about Median =
Median
Σ | d | 200
Average Deviation about Mean = = = 4.44 approx.
N 45
4.4
Coefficient of A.D. about Median = = 0.2.
22
Advantages of Average Deviations
1. Average deviation takes into account all the items of a series and
hence, it provides sufficiently representative results.
2. It simplifies calculations since all signs of the deviations are taken
as positive.
3. Average Deviation may be calculated either by taking deviations
from Mean or Median or Mode.
4. Average Deviation is not affected by extreme items.
5. It is easy to calculate and understand.
6. Average deviation is used to make healthy comparisons.
Disadvantages of Average Deviation
1. It is illogical and mathematically unsound to assume all negative
signs as positive signs.
2. Because the method is not mathematically sound, the results obtained
by this method are not reliable.

96 PAGE
© Department of Distance & Continuing Education, Campus of Open Learning,
School of Open Learning, University of Delhi
BUSINESS STATISTICS

3. This method is unsuitable for making comparisons either of the Notes


series or structure of the series.
This method is more effective during the reports presented to the general
public or to groups who are not familiar with statistical methods.

3.6.4 Standard Deviation


7KHVWDQGDUGGHYLDWLRQZKLFKLVVKRZQE\*UHHNOHWWHUı UHDGDVVLJPD 
is extremely useful in judging the representativeness of the mean. The
concept of standard deviation which was introduced by Karl Pearson, has
a practical significance because it is free from all defects which exists
in case of range, quartile deviation or average deviation.
Standard deviation is calculated as the square root of average of squared
deviations taken from actual mean. It is also called root mean square
deviation. The square of standard deviation i.e. ı2 is called ‘variance’ in
statistics.
Calculation of standard deviation in case of raw data
There are four ways of calculating standard deviation for raw data:
(i) When actual values are considered;
(ii) When deviations are taken from actual mean;
(iii) When deviations are taken from assumed mean; and
(iv) When ‘step deviations’ are taken from assumed mean.
(i) When the actual values are considered :

ΣX 2
ı  − ( X ) 2 where N = Number of the items,
N
ΣX 2
or ı2 = − ( X )2 X = Given values in the series
N
X = Arithmetic mean of the values
We can also write the formula as follows :
2
ΣX 2 ⎛ ΣX ⎞ ΣX
ı  −⎜ ⎟ where X =
N ⎝ N ⎠ N

PAGE 97
© Department of Distance & Continuing Education, Campus of Open Learning,
School of Open Learning, University of Delhi
B.COM. (PROGRAMME)

Notes Steps to calculate ı


(i) Compute simple mean of the given values.
(ii) Square the given values and aggregate them.
(iii) Apply the formula to find the value of standard deviation.
Example 9 : Suppose the values are given 2, 4, 6, 8, 10. We want to
apply the formula
ΣX 2
ı  − ( X )2
N

Solution : We are required to calculate the values of. They are calculated
as follows :
X X2
220
ı − (6)2
5
2 4 = 44 − 36 = 8 = 2.828

4 16 9DULDQFH ı2) = ( 8) 2 = 8
ΣX 30
6 36 X = = =6
N 5
8 64
10 100
N = 5 Ȉ; = 220
2

There are certain specific problems, where the method can be applied. It
is different type of problem which is given as follows :
(ii) When the deviations are taken from actual mean

Σx 2
ı  where N = No. of items and x = ( X − X )
N
Steps to Calculate ı

(i) Compute the deviations of given values from actual mean i.e., ( X − X )
and represent them by x.
(ii) Square these deviations and aggregate them.

98 PAGE
© Department of Distance & Continuing Education, Campus of Open Learning,
School of Open Learning, University of Delhi
BUSINESS STATISTICS

Notes
Σx 2
(iii 8VH WKH IRUPXOD ı 
N
Example 10 : We are given values as 2, 4, 6, 8, 10. We want to find
out standard deviation.

X x = (X – X ) x2
2 2 – 6 = –4 (– 4 )2 = 16
4 4 – 6 = –2 (–2)2 = 4
6 6 – 6 = 0 = 0
8 8 – 6 = + 2 (2)2 = 4
10 10 – 6 = + 4 (4)2 = 16
N = 5 Ȉx2 = 40
⎛ ΣX 30 ⎞
X = 6⎜ = ⎟
⎝ N 5 ⎠

Σx 2 40
ı  = = 8 = 2.828
N 5
(iii) When the deviations are taken from assumed mean
2
Σdx 2 ⎛ Σdx ⎞
ı  −⎜ ⎟
N ⎝ N ⎠
where, N = No. of items.
dx = deviations from assumed mean i.e., (X – A).
A = assumed mean
Steps to Calculate :
(i) We consider any value as assumed mean. The value may be given
in the series or may not be given in the series.
(ii) We take deviations from the assumed value i.e., (X – A), to obtain
dx IRU WKH VHULHV DQG DJJUHJDWH WKHP WR ILQG Ȉdx.
(iii) We square these deviations to obtain dx2 and aggregate them to find
Ȉdx2.
(iv) Apply the formula given above to get standard deviation.

PAGE 99
© Department of Distance & Continuing Education, Campus of Open Learning,
School of Open Learning, University of Delhi
B.COM. (PROGRAMME)

Notes Example 11 : Suppose the values are given as 2, 4, 6, 8 and 10. We can
obtain the standard deviation as:
X dx = (X – A) dx2
2 – 2 = (2 – 4) 4
assumed mean (A) 4 0 = (4 – 4) 0
6 + 2 = (6 – 4) 4
8 + 4 = (8 – 4) 16
10 + 6 = (10 – 4) 36
N = 5 Ȉdx = 10 Ȉdx2= 60
2 2
Σdx 2 ⎛ Σdx ⎞ 60 ⎛ 10 ⎞
ı  −⎜ ⎟ = − ⎜ ⎟ = 12 − 4 = 8 = 2.828.
N ⎝ N ⎠ 5 ⎝5⎠

(iv) When step deviations are taken from assumed mean


2
Σdx 2 ⎛ Σdx ⎞
ı  −⎜ ⎟ ×i
N ⎝ N ⎠
where i = Common factor, N = Number of items, dx = Step-deviations
X − A⎞
= ⎛⎜ ⎟
⎝ i ⎠
Steps to Calculate ı :
(i) We consider any value as assumed mean from the given values or
from outside.
(ii) We take deviation from the assumed mean i.e., (X – A).
(iii) We divide the deviations obtained in step (ii) with a common factor
to find step deviations and represent them as dx and aggregate them
WR REWDLQ Ȉdx.
(iv) We square the step deviations to obtain dx2and aggregate them to
ILQG Ȉdx2.
Example 12 : We continue with the same example to understand the
computation of Standard Deviation.

100 PAGE
© Department of Distance & Continuing Education, Campus of Open Learning,
School of Open Learning, University of Delhi
BUSINESS STATISTICS

Notes
⎛d ⎞
X d = (X – A) dx = ⎜ ⎟ ,i = 2 dx 2
⎝i⎠
2 –2 –1 1
A= 4 0 0 0
6 +2 1 1
8 +4 2 4
10 +6 3 9
N = 5 Ȉdx = 5 Ȉdx = 15
2

2
Σdx 2 ⎛ Σdx ⎞
ı  −⎜ ⎟ × i where N = 5, i = 2, dx   Ȉdx2 = 15
N ⎝ N ⎠
2
15 ⎛ 5 ⎞
ı   − ⎜ ⎟ × 2 = 3 − 1 × 2 = 2 × 2 = 1.414 × 2 = 2.828.
5 ⎝5⎠
Note : We can notice an important point that the standard deviation value
is identical by four methods. Therefore, any of the four formulae can be
applied to find the value of standard deviation. But the suitability of a
formula depends on the magnitude of items in a question.
σ
Coefficient of Standard-deviation =
X
,Q WKH DERYH JLYHQ H[DPSOH ı = 2.828 and
σ 2.828
Therefore, coefficient of standard deviation = = = 0.471
X 6
Coefficient of Variation or C.V.
σ 2.828
= ×100 = ×100 = 47.1%
X 6
Generally, coefficient of variation is used to compare two or more series.
If coefficient of variation (C.V.) is more in one series as compared to
the other, there will be more variations in that series, lesser stability or
consistency in its composition. If coefficient of variation is lesser as
compared to other series, it will be more stable or consistent. Moreover,
that series is always better where coefficient of variation is lesser or
coefficient of standard deviation is lesser.

PAGE 101
© Department of Distance & Continuing Education, Campus of Open Learning,
School of Open Learning, University of Delhi
B.COM. (PROGRAMME)

Notes Example 13 : Suppose we want to compare two firms where the salaries
of the employees are given as follows:
Firm A Firm B
No. of workers 100 100
Mean salary (Rs.) 100 80
Standard-deviation (Rs.) 40 45
Solution : We can compare these firms either with the help of coefficient
of standard deviation or coefficient of variation. If we use coefficient of
variation, then we shall apply the formula :
⎛σ ⎞
C.V. = ⎜ ×100 ⎟
⎝X ⎠

Firm A Firm B
40 45
C.V. = ×100 = 40% C.V. = ×100 = 56.25%
100 80
X = 100, σ = 40 X    ı  
Because the coefficient of variation is lesser for firm A as compared to
firm B, therefore, firm A is better.
Calculation of standard-deviation in discrete and continuous series
We use the same formula for calculating standard deviation for a continuous
series and a discrete series. The only difference that in discrete series,
values and frequencies are given whereas in a continuous series, class-
intervals and frequencies are given. When the mid-points of these class-
intervals are obtained, a continuous series takes the shape of a discrete
series. Alphabet X denotes values in a discrete series and mid points in
a continuous series.
When the deviations are taken from actual mean
We use the same formula for calculating standard deviation for a continuous
series:

Σfx 2
ı 
N

102 PAGE
© Department of Distance & Continuing Education, Campus of Open Learning,
School of Open Learning, University of Delhi
BUSINESS STATISTICS

ZKHUH 1  1XPEHU RI WKH LWHPV Ȉf) Notes


f = Frequencies corresponding to different values or class-intervals.
x = Deviations from actual mean.
X = Values in a discrete series and mid-points in a continuous series.
Step to calculate ı
(i) Compute the arithmetic mean by applying the required formula.
(ii) Take deviations from the arithmetic mean and represent these
deviations by x.
(iii) Square the deviations to obtain values of x2.
(iv) Multiply the frequencies of the different class-intervals with x2 to
find fx2. Aggregate fx2 FROXPQ WR REWDLQ Ȉfx2.
(v) Apply the formula to obtain the value of standard deviation.
Σfx 2
If we want to calculate variance then we can take σ 2 =
N
Example 14 : We can understand the procedure by taking an example :
Class Intervals Frequency (f) Midpoints (m) fm
10 – 14 5 12 60
15 – 19 10 17 170
20 – 24 15 22 330
25 – 29 10 27 270
30 – 34 5 32 160
N= 45 Ȉfm = 990

Σfm 990
Therefore, X = = = 22 where, N   Ȉ fm = 990
N 45
Calculation of Standard Deviation
Class Mid Deviations from
Intervals points actual median = 22
f X x x2 f x2
10 – 14 5 12 –10 100 500
15 – 19 10 17 –5 25 250

PAGE 103
© Department of Distance & Continuing Education, Campus of Open Learning,
School of Open Learning, University of Delhi
B.COM. (PROGRAMME)

Notes Class Mid Deviations from


Intervals points actual median = 22
20 – 24 15 22 0 0 0
25 – 29 10 27 +5 25 250
30 – 34 5 32 + 10 100 500
N = 45 Ȉfx = 1500
2

Σfx 2
ı  where, N = 45, Σfx 2 = 1500
N

1500
ı   = 33.33 = 5.77 approx.
45
When the deviations are taken from assumed mean
In some cases, the value of simple mean may be in fractions, then it
becomes time consuming to take deviations and square them. Alternatively,
we can take deviations from the assumed mean.
2
Σfdx 2 ⎛ Σfdx ⎞
ı  −⎜ ⎟
N ⎝ N ⎠
where N = number of items,
dx = deviations from assumed mean (X – A),
f = frequency of the different groups,
A = assumed mean and
X = values or mid points.
Step to calculate ı
(i) Take the assumed mean from the given values or mid points.
(ii) Take deviations from the assumed mean and represent them by dx.
(iii) Square the deviations to get dx2.
(iv) Multiply f with dx of different groups to obtain fdx and add them
XS WR JHW Ȉfdx.
(v) Multiply f with dx2 of different groups to obtain fdx2 and add them
XS WR JHW Ȉfdx2.
(vi) Apply the formula to get the value of standard deviation.

104 PAGE
© Department of Distance & Continuing Education, Campus of Open Learning,
School of Open Learning, University of Delhi
BUSINESS STATISTICS

Example 15 : We can understand the procedure with the help of an Notes


example.
Class Frequency Mid Deviations
Intervals point from
assumed
Mean = (17)
f x dx dx2 fdx fdx2
10 – 14 5 12 – 5 25 – 25 125
15 – 19 10 17 0 0 0 0
20 – 24 15 22 + 5 25 75 375
25 – 29 10 27 + 10 100 100 1000
30 – 34 5 32 + 15 225 75 1125
N = 45 Ȉ fdx =225 Ȉfdx2= 2625

2
Σfdx 2 ⎛ Σfdx ⎞
ı  −⎜ ⎟ where, N = 45, Σfdx 2 = 2625, Σfdx = 225
N ⎝ N ⎠
2
2625 ⎛ 225 ⎞
?  ı −⎜ ⎟ = 58.33 − 25 = 33.33 = 5.77 approx.
45 ⎝ 45 ⎠
When the step deviations are taken from the assumed mean
2
Σfdx 2 ⎛ Σfdx ⎞
ı  −⎜ ⎟ ×i
N ⎝ N ⎠
ZKHUH 1  1XPEHU RI WKH LWHPV Ȉf),
i = common factor,
f = frequencies corresponding to the different groups,
⎛ X − A⎞
dx = step-deviations ⎜ ⎟
⎝ i ⎠
Steps to calculate ı
(i) Take deviations from the assumed mean of the calculated mid-points
and divide all deviations by a common factor (i) and represent these
values by dx.
(ii) Square these step deviations dx to obtain dx2 for different groups.

PAGE 105
© Department of Distance & Continuing Education, Campus of Open Learning,
School of Open Learning, University of Delhi
B.COM. (PROGRAMME)

Notes (iii) Multiply f with dx of different groups to find fdx and add them to
REWDLQ Ȉfdx.
(iv) Multiply f with dx2 of different groups to find fdx2for different groups
DQG DGG WKHP WR REWDLQ Ȉfdx2.
(v) Apply the formula to get standard deviation.
Example 16 : Suppose we are given the series and we want to calculate
standard deviation with the help of step deviation method. According to
the given formula, we are required to calculate the value of i, N, Ȉfdx
DQG Ȉfdx2.
Class Frequency Mid Deviations i = 5
Intervals point from ⎛ X – A⎞
assumed ⎜⎝ i ⎟⎠
mean (22)
f x X dx dx2 fdx fdx2
10 – 14 5 12 – 10 – 2 4 – 10 20
15 – 19 10 17 – 5 – 1 1 – 10 10
20 – 24 15 22 + 0 0 0 0 0
25 – 29 10 27 + 5 + 1 1 10 10
30 – 34 5 32 + 10 + 2 4 10 20
N =45 Ȉfdx = 0 Ȉfdx2= 60
2
Σfdx 2 ⎛ Σfdx ⎞
ı  −⎜ ⎟ × i, where, N = 45, i Ȉfdx Ȉfdx2= 60
N ⎝ N ⎠
2
60 ⎛ 0 ⎞ 4
? ı  −⎜ ⎟ ×5 = × 5 = 1.33 × 5 = 1.154 × 5 = 5.77 approx.
45 ⎝ 45 ⎠ 3

Advantages of Standard Deviation


(i) Standard deviation is the best measure of dispersion because it
takes into account all the items and is capable of future algebraic
treatment and statistical analysis.
(ii) It is possible to calculate standard deviation for two or more series.
(iii) The measure is most suitable for making comparisons among two
or more series about variability.

106 PAGE
© Department of Distance & Continuing Education, Campus of Open Learning,
School of Open Learning, University of Delhi
BUSINESS STATISTICS

Disadvantages Notes
(i) It is difficult to compute.
(ii) It assign more weights to extreme items and less weights to items
that are nearer to mean. It is because of this fact that the squares
of the deviations which are large in size would be proportionately
greater than the squares of those deviations which are comparatively
small.

3.6.5 Mathematical Properties of Standard Deviation


(i) If sum of deviations of given distribution is calculated from arithmetic
mean and then the root mean square deviation is minimum, i.e.,
¥ = Minimum.

(ii) If different values are increased or decreased by a constant, the


standard deviation will remain the same. Whereas if different
values are multiplied or divided by a constant than the standard
deviation will be changed. It means that standard deviation is free
(not affected) from change in origin but dependent on change in
scale.
(iii) Combined standard deviation can be obtained for two or more series
with formula given below :

N1σ12 + N 2σ22 + N1d12 + N 2 d 22


ı12 =
N1 + N 2
where : N1 represents number of items in first series,
N2 represents number of items in second series,
ı12 represents variance of first series,
ı22 represents variance of second series,
d1 represents the difference between X12 − X1 ,

d2 represents the difference between X12 − X 2 ,


X 1 represents arithmetic mean of first series,
X 2 represents arithmetic mean of second series,
X 12 represents combined arithmetic mean of both the series.

PAGE 107
© Department of Distance & Continuing Education, Campus of Open Learning,
School of Open Learning, University of Delhi
B.COM. (PROGRAMME)

Notes Example 17: Find the combined standard deviation of two series, from
the below given information:
First Series Second Series
No. of items 10 15
Arithmetic means 15 20
Standard deviation 4 5
Solution : Since we are considering two series, therefore combined
standard deviation is computed by the following formula :

N1σ12 + N 2σ22 + N1d12 + N 2 d 22


ı12 =
N1 + N 2

where : N1= 10, N2 = 15, X1 = 15, X 2 = 20, σ1 = 4, σ2 = 5


X1 N1 + X 2 N 2
X12 =
N1 + N 2
(15 ×10) + (20 ×15) 150 + 300 450
or X12 = = = = 18
10 + 15 25 25
d1 = ( X 12 − X 1 ) = 18 − 15 = 3 and d 2 = ( X 12 − X 2 ) = 18 − 20 = −2.

By plying the formula of combined standard deviations, we get:

10(4) 2 + 15(5)2 + 10(18 − 15) 2 + 15(18 − 20) 2


ı12 =
10 + 15

(10 ×16) + (15 × 25) + (10 × 9) + (15 × 4)


=
25

160 + 375 + 90 + 60 685


= = = 27.4 = 5.2 approx.
25 25
(iv) Standard deviation of ‘n’ natural numbers can be computed as :

1
   ı  ( N 2 − 1) where, N represents number of items
12

108 PAGE
© Department of Distance & Continuing Education, Campus of Open Learning,
School of Open Learning, University of Delhi
BUSINESS STATISTICS

(v) Standard deviation for normal distribution is as follows: Notes


X ± σ covers 68.27% of items,
X ± 2σ covers 95.45% of items,
X ± 3σ covers 99.73% of items.

*The diagram is taken from Chegg.com. this is added by reviewer.


Example 18 : You are heading a rationing department in a State affected
by food shortage. Local investigators, submit the following report:
Daily calorie value of food available per adult during current period :
Area Mean Standard Deviation
A 2,500 400
B 2,000 200
The estimated requirement of an adult is taken as 2,800 calories daily
and the absolute minimum is 1,350. Comment on the reported figures,
and determine which area, in your opinion, need more urgent attention.

Solution: We know that X ± σ covers 68.27% of items, X ± 2σ covers


95.45% of items, X ± 3σ covers 99.73% cases. In the given problem if
we take into consideration 99.73%, i.e., almost the whole population, the
limits; would be X ± 3σ.
For Area A these limits are :

PAGE 109
© Department of Distance & Continuing Education, Campus of Open Learning,
School of Open Learning, University of Delhi
B.COM. (PROGRAMME)

Notes X  ı     î    


X ± ı   ±  î    
For Area B these limits are :
X + ı     î    
X – ı   ±  î    
It is clear from above limits that in Area A there are some persons who
are getting 1300 calories, i.e. below the minimum which is 1,350. But
in case of area B there is no one who is getting less than the minimum.
Hence area A needs more urgent attention.
(vi) Relationship between quartile deviation, average deviation and standard
deviation is given as:
Quartile deviation = 2/3 Standard deviation
Average deviation = 4/5 Standard deviation
(vii) We can also compute corrected standard deviation by using the
following formula :

Corrected ΣX 2
&RUUHFWHG ı  − (corrected X ) 2
N
Corrected ΣX
(a) Compute corrected X =
N
ZKHUH FRUUHFWHG Ȉ;  Ȉ;  FRUUHFW LWHPV ± :URQJ LWHPV
ZKHUH Ȉ;  N.X
(b &RPSXWHFRUUHFWHGȈ;2 Ȉ;2 + (Each correct item)2 – (Each
wrong item)2
ZKHUH Ȉ;2 = Nσ2 + NX 2
Example 19 : (a) Find out the coefficient of variation of a series for
which the following results are given:
1   Ȉ;ƍ   Ȉ;ƍ2   ZKHUH;ƍ GHYLDWLRQIURPWKHDVVXPHG
average 5.
(b) For a frequency distribution of marks, in statistics of 100 candidates
(grouped in class intervals of 0–10, 10–20) the mean and, standard

110 PAGE
© Department of Distance & Continuing Education, Campus of Open Learning,
School of Open Learning, University of Delhi
BUSINESS STATISTICS

deviation, were found to be 45 and 20. Later it was discovered that the Notes
score 54 was misread as 64 in obtaining frequency distribution. Find
out the correct mean and correct standard deviation of the frequency
distribution.
(c) Can, coefficient of variation be greater than 100%? If so, when?
Solution : (a) We want to calculate, coefficient of variation, which is =
σ
×100.
X
Therefore, we are required to calculate mean and standard deviation.
ΣX ′
Calculation of simple mean X = A + = where, A = 5, N Ȉ;ƍ 
N
25
X = 5+ = 5.5
50
Calculation of standard deviation
2 2
ΣX ′ 2 ⎛ ΣX ′ ⎞ 500 ⎛ 25 ⎞
ı  − ⎜ ⎟ = − ⎜ ⎟ = 5 − 0.25 = 4.75 = 2.179
N ⎝ N ⎠ 50 ⎝ 50 ⎠
Calculation of Coefficient of variation
σ 2.179 217.9
C.V. = ×100 = ×100 = = 39.6%
X 5.5 5.5

(b) Given X ı N = 100, wrong value = 64, correct value = 54.
Since this is a case of continuous series, therefore, we will apply the
formulae for mean and standard deviation that are applicable in continuous
series.
Calculation of correct Mean
Σfx
X= or NX = ΣfX
N
By substituting the values, we get 100 × 45 = 4500
Correct ΣfX = 4500 – 64 + 54 = 4490

Correct Σfx 4490


? Correct X = = = 44.9
N 100

PAGE 111
© Department of Distance & Continuing Education, Campus of Open Learning,
School of Open Learning, University of Delhi
B.COM. (PROGRAMME)

Notes Calculation of correct ı


ΣfX 2 ΣfX 2
ı  − ( X ) 2 RU  ı2 = − ( X )2
N N

ZKHUH ı   1   X = 45.


ΣfX 2
(20)2 = − (45) 2
100

ΣfX 2
or 400 = − 2025
100

ΣfX 2
or 400 + 2025 =
100
or 2425 × 100 = ΣfX 2 = 242500

? Correct ΣfX 2 = 242500 – (64)2 + (54)2 = 242500 – 4096 + 2916 =


242500 – 1180 = 241320

Correct ΣfX 2
&RUUHFW ı  − (Correct ( X )) 2
N

241320
= − (44.9) 2 = 2413.20 − 2016.01 = 397.19 = 39.9 approx.
100

(c) The formulae for the computation of coefficient of variation is = { σ


X }
× 100 .

Hence, coefficient of variation can be greater than 100% only when the
value of standard deviation is greater than the value of mean.
This will happen when data contains a large number of small items and
few items are quite large. In such a case the value of simple mean will
be pulled down and the value of standard deviation will go up.
Similarly, if there are negative items in a series, the value of mean will
come down and the value of standard deviation shall not be affected
because of squaring the deviations.
Example 20 : In a distribution of 10 observations, the value of mean
and standard deviation are given as 20 and 8. By mistake, two values

112 PAGE
© Department of Distance & Continuing Education, Campus of Open Learning,
School of Open Learning, University of Delhi
BUSINESS STATISTICS

are taken as 2 and 6 instead of 4 and 8. Find out the value of correct Notes
mean and variance.
Solution : We are given; N = 10, X    ı = 3
Wrong values = 2 and 6 and Correct values = 4 and 8
Calculation of correct Mean

ΣX
X = or X = ΣX
N
? ȈX = 10 × 20 = 200
%XW ȈX LV LQFRUUHFW 7KHUHIRUH ZH VKDOO ILQG FRUUHFW Ȉ;
&RUUHFW ȈX = 200 – 2 – 6 + 4 + 8 = 204

Correct ΣX 204
Correct Mean = = = 20.4
N 10
Calculation of correct variance

ΣX 2
ı2 = − ( X )2
N

ΣX 2
RU ı 2
= − ( X )2
N

ΣX 2
or (8)2 = − (20) 2
10

ΣX 2
or 64 = − 400
10

ΣX 2
or 64 + 400 =
10
RU ȈX2 = 4640
%XW WKLV LV ZURQJ DQG KHQFH ZH VKDOO FRPSXWH FRUUHFW ȈX2
&RUUHFW ȈX2 = 4640 –22 – 62 + 42 + 82
= 4640 – 4 – 36 + 16 + 64
= 4680

PAGE 113
© Department of Distance & Continuing Education, Campus of Open Learning,
School of Open Learning, University of Delhi
B.COM. (PROGRAMME)

Notes
Correct ΣX 2
&RUUHFW ı 2
= − Correct ( X ) 2
N
4680
= − (20.4) 2 = 468 − 416.16 = 51.84
10

3.6.6 Graphic Method


The concept of Lorenz-curve was devised by Max-o-Lorenz. It is also
called a cumulative percentage curve, in which the percentage of the items
is combined with the percentage of other items as wealth, profits, etc.
The Lorenz-curve can be drawn in the following manner:
(i) Size of the items and the frequencies are converted into percentages.
(ii) Cumulative percentages are obtained both for the items and frequencies.
(iii) We take cumulative frequency on OX-axis. The values on OX-axis
starts from 100% and decreases to 0%.
(iv) We take into account cumulative item percentages and divide the
axis in equal parts in such a manner that different parts on both
the axis are equal. On OY-axis, we start from 0% and increases to
100%.
(v) Join 0% of OX-axis and 100% of OY-axis by a straight line which
is called line of Equal Distribution.
(vi) We plot points of different series, on the basis of cumulative item
percentages and cumulative frequency percentages and join these
points to draw a ‘Lorenz curve’ for different series.
(vii) The ‘Lorenz-curves’ are compared with the line of Equal Distribution
and the distance between them will determine variability. The
Lorenz-curve located far away from the line of equal distribution
is more variable.
Example 21 : Draw Lorenz curves and interpret the results of the below
given profits of companies located in two different sites.
No. of companies
Profit earned (Rs. lakhs) Area A Area B
10 4 24

114 PAGE
© Department of Distance & Continuing Education, Campus of Open Learning,
School of Open Learning, University of Delhi
BUSINESS STATISTICS

No. of companies Notes


Profit earned (Rs. lakhs) Area A Area B
20 10 18
30 6 5
40 5 3
Solution :
Profits No. of Percentage Cumulative
earned Companies
(Rs. lakhs) Area A Area B Percentage
X fA fB X fA fB CX CfA CfB
10 4 24 10 16 48 10 16 48
20 10 18 20 40 36 30 56 84
30 6 5 30 24 10 60 80 94
40 5 3 40 20 06 100 100 100
Total: 100 25 50 100 100 100
Solution :
Mode Median Mode
Median Mean
Mean

POSITIVE SKEWNESS NO SKEWNESS NEGATIVE SKEWNESS


X > MEDIAN > MODE X = MEDIAN = MODE X < MEDIAN < MODE

Figure
We notice in the above diagram that the Lorenz curve for Area B
companies is away from the line of equal distribution in comparison
with Lorenz curve for Area A. Therefore, we can conclude that there is
more variability in Area B companies as compared to Area A companies.

3.7 Revisionary Problems


Example 22 : Compute (a) Inter-quartile range, (b) Semi-quartile range,
and (c) Coefficient of quartile deviation from the following data:

PAGE 115
© Department of Distance & Continuing Education, Campus of Open Learning,
School of Open Learning, University of Delhi
B.COM. (PROGRAMME)

Notes Farm Size (acres) No. of firms Farm Size (acres) No. of firms
0–40 394 161–200 169
41–80 461 201–240 113
81–120 391 241 and over 148
121–160 334
Solution : In this case, the real limits of the class intervals can be
obtained by subtracting 0.5 from the lower limits of the class intervals
and adding 0.5 to the upper limits of the different class intervals. This
adjustment is necessary to calculate median and quartiles of the series.
Farm Size (acres) No. of firms Cumulative frequency
(c.f.)
0–40 394 394
41–80 461 855
81–120 391 1246
121–160 334 1580
161–200 169 1749
201–240 113 1862
241 and over 148 2010
N = 2010
N 4 − c. f 0
Q1 = l1 + ×i
f

n 2010
' = = 502 th item
4 4
Q1 lies in the cumulative frequency of the group 41–80, where the real
limits of class intervals are 40.5–80.5 and l1 = 40.5, f = 461, i = 40, c.f0
n
= 394, = 502.5
4
502.5 − 394
? Q1 = 40.5 + × 40 = 40.5 + 9.4 = 49.9 acres
461
3n
− c. f 0
Similarly, Q3 = l1 + 4 ×i
f

116 PAGE
© Department of Distance & Continuing Education, Campus of Open Learning,
School of Open Learning, University of Delhi
BUSINESS STATISTICS

3n 3 × 2010 Notes
= ×1507.5 th item
4 4
Q3 lies in the cumulative frequency of the group 121–160, where the real
limits of the class interval are 120.5–160.5 and l1 = 120.5, i = 40, f =
3n
334, = 1507.5, c.f. = 1246
4
1507.5 − 1246
? Q3 = 120.5 + × 40 = 120.5 + 31.3 = 151.8 acres
334
Inter-quartile range = Q3 – Q1 = 151.8 – 49.9 = 101.9 acres
Q3 − Q1 151.8 − 49.9
Semi-quartile range = = = 50.95 approx.
2 2
Q3 − Q1 151.8 − 49.9 101.9
Coefficient of quartile deviation = = = = 0.5
Q3 + Q1 151.8 + 49.9 201.7
approx.
Example 23 : Calculate mean and coefficient of mean deviation about
mean from the following data:
Marks less than No. of students
10 4
20 10
30 20
40 40
50 50
60 56
70 60
Solution : In this question, we are given less than type series alongwith
the cumulative frequencies. Therefore, we are required first of all to find
out class intervals and frequencies for calculating mean and coefficient
of mean deviation about mean.

PAGE 117
© Department of Distance & Continuing Education, Campus of Open Learning,
School of Open Learning, University of Delhi
B.COM. (PROGRAMME)

Notes Marks No. of Mid Deviations from Step Deviation


students points assumed Mean from mean (35)
Deviations (ignoring signs)
(A = 35) i = 10
⎛X–A⎞
f X Xƍ dx = ⎜ ⎟ |dx| fdx f|dx|
⎝ i ⎠
0–10 4 5 – 30 – 3 3 – 12 12
10–20 6 15 – 20 – 2 2 – 12 12
20–30 10 25 – 10 – 1 1 – 10 10
30–40 20 35 0 0 0 0 0
40–50 10 45 + 10 + 1 1 + 10 10
50–60 6 55 + 20 + 2 2 + 12 12
60–70 4 65 + 30 + 3 3 + 12 12
N = 60 Ȉfdx = 0 Ȉf |dx| = 68
Σfdx
X = A+ ×i
N
where, N = 60, A = 35, i   Ȉfdx = 0
⎛ 0 ⎞
? X = 35 + ⎜ ×10 ⎟ = 35
⎝ 60 ⎠
Σf | dx | 68
M.D. about mean = × i = ×10 = 11.33
N 60
M.D. about mean 11.33
Coefficient of M.D. about mean = = = 0.324 approx.
mean 35
Example 24 : Calculate standard deviation from the following data:
Class interval Frequency
–30 to –20 5
–20 to –10 10
–10 to 0 15
0 to 10 10
10 to 20 5
N = 45

118 PAGE
© Department of Distance & Continuing Education, Campus of Open Learning,
School of Open Learning, University of Delhi
BUSINESS STATISTICS

Solution : Notes
Calculation of Standard Deviation
Class Frequency Mid Deviations Step
points from Deviations
Intervals assumed when i = 10
Mean
(A = –5)
( X – A)
f X Xƍ dx = dx2 fdx fdx2
i
–30 to –20 5 – 25 – 20 – 2 4 – 10 20
–20 to –10 10 – 15 – 10 – 1 1 – 10 10
10 to 0 15 – 5 + 0 0 0 0 0
0 to 10 10 5 + 10 1 1 10 10
10 to 20 5 15 + 20 2 4 10 20
N = 45 Ȉfdx = 0 Ȉfdx = 60
2

2
Σfdx 2 ⎛ Σfdx ⎞
ı  −⎜ ⎟ ×i
N ⎝ N ⎠
where N = 45, i   Ȉfdx   Ȉfdx2 = 60
2
60 ⎛ 0 ⎞ 60
? ı  − ⎜ ⎟ ×10 = ×10 = 1.33 ×10 = 1.153
45 ⎝ 45 ⎠ 45
Example 25 : For two firms A and B belonging to same industry, the
following details are available:
Firm A Firm B
Number of Employees: 100 200
Average wage per month: Rs. 240 Rs. 170
Standard deviation of the wage per month: Rs. 6 Rs. 8
Find (i) Which firm pays out larger amount as monthly wages?
(ii) Which firm shows greater variability in the distribution of
wages?
(iii) Find average monthly wage and the standard deviation of the
wages of all employees firms.
Solution : (i) For finding out which firm pays larger amount, we have
WR ILQG RXW Ȉ;

PAGE 119
© Department of Distance & Continuing Education, Campus of Open Learning,
School of Open Learning, University of Delhi
B.COM. (PROGRAMME)

Notes
ΣX
X = RU ȈX = NX
N
Firm A : N = 100, X = 240 ? ȈX = 100 × 240 = 24000
Firm B : N = 200, X = 170 ? ȈX = 200 × 170 = 34000
Hence firm B pays larger amount as monthly wages.
(ii) For finding out which firm shows greater variability in the distribution
of wages, we have to calculate coefficient of variation
σ 6
Firm A : C.V. = ×100 = ×100 = 2.50
X 240
σ 8
Firm B : C.V. = ×100 = ×100 = 4.71
X 170
Since coefficient of variation is greater for firm B, hence it shows greater
variability in the distribution of wages.

N1 X1 + N 2 X 2
(iii) Combined wage : X12 =
N1 + N 2

where, N1 = 100, X1 = 240, N2 = 200, X 2 = 170

(100 × 240) + (200 ×170) 24000 + 34000


Hence X12 = = = 193.33
100 + 200 300
Combined Standard Deviation

N1σ12 + N 2σ22 + N1d12 + N 2 d 22


ı12 =
N1 + N 2

where N1   ı1   ı2 = 8, d1 = ( X1 − X12 ) = 240 – 193.3 = 46.7


and d 2 = ( X 2 − X12 ) = 170 – 193.3 = – 23.3

(100)(36) + (200)(64) + (100)(46.7) 2 + (200)(−23.3) 2


ı12 =
100 + 200
3600 + 12800 + 218089 + 108578 343, 067
= = = 33.81
300 300

120 PAGE
© Department of Distance & Continuing Education, Campus of Open Learning,
School of Open Learning, University of Delhi
BUSINESS STATISTICS

Example 26 : From the following frequency distribution of heights of Notes


360 boys in the age-group 10–20 years, calculate the:
(i) arithmetic mean;
(ii) coefficient of variation; and
(iii) quartile deviation.
Height (cms) No. of boys Height (cms) No. of boys
126–130 31 146–150 60
131–135 44 151–155 55
136–140 48 156–160 43
141–145 51 161–165 28
Solution :

Calculation of X , Q.D., and C.V.


Heights m.p. (X – 143)/5
X f dx fdx fdx2 c.f.
126–130 128 31 – 3 –93 279 31
131–135 133 44 – 2 –88 176 75
136–140 138 48 – 1 –48 48 123
141–145 143 51 0 0 0 174
146–150 148 60 +1 +60 60 234
151–155 153 55 +2 +110 220 289
156–160 158 43 +3 +129 387 332
161–165 163 28 +4 +112 448 360
N = 45 Ȉfdx = 182 Ȉfdx = 1618
2

Σfdx
(i) X = A + ×i where, N = 360, A = 143, i   Ȉfdx = 182
N
182
? X = 143 + × 5 = 143 + 2.53 = 145.53.
360
σ
(ii) C.V. = ×100
X
2 2
Σfdx 2 ⎛ Σfdx ⎞ 1618 ⎛ 182 ⎞
ı  −⎜ ⎟ ×i = −⎜ ⎟ ×5
N ⎝ N ⎠ 360 ⎝ 360 ⎠

PAGE 121
© Department of Distance & Continuing Education, Campus of Open Learning,
School of Open Learning, University of Delhi
B.COM. (PROGRAMME)

Notes
= 4.494 − 0.506 × 5 = 2.00 × 5 = 10
10
C.V. = ×100 = 6.87 per cent
145.53
Q3 − Q1
(iii) Q.D. =
2
N 360
Q1 = Size of th observation = = 90th observation
4 4
Q1 lies in the class 136–140. But the real limit of this class is
135.5–140.5.
N 4 − cf 0 90 − 75
Q1 = l1 + × i = 135.5 + × 5 = 135.5 + 1.56 = 137.06
f 48
3N 360
Q3 = Size of th observation = 3 × = 270th observation
4 4
Q3 lies in the class 151–155. But the real limit of this class is
150.5–155.5.
3 N 4 − cf 0 270 − 234
Q3 = l1 + × i = 150.5 + × 5 = 150.5 + 3.27 = 153.77.
f 55

Q3 − Q 1 153.77 − 137.06
Q.D. = = = 8.355.
2 2

3.8 Summary
While averages summarize and present data in a single number, variation is
studied to get a better idea of the nature of data. Variation can be absolute
or relative. Absolute variation refers to the amount of variation in a set of
data while relative variation serves to compare variability across different
sets of data. The ‘distance’ measures of variation include range and partial
ranges including inter-quartile range and inter-percentile range which are
used in addition to or as surrogates for range. Range is commonly used
in reporting price movements, quality control, etc. Coefficient of range
is a relative measure. The measures involving deviations include quartile

122 PAGE
© Department of Distance & Continuing Education, Campus of Open Learning,
School of Open Learning, University of Delhi
BUSINESS STATISTICS

deviation and its co-efficient: Mean deviation and its coefficient; and Notes
standard deviation, variance and coefficient of variation.
Quartile deviation is a quick, inspectional measure of variability and used
when there are scattered or extreme values included in the data. A measure
based on each observation in the data is the mean deviation which is equal
to the sum of absolute deviations of the various observations from their
mean or median. The relative measure related to this is the coefficient of
mean deviation. Standard deviation is also based on all observations. It
is the best measure of variation as it possesses mathematical properties.
Coefficient of standard deviation is sometimes used instead of coefficient
of variation. All coefficients are pure numbers and there are no units
associated with them. Hence they are used for making comparisons of
variability.
Graphically, Lorenz curve is used to describe inequalities of income.
The extent of departure of the curve of actual distribution of income
from the line of equal distribution indicates the degree of inequalities
of income. Well-defined relationship exists between values of quartile
deviation, mean deviation and standard deviation in the case of normal
distributions. The relationship works well even for distributions which
deviate moderately from normality.

3.9 Self-Assessment Questions

Exercise 1 : True and False Statements


(i) Measures of variation attempt to present in a single number the
amount of variation in a set of data.
(ii) All measures of relative variation are pure numbers with no units
attached to them.
(iii) Range cannot be negative.
(iv) Range cannot be determined in open-ended frequency distributions.
(v) Coefficient of range cannot be greater than 1.
(vi) Quartile deviation is same as semi inter-quartile range.
(vii) Only absolute values of deviations are considered in the calculation
of mean deviation.

PAGE 123
© Department of Distance & Continuing Education, Campus of Open Learning,
School of Open Learning, University of Delhi
B.COM. (PROGRAMME)

Notes (viii) Since the sum of absolute deviations measured from median is
the minimum, this serves as the most appropriate average for
calculating mean deviation.
(ix) Since the sum of deviations of a set of values from their mean is
equal to zero, it follows that mean deviation from mean would
always be equal to zero.
(x) Mean deviation can never be negative.
(xi) Mean deviation cannot be calculated for distributions with open-
ended classes.
(xii) The arithmetic mean is used for measuring deviations in calculating
standard deviation due to its least squares property.
(xiii) Standard deviation cannot be equal to zero.
(xiv) Standard deviation can never exceed the arithmetic mean.
(xv) Standard deviation is positive or negative depending upon the sign
of deviations of various values from their mean.
(xvi) Variance is the square root of standard deviation.
(xvii) Coefficient of variation is always expressed as a percentage.
(xviii) Coefficient of standard deviation is equal to the ratio of standard
deviation to arithmetic mean of the data.
(xix) Coefficient of variation expresses arithmetic mean as a percentage
of standard deviation.
(xx) If each of the values of a set of data is increased by 5, the mean
and standard deviation would both increase by 5.
(xxi) If each of the values of a set of data is multiplied by –5, the
standard deviation would also be multiplied by the same number
and hence become negative.
(xxii) If each of the values of a set of data is increased by K, the
coefficient of variation would also increase by K.
(xxiii) When each value of a given set of data is multiplied by K, the
revised coefficient of variation would be K times the original
coefficient value.
(xxiv) The combined standard deviation of two sets of data will always
lie between the standard deviation values of the two sets.

124 PAGE
© Department of Distance & Continuing Education, Campus of Open Learning,
School of Open Learning, University of Delhi
BUSINESS STATISTICS

(xxv) The standard deviation of a distribution is approximately equal to Notes


1.25 times the mean deviation and 1.5 times the quartile deviation.
(xxvi) Standard deviation is exactly equal to one-sixth of the range.
(xxvii) In a normal distribution, the percentage of values included between
and is 99.73.
(xxviii) Calculating the Z-scores is standardizing the data values.
(xxix) The sum total of all Z-scores for a given set of values ranges
between ±3.
Ans.
(i) T (ii) T (iii) T (iv) T (v) T (vi) T (vii) T
(viii) T (ix) F (x) T (xi) T (xii) T (xiii) F (xiv) F
(xv) F (xvi) F (xvii) T (xviii) T (xix) F (xx) F (xxi) F
(xxii) F (xxiii) F (xxiv) F (xxv) T (xxvi) F (xxvii) F (xxviii) T
(xxix) F

Exercise 2 : Questions and Answers


(i) What is range? What are its limitations as a measure of variation?
Give examples where range can be used satisfactorily for measuring
variation.
(ii) What are quartiles? How are they used for measuring variation?
(iii) Define mean deviation. How does it differ from standard deviation?
(iv) Define mean deviation, standard deviation and inter-quartile range
of a frequency distribution. Why is standard deviation considered
as the most appropriate measure of variation? Give an example in
which you would prefer an alternative measure of variation.
(v) State and explain the properties of standard deviation and variance.
Do you agree that standard deviation is always positive and never
negative or zero?
(vi) What is coefficient of variation? How is it different from coefficient
of standard deviation and variance?
(vii) Explain the relationship between quartile deviation. mean deviation and
standard deviation in the case of normal distribution. Also, discuss
the empirical relationship between mean and standard deviation.

PAGE 125
© Department of Distance & Continuing Education, Campus of Open Learning,
School of Open Learning, University of Delhi
B.COM. (PROGRAMME)

Notes (viii) What is Lorenz curve? How is it obtained? Discuss its significance
as a tool of studying variation.
(ix) Determine (i) weekly and (ii) monthly range of gold prices (per 10
gm) from the following data for a month:
Week High Low
1 28,122 27,880
2 29,208 28,890
3 28,890 28,706
4 29,225 28,930
(x) The heights of 11 men are measured as 65, 68, 70, 69, 58, 66, 71,
65, 67, 69 and 73 inches. Calculate the range. If the shortest and
the tallest of them are omitted, what is the percentage change in
range?
(xi) Draw a “less than ogive” from the following data and obtain the
lower and upper quartiles there from. Also, calculate the values of
quartile deviation and its coefficient.
Wages (in Rs.) No. of workers
5,000 or more Nil
4,500 or more 4
4,000 or more 18
3,500 or more 38
3,000 or more 60
2,500 or more 75
2,000 or more 85
1,500 or more 93
1,000 or more 100
(xii) The following table shows the percentage of different age groups
to the total population of a certain country:
Age group Percentage of the total
(years) population
0–14 42.0
15–19 8.7
20–24 7.9
25–29 7.4

126 PAGE
© Department of Distance & Continuing Education, Campus of Open Learning,
School of Open Learning, University of Delhi
BUSINESS STATISTICS

Age group Percentage of the total Notes


(years) population
30–39 12.6
40–49 9.3
50–59 6.1
60 and above 6.0
You are required to find the age limits within which the middle
50 percent of the population lies. Also, calculate (i) inter-quartile
range, (ii) quartile deviation, and (iii) co-efficient of quartile
deviation. (Note that in census, the age is recorded as on last
birthday).
(xiii) The distribution of marks of 1200 students appeared in an entrance
examination is given below:
Marks: 20–30 20–40 20–50 20–60 20–70 20–80 20–90 20–100
No. of 30 90 210 420 720 945 1080 1200
students:
(xiv) In the following data, two class frequencies are missing:
Class interval : 100–110 110–120 120–130 130–140 140–150
Frequency: 4 7 15 ? 40
Class interval: 150–60 160–170 170–180 180–190 190–200
Frequency: ? 16 10 6 3
However, it is possible to ascertain that the total frequency is 150
and that the median is equal to 146.25. You are required to find
the missing frequencies. Having obtained these, calculate mean,
standard deviation and the coefficient of variation.
(xv) Find two numbers whose mean is 12 and standard deviation is 4.
(xvi) Find the mean and standard deviation of the first 13 natural
numbers.
(xvii) The mean and variance of the following continuous distribution
are 61 and 15.9, respectively. The distribution, after taking step-
deviations, is as follows:
dƍ –3 –2 –1 0 1 2 3
f 10 15 25 25 10 10 5
You are required to determine actual class intervals.

PAGE 127
© Department of Distance & Continuing Education, Campus of Open Learning,
School of Open Learning, University of Delhi
B.COM. (PROGRAMME)

Notes (xviii) The standard deviation of a set of 10 numbers was calculated


as equal to 5 while their arithmetic mean was found to be 12. It
was discovered later on that an item was recorded as 5 instead of
15. Rectify the error and determine the correct value of standard
deviation.
(xix) The mean of 5 observations is 4.4 and the variance is 8.24. If
three of the observations are 1, 2 and 6, find the other two.
(xx) Mean, median and variance of a set of 5 numbers are known to
be 12,11 and 9.2, respectively. If two of the numbers are 8 and
16, determine the remaining numbers.
(xxi) The following is the record of the number of bricks laid each day
for 10 days by two brick layers A and B. Calculate the co-efficient
of variation in each case and discuss the relative consistency of
the two brick layers.
A: 700 675 725 625 650 700 650 700 600 650
B: 550 600 575 550 650 600 550 525 625 600
If each of the values in respect of worker A is decreased by 10
and each of the values in respect of worker B is increased by
50, how will it affect the results obtained earlier?
(xxii) A purchasing agent obtained samples of lamps from two suppliers.
He had the samples tested in his own laboratory for the length
of life, with the following results:
Length of life Samples from
(in hours)
Company A Company B
700 – 900 10 3
900 – 1,100 16 42
1,100 – 1,300 26 12
1,300 – 1,500 8 3
(a) Which company’s lamps have greater average life?
(b) Which company’s lamps have more uniform life?
(xxiii) A sample of 35 values has mean 80 and standard deviation equal
to 4. A second sample of 65 values has mean 70 and standard
deviation equal to 3. Find the mean and standard deviation of
the combined set of 100 values.

128 PAGE
© Department of Distance & Continuing Education, Campus of Open Learning,
School of Open Learning, University of Delhi
BUSINESS STATISTICS

(xxiv) Particulars regarding the incomes of employees of two factories Notes


are given below:
Factory No. of Average Variance
Employees Income (Rs.)
A 600 475 180
B 500 586 140
(a) In which factory is the variation in income greater?
(b) What is the wage bill of each of the-factories?
(c) What is the average income of all employees put together?
(d) What is the combined standard deviation of incomes of
the two sets of employees?
(xxv) The number of employees, wages per employee and the variance
of wage for two factories are given here:
Factory A Factory B
No. of employees 50 100
Average wages per day (Rs.) 120 85
Variance of wages 9 16
(a) In which factory is there greater variation in the distribution
of wages?
(b) Suppose in factory B, the wages of an employee were
recorded as Rs. 120 instead of Rs.100. What would be
the corrected variance for factory B? Will it change the
conclusion drawn in (a)?
(xxvi) Fill in the blanks:
Group 1 Group 2 Group 3 Combined
Number 70 30 ? 150
Mean 140 ? 146 145
Variance ? 48 56 78
(xxvii) For a set of 100 values, the standard deviation is known to be 14.4
and the co-efficient of variation is 40%. Calculate the arithmetic
mean.
(xxviii) If a 20 is subtracted from every observation in a data set. Then
the co-efficient of variation of the resulting data set is 20%. If a

PAGE 129
© Department of Distance & Continuing Education, Campus of Open Learning,
School of Open Learning, University of Delhi
B.COM. (PROGRAMME)

Notes 40 is added to every observation of the same data set, then the
co-efficient of variation of the resulting set of data is 10%. Find
the mean and standard deviation of the original set of data.
(xxix) A set of 40 numbers has mean and standard deviation equal to
DQG ı UHVSHFWLYHO\ ,I HDFK RI WKH YDOXHV RI WKH VHW LV PXOWLSOLHG
by 16, the co-efficient of variation works out to be 25% while
if each value of the set is increased by 16, the co-efficient of
variation becomes 20%.Find the mean and standard deviation of
the set of numbers.
(xxx) Two groups of workers, consisting of 30 and 50 persons, have
the same mean wages but different standard deviations. The
respective standard deviations are Rs. 16 and Rs. 12. Obtain the
combined standard deviation of their wages.
Ans.
(x) 15.6 (xi) Q1 = 2500, Q3 = 3825,
QD = 662.5 CQD =
0.209
(xii) 28.214, 14.107, 0.612 (xiii) 13.575, 0.206
(xiv) 147.33, 19.198, 13.03% (xv) 8,16
(xvi) 7, 3.742 (xvii) 20-40, 40-60 etc.
(xviii) 4.47 (xix) 4, 9
(xx) 15, 10 (xxi) 5.568%, 6.38%,
5.562%, 5.876%
(xxii) 1106.67, 1050, 16.65%, (xxiii) 73.5, 5.85
11.86%
(xxiv) A, 285000, 293000, (xxv) 2.5%, 4.71%, 5.96,
525.45, 56.72 2.88%
(xxvi) 50,155, 38 (xxvii) 36
(xxviii) 80, 12 (xxix) 64, 16
(xxx) 13.64

130 PAGE
© Department of Distance & Continuing Education, Campus of Open Learning,
School of Open Learning, University of Delhi
L E S S O N

4
Skewness and Kurtosis
STRUCTURE
4.1 Learning Objectives
4.2 Tests of Skewness
4.3 Nature of Skewness
4.4 Characteristics of Skewness
4.5 Methods of Skewness
4.6 Measures of Kurtosis
4.7 Comparison among Variation, Skewness and Kurtosis
4.8 Summary
4.9 Self-Assessment Questions

4.1 Learning Objectives


After reading this lesson, you should be able to :
‹ Understand the meaning of skewness and kurtosis.
‹ Distinguish between skewness, variation and kurtosis.
‹ Compute the skewness and kurtosis.
‹ Comment upon the nature of distribution with the help of measures of skewness and
kurtosis.

4.2 Tests of Skewness


Measures of Skewness and Kurtosis, like measures of central tendency and dispersion,
study the characteristics of a frequency distribution. Averages tell us about the central
value of the distribution and measures of dispersion tell us about the concentration of
the items around a central value. These measures do not reveal whether the dispersal of
value on either side of an average is symmetrical or not. If observations are arranged
in a symmetrical manner around a measure of central tendency, we get a symmetrical

PAGE 131
© Department of Distance & Continuing Education, Campus of Open Learning,
School of Open Learning, University of Delhi
B.COM. (PROGRAMME)

Notes distribution, otherwise, it may be arranged in an asymmetrical order which


gives asymmetrical distribution. Thus, skewness is a measure that studies
the degree and direction of departure from symmetry.
A symmetrical distribution, when presented on the graph paper, it gives
a ‘symmetrical curve’, where the value of mean, median and mode are
exactly equal. On the other hand, in an asymmetrical distribution, the
values of mean, median and mode are not equal.
When two or more symmetrical distributions are compared, the difference
in them are studied with ‘Kurtosis’. On the other hand, when two or more
symmetrical distributions are compared, it will give different degrees of
Skewness. These measures are mutually exclusive i.e. the presence of
skewness implies absence of kurtosis and vice versa.
It means that if a distribution looks equal or same from both sides i.e
left and right of the center value/point then the distribution is called
symmetrical and skewness is the measure of this symmetry. If the
distribution is skewed on either side that means right-tailed or left tailed
in a normal distribution then it can be measured by Kurtosis.
There are certain tests to know whether skewness does or does not exist
in a frequency distribution. They are :
1. In a skewed distribution, values of mean, median and mode would
not coincide. The values of mean and mode are pulled away and
the value of median will be at the center. In this distribution, Mean-
Mode = 2/3 (Median – Mode).
2. Quartiles will not be equidistant from median.
3. When the asymmetrical distribution is drawn on the graph paper,
it will not give a bell shaped-curve.
4. Sum of the positive deviations from the median is not equal to sum
of negative deviations.
5. Frequencies are not equal at points of equal deviations from the
mode.

132 PAGE
© Department of Distance & Continuing Education, Campus of Open Learning,
School of Open Learning, University of Delhi
BUSINESS STATISTICS

4.3 Nature of Skewness Notes

Skewness can be positive or negative or zero.


1. When the values of mean, median and mode are equal, there is no
skewness.
2. When mean > median > mode, skewness will be positive.
3. When mean < median < mode, skewness will be negative.

4.4 Characteristics of Skewness


1. It should be a pure number in the sense that its value should be
independent of the unit of the series and also degree of variation
in the series.
2. It should have zero-value, when the distribution is symmetrical.
3. It should have a meaningful scale of measurement so that we could
easily interpret the measured value.

4.5 Methods of Skewness


Skewness can be studied graphically and mathematically. When we study
skewness graphically, we can find out whether skewness is positive or
negative or zero. We cannot find out value of coefficient of skewness.
This can be shown with the help of a diagram :

PAGE 133
© Department of Distance & Continuing Education, Campus of Open Learning,
School of Open Learning, University of Delhi
B.COM. (PROGRAMME)

Notes Mathematically skewness can be studied as :


(a) Absolute Skewness.
(b) Relative or coefficient of skewness.
When the skewness is presented in absolute item i.e., in units, it is absolute
skewness. If the value of skewness is obtained in ratios or percentages,
it is called relative or coefficient of skewness.
When skewness is measured in absolute terms, we can compare one
distribution with the other, if the units of measurement are not different.
When it is presented in ratios or percentages, comparison become easy.
Relative measures of skewness is also called coefficient of skewness.
Mathematical measures of skewness can be calculated on the basis of:
(a) Bowley’s Method
(b) Karl-Pearson’s Method
(c) Kelly’s method

4.5.1 Bowley’s Method


Bowley’s method of skewness is based on the values of median, lower
and upper quartiles. This method suffer from the same limitations which
are in the case of median and quartiles.
Wherever positional measures are called for, skewness should be measured
by Bowley’s method. This method is, also used in case of ‘open-end
series’, where the importance of extreme values is ignored.
Absolute skewness = Q3 + Q1– 2 Median
Q2 + Q1 − 2 Median
Coefficient of Skewness =
Q3 − Q1
Coefficient of skewness lies within the limit ±1. This method is quite
convenient for determining skewness where one has already calculated
quartiles.
For example, if the class intervals and frequencies are given as follows :
Class Intervals Frequencies
Below 10 5
10 – 20 10

134 PAGE
© Department of Distance & Continuing Education, Campus of Open Learning,
School of Open Learning, University of Delhi
BUSINESS STATISTICS

Class Intervals Frequencies Notes


20 – 30 15
30 – 40 10
above 40 5
In this case, if we want to calculate, coefficient of skewness on the basis
of this method, then we are required to calculate the values of Median,
Q3 and Q1:
Calculation of Coefficient of Skewness on the basis of Median and
Quartiles
Class Frequency Cumulative Calculations
Intervals ƒ frequency (c.ƒ.)
Below 10 5 5 n 2 − c. f .
Median = l1 + ×i
f
10–20 10 15 45
n 2= = 22.5 , It lies in the
2
cumulative
20–30 15 30 frequency 30, corresponding to
class (20–30)
30–40 10 40
22.5 − 15
? Median = 20 + ×10
15
40 and 5 45 7.5
above = 20 + ×10 = 25
15
N = 45

3n 4 − c. f . 45
Q1 = l + ×i n 4= = 11.25 , lies in the cumulative
f 4
Absolute Skewness = Q3 + Q1 frequency, corresponding to class interval (10
– 2 median – 20)
where, Q3 = 33.75, Q1= 16.75,
11.25 − 5
Median = 25 Q1 = 10 + ×10 = 16.25
10
? Ab. Skewness = 33.75 +
3n 4 − c. f .
16.25 – 2(25) Q3 = l + ×i
f
= 50 – 50 = 0 3n/4 = 33.75, that lies in the cumulative

PAGE 135
© Department of Distance & Continuing Education, Campus of Open Learning,
School of Open Learning, University of Delhi
B.COM. (PROGRAMME)

Notes
Q3 + Q1 − 2(Median)
Coefficient of Skewness =
Q3 − Q1
frequency 40, corresponding to group \ (30–40)
Now we have, Q3 = 33.75, Q1 = 16.25,
33.75 − 30
Q3 = 30 + ×10 = 33.75
10
Median 25
33.75 + 16.25 − 2(25) 0
? Coefficient of Skewness = = =0
33.75 − 16.25 17.5

4.5.2 Karl-Pearson’s (Pearsonian Coefficient of Skewness)


Karl Pearson has suggested two formulae:
(i) where the relationship of mean and mode is established;
(ii) where the relationship between mean and median is established.
When the values of When the values of
Mean and Mode are related Mean and Median are related
Absolute skewness = Mean – Mode Absolute skewness = 3(Mean –
Median)
Coefficient of skewness = Coefficient of skewness =
Mean – Mode 3(Mean – Median)
σ σ
Coefficient of skewness generally Coefficient of skewness generally
lies within ±1 lies within ±3
Calculation of Coefficient of skewness by using the following formula:
Mean – Mode
Coefficient of skewness =
σ
ΣX 105
Given X values are = 12, 18, 18, 22, 35, and N = 5 ? X = = = 21
N 5
and Mode = 18

136 PAGE
© Department of Distance & Continuing Education, Campus of Open Learning,
School of Open Learning, University of Delhi
BUSINESS STATISTICS

X (X – 21) Notes
X x 2

12 –9 81 Σx 2
σ= where x = ( X − X )
N

296
18 –3 9 ∴ σ= = 59.2 = 7.7
5
18 –3 9
22 + 1 1
35 + 14 196
N = 5 Ȉx = 296
2

Mean – Mode
? Coefficient of skewness =
σ
Substitute Mean = 21, Mode =18, Standard deviation = 7.7.
21 − 18 3
? SK = = = + 0.4
7.7 7.7
Calculation of Karl-Pearson’s coefficient of skewness by using the
following formula:
3(Mean – Median)
Coefficient of skewness =
σ
For the given data X = 12, 18, 18, 22, 35
 0HDQ  0HGLDQ  ı 
3(21 − 18) 3 × 3 9
? Coefficient of skewness = = = = 1.12
7.7 7.7 7.7

4.5.3 In Text Questions


Example 1 : Calculate appropriate measure of skewness from the following
income distribution:
Monthly income (Rs.) Frequency
Upto–100 9
101–150 51

PAGE 137
© Department of Distance & Continuing Education, Campus of Open Learning,
School of Open Learning, University of Delhi
B.COM. (PROGRAMME)

Notes Monthly income (Rs.) Frequency


151–200 120
201–300 240
301–500 136
501–750 33
751–1000 9
above 1000 2
N = 600
Solution : In this problem, the open-ends series is given with inclusive
class-intervals. Hence, Bowley’s measure of skewness is better, because
it is based on Quartiles and not affected by extreme class intervals.
Calculation of coefficient of skewness based on quartiles and
median
Monthly income (Rs.) Frequency Cumulative Frequency
f c.f.
Upto 100 09 9
101–150 51 60
151–200 120 180
201–300 240 420
301–500 136 556
501–750 33 589
751–1000 09 598
Above 1000 02 600
N = 600
Q3 + Q1 − 2 Median
Coefficient of skewness =
Q3 − Q1

n 2 − cf
Median = L1 + ×i
f

600
?N/2 = = 300; It lies in the cumulative frequency 420,
2
which is corresponding to group 201 – 300.

138 PAGE
© Department of Distance & Continuing Education, Campus of Open Learning,
School of Open Learning, University of Delhi
BUSINESS STATISTICS

But the real limits of the class interval are 200.5 – 300.5 Notes
300 − 180 120
? Median = 200.5 + ×100 = 200.5 + ×100 = 250.5
240 240
n 4 − cf
Q1 = L1 + ×i
f

600
N/4 = = 150. It lies in the cumulative frequency 180, which is
4
corresponding to class interval 151–200.
But the real limits of this class-interval are 150.5–200.5.
150 − 60 90
? Q1 = 150.5 + × 50 = 150.5 + × 50 = 150.5 + 37.5 = Rs. 188
120 120
3n 4 − cf
Q3 = L1 + ×i
f
where 3n/4 is used to find out upper quartile group.
3 × 600
? 3n 4 = = 450 . It lies in the cumulative frequency 556, which is
4
corresponding to group 301 – 500.
The real limits of this class interval are 300.5–500.5
450 − 420 30
? Q 3 = 300.5 + × 200 = 300.5 + × 200 = 300.5 + 44.12 = Rs. 344.62
136 136
Hence, Coefficient of skewness =
344.62 + 188 − 2(250.5) 532.62 − 501 31.62
= = = + 0.2 approx.
344.62 − 188 156.62 156.62
Example 2 : Calculate the appropriate measure of skewness from the
following cumulative frequency distribution :
Age (under years) : 20 30 40 50 60 70
No. of persons : 12 29 48 75 94 106
Solution: In this problem, we are given the upper limits of classes along
with the cumulative frequency. Therefore, we have to find out the lower
limits and frequencies for the given data.

PAGE 139
© Department of Distance & Continuing Education, Campus of Open Learning,
School of Open Learning, University of Delhi
B.COM. (PROGRAMME)

Notes Age (years) Number of Persons Cumulative


Frequency (f) Frequency (c.f.)
Below 20 12 12
20–30 17 29
30–40 19 48
40–50 27 75
50–60 19 94
60–70 12 106
N = 106
Because the lower limit of the first group is not given, the appropriate
measure of skewness is Bowley’s method. It is based on quartiles and
median and is not influenced by extreme class-intervals.

⎛ Q + Q1 − 2 Median ⎞
Bowley’s coefficient of skewness = ⎜ 3 ⎟
⎝ Q3 − Q1 ⎠
Thus, we have to calculate the values of Q3, Q1 and median.
n 2 − c. f .
Median = L1 + ×i
f
N 106
Median has items or or 53 items below it.
2 2
Therefore, it lies in the cumulative frequency 75, which is corresponding
to the class-interval (40–50). Hence, median group is (40–50).
N
where L1 = 40, i = 10, f = 27, = 53, c.f. = 48
2
53 − 48 5
? Median = 40 + ×10 = 40 + ×10 = 40 + 1.9 = 41.9
27 27
x
− c. f
Q1 = L1 + 4 ×i
f
N 106
Q1has or or 26.5 items below it.
4 4

140 PAGE
© Department of Distance & Continuing Education, Campus of Open Learning,
School of Open Learning, University of Delhi
BUSINESS STATISTICS

It lies in the cumulative frequency 29, which is corresponding to the- Notes


class-interval 20–30.
Therefore, Q1 group is 20–30.
N
where L1= 20, = 26.5, i = 10, c.f. = 12, f = 17
4
26.5 − 12
? Q1 = 20 + ×10 = 20 + 8.53 = 28.53
17
3N
− c. f .
Q3 = l1 + 4 ×i
f
3N 3 ×106
Q3 has or or 79.5 items below it.
4 4
It lies in the cumulative frequency 94, which is corresponding to the
group 50–60.
Therefore, Q3 group is 50–60.
79.5 − 75 4.5
? Q3 = 50 + ×10 = 50 + × 10 = 50 + 2.37 = 52.37
19 19
Q3 + Q1 − 2 Median
Coefficient of skewness =
Q2 − Q1
where Q3 = 52.37, Q1 = 28.53, median = 41.9
? Coefficient of Skewness =
52.37 + 28.53 − 2(41.9) 80.90 − 83.8 −2.90
= = = −0.12
52.37 − 28.53 23.84 23.84
Example 3 : Calculate the Karl-Pearson’s coefficient of skewness from
the following data :
Marks (above) : 0 10 20 30 40 50 60 70 80
No. of Students : 150 140 100 80 80 70 30 14 0
Solution : Thus formula of Karl-Pearson is applied to find out coefficient
of skewness.
3(mean – median)
SK =
σ

PAGE 141
© Department of Distance & Continuing Education, Campus of Open Learning,
School of Open Learning, University of Delhi
B.COM. (PROGRAMME)

Notes n
− c. f .
Median = L1 + 2 ×i
f
N 150
Median has
or or 75 items below it. It lies in the cumulative
2 2
frequency 80, which is corresponding to the group (40–50). Therefore,
median group is 40–50.
N
where, L1 = 40, = 76, f = 10, c. f . = 70, i = 10
2
75 − 70
Median = 40 + + 10 = 45.
10
Calculation of Mean and Standard Deviation
Marks Frequency Mid Deviations i = 10 dx2 fdx fdx2
f points from
Assumed
Mean
X (X – 45)
⎛ X – 45 ⎞
dx = ⎜ ⎟
⎝ 10 ⎠
0–10 10 5 – 40 – 4 16 – 40 160
10–20 40 15 – 30 – 3 9 – 120 360
20–30 20 25 – 20 – 2 4 – 40 80
30–40 0 35 – 10 – 1 1 0 0
40–50 10 45 0 0 0 0 0
50–60 40 55 + 10 + 1 1 40 40
60–70 16 65 + 20 + 2 4 32 64
70–80 14 75 + 30 + 3 9 42 126
N= 150 Ȉfdx = –86 Ȉ fdx2 = 830

Σfd
X = A+ × i , where A = 45, N = 150, i  Ȉfdx = –86.
N
−86
? X = 45 + ×10 = 45 − 5.73 = 39.27
150
2
Σfdx 2 ⎛ Σfdx ⎞
 ı −⎜ ⎟ ×i
N ⎝ N ⎠

142 PAGE
© Department of Distance & Continuing Education, Campus of Open Learning,
School of Open Learning, University of Delhi
BUSINESS STATISTICS

where, N = 150, i  Ȉfdx2  Ȉfdx = – 86. Notes


2
830 ⎛ −86 ⎞
? ı −⎜ ⎟ ×10 = 5.5333 − (−.57) ×10
2
150 ⎝ 150 ⎠

 RU ı 5.5333 − 0.3249 ×10 = 5.2048 ×10 = 22.8


3(Mean – Median)
Coefficient of skewness =
σ
 ZKHUH ı  0HDQ  0HGLDQ 
? Coefficient of skewness
3(39.27 − 45.00) 3(−5.73) −17.19
= = = −0.754
= 22.8 22.8 22.8
Hence, Coefficient of skewness is – 0.754.
Example 4 : (a) In a frequency distribution the coefficient of skewness
based on quartiles is 0.6. If the sum of the upper and lower quartile is
100 and median is 38, find the values of lower and upper quartiles. Also
find out the value of middle 50% items.
(b) In a certain distribution, the following results were obtained:
Coefficient of variation = 40%
X = 25.
Mode = 20.
Mean – Mode
Find out the Coefficient of skewness, by applying
Standard Deviation
Solution : (a) Since Bowley’s method is based on quartiles, we shall
use the following formula :
Q3 + Q1 − 2(Median)
Coefficient of skewness =
Q3 − Q1

where coeff. of SK. = +0.6, median = 38, (Q3 + Q1) = 100.


By substituting the values in the formula, we get
100 − 2(38)
+ 0.6 =
(Q3 − Q1 )

PAGE 143
© Department of Distance & Continuing Education, Campus of Open Learning,
School of Open Learning, University of Delhi
B.COM. (PROGRAMME)

Notes By cross multiplying, we get:


24
0.6 (Q3 – Q1) = 100 – 76 = 24 or Q3 – Q1 = = 40.
0.6
We can solve the below given simultaneous equations :
Q3 + Q1 = 100 ...(i)
Q3 – Q1 = 40 ...(ii)
or 2Q3 = 140 (adding the equations)
Q3 = 70
Since Q3 + Q1 = 100
? Q1 = 100 – 70 = 30.
Hence the lower and upper quartiles are 30 and 70.
The value of middle 50% items can be obtained with the help of (Q3 – Q1).
? The value of middle 50% items is (70 – 30) = 40.
(b) In this problem, the value of standard-deviation is missing. We can
FDOFXODWH ı E\ DSSO\LQJ WKH IROORZLQJ IRUPXOD
σ
C.V. = ×100
X

We are given, C.V. = 40%, X = 25


σ
? 40 = ×100 RU ı 
25
Mean – Mode
Coefficient of skewness =
σ
or 2Q3 = 140 (adding the equations)
Q3 = 70
Since Q3 + Q1= 100
? Q1 = 100 – 70 = 30
Hence the lower and upper quartiles are 30 and 70.
The value of middle 50% items can be obtained with the help of (Q3 – Q1)
? The value of middle 50% items is (70 – 30) = 40.

144 PAGE
© Department of Distance & Continuing Education, Campus of Open Learning,
School of Open Learning, University of Delhi
BUSINESS STATISTICS

(b) In this problem, the value of standard-deviation is missing. We can Notes


FDOFXODWH ı E\ DSSO\LQJ WKH IROORZLQJ IRUPXOD
σ
C.V. = ×100
X

We are given, C.V. = 40%, X = 25


σ
? 40 = ×100 RU ı 
25
Mean – Mode
Coefficient of skewness =
σ
:H DUH JLYHQ 0HDQ  0RGH  ı 
25 − 20
Coefficient of skewness = = 0.5
20
Hence Coefficient of skewness is = + 0.5
Example 5 : What is the relationship between Mean, Median and Mode in:
(a) Symmetrical curve.
(b) A negatively skewed curve.
(c) A positively skewed curve.
From the marks obtained by 120 students each in section A and B of a
class, the following measures are secured :
Section A Section B
Mean = 47 Marks Mean = 48
Standard deviation = 15 marks Standard deviation = 15 marks
Mode = 52 Mode = 45.
Find out the coefficient of skewness and determine the degree of skewness
and in which distribution, the marks are more skewed.
Solution : The relationship between mean, median and mode, in different
cases, can be established as :
(a) In a symmetrical curve, there is no skewness. Therefore the value
of mean = median = mode.
(b) In a negatively skewed curve, the value of mean is less than median
is less than mode. In other words, mean < median < mode.

PAGE 145
© Department of Distance & Continuing Education, Campus of Open Learning,
School of Open Learning, University of Delhi
B.COM. (PROGRAMME)

Notes (c) In a positively skewed curve, the value of mean is greater than median
is greater than mode. In other words, mean > median > mode.
In the given problem, for finding out the degree of skewness, we have
to compute the coefficient of skewness,
ZKHUH ȕ2 = 3, Mesokurtic Curve
ȕ2< 3, Platykurtic Curve
ȕ2>Leptokurtic Curve

4.6 Measures of Kurtosis


0HDVXUH RI NXUWRVLV LV GHQRWHG E\ ȕ2 DQG LQ D QRUPDO GLVWULEXWLRQ ȕ2= 3.
,I ȕ2 is greater than 3, the curve is more peaked and is named as leptokurtic,
LI ȕ2 is less than 3, the curve is flatter at the top than the normal, and
is named as platykurtic. Thus kurtosis is measured by:
Σfx 4
μ4 n
 ȕ2 = = where x = ( X − X )
μ 2 ⎛ Σfx 2 ⎞2
⎜ ⎟
⎝ n ⎠
R.A. Fisher had introduced another notation Greek letter gamma,
symbolically,
μ4
 Ȗ2 ȕ2 – 3 = = 3.
μ 22

,Q WKLV FDVH RI D QRUPDO GLVWULEXWLRQ Ȗ2 LV ]HUR ,I Ȗ2 is more than zero


SRVLWLYH  WKHQ WKH FXUYH LV SODW\NXUWLF DQG LI Ȗ2 is less than 0 (negative)
then the curve is leptokurtic.
Σfx 4
It may be noted that μ4 = is an absolute measure of kurtosis, but
n
μ4
ȕ2 = LV D UHODWLYH PHDVXUH RI NXUWRVLV /DUJHU WKH YDOXH RI Ȗ2 in a
μ 22
frequency distribution, the greater is its departure from normality.
6NHZQHVV DQG NXUWRVLV ȕ1 DQG ȕ2are measures of symmetry and normality

146 PAGE
© Department of Distance & Continuing Education, Campus of Open Learning,
School of Open Learning, University of Delhi
BUSINESS STATISTICS

UHVSHFWLYHO\ ,I ȕ2  WKH GLVWULEXWLRQ LV V\PPHWULFDO DQG LI ȕ2 = 3, the Notes


distribution curve is mesokurtic.

4.7 Comparison Among Variation, Skewness, Kurtosis


Dispersion, Skewness and Kurtosis are different characteristics of frequency
distribution. Dispersion studies the scatter of the items round a central
value or among themselves. It does not show the extent to which deviations
cluster below an average or above it. This is studied by skewness. In other
words, this tells us about the cluster of the deviations above and below
a measure of central tendency. Kurtosis studies the concentration of the
items at the central part of a series. If items concentrate too much at the
centre, the curve becomes ‘LEPTOKURTIC’ and if the concentration at
the centre is comparatively less, the curve becomes ‘PLATYKURTIC’.
Example 6 : From the following data given below, calculate the value
of kurtosis and find out the nature of distribution:
X : 0–10 10–20 20–30 30–40 40–50
f : 5 10 15 10 5
Solution :
Σfx 1125
Calculation of Mean = = = 25
N 45
μ4 Σfx 4
 &DOFXODWLRQ RI ȕ2 = where, μ 4 = = 40000
μ 22 N

Σfx 2 60000
μ2 = = = 133.33
N 45
μ4 40000
? ȕ2 = = =3
μ 2 (133.33) 2
2

6LQFH WKH YDOXH RI ȕ2 = 3, the distribution curve is mesokurtic.

PAGE 147
© Department of Distance & Continuing Education, Campus of Open Learning,
School of Open Learning, University of Delhi
B.COM. (PROGRAMME)

Notes (CALCULATION OF ȕ2)


X f Mid
Classes Frequency Points (X – 25)
X fx x x2 x3 x4 fx2 fx3 fx4
0–10 5 5 25 – 20 400 –8000 160000 2000 –40000 800000
10–20 10 15 150 – 10 100 –1000 10000 1000 –10000 100000
20–30 15 25 375 0 0 0 1 0 0 0
30–40 10 35 350 + 10 100 1000 10000 1000 10000 100000
40–50 5 45 225 + 20 400 8000 160000 2000 40000 800000
N = 45 Ȉfx = Ȉfx2 0 1800000
1125 =6000

4.8 Summary
Both skewness and kurtosis are related to the shape of the frequency
curve. Skewness means lack of symmetry, which implies that the mean,
median and mode are unequal in such a case. Skewness is positive when
its longer tail is to the right and negative when it is on the left. There
are three measures of skewness, given by Karl Pearson which is based
on averages and standard deviation; by Bowley which uses median and
quartiles; and by Kelly, based on median and the tenth and ninetieth
percentiles. Kurtosis refers to relative height of the frequency curve.
Distributions can be mesokurtic, leptokurtic and platykurtic on this basis.

4.9 Self-Assessment Questions

Exercise 1 : True or False Statements


(i) Since mode is the point corresponding to maximum concentration of
frequencies, its value is always higher than the mean and median.
(ii) Skewness and kurtosis are both indicative of the nature of dispersion.
(iii) All distributions are either positively skewed or negatively skewed.
(iv) For a symmetrical distribution, the sum of positive and negative
deviations from median is always equal to zero.
(v) If the excess of mean over mode is negative, it implies that skewness
is negative.

148 PAGE
© Department of Distance & Continuing Education, Campus of Open Learning,
School of Open Learning, University of Delhi
BUSINESS STATISTICS

(vi) A longer tail to the right indicates positive skewness while a longer Notes
tail to the left indicates negative skewness in the data.
(vii) For any skewed distribution, Mean – Mode = 3(Mean – Median).
(viii) Positive skewness is indicated when > Me> Mo and negative
skewness when < Me < Mo.
(ix) In a highly skewed distribution, the value of second quartile may be
different from that of the median.
(x) In every distribution, the lower and upper quartiles are equidistant
from median.
(xi) Bowley’s measure of skewness can vary between ±3.
(xii) Negatively skewed distributions are usually platykurtic.
(xiii) Bowley’s measure of skewness is more appropriate to use in an
open-ended distribution.
(xiv) A distribution more peaked than normal distribution is called
platykurtic distribution.
(xv) Kelly’s measure of kurtosis can vary between the limits of – 0.2631
to +0.2369.
(xvi) The five-point summary of a distribution includes mean, median,
mode, lower quartile and upper quartile.
(xvii) A distribution with lower quartile = 127.8, median = 135.2 and
upper quartile = 148.8 has negative skewness.
Ans.
(i) F (ii) T (iii) F (iv) T (v) T (vi) T (vii) F
(viii) T (ix) F (x) F (xi) F (xii) F (xiii) T (xiv) F
(xv) T (xvi) F (xvii) F

Exercise 2. Questions and Answers


(i) What is skewness? What are the tests of skewness? Distinguish
between positive and negative skewness. Give examples of cases
where positively and negatively skewed distributions may be obtained.
(ii) Draw rough sketches to show asymmetrical distribution, a negatively
skewed distribution and a positively skewed distribution. Also, show
the relative location of mean, median and mode in each case.

PAGE 149
© Department of Distance & Continuing Education, Campus of Open Learning,
School of Open Learning, University of Delhi
B.COM. (PROGRAMME)

Notes (iii) State the empirical relationship between mean, median and mode for
unimodal frequency curves that are moderately skewed.
(iv) Explain the measures of skewness given by Karl Pearson and Bowley.
(v) What is kurtosis? How is it measured in terms of Kelly’s formula
and the beta co-efficient?
(vi) “Averages, measures of variation, skewness and kurtosis are complementary
in understanding a frequency distribution.” Explain.
(vii) For the distribution of daily wages of a factory employing 880 workers,
the co-efficient of quartile deviation is 3/5 and the co-efficient of
skewness based on quartiles is 1/3. The median wage is known to
be Rs. 90. Calculate the lower and upper quartile wages.
(viii) In a symmetrical distribution, the mean, standard deviation and
range of marks for a group of 20 students are 40, 12 and 60. Find
the standard deviation of marks if the students with highest and
lowest marks are excluded.
(ix) Given that median = 133.5 and mode = 134, obtain the missing
frequencies for the following distribution and then calculate Bowley’s
co-efficient of skewness:
Class Interval Frequency
100–110 8
110–120 32
120–130 ?
130–140 ?
140–150 ?
150–160 12
160–170 8
Total 460
(x) For a distribution, Bowley’s co-efficient of skewness is 0.6. If the
sum of the upper and the lower quartiles is 100 and median is 38,
find the values of the upper and lower quartiles.
(xi) For a distribution, Bowley’s co-efficient of skewness is – 0.36, lower
quartile is 8.6 and median is 12.3. Calculate the co-efficient of
quartile deviation for this distribution.

150 PAGE
© Department of Distance & Continuing Education, Campus of Open Learning,
School of Open Learning, University of Delhi
BUSINESS STATISTICS

(xii) The following table gives the distribution of monthly wages of 500 Notes
workers in a factory:
Monthly Wages No. of Workers
(in Rs.)
1,500 – 2,000 10
2,000 – 2,500 25
2,500 – 3,000 145
3,000 – 3,500 220
3,500 – 4,000 70
4,000 – 4,500 30
Compute average monthly wage, mode, standard deviation and Karl
Pearson’s co-efficient of skewness.
(xiii) Given that median = 46 and mode = 37, find the missing frequencies
of the following distribution and also calculate Karl Pearson’s co-
efficient of skewness:
Class 20–30 30–40 40–50 50–60 60–70 70–80 80–90 Total
Interval :
Frequency : 12 ? ? ? 12 9 7 100
(xiv) Consider the following data about two distributions:
Distribution A Distribution B
Mean 120 110
Median 110 120
Standard deviation 10 10
Examine the following statements, stating with reasons whether each
of them is true or false:
(a) Distribution A has the same degree of variation as distribution
B has.
(b) Distribution A has the same degree of skewness as distribution
B has.
(xv) Karl Pearson’s co-efficient of skewness of a distribution is 0.40. Its
standard deviation is 8 and mean is 30. Find the median and mode
of the distribution.
(xvi) For a moderately skewed distribution of the retail prices of children’s
shoes, it is found that the mean price is Rs. 180 and the median

PAGE 151
© Department of Distance & Continuing Education, Campus of Open Learning,
School of Open Learning, University of Delhi
B.COM. (PROGRAMME)

Notes price is Rs. 164. If the co-efficient of variation is 20%, find the
Karl Pearson’s co-efficient of skewness.
(xvii) For a distribution, mean = 65, median = 70 and co-efficient of
skewness = –0.6. Find the mode and co-efficient of variation.
(xviii) Using mean and median, calculate Karl Pearson’s co-efficient of
skewness for the following distribution:
Marks : 10-100 10-80 10-60 10-50 10-40 10-30 10-20
Frequency : 100 88 73 57 35 17 5
(xix) Given, mean = 50, co-efficient of variation = 40% and J = –0.4.
Find mode, median and standard deviation.
(xx) The sum of 20 observations is 300 and the sum of their squares is
5000. Find the co-efficient of variation and co-efficient of skewness,
given further that median =15.
(xxi) Pearson’s co-efficient of skewness for a data distribution is 0.5 and
co-efficient of variation is 40%. Its mode is 80. Find the mean and
median of the distribution.
Ans.
(ix) –0.115 (x) 70, 30 (xi) 0.24
(xii) 3.345, 3.167, (xiii) 26, 20, 14, 0.7 (xiv) (a) False
503.46, 0.354 CVA< CVB
(b) True
(xv) 38.93, 36.8 (xvi) 1.33 (xvii) 80, 38.46%
(xviii) 0.12 (xix) 58, 52.67, 20 (xx) 33.33%, 0
(xxi) 100, 93.33

152 PAGE
© Department of Distance & Continuing Education, Campus of Open Learning,
School of Open Learning, University of Delhi
L E S S O N

5
Moments
STRUCTURE
5.1 Learning Objectives
5.2 Concept of Central Moments
5.3 Sheppard’s Method
5.4 &RHI¿FLHQWV RI 0RPHQWV
5.5 Summary
5.6 Self-Assessment Questions

5.1 Learning Objectives


After reading this lesson, you should be able to:
‹ Understand the meaning of moments.
‹ Compute central moments by different methods.
‹ Comprehend Shepard’s corrections of moments for Grouping errors.
‹ &RPPHQW XSRQ WKH QDWXUH RI GLVWULEXWLRQ ZLWK WKH KHOS RI Į DQG ȕ

5.2 Concept of Central Moments


The concept of moments has crept into the statistical literature from mechanics. In mechanics,
this concept refers to the turning or the rotating effect of a force whereas it is used to
describe the peculiarities of a frequency distribution in statistics. We can measure the central
tendency of a set of observations by using moments. Moments also help in measuring the
scatteredness, asymmetry and peakedness of a curve for a particular distribution.
Moments refers to the average of the deviations from mean or some other value, raised
to a certain power. The arithmetic mean of various powers of these deviations in any
distribution is called the moments of the distribution about mean. Moments about mean are
generally used in statistics. We use a greek alphabet μ read as mu for these moments. We
shall understand the first four moments about mean in the lesson, i.e., μ1, μ2, μ3 and μ4.

PAGE 153
© Department of Distance & Continuing Education, Campus of Open Learning,
School of Open Learning, University of Delhi
B.COM. (PROGRAMME)

Notes We can compute central moments in the following ways :


1. Direct method
2. Short-cut method
3. Step-deviation method

5.2.1 Direct Method


(i) Calculate arithmetic mean (X).
(ii  &DOFXODWH WKH VXP RI GHYLDWLRQV Ȉ[ IURP DULWKPHWLF PHDQ
(iii) Calculate the sum of x2, x3 and x4.
In case of frequency distributions multiply the individual value of
x2, x3 and x4 with corresponding frequencies and find out the sum
of fx2, fx3, and fx4.
(iv) Apply the following formulae rule.
Σx
μ1 = =0
N

Σx 2
μ2 =
N

Σx 3
μ3 =
N

Σx 4
μ4 =
N
In case of frequency distribution apply:
Σfx
μ1 =
N

Σfx 2
μ2 =
N

Σfx3
μ3 =
N

154 PAGE
© Department of Distance & Continuing Education, Campus of Open Learning,
School of Open Learning, University of Delhi
BUSINESS STATISTICS

Notes
Σfx 4
μ4 =
N
Let us take an example to understand the computation of the moments
about mean
Example 1 : Calculate the first four moments about the mean from the
following set of numbers 2, 3, 7, 8, 10.
Solution :
Calculation of Moments

(X – X )
X x x2 x3 x4
2 –4 16 –64 256
3 –3 9 –27 81
7 1 1 1 1
8 2 4 8 16
10 4 16 64 256
ȈX = 30 0 46 –18 610
ΣX 30
X = = = 6 , where N = 5
N 5
Moments of the data can be computed by using the values calculated
above.
Σx 0
μ1 = = =0
N 5

Σx 2 46
μ2 = = = 9.2
N 5

Σx3 −18
μ3 = = = −3.6
N 5
Σx 4 610
μ4 = = = 122
N 5
Therefore, the first four central moments about the mean are : 0, 9.2,
–3.6 and 122 respectively.

PAGE 155
© Department of Distance & Continuing Education, Campus of Open Learning,
School of Open Learning, University of Delhi
B.COM. (PROGRAMME)

Notes Example 2 : From the marks distribution of 100 candidates, compute


the first four moments about mean.
Marks Number of Candidates
0–10 10
10–20 15
20–30 25
30–40 25
40–50 10
50–60 10
60–70 5
Solution :
Calculation of Moments
(Mid
Value) (X – X )
Marks X f fX x fx fx2 fx3 fx4
0–10 5 10 50 – 26 – 260 6,760 – 1,75,760 45,69,760
10–20 15 15 225 – 16 – 240 3,840 – 61,440 9,83,040
20–30 25 25 625 – 6 – 150 900 – 5,400 32,400
30–40 35 25 875 4 100 400 1,600 6,400
40–50 45 10 450 14 140 1,960 27,440 3,84,160
50–60 55 10 550 24 240 5,760 1,38,240 33,17,760
60–70 65 5 325 34 170 5,780 1,96,520 66,81,680
1,59,75,200 N = 100 3100 0 25,400 1,21,200

ΣfX 3100
? X = = = 31 marks.
N 100
Now, we can calculate the moments about mean as follows :
Σfx 0
μ1 = =
N 100

Σfx 2 25, 400


μ2 = = = 254
N 100

Σfx3 1, 21, 200


μ3 = = = 1, 212
N 100

156 PAGE
© Department of Distance & Continuing Education, Campus of Open Learning,
School of Open Learning, University of Delhi
BUSINESS STATISTICS

Notes
Σfx 4 1,59, 75, 200
μ4 = = = 1,59, 752
N 100
Therefore, the Central Moments are : 0, 254, 1212, 159752 respectively.

5.2.2 Short-Cut-Method
If the arithmetic mean is in fractions then, it is difficult to calculate
deviations (x) from arithmetic mean. Short-cut method is used in such
cases.
(i) Take any value as an arbitrary mean (A).
(ii) Calculate deviations (d) from A and calculate the first four moments
in the similar way as done in direct method.
These moments are called moments about an arbitrary origin which are
represented by the greek word v read as nu. The formulae for these
moments are :

Σ( X − A) Σd
v1 = = where d = X – A
N Ν
Σ( X − A) 2 Σd 2
v2 = =
N Ν
Σ( X − A)3 Σd 3
v3 = =
N Ν
Σ( X − A) 4 Σd 4
v4 = =
N Ν
In case of frequency distribution,

Σf ( X − A) Σfd
v1 = =
N N

Σf ( X − A) 2 Σfd 2
v2 = =
N N
Σf ( X − A)3 Σfd 3
v3 = =
N N

PAGE 157
© Department of Distance & Continuing Education, Campus of Open Learning,
School of Open Learning, University of Delhi
B.COM. (PROGRAMME)

Notes
Σf ( X − A) 4 Σfd 4
v4 = =
N N
After calculating moments about an arbitrary origin convert them into
Moments about mean by using the following equations:
μ 1 = v 1– v 1 = 0
μ 2 = v 2 – v 12 ı2
μ3 = v3 – 3v2v1 + 2v13
μ4 = v4 – 4v3.v1 + 6v2.v12 – 3v14
We can calculate the Moments about an arbitrary origin from Moments
about the mean by this relationship:
v1 = μ1+d where d is the difference between the
v2 = μ2 + d2 mean and origin about which the Moments
v3 = μ3 + 3μ2d + d3 are to be calculated.
v4 = μ4 + 4μ3d + 6μ2d2 + d4 ?d = X – A
Example 3 : We are given the following set of numbers 1, 3, 7, 9, 10.
Calculate the first four moments about the origin 4.
Solution :
Calculation of First Four Moments about A = 4
X d = (X – A) d2 d3 d4
1 – 3 9 – 27 81
3 –1 1 – 1 1
7 3 9 27 81
9 5 25 125 625
10 6 36 216 1296
N = 5 10 80 340 2084
Σd 10
v1 = = =2
N 5

Σd 2 80
v2 = = = 16
N 5
Σd 3 340
v3 = = = 68
N 5

158 PAGE
© Department of Distance & Continuing Education, Campus of Open Learning,
School of Open Learning, University of Delhi
BUSINESS STATISTICS

Notes
Σd 4 2084
v4 = = = 416.8
N 5
Therefore the Moments about an arbitrary origin are 2, 16, 68 and 416.8
respectively.
Example 4 : Calculate first four moments about mean for the distribution
of heights of the following 100 students.
Heights (Inches) 61 64 67 70 73
Number of Students 5 18 42 27 8
Solution :
Calculation of Central Moments (short-cut method)
Heights No. of A = 67 f × d fd × d fd2 × d fd3 × d
students
X f d = (X – 67) fd fd2 fd3 fd4
61 5 –6 –30 180 –1,080 6,480
64 18 –3 –54 162 – 486 1,458
67 42 0 0 0 0 0
70 27 +3 81 243 729 2,187
73 8 +6 48 288 1,728 10,368
N = 100 45 873 891 20,493
Now we can substitute the calculated values in the formulae
Σfd 45
v1 = = = 0.45
N 100

Σfd 2 873
v2 = = = 8.73
N 100

Σfd 3 891
v3 = = = 8.91
N 100

Σfd 4 20493
v4 = = = 204.93
N 100
Moments about mean can be calculated as follows :
μ1 = v1 –v1 = 0 = 0.45 – 0.45 = 0

PAGE 159
© Department of Distance & Continuing Education, Campus of Open Learning,
School of Open Learning, University of Delhi
B.COM. (PROGRAMME)

Notes μ 2 = v 2 – v 12 ı2 = 8.73 – (0.45)2 = 8.73 – 0.2025 = 8.5275


μ3 = v3 – 3v2v1 + 2v13 = 8.91 – 3(8.73 × 0.45) + 2(0.45)3 = 8.91 – 11.7855
+ 0.18225 = – 2.6932
μ4 = v4 – 4v3. v1 + 6v2.vl2 – 3v14
= 204.93 – 4 × 8.91 × 0.45 + 6 × 8.73 × (0.45)2 – 3 × (0.45)4 =
204.93 – 16.038 + 10.60695 – 0.1230 = 199.3759
Hence the Central Moments are : 0,8.5275, –2.6932 and 199.3759.
If we are given the values of central moments and were interested in
finding the Moments about an arbitary origin (A = 67). Then we can
calculate as follows :

Σfd 45
X = A+ = 67 + = 67.45
N 100
D = ( X − A) = 67.45 − 67 = 0.45

v1 = μ1 + d = 0 + 0.45 = 0.45

v2 = μ 2 + d 2 = 8.5275 + (0.45) 2 = 8.5275 + .2025 = 8.73

v3 = μ3 + 3μ 2 d + d 3
= −2.6932 + 3 × 8.5275 × (0.45) + (0.45)3 = −2.6932 + 11.512125 + 0.091125 = 8.91.

v4 = μ 4 4μ3d + 6μ 2 d 3 + d 4
= 199.3759 – (4 × –2.69325 + 0.45) + 6 × 8.5275 × (0.45)2 + (0.45)4
= 199.3759 – 4.84785 + 10.36091 + 0.041006 = 204.93
? Moments about an arbitrary origin (67) are : 0.45, 8.73, 8.91 and
204.93.

5.2.3 Step-Deviation-Method
It is the most appropriate method to calculate central moments in problems
of continuous frequency distributions with equal class-intervals. Step-
deviation method is similar to short cut method. The only difference is
that in case of step-deviation method, we take a common factor from
among the deviations (d)which are taken from assumed mean (A).

160 PAGE
© Department of Distance & Continuing Education, Campus of Open Learning,
School of Open Learning, University of Delhi
BUSINESS STATISTICS

(i) Calculate deviations (d) from arbitrary origin (A). Notes


(ii) Take a common factor from all the values of d and find out the
sum of d’.
(iii) Find out the values of dƍ2, dƍ3 and dƍ4 and their aggregates.
(iv) Calculate the value of v1, v2, v3 and v4 by using the formulae.
(v) Convert the calculated Moments about an arbitrary origin into Moments
about the mean with the help of these relationship:
μ1 = v1 − v1 = 0

μ2 = v2 − v12 = σ2

μ3 = v3 − 3v2v1 + 2v13

μ4 = v4 − 4v3 .v1 + 6v2 .v12 + 3v14

We shall understand the computation of the first four moments about an


arbitrary origin by step deviation method.
Example 5 : Calculate first four moments about mean with the help of
moments about an assumed mean 35 from the following data :
Class Frequency
0–10 4
10–20 10
20–30 21
30–40 32
40–50 21
50–60 7
60–70 5

PAGE 161
© Department of Distance & Continuing Education, Campus of Open Learning,
School of Open Learning, University of Delhi
B.COM. (PROGRAMME)

Notes Solution :
Calculation of Moments about arbitrary Mean
(Mid Points) A = 35, C = 10

Class X f ⎡ X – A⎤ fdƍ1 fdƍ2 fdƍ3 fdƍ4


dƍ = ⎢
⎣ C ⎥⎦
0–10 5 4 – 3 – 12 36 – 108 324
10–20 15 10 – 2 – 20 40 – 80 160
20–30 25 21 – 1 – 21 21 – 21 21
30–40 35 32 0 0 0 0 0
40–50 45 21 1 21 21 21 21
50–60 55 7 2 14 28 56 112
60–70 65 5 3 15 45 135 405
N= 100 –3 191 3 1043
First of all, we shall calculate the first four moments about an arbitrary
mean by substituting the values.
Σfd ′ −3
v1 = ×C = ×10 = −0.3
N 100
Σfd ′2 191
v2 = ×C2 = ×100 = 191
N 100
Σfd ′3 3
v3 = × C3 = ×103 = 30
N 100
Σfd ′4 1043
v4 = ×C4 = ×104 = 1, 04,300
N 100
From these we get the central moments as below :
μ1 = v1 − v1 = −0.3 − (−0.3) = 0

μ 2 = v2 − v12 = σ2 = 191 − (−0.3) 2 = 191 − 0.09 = 190.91

μ3 = v3 − 3v2v1 + 2v13
= 30 – 3 × 191 (–0.3) + 2 (–0.3)3 = 30 + 171.9 – 0.054 = 201.846
μ 4 = v4 − 4v3 .v1 + 6v2 .v12 − 3v14

= 1, 04,300 − 4 × 30(−0.3) × 6 ×191(−0.3) 2 − 3(−0.3) 4


= 1, 04,300 + 36 + 103.14 − 0.0243 = 1, 04, 439.12
162 PAGE
© Department of Distance & Continuing Education, Campus of Open Learning,
School of Open Learning, University of Delhi
BUSINESS STATISTICS

5.3 Sheppard’s Method Notes

When data are grouped into frequency distributions, the individual


values lose their identity. While calculating moments, it is assumed that
the frequencies are concentrated at the mid-points of the classes for a
continuous frequency distribution.
Let us understand by assuming a class of 10–20 whose relative frequency
is 20.
To compute Moments or arithmetic mean, we always take the mid-point
10 + 20
of the class 10–20 which is = 15. But in reality it may be just
2
possible that more than half the values for the class 10–20 are more than
15. Due to this assumption the grouping errors enter into the calculation
of Moments. To remove these errors W.F. Sheppard introduced some
corrections which are known as Sheppard’s corrections. These are :
The first and third Moment needs no corrections.
1 2
μ2 (corrected) = μ2(uncorrected) – i where i is the class-interval.
12
1 7 4
μ4(corrected) = μ4 (uncorrected) – μ 2 (uncorrected) i 2 + i
2 240
Sheppard’s corrections are applied only under these conditions :
(i) When the frequency distribution is continuous.
(ii) When frequency tapers off to zero in both directions.
(iii) When the frequencies are not less than 1000.
(iv) When the frequency distribution is not J-shaped or U-shaped or
skewed.
(v) When the class-interval is uniform.
Let us understand with the help of an example.
Example 6 : Applying Sheppard’s corrections, find out the corrected values
of the moments where the class interval is 10 and μ1 = 0, μ 2 = 254, μ3 = 1212
and μ 4 = 1,59, 752.

PAGE 163
© Department of Distance & Continuing Education, Campus of Open Learning,
School of Open Learning, University of Delhi
B.COM. (PROGRAMME)

Notes Solution : We are given all the values of four moments and class-interval.
μ1 and μ3needs no correction.
1 2 1 100
μ2 (corrected) = μ 2 − i = 254 − ×102 = 254 − = 245.667
12 12 12
1 7 4 1 7
μ4 (corrected) = μ 4 − μ 2i 2 + i = 1,59752 − × 254 ×102 + ×104
2 240 2 240
= 1,59752 – 12,700 + 291.667 = 147,343.667.
Therefore, the corrected values of four moments are 0,245.667, 1,212
and 147,343.667 respectively.

5.4 Coefficients of Moments


7KHUH DUH WKUHH FRHIILFLHQWV ZKLFK DUH XVHG LQ SUDFWLFH 7KH\ DUH Į $OSKD 
ȕ %HWD  Ȗ *DPPD FRHIILFLHQWV 7KHVH FRHIILFLHQWV DUH FDOFXODWHG RQ
individual relationships of various Moments. The formulae are :
Alpha-Coefficients
μ1 μ2
α1 = =0 α2 = =1
σ σ2

μ3 μ3 μ4 μ4
α3 = = , and α4 = =
σ3 μ2
32
σ4 μ 22
Beta-Coefficients
μ32 μ4
β1 = = α 22 and β2 = = α4
μ32 μ 22
Gamma-Coefficients

μ 4 − 3μ 22
γ1 = β1 = α3 and γ 2 = β2 − 3 =
μ 22
%HWD&RHIILFLHQWV ȕ1 DQG ȕ2) are used to find the skewness and kurtosis
of a distribution. Let us take an illustration to understand coefficients.
Example 7: The values of μ1 , μ 2 , μ3 , and μ 4 , are 0, 9.2, 3.6 and 122
respectively. Find out the skewness and kurtosis of the distribution.

164 PAGE
© Department of Distance & Continuing Education, Campus of Open Learning,
School of Open Learning, University of Delhi
BUSINESS STATISTICS

Solution : To comment upon skewness and kurtosis of the distribution, Notes


ZH VKDOO FDOFXODWH WKH YDOXHV RI ȕ1 DQG ȕ2.
μ32
(3.6)2 12.96
ȕ1 = 3 = = = 0.0166
μ 2 (9.2) 2
778.688

μ4 122 122
ȕ2 = = = = 1.4
μ 2 (9.2)
2 2
84.64
Hence the distribution is positively skewed and the curve is platykurtic
or flat at the top.
Example 8 : Calculate the first four Moments about an arbitrary origin.
Convert them into Moments about mean. Applying Sheppard’s corrections,
calculate corrected Moments and beta coefficients from the following data:
Experience No. of Employees
(years)
0–1 15
1–2 22
2–3 45
3–4 35
4–5 30
5–6 20
6–7 16
7–8 10
8–9 5
9–10 2

PAGE 165
© Department of Distance & Continuing Education, Campus of Open Learning,
School of Open Learning, University of Delhi
B.COM. (PROGRAMME)

Notes Solution :
Calculation of Moments
Experience Mid No. of Let A = 4.5
Points Employees
(Years) X f d = (X – A) fd fd2 fd3 fd4
0–1 0.5 15 –4 – 60 240 – 960 3,840
1–2 1.5 22 – 3 – 66 198 – 594 1,782
2–3 2.5 45 – 2 – 90 180 – 360 720
3–4 3.5 35 – 1 – 35 35 – 35 35
4–5 4.5 30 0 0 0 0 0
5–6 5.5 20 1 20 20 20 20
6–7 6.5 16 2 32 64 128 256
7–8 7.5 10 3 30 90 270 810
8–9 8.5 5 4 20 80 320 1,280
9–10 9.5 2 5 10 50 250 1,250
200 – 139 957 – 961 9,993
Σfd −139
We can find out v1 = = = −0.695
N 200
Σfd 2 957
v2 = = = 4.785
N 200

Σfd 3 −961
v3 = = = −4.805
N 200

Σfd 4 9993
v4 = = = 49.965
N 200
Computed moments are moments about an arbitrary point, 4.5. The central
moments are calculated below:
μ1 = v1 – v1 = – 0.695 – (–0.695) = 0
μ2 = v2 – v12= 5.985 – (–0.695)2 = 5.985 – 0.483 = 5.502
μ3 = v3 – 3v2v1 + 2v13
= – 4.805 – 3 (5.985) × (– 0.695) + 2(–0.695)3 = – 4.805 + 12.479
– 0.671 = 7.003
μ4 = v4 – 4v3.v1 + 6v2.v12 – 3v14
= 49.965 – 4 (– 4.805) × (– 0.695) + 6 × 5.985 (–0.695)2 – 3(– 0.695)4
= 49.965 – 13.358 + 17.345 – 0.700 = 53.252.

166 PAGE
© Department of Distance & Continuing Education, Campus of Open Learning,
School of Open Learning, University of Delhi
BUSINESS STATISTICS

Applying Sheppard’s corrections, we have Notes


1 2
μ2 (corrected) = μ 2 − i
12
1
= 5.502 − × (1) 2 = 5.502 − 0.083 = 5.419
12
1 7
μ4 (corrected) = μ4 – μ 2i 2 + × i4
12 240
1 7
= 53.252 − × 5.502 × (1) 2 × × (1) 4
2 240
7
= 53.252 − 2.751 + = 53.252 − 2.751 + 0.029 = 50.53
240
Beta Constants

μ32
(7.003) 2 49.042
 ȕ1 = 3 = = = 0.31
μ 2 (5.419) 2 159.132

μ4 50.33 50.33
 ȕ2 = = = = 1.714
μ 22 (5.419) 2 29.336
Therefore the central moments after correction are 0, 5.419, 7.003 and
 ȕ1  DQG ȕ2 = 1.714.

5.5 Summary
Moments provide a useful method to study various characteristics of a
set of data. Moments calculated about mean are called central moments.
There can also be moments about any given value A. When A = 0,
moments calculated are called moments about origin. It is possible to
convert moments about A into central moments and vice versa. The first
moment about zero is equal to mean and the first moment about mean
is equal to zero.
The second central moment is the variance of the distribution. The beta
and g-statistics are calculated with central moments and are used to learn
DERXW VNHZQHVV DQG NXUWRVLV ȕ1 and g1 are measures of skewness while
ȕ2 and g2 measure kurtosis. If the third central moment is positive, the

PAGE 167
© Department of Distance & Continuing Education, Campus of Open Learning,
School of Open Learning, University of Delhi
B.COM. (PROGRAMME)

Notes skewness is positive while if it is negative, the skewness is negative. For


D V\PPHWULFDO GLVWULEXWLRQ LW LV HTXDO WR ]HUR ȕ1 is always positive but
WKH JUHDWHU LWV YDOXH WKH PRUH WKH VNHZQHVV ,I ȕ2= 3, the distribution is
PHVRNXUWLF LI ȕ2 is less than 3, the distribution is platykurtic while if it
is greater than 3, the distribution in question is leptokurtic.

5.6 Self-Assessment Questions

Exercise 1 : True and False Statements


(i) The first moment about mean is always equal to zero.
(ii) The moments of even order can never assume negative values.
(iii) The fourth moment about mean always exceeds the third moment
which, in turn, always exceeds the second moment.
(iv) The second moment about origin is equal to the variance of the
distribution.
(v) A negative value of the third moment about an arbitrary point does
not necessarily mean that the distribution has negative skewness.
(vi) Sheppard’s corrections aim at removing the effect of grouping error.
(vii) The corrected values of moments, obtained after applying Sheppard’s
corrections, are always higher than the corresponding uncorrected
values of the moments.
(viii) All symmetrical distributions are not mesokurtic, but all mesokurtic
distributions are symmetrical.
(ix) A look at the value of the fourth central moment reveals whether
the distribution is mesokurtic or not.
(x  :KHQ ȕ1 !  WKH VNHZQHVV LV SRVLWLYH DQG ZKHQ ȕ1< 0, the skewness
is negative.
Ans.
(i) T (ii) T (iii) F (iv) F (v) T (vi) F (vii) F
(viii) F (ix) F (x) F

168 PAGE
© Department of Distance & Continuing Education, Campus of Open Learning,
School of Open Learning, University of Delhi
BUSINESS STATISTICS

Exercise 2 : Questions and Answers Notes


(i) Define moments. Establish the relationship between moments about
arbitrary origin and central moments and vice versa.
(ii) Explain how moments help in determining the shape of a frequency
distribution. In this context, explain the calculation and interpretation
of beta and gamma statistics.
(iii) Explain clearly the difference between skewness and kurtosis. Is it
FRUUHFW WR VD\ WKDW D GLVWULEXWLRQ ZKLFK KDV ȕ2 equal to 3 must be
symmetrical?
(iv  ³$ GLVWULEXWLRQ IRU ZKLFK ȕ1 equal to 0, is necessarily a normal
distribution.” Do you agree? Explain.
(v) What are Sheppard’s corrections for moments? State the conditions
under which they are applied.
(vi) The following is the distribution of heights of the students of a
class:
Height (cms) No. of Students
120–125 3
125–130 7
130–135 25
135–140 30
140–145 25
145–150 7
150–155 3
(a) Calculate Karl Pearson’s co-efficient of skewness using (i)
mean and mode, and (ii) mean and median.
(b) Calculate Bowley’s co-efficient of skewness.
(vii) Calculate (a) central moments, (b) moments about A = 6, and (c)
moments about origin for the following data:
9, 4, 6, 9, 11, 13, 8, 4
(viii) Calculate four central moments for the following distribution:
Class Interval Frequency
10–20 8
20–30 12
30–40 15

PAGE 169
© Department of Distance & Continuing Education, Campus of Open Learning,
School of Open Learning, University of Delhi
B.COM. (PROGRAMME)

Notes Class Interval Frequency


40–50 8
50–60 5
60–70 2
Total 50
(ix) Given, n =  Ȉ(X ±  ±  Ȉ X – 10)2  DQG Ȉ(X – 10)3
= – 480. Calculate the following:
(a) Arithmetic mean
(b) Variance
(c) Third moment about 10
(d) A measure of relative skewness based on moments
(x) For the following distribution, calculate first four (i) moments about
45 and (ii) central moments. Also, comment on the shape of the
distribution using gamma statistics.
Class Interval Frequency
10–20 2
20–30 5
30–40 12
40–50 20
50–60 8
60–70 2
70–80 1
(xi) Given the following distribution:
Class Interval Frequency
10–20 2
20–30 6
30–40 12
40–50 20
50–60 12
60–70 6
70––80 2
(a) Calculate mean, median and mode and also find the value of
Karl Pearson’s co-efficient of skewness.

170 PAGE
© Department of Distance & Continuing Education, Campus of Open Learning,
School of Open Learning, University of Delhi
BUSINESS STATISTICS

(b) Calculate the first four central moments and the beta co-efficient Notes
of skewness.
(c) Comment on the results.
(xii) You are given the following frequency distribution:
Class Interval Frequency
80–100 12
100–120 15
120–140 20
140–160 38
160–180 60
180–200 33
200–220 14
220–240 8
(a) Calculate moments about 170.
(b) Convert these to central moments and obtain the values of
beta and gamma statistics. Also, comment on the shape of
the distribution.
(xiii) For the following distribution, calculate first four moments about
A = 150 and obtain central moments from these. Also, calculate
beta co-efficients and comment on the skewness and kurtosis.
Class Interval Frequency
80–100 3
100–120 5
120–140 20
140–160 16
160–180 10
180–200 4
200–220 2
(xiv) The first four moments of a distribution are 2, 20, 40, and 500,
respectively. Comment on the shape of the distribution.
(xv) The first four moments of a distribution are calculated as 6; 235;

PAGE 171
© Department of Distance & Continuing Education, Campus of Open Learning,
School of Open Learning, University of Delhi
B.COM. (PROGRAMME)

Notes 1,248; and 24,680. You are required to examine the skewness and
kurtosis of the distribution.
(xvi) From the following information about two distributions, find which
is more skewed?
Distribution Second Moment Third Moment
A 16 –15.7
B 40 25.8
(xvii) For a mesokurtic distribution, the fourth central moment is 768.
Obtain its standard deviation.
(xviii) The first four moments of a distribution about the value 4 of the
variable are –1.5, 17, –30 and 108. Its mean is given to be 2.5.
You are required to:
(a) Calculate the central moments and moments about origin.
(b) Determine the co-efficient of variation and variance of the
distribution.
(c) Examine the shape of the distribution.
(xix  )RU D PHVRNXUWLF GLVWULEXWLRQ ȕ1= 0.004 and μ3= 16. Calculate the
value of its fourth central moment.
(xx) For a mesokurtic distribution, co-efficient of variation = 40% and
arithmetic mean = 40. Find the value of its fourth central moment.
(xxi) If variance = 42, then what values of μ4 would make a distribution
(i) mesokurtic, (ii) platykurtic, and (iii) leptokurtic?
(xxii) The first two moments about 40 for a set of 25 values were calculated
as equal to 65 and 2,985, respectively. Test if the calculations are
consistent.
(xxiii) You are given here the results of calculations in respect of a
negatively skewed distribution:
 1  PHDQ  YDULDQFH  ȕ1  DQG ȕ2 = 2.4.
It was discovered later on that an item 12 was wrongly recorded as
2. Find the corrected values of mean, variance and the two beta
constants.
(xxiv)For a mesokurtic distribution, it is known that the first moment about

172 PAGE
© Department of Distance & Continuing Education, Campus of Open Learning,
School of Open Learning, University of Delhi
BUSINESS STATISTICS

32 is 28 while the fourth moment about 60 is 62,208. Determine Notes


the values of mean and co-efficient of variation.
(xxv) Apply Sheppard’s corrections to the following moments :
First moment = – 7 Third moment = 873
Second moment = 193 Fourth moment = 87,504
Width of class interval = 10
Ans.
(vi) 0, 0, (vii) 0, 9, 2.25, 154.5, (viii) 0, 179.4, 951,
2, 13, 64.25, 80, 354,
404.5, 8, 73,
730.25, 7778.5
(ix) 6, 271, –48, (x) –2.6, 150, –1, (xi) 45, 45, 45, 0, 0,
0.733 100, 75,000, 0, 180, 0, 90000, 0;
143.24,
(xii) –8.6, 1212, –40, (xiii) –5, 740, – 6000, (xiv) 1, 2.39;
400, 44, 40000, 1544000, 0, 715,
0, 1, 138.04, 4850, 15.33,
–10, 402.5, 35, 125, 0.0644, 3;
71, 667, 0.0734,
–0.271, 2.758,
–0.242;
(xv) Low positive (xvi) A, (xvii) 4;
skewness,
platykurtic;
(xviii) 0, 14.75, 39.75, (xix) 4800; (xx) 1,96,608;
142.3125, 2.5,
21, 166, 1132,
183.3%, 21,
0.702, –2.35;
(xxi) 5.292, < 5.292, (xxii) No, 0; (xxiii) 14.1, 228.59,
> 5.292; 0.771, 2.042;
(xxiv) 60, 20%; (xxv) 1.357, 154.578

PAGE 173
© Department of Distance & Continuing Education, Campus of Open Learning,
School of Open Learning, University of Delhi
UNIT-2

PAGE 175
© Department of Distance & Continuing Education, Campus of Open Learning,
School of Open Learning, University of Delhi
L E S S O N

1
Theory of Probability
STRUCTURE

1.1 Learning Objectives


1.2 Probability Distribution
1.3 Basic Terminology in Probability
1.4 Methods of Assigning Probability
1.5 Computation of Probability
1.6 Laws of Probability
1.7 Bayes’ Theorem
1.8 Expected Value
1.9 Summary
1.10 Self-Assessment Questions

1.1 Learning Objectives


After reading this chapter, you should be able to:
‹ Define probability.
‹ Understand the concept of experiment, sample space, events and their relationships.
‹ Describe the classical, relative frequency and personalistic approaches of probability.
‹ Compute probabilities with the help of addition, multiplication, conditional and Bayes’
theorem.

1.2 Probability Distribution


Jacob Bernoulli, Abraham de Moivre, Reverend Thomas Bayes and Joseph Lagrange
significantly contributed to develop the probability formulas and techniques. However, in

PAGE 177
© Department of Distance & Continuing Education, Campus of Open Learning,
School of Open Learning, University of Delhi
B.COM. (PROGRAMME)

Notes the early nineteenth century Pierre Simon and Marquis de Laplace
popularize the general theory of probability by compiling all these earlier
ideas.
Probability is the branch

Second Die
of mathematics concerned 6
with the outcomes of random
events. Probability refers to 5
the chance or potential of
4
an outcome. It explains the
likelihood of occurrence of 3
specific event(s). Thus, the
probability is defined as the 2
degree to which something is 1
likely to occur. The probability
that an event will occur ranges
1 2 3 4 5 6
from 0 to 1, where 0 denotes First Die
impossible event while 1 sample space for pair of dice experiment
denotes certain event.
Figure 1
Initially, probability theory
was applied in gambling to determine the chance of losing or winning,
eventually in the nineteenth century the concept of probability was used
in the insurance industry to quantify the risk of loss in order to calculate
insurance premiums to be charged from policyholder. Over the period
of time, its application extended to social and economic problems. For
instance – Most of the decision-making situations in business management
involve uncertainty. Since uncertainty is present and is an important aspect
in determining the consequences of various Jacob Bernoulli, Abraham
de Moivre, Reverend Thomas Bayes and Joseph Lagrange significantly
contributed to develop the probability formulas and techniques. However,
in the early nineteenth century Pierre Simon and Marquis de Laplace
popularize the general theory of probability by compiling all these earlier
ideas.
Probability is the branch of mathematics concerned with the outcomes
of random events. Probability refers to the chance or potential of an
outcome. It explains the likelihood of occurrence of specific event(s).

178 PAGE
© Department of Distance & Continuing Education, Campus of Open Learning,
School of Open Learning, University of Delhi
BUSINESS STATISTICS

Thus, the probability is defined as the degree to which something is Notes


likely to occur. The probability that an event will occur ranges from 0
to 1, where 0 denotes impossible event while 1 denotes certain event.
Initially, probability theory was applied in gambling to determine the
chance of losing or winning, eventually in the nineteenth century the
concept of probability was used in the insurance industry to quantify the
risk of loss in order to calculate insurance premiums to be charged from
policyholder. Over the period of time, its application extended to social
and economic problems. For instance – alternative courses of action,
it is imperative to get proper appreciation of it, draw a mathematical
picture of it and attempt to measure it in numerical terms. There are
many advantages in having a numerical measure for uncertainty. Besides
facilitating understanding and allowing analysis, it helps in communication
between executives. Verbally, a manager at a meeting might indicate
that he is “fairly sure” about the success of a particular project. This
phrase might mean something quite different to the other executives at
the meeting. ‘Fairly sure’ might mean that success will occur 9 out of
10 times to one decision maker (implying that he is 90 per cent sure)
while the same phrase might indicate 7 times out of 10 to another.
Numbers remove such confusion. Besides this, an important advantage
of a numerical measure is the ability to use mathematics for analysis.
Uncertainty is expressed in numerical terms by the theory of probability
as probability is at once the language and measure of uncertainty. In this
lesson, we are going to study as to how the probability also provides a
foundation for the whole of the analytical statistics that we are going to
learn in the course of these lessons.
The theory of probability takes on practical value when it is defined in
relation to an experiment. Such an experiment might be tossing a coin,
taking out a card out of a standard deck of playing cards, tossing a six-
faced die, observing the number of defectives in a lot of electric bulbs,
tossing a pair of dice, drawing a ball from an urn containing balls,.. and
so on. Once the experiment has been defined, all possible outcomes from
the experiment are identified. This exhaustive set of outcomes constitutes
the sample space, S. The sample space is a key concept and an important
base of probability theory.

PAGE 179
© Department of Distance & Continuing Education, Campus of Open Learning,
School of Open Learning, University of Delhi
B.COM. (PROGRAMME)

Notes One of the simplest sample spaces can be the set of outcomes when
a pair of coins is tossed. It consists of four outcomes which can be
conveniently represented as :
S = {HH, HT, TH, TT}
where H denotes a head and T denotes a tail.
We can consider the case of a manufacturer who produces electric bulbs
in large batches. From each batch, a sample of 80 items is selected at
random, and the number of defective items are recorded. Although the
number of defectives in any sample cannot be predicted with certainty, all
of the possible outcomes may be known. The number of defective items
in a sample can be any integer from 9 to 80. Here the sample space is :
S = {0, 1, 2, 3, .......80}
In the same manner, when a pair of dice is tossed, a total of 36 outcomes
are possible. This can be represented as shown in figure 1
S = {(1,1), (1,2), (1,3), (1,4), (1,5), (1,6), (2,1), (2,2), (2,3) (2,4), (2,5),(2,6),
(3,1), (3,2), (3,3), (3,4), (3,5), (3,6), (4,1), (4,2), (4,3), (4,4), (4,5), (4,6),
(5,1), (5,2), (5,3), (5,4), (5,5), (5,6), (6,1), (6,2), (6,3), (6,4), (6,5), (6,6)}
Or simply Sample Space can be calculated as:
S = (all possible outcome of Die 1) × (all possible of outcome of Die 2)
In all the three examples, the number of outcomes from the experiment
are known to be finite. While in most cases it is so but it is not a rule.
The number of outcomes can be infinite as well. For example, it we
consider the experiment of observing the life-time of an electric bulb
in hours, the outcome can be any real, non-negative number. Thus, this
sample space contains an infinite number of sample points.

1.3 Basic Terminology in Probability

1.3.1 Random Experiment


A random experiment is any operation that can be repeated indefinitely
and has a well-defined collection of possible outcomes, whose outcome
cannot be predicted with certainty. Each performance of an experiment
is called trial.

180 PAGE
© Department of Distance & Continuing Education, Campus of Open Learning,
School of Open Learning, University of Delhi
BUSINESS STATISTICS

Examples of random experiment are tossing a coin, rolling a die, drawing Notes
a card from a deck.

1.3.2 Sample Space


The sample space symbolized as ‘S’ is a collection or set of all possible
outcomes of a random experiment. Each element of a sample space is
called a sample point. For example – The sample space of rolling a die
is S = {1, 2, 3, 4, 5, 6}, here rolling a die is a trial of the experiment
and this sample space has total six sample points.
Note: In set theory, the sample space associated with an experiment serves
the same purpose as the universal set.
Example 1: Determine the sample space if two coins are tossed
simultaneously.
Solution : Let H denotes Head and T denote Tail, the possible outcomes
are:
Coin 1 Coin 2
H H
H T
T H
T T
Thus, the sample space is S = {HH, HT, TH, TT}
Example 2 : Determine the sample space if one coin is tossed twice in
succession.
Solution : Tossing a coin twice in succession gives the same sample
space as the tossing of two coins simultaneously.
Note this experiment will hold true for tossing a coin n number of times.

1.3.3 Event
An event refers to any set of possible outcomes in a sample space. If
the sample space for an event has the elements S1, S2, S3,...Sn, an event
in the sample space S would be any one, or collection of S1, S2, S3;...Sn.
In a sample space, every combination of sample points may be defined

PAGE 181
© Department of Distance & Continuing Education, Campus of Open Learning,
School of Open Learning, University of Delhi
B.COM. (PROGRAMME)

Notes as an event. In an experiment of counting defective items in a sample,


the set of all possible outcomes having less than 10 defective items can
be represented by the event A.
Each sample point does not have to identify a separate event. The faces
of a die provide a sample space of six outcomes. If the occurrence of
each face identifies a different event, there are six possible events. On
the other hand, suppose that an even number represents a gain of Rs.
100 to a person X and an odd number represents a loss of Rs. 100 to the
person Y. In this case, there are six outcomes but only two events – gain
of Rs. 100 to X, and loss of Rs. 100 to Y.
Every event A is a subset of the sample space and every event is a
collection of the elements of a sample space. Events can be classified
as being elementary or compound. Elementary events are said to be
those which have a single sample point whereas compound events are
those which contain more than one sample point. Thus, whereas the
compound events can be decomposed, the elementary events cannot be.
The appearance of 3 on a die is an elementary event while the appearance
of an even number on the die is a compound event (as it contains three
sample points).
Now, keeping in mind the definitions of experiment, sample space and
events, we introduce some more concepts.
1.3.3.1 Simple Event
A simple event is a subset of sample space consisting of a single sample
point. In other words, the single outcome of a random experiment is called
a simple event. For example – the event of getting {4} from rolling a
die, getting {H} from tossing a coin, and drawing {Ace of Spade} from
a deck of 52 cards.
1.3.3.2 Compound Event
A compound event is more complex than a simple event since it involves
the possibility of several outcomes. Compound events can also be thought
of as a composite of two or more simple occurrences. A compound event
is a subset of sample space consisting of more than one sample point.
For example – the event {1,3,5} from rolling a die, getting {face cards}
from a deck of 52 cards.

182 PAGE
© Department of Distance & Continuing Education, Campus of Open Learning,
School of Open Learning, University of Delhi
BUSINESS STATISTICS

1.3.3.3 Mutually Exclusive and Overlapping Events Notes


Two events are mutually exclusive if the occurrence of one event precludes
the occurrence of the other. For example, the events that (i) an employee
would be late, and (ii) the employee would be absent, on a particular
day, are mutually exclusive since both cannot occur simultaneously. An
employee cannot be both late and absent on a particular day. Similarly,
suppose we consider a box in which 20 cards, marked 1 to 20 are placed
and a card is drawn at random. If A be the event that the number on
the card is divisible by 3 and B be the event that the number would be
divisible by 7, then the events A and B are mutually exclusive. This is
because for A to occur, the number would be one of 3, 6, 9, 12, 15
or 18, and B to occur, it should be one of 7 or 14. Since no number is
common to them, they are mutually exclusive.
On the other hand, two or more events which are not mutually exclusive
are called overlapping events. In the above example of cards, suppose A
represents the event that the number on the card chosen is divisible by
3 and B represents the event that the number is divisible by 5, then for
A to occur the number must be either 3, 6, 9, 12, 15 or 18, and for B
to occur, it must be one of 5, 10, 15 and 20. Note that if the number 15
is obtained, it implies that both A and B have taken place. Thus A and
B are not mutually exclusive.
We can use Venn diagram to depict mutually exclusive and overlapping
events. This is shown in figure 2. Part (a) of the figure shows the mutually
exclusive events A and B, each of them defined over the sample space
S. Note that A and B have no sample points in common. On the other
hand part (b) of the figure shows overlapping events A and B, as they
have some common sample points.

PAGE 183
© Department of Distance & Continuing Education, Campus of Open Learning,
School of Open Learning, University of Delhi
B.COM. (PROGRAMME)

Notes 1.3.3.4 Complementary Events


Events are said to be complementary when the sample space is partitioned
into the segment that represents the occurrence of an event A, and the
segment that is not a part of A. Thus, the complement of an event A is
the collection of all possible outcomes that are not contained in event
A. For example, in the toss of a coin, appearance of head and tail
are complementary to each other. Complementary events are shown in
figure 3. Here A and are complementary to each other. The events of a
person being able to hit a target, and not being able to hit the target are
complementary, and so are the events of the appearance of a head and
a tail on tossing a coin.
1.3.3.5 Independent and Dependent Events
Two events are said to be independent if the occurrence of one event in
no way influences the occurrence of the other event. For example, we
toss a six-faced die and call the event of appearance of an even number
as the event A and the appearance of an odd number as the event B.
Now, suppose that in the first toss we get an even number. If we toss
the die the second time, we can still get an even or an odd number and
their chances are not influenced by the result of the first trial. Thus, the
appearance of an even number in the first trial and the appearance of
an even number in the second trial is an example of independent events.
Similarly, if we pick a card at random from a deck of playing cards,
note its suit and put it back and then draw one card from the deck, then
the chances of a king card, for example, in the second trial is not at all
affected by the card we had drawn at the first trial. But if we take out
a card and do not replace it back, then the chances of drawing a king
card in the second trial are certainly affected by the card we had drawn
in the first trial. If it were a king card the first time, then only 3 king
cards remain in the 51 cards while if a non-king card was drawn then
we would have 4 king cards in the lot of 51 cards. So the chances of a
king card in the second trial are dependent upon the results of the first
trial. This time, the events of a king card in the first trial and king card
in the second trial are not independent because the outcome in one trial
is in some way influenced by the outcome of the previous trial.

184 PAGE
© Department of Distance & Continuing Education, Campus of Open Learning,
School of Open Learning, University of Delhi
BUSINESS STATISTICS

1.4 Methods of Assigning Probability Notes

There are three basic ways of assigning probability to an event. They are :
(i) Classical approach,
(ii) Relative frequency approach, and
(iii) Subjective approach.

1.4.1 Classical Approach


Classical probability is the oldest and simplest form of probability that
has equal odds of something happening. It originated with the games of
chance. According to this theory, if there are n outcomes of an experiment
which are mutually exclusive and equally likely to occur (all outcomes
having an equal chance to occur), then the probability of each sample
point is 1/n. Thus, if a fair die is rolled, each of six numbers 1, 2,... 6,
is equally likely to occur and the probability that a given number, say 5,
would occur is 1/6. From this, the classical interpretation of probability
is: if the sample space of an experiment has n(S) equally likely, mutually
exclusive outcomes and if an event A, defined on this sample space has
n(A) sample points, then the probability that event A would occur is the
ratio of n(A) to n(S).
n(A)
Thus, P(A) =
n(S)
Note: ‘n’ denotes the number of sample point.
To illustrate, we consider the following examples.
Example 3 : A six-faced die is tossed once. Find the probability that
the number obtained on tossing is (i) and odd number, (ii) a number
greater than 2.
Solution : Let A : the event that the number is an odd number, and
B : the event that the number is greater than 2.
From the given information, n(S) = 6 (as there are six possible outcomes)
n(A) = 3 (being numbers 1, 3 and 5), and
n(B) = 4 (being numbers 3, 4, 5 and 6)

PAGE 185
© Department of Distance & Continuing Education, Campus of Open Learning,
School of Open Learning, University of Delhi
B.COM. (PROGRAMME)

Notes n(A) 3 1
? P(A) = = = ; and
n(S) 6 2

n(A) 4 2
P(B) = = =
n(S) 6 3
Example 4 : A card is drawn from a deck of playing cards at random.
Find the chance that (i) it is a face card, (ii) it is a black ace card.
Solution : Let A : the event that the card is a picture card
B : the event that the card is black ace card
we have, n(S) = 52 (there being 52 cards)
n(A) = 12 (there being 4J, 4Q and 4K cards with faces)
n(B) = 2 (there being 2 black aces)

12 3
? P(A) = = , and
52 13
2 1
P(B) = =
52 26
Example 5 : Find the probability that a leap year selected at random
shall contain 53 Sundays.
Solution : Like every year, a leap year would have 52 full weeks. The
remaining two days of the years could be:
Sunday and Monday, Monday and Tuesday, Tuesday and Wednesday,
Wednesday and Thursday,
Thursday and Friday, Friday and Saturday, or Saturday and Sunday.
We observe here that n(S) = 7. Since two of the above combinations
have a Sunday included, we haven(A) = 2.
n(A) 2
Therefore, P(A) = = .
n(S) 7
The classical theory, under the assumption of equally likely outcomes,
depends on logical reasoning. It does very well when we are concerned
with balanced coins, perfect dice, wellshuffled pack of cards and all
those situations where all outcomes are equally likely. However, problems
are immediately encountered when we have to deal with the unbalanced

186 PAGE
© Department of Distance & Continuing Education, Campus of Open Learning,
School of Open Learning, University of Delhi
BUSINESS STATISTICS

coins, loaded dice and so on. In such situations, we have to depend on Notes
the relative frequency approach.
Limitations of Classical Approach:
1. The classical probability cannot be applied in situations in which
the possibilities that arise cannot be regarded as equally likely.
Thus, it has serious problem while dealing with decision-making
problems in management.
2. It fails to calculate the probability while dealing with infinite sample
space.
3. It makes assumptions about scenarios that are extremely unlikely but
may theoretically occur such as coin landing on edge.

1.4.2 Relative Frequency Approach


The origin of this approach dates back to 1800s when the statistical
data of birth and death were collected to assess the risk of losses in life
insurance. This approach uses the relative frequencies of past occurrence
of an event as probabilities and uses the same to predict the probability of
similar future events. It is based on the actual observation. For example,
if we were interested in the probability of 50 or fewer customers arriving
at a super market before 10 a.m. we would pick a trial number of days
(n) and count how often 50 or fewer actually did arrive before 10 a.m.
Our probability assessment would be the ratio of days when 50 or fewer
customers arrived to the total number of days observed. Similarly, to have
an idea of the probability that a head would appear on tossing a coin,
we may actually toss the coin a number of times, say 1000, and find the
number of times a head appears. If 536 times a head has appeared, then
the probability of head to occur will be taken to be the ratio of the two :
536/1000. Naturally, if the coin is a fair one then the ratio will approach
to 0.5 as we continuously increase the number of trials. And if the coin
is not a fair one then the chances of a head would tend to approach the
true probability of the head occurring on this coin depending upon how
biased is the coin.
Formally, the probability assessment for event A using the relative
frequency approach is given by :

PAGE 187
© Department of Distance & Continuing Education, Campus of Open Learning,
School of Open Learning, University of Delhi
B.COM. (PROGRAMME)

Notes Number of times A occurs


P(A) =
Number of trials, n
It can be easily visualised that when the number of trials increases, we get
better and better estimate of the true probability of the event in question.
Both the classical and the relative frequency approaches to probability
are objective in nature. The classical definition is objective in the sense
that it is based on deduction from a set of assumptions while the relative
frequency approach is objective because the probability is derived from
repeated empirical observations. However, both the theories fail when we
are dealing with unique events. For example to determine the probability
that a certain student will succeed in a particular examination, we can
apply none of the two. This is because it cannot be ruled that for every
student the events of succeeding and not succeeding are equally likely.
Similarly, we cannot subject the candidate to appear in the examination
several times to estimate his probability of success. In such cases, we
have the personalistic approach to probability.
Limitations of Relative Frequency Approach
1. The number of trials requires to be large enough to get reliable
probability.
2. The experiment conditions may not remain homogenous.
3. The ratio m/n cannot attain a unique value irrespective of number
of trials.

1.4.3 Personalistic Approach


The approach views that the probability of an event is a measure of the
degree of belief that an investigator has in the happening of it. It grants
that the probability of the same event may be assigned differently by
different investigators according to the confidence each one has in its
happening. In fact, subjective probability can be defined as the probability
assigned to an event based on readily available evidence. For example,
an investigator perceives 80 per cent chance of a candidate succeeding
in an examination, while another might estimate the chances to be 95
per cent. Thus, whereas the chances of a candidate succeeding in an

188 PAGE
© Department of Distance & Continuing Education, Campus of Open Learning,
School of Open Learning, University of Delhi
BUSINESS STATISTICS

examination may be placed at 80 per cent by one person, another might Notes
estimate the chances to be 95 per cent. Accordingly, the two would assign
a probability 0.80 and 0.95 respectively for the event to happen.
In may be mentioned that the three approaches to probability definitions
are not competitive rather they are complementary in nature.

1.5 Computation of Probability


As we have already seen, the probability of an event is defined as the
ratio of the number of favourable outcomes (for the event) to the total
number of possible outcomes. Little difficulty is experienced when the
total and favourable outcomes are small in number but when they are
large, we may require the use of counting techniques to identify their
number. Therefore, we first state the method of obtaining permutations
and combinations.
In the study of probability, having an adequate understanding of permutation
and combination is one of the prerequisites. Many problems in problems
can be simplified by using Permutation and Combination concepts.
Permutations: The term permutation refers to a mathematical computation
of the number of possible arrangements of a given collection. Simply
said, a permutation is a term that defines the number of different ways
something can be ordered or arranged. For example, there are 5 chairs and
3 persons are to be seated. The first person can be seated in 5 different
ways; once a seat is taken by the first person the second person can take
in 4 different ways and similarly third person can take seat in 3 different
ways. Thus, to find the number of ways for arranging 3 persons in 5
chairs, we multiply the options available to us. We do it in 5 × 4 × 3
ways, i.e., it can be done in 60 ways.
When we generalize this, we obtain n possibilities for the first chair, n-1
options for the second, and n-2 options for the third chair. Thus, the total
number of permutations (arrangements) of r individuals in n seats is:
n
Pr = n! / (n - r)!
Example 6: How many words can be formed using the letters of the
word GOAL?

PAGE 189
© Department of Distance & Continuing Education, Campus of Open Learning,
School of Open Learning, University of Delhi
B.COM. (PROGRAMME)

Notes Solution: There are total 4 distinct letters in the given word. Thus, n is 4
All 4 letters are to be taken to make all possible arrangement. Thus, r = 4
4! 4!
Therefore, nPr = 4P4 = = = 4! {0! is always 1}
4!-4! 0!

The word GOAL can be arranged or formed using all letters of GOAL
in 4! ways or 24 ways.
Example 7: How many words can be formed using the letters of the
word GOAL starting with G and ending with L?
Solution: If we fix two letters G and L at the beginning and at the end
respectively, then we are left with only two distinct letters with two
empty spaces as shown below:
G _ _ L
Therefore remaining 2 letters can be arranged or formed in 2P2 ways or
2! ways.
Example 8: How many words can be formed using the letters of the
word MATHEMATICS?
Solution: Here, we need to make some adjustments in our solution because
all the letters of the given word is not unique such as the letters M, A,
and T are being repeated twice.
Hence, The required number of arrangements can be calculated by dividing
the nPr by the number of times each letter is repeated,
11
Required number of arrangements = P11 ÷ 2!×2!×2!
Here first 2! is used for M being repeated twice, second 2! for A being
repeated twice and third 2! for T being repeated twice.
Combinations: Combination is a process of selection of elements from
a set of elements in which (unlike permutations) the order of selection
does not matter. Combination is a special type of permutation selection
in which the order does not matter. So, the number of permutations is
always greater than the number of combinations for selecting ‘r’ elements
out of ‘n’ elements. It is feasible to count the number of combinations in
smaller circumstances, but the potential of a set of combinations increases
with the number of groups of elements or sets. As a result, a formula has

190 PAGE
© Department of Distance & Continuing Education, Campus of Open Learning,
School of Open Learning, University of Delhi
BUSINESS STATISTICS

been developed to determine the number of things that can be selected. Notes
Thus, A combination is created by selecting r items from a group of n
items without replacing them and without regard to their order.
n n!
Cr =
r!( n - r )!

Example 9: A committee consisting of 3 executives need to be formed


from 10 available executives. How many ways the committee can be
formed?
Solution: The number of possible combinations are.
n 10 10! 10! 10*9*8
Cr = C3 = = = = 120
3!(10-3)! 3!7! 3*2*1

Example 10: A team of 2 players is to be formed from 2 available


players. In how many ways the team can be formed?
Solution: Number of players available is 2 Thus, n = 2
Team consisting of number of players is 2 Thus, r = 2
n 2! 2! 2!
C r = 2C 2 = = = = 1
2!( 2-2)! 2!0! 2!

(1) If a job can be done in m ways and another job can be done in n
ways, then the total number of ways in which both of them can be done
is m × n. This is the fundamental multiplication rule.
Example 11 : A man can go from city A to city B by three routes and
come back by any of four routes, in how many ways can he perform his
to and fro journey.
Solution : He can perform the journey in a total of 3 × 4 = 12 different
ways.
Example 12 : Three balanced dice are tossed. Find the chance that the
sum of digits on the two would be equal to 10.
Solution : Total number of ways in which three dice can fall = 6 × 6 ×
6 = 216. Total number of ways in which a total of 10 can appear = 27
(as shown below)
(1, 3, 6), (1, 4, 5), (1, 5, 4), (1, 6, 3), (2, 2, 6), (2, 3, 5), (2, 4, 4),
(2, 5, 3), (2, 6, 2), (3, 1, 6), (3, 2, 5), (3, 3, 4), (3, 4, 3), (3, 5, 2),

PAGE 191
© Department of Distance & Continuing Education, Campus of Open Learning,
School of Open Learning, University of Delhi
B.COM. (PROGRAMME)

Notes (3, 6, 1), (4, 1, 5), (4, 2, 4), (4, 3, 3), (4, 4, 2), (4, 5, 1), (5, 1, 4),
(5, 2, 3), (5, 3, 2), (5, 4, 1), (6, 1, 3), (6, 2, 2), (6, 3, 1)
27 1
Accordingly, P (total of 10) = =
216 8

(2) The total number of arrangements of n distinct objects considered


all at a time is equal to n!
Thus, nPn = n !
Example 13 : In how many ways can the letters in the word DELHI be
arranged ?
Solution : Since all the 5 letters are different, they can be arranged in
5! = 120 ways.
(3) The total number of arrangements of n distinct objects taken r at a
time equals.
n!
n
Pr =
(n − r )

Example 14 : A car dealer has 4 places in his showroom. It has just


received a consignment of 10 cars of different shades. In how many ways
can he arrange cars in the showroom ?
Solution :

nP 10! 10 × 9 × 8 × 7 × 6!
r = 10
P4 = = = 5040
(10 − 40)! 6!
(4) If out of n objects, k1 are alike, k2 are alike, k3 are alike....and so on
such that k1 + k2 + k3 + ......... = n, the number of arrangements of the
n objects would be equal to:
n n!
Pk1, k2 , k3 .... =
k1 !k2 !k3 !,....

Example 15 : In how many ways can the “letters in the word STATISTICS
be arranged ?
Solution : Here n = 10, k1 (S) = 3, k2(T) = 3, k3(I) = 2, k4(A) = 1 and
k5(C) = 1

192 PAGE
© Department of Distance & Continuing Education, Campus of Open Learning,
School of Open Learning, University of Delhi
BUSINESS STATISTICS

Accordingly Notes
10P 10!
3, 3, 2, 1, 1 = 3!3!2!1!1! = 50400.

(5) Out of a total of n distinct objects, the number of combinations of r


objects can be obtained as follows:

nC = n!
r or n !/ (n − r )!r !
(n − n)!r !
Example 16 : In how many ways can a committee of 3 persons be chosen
out of a total of 10 persons?
Solution : Here n = 10 and r = 3. The total number of committees would
be :

nC = 10! 10 × 9 × 8 × 7!
r
10
C3 = = = 120
(10 − 3)!3! 7! × 3 × 2
Example 17 : A committee of four is to be selected randomly out of a
total of 10 executives, 3 of which are chartered accountants. Find the
probability that the committee would include exactly 2 CAs.
Solution : The committee of 4 executives can be selected out of a total
of 10 executives in 10C4 ways. The number of ways in which 2C As can
be selected out of 3 is equal to 3C2 while the number of ways in which
2 executives out of a total of 7 executives is equal to 7C2.
3
C2 × 7 C2 63
? P (committee includes exactly 2 CAs) = 10
=
C4 210
Example 18 : Two cards are drawn at random from a well-shuffled deck
of cards. Find the probability that both are ace cards.
Solution : No. of ways in which 2 cards can be selected out of 52 cards
52 × 51× 50!
= 52
C2 = = 1326
50!2
No. of ways in which 2 aces can be selected out of 4 ace cards =
4!
4
C2 = =6
2!2!

PAGE 193
© Department of Distance & Continuing Education, Campus of Open Learning,
School of Open Learning, University of Delhi
B.COM. (PROGRAMME)

Notes 6 1
? P(2 ace cards) = =
1326 221

1.6 Laws of Probability


The probability associated with any event represents the likelihood of that
event occurring on a particular trial of an experiment. This probability
also measures the perceived uncertainty about whether the event will
occur. If we are not uncertain at all, we assign the event a probability of
zero or one. If the event be A, then P(A) = 0 means that event A would
not occur, while P(A) = 1 indicates that event A would definitely occur.
Thus, for any event, the probability would range between zero and one.
Probability is non-negative concept. Symbolically,
Rule 1 : 0 d P (A) d 1
Example 19 : (i) Determine the probability that a 7 would appear on a
six-faced die tossed once, (ii) Determine the probability that an even or
an odd number would appear on tossing a die.
Solution :
0
(i) P(7) = = 0 (since a 7 does not exist, therefore there is no question
6
of its occurrence)
6
(ii) P(even or odd number) = = 1 (since the number appearing must
6
be an even or an odd one)
Rule 2 : The probability of the complement of event A is one minus the
probability of event A.
Symbolically,
P(A) = 1–P(A)
To be able to hit and not be able to hit a target for example, are
complementary events. If the probability of a person to hit a target is
given to be 3/5, then the probability that he would not be able to hit the
target would be :

194 PAGE
© Department of Distance & Continuing Education, Campus of Open Learning,
School of Open Learning, University of Delhi
BUSINESS STATISTICS

3 2 Notes
P(not hitting the target) = 1 − =
5 5

1.6.1 Addition Rule


When making a decision involving probabilities, we often need to combine
event probabilities with some event of interest. Here we first consider
the calculation of probability that event A or B, each of them being
defined on the sample space would occur. We use the addition rules of
probability for this purpose.
Rule 3 : When the events are mutually exclusive, the probability of
occurrence of either of them is given by the sum of their individual
probabilities. For two events A and B which are mutually exclusive,
P(A or B) = P(A) + P (B)
Alternatively, P(A ‰ B) = P(A) + P(B)
where (A ‰B) reads A union B and means A or B. Thus, for the mutually
exclusive events, the probability that either one of them would occur is
given by the sum of their individual probabilities. This rule is known as
the special rule of addition. In general terms, the rule is:
P(A ‰ B ‰ C ‰...K) = P(A) + P(B) + P(C) + ...+ P(K)
Example 20 : A box contains 20 discs numbered 1 to 20. A disc is
selected at random. Find the probability that the number on the disc is
divisible by 5 or 7.
Solution : Let A be the event that the number is divisible by 5, and B
be the event that the number is divisible by 7. Since there is no number
which is common in these, the events A and B are mutually exclusive.
Accordingly.
4 2 4 2 6 3
P(A) = , P(B) = and P(A ‰ B) = + = =
20 20 20 20 20 10
Rule 4 : When the events are overlapping : when two events A and B are
overlapping, then the probability that either A or B or both of them would
occur is given by the sum of individual probabilities of events A and B
to occur minus the probability of their joint occurrence. Symbolically,

PAGE 195
© Department of Distance & Continuing Education, Campus of Open Learning,
School of Open Learning, University of Delhi
B.COM. (PROGRAMME)

Notes P(A or B or Both) = P(A) + P(B) – P(A and B)


Alternatively, P (A ‰ B) = P(A) + P(B) – P(A ˆ B)
When they are three overlapping events A, B, and C, we have,
P(A ‰ B ‰ C) = P(A) + P(B) + P(C) – P(A ˆ B) – P(A ˆ C) –
P(B ˆ C) + P(A ˆ B ˆ C)
Example 21 : A box contains 20 discs numbered 1 through 20. A disc is
selected at random. Find the chance that its number is divisible by 3 or 5.
Solution : Let A be the event that the number is divisible by 3, and B
the event that the number is divisible by 5.
Here six numbers 3, 6, 9, 12, 15 and 18 are divisible by 3 and four
numbers 5, 10, 15 and 20 are divisible by 5. We notice that the number
15 is included in both the lists. Thus, we have,
6 4 1
P(A) = , P(B) = , and P(A ˆ B) =
20 20 20
6 4 1 9
Accordingly, P(A ‰ B) = + − =
20 20 20 20
Example 22 : A survey conducted to know the smoking habits of 500
persons yielded the following results :
Cigarette Brand No. of smokers
A 140
B 175
C 100
A and B 45
A and C 38
B and C 44
A and B and C 18
Find the probability that a person selected at random from the above
group would be:
(i) a smoker of brand A or B,
(ii) a smoker of A or B or C.
(iii) a non-smoker.

196 PAGE
© Department of Distance & Continuing Education, Campus of Open Learning,
School of Open Learning, University of Delhi
BUSINESS STATISTICS

140 175 Notes


Solution: From the given information, P(A) = , P(B) = , P(C)
500 500
100 45 38 44
= , P(A ˆ B) = , P(A ˆ C) = , P(B ˆ C) = , and
500 500 500 500
18
P(A ˆ B ˆ C) =
500
Accordingly,
140 175 45 270
(i) P(A ‰ B) = P(A) + P(B) – P(A ˆB) = + − = = 0.54
500 500 500 500
(ii) P(A ‰ B ‰ C) = P(A) + P(B) + P(C) – P(A ˆ B) – P(A ˆ C) –
P(B ˆ C) + P(A ˆ B ˆ C)
140 175 100 45 38 44 18 306
= + + − − − + = = 0.612
500 500 500 500 500 500 500 500
306 194
(iii) P (non-smoker) = 1 – P (smoker) = 1 − = = 0.388
500 500

1.6.2 Conditional Probability


In dealing with probability we often need to determine the chances of two
or more events occurring either at the same time or in succession. For
example, a quality control manager for a manufacturing company may be
interested in the probability of selecting two successive defectives from
an assembly line. In other instances, the decision maker may know that
an event has occurred and may want to know the chances of a second
event occurring. For example, the market research organization engaged
by a company may give a favourable report for a high sales figure for a
new product to be introduced by the company. The company managing
director might well be interested to know the probability of making high
sales given a favourable report.
These situations require tools different from those presented above in
context of addition rules. Specifically we need to understand the rules for
conditional probability and multiplication of probabilities. To understand,
suppose that the employees of an organization are cross-classified according
to sex and rank as follows :

PAGE 197
© Department of Distance & Continuing Education, Campus of Open Learning,
School of Open Learning, University of Delhi
B.COM. (PROGRAMME)

Notes Officer Clerk Total


Males 400 300 700
Females 200 1000 300
Total 600 400 1000
If an employee is selected at random then the probability that the employee
would be a male = 700/1000. since out of the total of 1000 employees, a
total of 700 are males. This probability is unconditional in the sense that
we are not given any information about the type of employee selected.
Now, if it is given that an employee is selected at random and he is
an officer, then the probability that he would be a male shall be equal
to 400/600, because the focus would be only on the officer employees
which are 600 in all and of which there are 400 who are males. This
probability is conditional. If we let the event A to represent the event that
the employee would be a male, event A to represent that an employee
would be an officer, we can write the conditional probability as :
400 2
P(A/B) = =
600 3
Where (A/B) reads as event A given that event B has occurred. Upon a
closer look we can represent the probability as:
P(A ∩ B)
P(A/B) =
P(B)
In this, P(A ˆ B) represents the probability that both events A and B
would occur and P(B) is the probability for the event B to occur. Thus,
P(A/B) is the conditional probability of A given B and is defined provided
P(B) > 0.
For the above example, P(A ˆ B) = 400/1000 and P(B) = 600/1000. As
such,
400 /1000 400 2
P(A/B) = = =
600 /1000 600 3
Example 23 : Consider an experiment in which two successive draws are
to be made from an urn containing three white balls and five black balls.
Assume that the balls are drawn at random and that the ball chosen on
the first draw is not replaced. Find the probability that (i) the first ball
drawn is white, and (ii) the second one is black.

198 PAGE
© Department of Distance & Continuing Education, Campus of Open Learning,
School of Open Learning, University of Delhi
BUSINESS STATISTICS

Solution : (i) Let A be the event that the first ball drawn is white and Notes
B be the event that the second ball drawn is black. From the given
3
information, P(A) = , since there are three white balls in a total of
8
eight balls. (ii) To determine the conditional probability of B given A,
P (B/A), which is the probability of drawing a black ball on the second
draw after drawing a white ball for the first draw, it should be noted that
if A has already occurred, then there is a total of seven balls remaining
5
and five of them are black. Thus, P(B/A) = .
7

1.6.3 Multiplication Rule


From the conditional probability defined in the preceding paragraphs, since
P(A ∩ B)
P(A/B) =
P(B)

Rule 5 : P(A ˆ B) = P (B) × P (A/B)


Also P(A ˆ B) = P (A) × P (B/A)
This is called the multiplication rule for the non-independent events A and
B, and states that the joint probability of the events A and B is given by
the probability of the event A multiplied by the probability for event B
given that event A has occurred (or the probability of event B, multiplied
by the probability for event A given that event B has occurred).
Similarly, for events A, B, and C which are not independent, we have
P(A ˆ B ˆ C) = P (A) × P (B/A) × P (C/A ˆ B)
Example 24 : Two balls are selected one after the other from an urn
containing 7 black and 8 green balls. The first ball is not replaced before
the second one is drawn. Find the probability that both would be green.
Solution : Let A be the event that the first ball drawn is green and B be
the event that the second ball drawn is green. From the given information.
8 7
P(A) = , P(B/A) = (if a green ball is taken out there would
15 14
be 7 green balls in a total of 14 balls)

PAGE 199
© Department of Distance & Continuing Education, Campus of Open Learning,
School of Open Learning, University of Delhi
B.COM. (PROGRAMME)

Notes 8 7 4
? P(A ˆ B) = P (A) × P(B/A) = × =
15 14 15
Rule 6 : If the events A and B are independent, the probability that both
events occur can be determined by using P(A) and P(B). As mentioned
earlier, two events are independent if the occurrence of one has no
effect upon the occurrence of the other. More formally, if A and B are
independent,
P(A/B) = P(A), and P(B/A) = P(B).
If A and B are independent, the conditional probability of A, given B,
is the same as P(A), since the occurrence of the event B does not affect
the occurrence of the event B; P(A/B) = P(A).
The joint probability of independent events may be seen as the product
of the probabilities of the events A and B, since:
P(A ∩ B)
P(A/B) = = P(A) and P(A ˆ B) = P(A) × P(B)
P(B)
To generalize, for independent events A, B, C ... we have
P(A ˆ B ˆ C) = P(A) × P(B) × P (C) × ....
Example 25 : Two balls are selected one after the other from an urn
containing 7 black and 8 green balls. The first ball is replaced before
the second one is drawn. Find the probability that both would be green.
Solution : Let A and B be the events that the first and the second ball,
respectively, would be green.
From the given information,
8 8
P(A) = and P(B) =
15 15
Accordingly,
8 8 64
P(A ˆ B) = P(A) × P(B) = × =
15 15 225
If it is significant to note that the condition P(A ˆ B) = P(A) × P(B)
is satisfied then the events A and B are said to be independent, just as
when they are independent then this relation is satisfied. This condition

200 PAGE
© Department of Distance & Continuing Education, Campus of Open Learning,
School of Open Learning, University of Delhi
BUSINESS STATISTICS

can be employed to determine whether the given events A and B are Notes
independent.
Example 26 : For the data given in example, test whether the events of
an employee selected being male and an employee selected being clerk
are independent.
Solution : Let A be the event that an employee selected is male and B
be the event that an employee selected would be a clerk. From the given
information,
the number of employees who are males = 700
the number of employees who are clerks = 400
the number of employees who are males and clerks = 300
Accordingly
700 400 300
P(A) = , P(B) = , and P(A ˆ B) =
1000 1000 1000
700 400 300
Here since × ≠ , therefore the events A and B are
1000 1000 1000
not independent.
Now we shall discuss the theorem of total probability, also called as the
theorem of elimination.
Rule 7 : If H1, H2, ....Hn be n mutually exclusive events, each with a
non-zero probability, and E be an event defined on the same sample
space and can be associated with either of them, the total probability of
event E to occur is given by : P(E) = P(H1) × P(E/H1) + P(H2) × P(E/
H2) +....+ P(Hn) × P(E/Hn).
Alternatively,
n
P(E) = ∑ [P(Hi ) × P(E/Hi )]
i=1

In this formulation, of course, P(Hi) and P(E/Hi) must all be given.


To illustrate the application of this theorem, consider the following example.
Example 27 : Two sets of candidates are competing for the positions
of board of directors of a company. The chances for the first set to win
are 60% while the chances for the second set are 40%. If the first set

PAGE 201
© Department of Distance & Continuing Education, Campus of Open Learning,
School of Open Learning, University of Delhi
B.COM. (PROGRAMME)

Notes wins, the probability that a product will be introduced is 0.80 while if
the second set wins, the probability for the product to be introduced is
0.30. Determine the probability that the product will be introduced.
Solution : If H1 is the event that the first set wins,
H2 is the event that the second set wins, and
E is the event that the product is introduced, then
P(H1) = 0.60, P(E/H1) = 0.80, P(H2) = 0.40, P(E/H2) = 0.30
Accordingly,
P(E) = P(H1) × P(E/H1) + P(H2) × P(E/H2)
= 0.6 × 0.8 + 0.4 × 0.3 = 0.48 + 0.12 = 0.60
Thus, there is a sixty per cent chance that the product shall be introduced.

1.7 Bayes’ Theorem


Conditional probabilities provide a lot of good information for decision
makers. For instance, say the medical researchers are interested in
determining the probability of getting cancer by a person supposing he was
exposed to hazardous chemicals. That is, P(cancer/hazardous chemicals).
In such cases we use the conditional probability rule
P(A ∩ B)
P(A/B) =
P(B)

However, in many practical applications, decision makers may know that


an event has occurred but do not know what the chances were of that
event before the fact. This cannot be known by the use of conditional
probability rule directly. In such cases we employ an extension of
conditional probability called Bayes’ Theorem. This theorem deals with
the conditional probability of an event Hi, given the probability of E,
where E may have elements in each of the events H1, H2, H3, ....Hn with
no element of E in more than one Hi. The Venn diagram of the figure
displays such a condition.

202 PAGE
© Department of Distance & Continuing Education, Campus of Open Learning,
School of Open Learning, University of Delhi
BUSINESS STATISTICS

Notes
H2

H1

H3
H4

Bayes’ Theorem
Figure 4
We shall illustrate the concept with an example and then make a
generalization.
Example 28 : Box 1 contains 5 white balls and 3 red balls. Box 2
contains 4 white balls and 4 red balls. A box is selected at random and
one ball is randomly taken from that box. If the ball is white, what is
the probability that it came from box 1 ? box 2 ?
Solution : Let H1 : the box 1 is selected,
H2 : the box 2 is selected, and
E : the ball is white.
1 1
From the given information, P(H1) = , P(H2) = , P(E/H1) =
2 2
5 4
, and P(E/H2) = .
8 8
Here we wish to calculate P(H1/E) P(H2/E)
From the theorem of conditional probability,

P ( H E ) = P(HP(E)∩ E)
1 1 and P H 2 ( E ) = P(HP(E)∩ E)
2

P(H1ˆ E) is the probability of first selecting box 1 and then selecting


one white ball from it.
1 5 5
P(H1ˆ E) = P(H1) × P(E/H1) = × =
2 8 16

PAGE 203
© Department of Distance & Continuing Education, Campus of Open Learning,
School of Open Learning, University of Delhi
B.COM. (PROGRAMME)

Notes P(H2ˆ E) is the probability of first selecting box 2 and then selecting
one white ball from it.
1 4 4
P(H2ˆ E) = P(H2) × P(E/H2) = × =
2 8 16
Since the ball selected can be from box 1 or box 2, we have,
P(E) = P(H1ˆE) + P(H2 ˆ E)
= P(H1) × P(E/H1) + P(H2) × P(E/H2)

⎛1 5⎞ ⎛1 4⎞ 5 4 9
= ⎜ × ⎟+⎜ × ⎟ = + = .
⎝ 2 8 ⎠ ⎝ 2 8 ⎠ 16 16 16
Accordingly,

( E)
5
H1 P(H1 ∩ E) P(H1 ∩ E)
P = = = 16 = 5
P(E) P(H1 ∩ E) + P(H 2 ∩ E) 9 9
16

( )
4
H2 P(H 2 ∩ E) 4
Also, P = = 16 =
E P(E) 9 9
16
Notice here that naturally either box 1 or box 2 would have been
selected. When no information about the colour of the ball is known,
the probability that box 1 is selected is 1/2 and so is the probability
that box 2 is selected. Thus, P(H1) = 1/2 and P(H2) = 1/2 are the prior
probabilities. Having known later on that the ball selected is of the
white colour, we have revised these probabilities of P(H1/E) = 5/9 and
P(H2/E) = 4/9. These probabilities are known as posterior probabilities.
Thus the prior probabilities are transformed into posterior probabilities
by incorporating the additional information, with the help of conditional
and joint probabilities. The information in the above stated example can
be restated as follows :
Event Prior Conditional Joint Prob. Posterior
Prob. Prob. Prob.
(Hi) P(Hi) P(E/Hi) P(Hiˆ E) P(Hi/E)
H1 1/2 5/8 5/16 5/9
H2 1/2 4/8 4/16 4/9
Total, P(E) = 9/16

204 PAGE
© Department of Distance & Continuing Education, Campus of Open Learning,
School of Open Learning, University of Delhi
BUSINESS STATISTICS

We can formally state the Bayes’ theorem now as follows : If H1, H2, Notes
...Hn be mutually exclusive and collectively exhaustive events and E be
an event which is arbitrarily defined on this sample space such that P(E)
> 0, then the Bayes’ Therom states that:

( E) P(Hi ∩ E) n
P
Hi
=
P(E)
where in P (E) ∑ P(Hi ∩ E)
i =1

Example 29 : A company has two suppliers of raw materials used in


making cement. Vendor A supplies 30 per cent of raw materials while
vendor B supplies 70 per cent. Tests have shown the 40 per cent of
vendor A’s materials are poor quality whereas 5 per cent of vendor B’s
materials are poor quality. The cement company’s manager has just found
that there is a poor quality material in inventory. Which company most
probably supplied the material ?
Solution : Let H1 be the event that the material is supplied by vendor A,
H2 be the event that the material is supplied by vendor B,
E be the event that the material is of the poor quality.
Given:
Prior probabilities: P(H1) = 0.30, P(H2) = 0.70
Conditional probabilities: P(E/H1) = 0.40, OP(E/H2) = 0.05.
Joint probabilities: P(H1ˆ E) = P(H1) × P(E/H1) = 0.30 × 0.40 = 0.120
P(H2ˆ E) = P(H2) × P(E/H2) = 0.70 × 0.05 = 0.035
Total probability, P(E) = P(H1ˆ E) + P (H2ˆ E) = 0.120 + 0.035 = 0.155.
Posterior probabilities:

P ( H E ) = P(HP(E)∩ E) = 0.120
1 1
0.155
= 0.77

P ( H E ) = P(HP(E)∩ E) = 0.035
2 2
0.155
= 0.23

Thus, vendor A most likely supplied the poor quality material.

PAGE 205
© Department of Distance & Continuing Education, Campus of Open Learning,
School of Open Learning, University of Delhi
B.COM. (PROGRAMME)

Notes 1.8 Expected Value


Expected value of a discrete random variable is an
A discrete
important concept, which has its origin in gambling
random variable
and to which the probability is applied is the
is a type of
expected value. According to this, if an experiment
random variable
has n outcomes that are assigned the payoffs x1,
that can take
x2,............ xn occurring with probabilities p1, p2,....
on a finite
pn respectively, then the expected value is given by
or countable
μ = E (x) = x p + x p + ............. + xnpn number of
1 1 2 2
or μ = E (x   ™ [ipi distinct values
The expected value of a discrete random variable is no different from the
weightage average of all possible outcome corresponding to its probability
of occurrence. The term expected does not mean exact number of event
to be occurred rather it shows the average of any event to be occurred
in long run.
Properties of Expectation
1. The expected value of any constant is the constant itself. Symbolically,
E (c) = c
2. The expected value of the sum or difference of two random variables
is equal to the sum or difference of their respective expected values.
Symbolically, E (A ± B) = E (A) ± E (B)
3. The expected value of constant times a random variable is the
constant times if its expected value.
Example 30 : A player is engaged in the experiment of rolling a fair die.
The player recovers an amount of rupees equal to the number of dots on
the face that turns up, except when face 5 or 6 turns up in which case
the player will lose Rs. 5 or Rs. 6 respectively. What is the expected
value of the game to the player ?
Solution : From the given information we have :
Outcome: 1 2 3 4 5 6
Probability: 1/6 1/6 1/6 1/6 1/6 1/6
Payoff: 1 2 3 4 –5 –6

206 PAGE
© Department of Distance & Continuing Education, Campus of Open Learning,
School of Open Learning, University of Delhi
BUSINESS STATISTICS

The expected value of the games is : Notes


1 1 1 1 1 1 1
E(x) = 1× + 2 × + 3 × + 4 × − 5 × − 6 × = −
6 6 6 6 6 6 6
Thus, the player would expect to lose on an average Re. 1/6 or 17 p.
on each throw.
Example 31 : An oil company may bid for only one of two contracts
for oil drilling in two different areas A and B. It is estimated that a net
profit of Rs. 4,00,000 would be realized from the first field and Rs.
5,00,000 from the second field. Legal and other costs of bidding for the
first oil field are Rs. 1,02,500 and for the second one are Rs. 1,05,000.
The probability of discovering oil in the first field is 0.60 and in the
second is 0.70. The manager of the company wants to know as to for
which oil field should the manager bid ?
Solution : The expected values for the two contracts are calculated below :
Calculation of Expected Value
Investment Outcome Amount Probability Expected Value
A Success 4,00,000 0.6 2,40,000
Failure (1,02,500) 0.4 (41,000)
Total 1,99,000
B Success 5,00,000 0.7 3,50,000
Failure (1,05,000) 0.3 (31,500)
Total 3,18,500
Example 32 : A box contains 4 white, 8 green and 8 red marbles. A
player selects one marble at random. The player wins Rs. 6 if the marble
he selects is white, Rs. 2 if it is green, but must pay if it is red. How
much should he pay for a red marble if this is to be a fair game?
4
Solution: Probability of selecting a white ball = ,
20
8
Probability of selecting a green ball = ,
20
8
Probability of selecting a red ball = .
20

PAGE 207
© Department of Distance & Continuing Education, Campus of Open Learning,
School of Open Learning, University of Delhi
B.COM. (PROGRAMME)

Notes For a game to be fair, its net pay off must be equal to zero. We have,
Colour of Ball Payoff Probability Expected
Value
White 6 4/20 24/20
Green 2 8/20 16/20
Red –x (suppose) 8/20 –8x/20
0
24 16 8 x 40 8 x
? + − or Ÿ =
20 20 20 20 20
or x = 40/8 = Rs. 5.

1.8.1 Variance of Discrete Random Variable


The variance of a discrete random variable X is a measure of how far
the variable’s values deviate from the expected value. Symbolically,
Var (X) =E [(X - μ)2]
In words, the variance of a random variable is the average of the squared
deviations of the random variable from its mean (expected value).
We know,
ı  ¥9DU ;
The variance of a random variable is a quantity with units squared, whereas
the standard deviation has the same units as the random variable. As a
result, the standard deviation is easier to interpret.
The computation of variance by the above mentioned formula is very
helpful if μ is an integer, otherwise it becomes very cumbersome process.
Therefore, the short cut method of variance can also be used to simplify
the computation,
Var (E) = E (X2) – [E (X)]2
 ™ [L2SL ± ™ [LSL 2

1.9 Summary
Probability is the likelihood that something will happen. When we calculate
the probability of an event, we assign it a number between zero and

208 PAGE
© Department of Distance & Continuing Education, Campus of Open Learning,
School of Open Learning, University of Delhi
BUSINESS STATISTICS

one, depicting how likely it is to happen. There are three approaches to Notes
calculate, probability of an event. These are: (i) classical approach, where
the probability of an event is the ratio of number of favourable outcomes
to the number of total possible outcomes; (ii) relative frequency approach,
where an estimate of probability is given by the ratio of the number of
favourable outcomes to the number of trials made; and (iii) personalistic
approach, where the probability to an event is assigned by an individual
depending on his degree of belief in the occurrence of the event.
There are several theorems of probability, which are used to calculate
probabilities in different situations.
Theorem of Complementary events: This is used to determine the probability
of an event happening by subtracting the probability of the event not
happening from 1.
Theorem of Addition: It deals with the probability of occurrence of
either of the events when they are mutually exclusive or when they are
overlapping. According to this, the probability that either of the events
will happen is equal to the sum of their individual probabilities less the
probability of their joint occurrence.
Theorem of Multiplication: This theorem deals with the calculation of the
probability when our interest is in the occurrence of the events jointly.
For independent events, it uses multiplication of individual probabilities
while for events which are not independent, it uses conditional probability.
A conditional probability, is the likelihood that an event will happen,
given that another event has already happened.
A probability tree provides a useful way of the handling and analysing
conditional probabilities occurring at multiple levels. It represents the
given information through various branches on a set of chance nodes.
Bayes’ Theorem: This theorem provides a method of revising given
probabilities on the basis of additional information. This involves
transforming prior probabilities into posterior probabilities with the help
of conditional and joint probabilities.

PAGE 209
© Department of Distance & Continuing Education, Campus of Open Learning,
School of Open Learning, University of Delhi
B.COM. (PROGRAMME)

Notes 1.10 Self-Assessment Questions

Exercise 1: True or False Statements


(i) The outcomes of an experiment are known as sample space.
(ii) Two events are said to be mutually exclusive if the happening of
one does not affect the probability of happening of the other.
(iii) Mutually exclusive events may or may not be collectively exhaustive.
(iv) Overlapping events are same as non-independent events.
(v) Appearance of a heads and appearance of a tails in single trial of
a coin represent independent events.
(vi) Two mutually exclusive events are not necessarily complementary
events but two complementary events are mutually exclusive.
(vii) Overlapping events are those which can occur severally and jointly.
(viii) The classical approach defines probability of an event as the ratio
of number of favourable outcomes to the total number of trials.
(ix) Personalistic approach can be used to obtain probabilities of unique
events only.
(x) All three approaches to the definition of probability have one thing
in common, the probability is expressed as a ratio not exceeding
1.
(xi) For an experiment involving a toss of three coins, n(S) is equal to
6.
(xii) If P(A/B) = P(B), then A and B are said to be independent.
(xiii) In a toss of a die, the probability of getting a 5 shall be same as
the probability of getting 5 given that the number is odd.
(xiv) In the statistical sense, E 1, and E 2 are independent only when
P(E1ˆE2)= P(E1)× P(E2).
(xv) Bayes’ theorem is used to calculate revised probabilities called
posterior probabilities from prior probabilities, using conditional
and joint probabilities.
(xvi) The sum of posterior probabilities is equal to 1 as is the sum of
prior probabilities, in a given problem.

210 PAGE
© Department of Distance & Continuing Education, Campus of Open Learning,
School of Open Learning, University of Delhi
BUSINESS STATISTICS

Ans. Notes
(i) T (ii) F (iii) T (iv) F (v) F (vi) T
(vii) T (viii) F (ix) F (x) T (xi) F (xii) F
(xiii) F (xiv) T (xv) T (xvi) T
Exercise 2 : Questions and Answers
(i) What is probability? Explain the calculation of probability under
the classical approach.
(ii) Which probability approach would you use to calculate the following
probabilities? Give reasons also:
(a) The next toss of a fair coin will land on heads.
(b) India will win the next match with England.
(c) The sum of the faces of two dice will be eight.
(d) The success of a new product launched in the market.
(iii) “Complementary events are mutually exclusive but mutually exclusive
events may not be complementary.” Discuss with examples.
(iv) Distinguish between mutually exclusive and overlapping events. How
is the theorem of addition applied in both these cases?
(v) Distinguish clearly between mutually exclusive and independent
events. Can two events be mutually exclusive and independent
simultaneously? Do you agree that on tossing a coin once, the
appearance of heads and appearance of tails represent independent
as well as mutually exclusive events?
(vi) In each of the following cases, examine whether events are mutually
exclusive, overlapping, complementary, independent or not-independent:
(a) On a single toss of a die. appearance of 5 or 6 or appearance
of a number smaller than 4.
(b) A bank employee being an assistant manager or being a female.
(c) A claim adjuster in an insurance company being a male or
above 50 years of age.
(d) An employee being a clerk or a sportsman.
(e) A person in a hospital being a heart specialist or over 45
years of age or a lab technician.
(f) A two-shift factory employee working in morning shift or
evening shift.
(g) A teacher in a college working in the commerce department
or the chemistry department.

PAGE 211
© Department of Distance & Continuing Education, Campus of Open Learning,
School of Open Learning, University of Delhi
B.COM. (PROGRAMME)

Notes (h) A college employee being a teaching faculty or a member of


non-teaching staff.
(i) A factory employee being male or being a trade union member.
(j) Appearance of an odd number or appearance of a number
greater than 4 on a single toss of a die.
(k) Getting two defectives one after another from a lot of 50 units
of an item.
(vii) Explain the meaning of marginal, joint and conditional probability.
How can we obtain marginal probability of event E from the given
joint probabilities of events A and E, B and E, C and E, and D and
E, where A, B, C and D are the events to which E is related?
(viii) What is statistical independence? How can we ascertain whether
events A and B are statistically independent?
(ix) State and explain Bayes’ Theorem.
(x) A, B and C are competing for the award of a building contract. It is
believed that the chances of A’s getting the contract are one-half of
the combined chances of B and C’s getting it. Further B’s chances
are believed to be one-half of C’s chances. What is the probability
of each one getting the contract?
(xi) A survey on MBA students provided the following data for 2,018
students:
Age group Whether applied to more than one school
Yes No
23 or under 297 201
24–26 209 379
27–30 185 268
31–35 66 193
36 and over 51 169
(a) What is the probability that a randomly selected applicant is
23 or under?
(b) What is the probability that a randomly selected applicant is
older than 26?
(c) What is the probability that a randomly selected applicant
applied to more than one school?
(d) What is the probability that a randomly selected applicant is
above 36 and has not applied to more than one school?

212 PAGE
© Department of Distance & Continuing Education, Campus of Open Learning,
School of Open Learning, University of Delhi
BUSINESS STATISTICS

(e) What is the probability that a randomly selected applicant is Notes


under 27 and has applied to more than one school?
(xii) There are twenty tickets in a bag, numbered consecutively from
1 to 20. A ticket is selected at random. Find the chance that the
number on the ticket is:
(a) Greater than 14
(b) Divisible by 4
(c) Between 8 and 15, both inclusive.
(xiii) There are four shops and four applicants each of whom applies for
one shop at random. Find the probability that:
(a) Each of them applies for a different shop.
(b) Each of them applies for the same shop.
(xiv) A committee of 6 is to be chosen randomly from a group of 8 men
and 4 women. Determine the probability that it shall be composed
of:
(a) 4 men and 2 women
(b) 6 women
(c) 2 men and 4 women
(d) 6 men
(xv) Among the 90 pieces of mail delivered to an office, 50 are addressed
to the accounting department and 40 are addressed to the marketing
department. If two of these pieces of mail are delivered to the
managers’ office by mistake, and the selection is random, what are
the probabilities that:
(a) Both of them should have been delivered to the accounting
department;
(b) Both of them should have been delivered to the marketing
department;
(c) One should have been delivered to the accounting department
and the other to the marketing department?
(xvi) A bag contains 6 green, 7 blue and 2 red balls. Three balls are
chosen at random. Find the probability that (i) all of them are
green, (ii) both the red balls are included and (iii) the balls are all
of different colours.

PAGE 213
© Department of Distance & Continuing Education, Campus of Open Learning,
School of Open Learning, University of Delhi
B.COM. (PROGRAMME)

Notes (xvii) Two unbiased dice are tossed. What is the probability that the total
of numbers on them would be a multiple of 3?
(xviii) A pack contains 30 tickets numbered consecutively from 1 to 30.
A ticket is chosen at random from this. Find the chance that the
number on this would be (i) a multiple 6 or 7 and (ii) a multiple
of 3 or 5.
(xix) Five candidates A, B, C, D and E appear for an interview. Two
candidates D and E are eliminated in the first round of the interview.
A has twice the chance of being selected than B, and B has twice
the chance as C, in the final interview. D bets that either A or B
will be selected and E bets that either B or C will be selected. Who
is likely to win the bet?
(xx) Given the following probability table of television viewing frequencies
(X) and the income levels (Y):
Viewing Income levels (Y) Total
frequency (X) High Middle Low
Regular 0.10 0.15 0.05 0.30
Occasional 0.10 0.20 0.10 0.40
Rarely 0.05 0.05 0.20 0.30
Total 0.25 0.40 0.35 1.00
(a) What is the probability that a person is a low income individual
and views TV regularly?
(b) If an individual is at low income level, what is the probability
that he/she views TV regularly?
(c) What is the probability that given an individual does not have
high income, he/she rarely watches TV?
(d) If an individual occasionally watches TV, what is the probability
that he/she is a high income earner or a middle income earner?
(e) Is viewing TV regularly independent of earning high income?
Explain.
(xxi) The probability that a contractor will not get a plumbing contract
is 2/3 and the probability that he will get an electric contract is
5/9. If the probability of getting at least one contract is 4/5. What
is the probability that he will get both the contracts?

214 PAGE
© Department of Distance & Continuing Education, Campus of Open Learning,
School of Open Learning, University of Delhi
BUSINESS STATISTICS

(xxii) An unbiased die and a biased die are tossed together. Find the Notes
probability that the sum of digits obtained on them is even, given
that on the biased die, it is thrice as likely to show an even number
as an odd one when tossed once.
(xxiii) A six-faced die is so biased that the digits 1, 3 or 5 on it are
thrice as likely as the digits 2, 4 or 6, when tossed once. Find the
probability that in two tosses of this die, the sum of digits would
be odd.
(xxiv) A husband and wife appear in an interview for two vacancies for
the same post. The probability of the husband’s selection is 1/7 and
that of wife’s selection is 1/5. What is the probability that:
(a) Both of them will be selected?
(b) Only one of them will be selected?
(c) None of them will be selected?
(xxv) (a) In rolling a pair of dice, what is the probability of rolling a
total of 21 on the first two rolls?
(b) Given that P(A) = 0.65. P(B) = 0.80, P(A/B) = P(A) and P(B/A)
= 0.85. Is this a consistent assignment of probabilities?
(xxvi) An MBA applies for one job in two firms X and Y. The probability
of his being selected in firm X is 0.7 and his being rejected in firm
Y is 0.5. The probability of at least one of his applications being
rejected is 0.6. What is the probability that he will be selected in
one or both of the firms?
(xxvii) During a survey of road safety, it was found that 60 per cent of
accidents occur at night, 52 per cent are alcohol related, and 37
per cent are alcohol related and occur at night:
(a) What is the probability that an accident was alcohol related
given that it occurred at night?
(b) What is the probability that an accident occurred at night
given that it was alcohol related?
(xxviii) An advertising executive is studying television-viewing habits
of married men and women during prime time hours. On the basis
of past viewing records, the executive has determined that during
prime time, husbands are watching television 60% of the time.
It has also been determined that when the husband is watching
television, 40% of the time the wife is also watching. When the

PAGE 215
© Department of Distance & Continuing Education, Campus of Open Learning,
School of Open Learning, University of Delhi
B.COM. (PROGRAMME)

Notes husband is not watching television, 30% of the time the wife is
watching television. Find the probability that:
(a) If the wife is watching television, the husband is also watching
television.
(b) The wife is watching television during prime time.
(xxix)In a small factory, machines A, B and C manufacture 35%, 25% and
40% respectively of the total output. Of their output, respectively,
0.5, 4 and 2 per cent are defective. One item is drawn and found
to be defective. What are the respective probabilities that it was
produced by machines A, B and C?
(xxx) Reliance Industries Limited is determining whether it should submit
a bid for oil exploration contract. In the past, main competitor of
RIL, ONGC has submitted bids 66 per cent of the time. If ONGC
does not bid for oil exploration contract, the probability that RIL
will get the contract is 0.45. If ONGC does bid for oil exploration
contract, the probability that RIL will get the contract is 0.25:
(a) If Reliance Industries gets the contract, what is the probability
that ONGC did not bid?
(b) What is the probability that Reliance Industries will get the
contract?
Ans.
(x) 0.33, 0.22, 0.44, (xi) 498/2018, 932/ (xii) 0.3, 0.25, 0.4
2018, 808/2018,
169/2018, 507/
2018
(xiii) 3/32, 1/64 (xiv) 0 . 4 5 4 5 , 0 , (xv) 0.306, 0.195,
0.0303, 0.0303 0.4993
(xvi) 20/455, 13/455, (xvii) 0.33 (xviii) 9/30, 14/30
84/455
(xix) D is likely to win (xx) 0.0575, 0.01429, (xxi) 14/45
0.333, 0.75, No
(xxii) 0.5 (xxiii) 0.375 (xxiv) 1/35, 10/35,
24/35
(xxv) 20/1296, Not (xxvi) 0.8 (xxvii) 0.617, 0.712
consistent
(xxviii) 0.67, 0.36 (xxix) 0.362, 0.406, (xxx) 0.519, 0.318
0.232

216 PAGE
© Department of Distance & Continuing Education, Campus of Open Learning,
School of Open Learning, University of Delhi
L E S S O N

2
Probability Distributions
STRUCTURE
2.1 Learning Objectives
2.2 Probability Distribution
2.3 Binomial Distribution
2.4 Poisson Distribution
2.5 Normal Distribution
2.6 Summary
2.7 Self-Assessment Exercise

2.1 Learning Objectives


After reading this lesson, you should be able to :
‹ Understand the concept of probability distribution and random variables.
‹ Understand the characteristics and procedure of computing probabilities using binomial
and Poisson distribution.
‹ Comprehend the difference between discrete and continuous probability distribution.
‹ Understand normal distribution, properties of a normal curve and computation of
probabilities using z-values.
‹ Analyse the situations under which a Poisson distribution is treated a binomial or
normal distribution.

2.2 Probability Distribution


Probability distribution refers to a set of mathematical models of the relative frequencies of
a finite number of observations of a variable. It is a systematic arrangement of probabilities
of mutually exclusive and collectively exhaustive elementary events of an experiment.
Observed frequency distributions are based on actual observation and experimentation. We
can deduce mathematically a frequency distribution of certain populations based on the trend
of the known values. This kind of distribution on experience or theoretical considerations

PAGE 217
© Department of Distance & Continuing Education, Campus of Open Learning,
School of Open Learning, University of Delhi
B.COM. (PROGRAMME)

Notes is known as theoretical distribution or probability distribution or it can


be thought as a theoretical frequency distribution. These distributions
may not fully agree with actual observations or empirical distributions
based on sample observations. If the number of experiments is increased
sufficiently the observed distributions may come closer to theoretical or
probability distributions. Theoretical distributions are useful for situations
where actual observations or experiments are not possible. Moreover, it
can be used to test the goodness of fit. They provide decision makers with
a logical basis for making decisions and are useful in making predictions
on the basis of limited information or theoretical considerations.
There are two types of probability distributions: discrete and continuous:
1. Discrete Probability Distribution
2. Continuous Probability Distribution

2.2.1 Discrete Probability Distribution


A discrete probability distribution is a sort of probability distribution that
depicts all potential values of a discrete random variable corresponding to
its respective probabilities. In other words, a discrete probability distribution
describes the possibility of each potential value of a discrete random
variable occurring. A discrete probability distribution is the listing of all
possible discrete random variables which could occur if an experiment
was done i.e., the accepted discrete values are restricted to whole numbers
only. Hence, negative values, fractions, or decimals are not considered.
For example, rolling a die will give only finite and countable outcome
ranges from 1 to 6 thus this experiment will result in discrete probability
distribution with a probability of 1/6 for each possible outcome.
The discrete probability distribution is applied in numerous scientific
investigations as well as by finance specialists to assess discrete indicators
and to appraise the stock market.
The probability of discrete probability distribution can be a decimal value
but value of X i.e., event should be in whole number. For example, number
of votes a candidate gets in an election, appearance of advertisement on
TV, etc.

218 PAGE
© Department of Distance & Continuing Education, Campus of Open Learning,
School of Open Learning, University of Delhi
BUSINESS STATISTICS

2.2.2 Continuous Probability Distribution Notes

In a continuous probability distribution, the value may take any value


within a given range in other words, the variable may take on value
in decimal corresponding to its probability. For example, measuring
temperature of a city, measuring weight of athletes etc. It is different
from discrete probability in the sense that listing of actual values of
variable are not possible rather the distribution are listed out in the form
of range. The most important continuous probability distribution in the
field of statistics is normal probability distribution which is commonly
known as normal distribution.
Discrete probability distribution is applied where counting of values
is required while continuous probability distribution is applied where
measurement is required.

2.3 Binomial Distribution


The binomial distribution is a discrete probability distribution that
describes the number of successes in a fixed number of independent
Bernoulli trials (trials having only two possible outcomes) with a given
probability of success such as tossing a coin, hitting a target, producing
defective product etc.
The binomial distribution was discovered by James Bernoulli in 1700
to deal with the dichotomous classification of events. It is a probability
distribution expressing the probability of one set of dichotomous alternatives,
i.e., success or failure. The binomial probability distribution is developed
under some assumptions which are:
1. The binomial distribution is modelled for fixed number of trials.
2. Each trial is performed under homogenous conditions.
3. Each trial shall give only two possible outcomes i.e., success or
failure.
4. Possible outcomes are mutually exclusive.
5. The trials are independent i.e., the outcome of one trial does not
affect, in any way, the occurrence of succeeding trials.

PAGE 219
© Department of Distance & Continuing Education, Campus of Open Learning,
School of Open Learning, University of Delhi
B.COM. (PROGRAMME)

Notes 6. The probability of success denoted by p remains constant for all


trials. The probability of a failure denoted by q is equal to (1 – p).
7. This distribution is characterized by two parameters n i.e., number
of trials and p probability of successes in each trial.
8. The mean (μ) of a binomial distribution is np DQG YDULDQFH ı2) is
np(1–p) or npq.
9. The shape of the binomial distribution curve depends upon the value
of n and p.
Derivation of Binomial Distribution
Let’s assume a Bernoulli experiment is repeated n number of times i.e.,
the number of trials is n having two mutually exclusive possible outcomes
Success and Failure in each trial. Further, each trial of the experiment
is independent of each other. The probability of success is p remains
the same throughout the experiment. Since there are only two possible
outcomes Success or S and Failure or F and the P (S) is p, therefore,
the P (F) will be (1 – p) which may be re-written as q.
If X is the number of successes in n number of trials. Since it is a discrete
probability distribution, X may take any possible value ranging from 0
to n i.e., 0,1,2,3,…….,n. Now to find out the probability of obtaining
an exact number of successes let’s say r successes (r = 0,1,2,….,n) in
n trials with two possible outcomes, obtaining r successes and (n – r)
failures has probability
n
Cr pr (1 – p)n – r
Symbolically,
P (X = r) = nCr pr qn – r
If a trial of an experiment can result in success with probability p and
failure with probability q = (1 – p), the probability of exactly successes
in n trials is given by
P(x) = nCx pxqn-x where x = 0, 1, 2...n where P(x) = Probability of x
successes
n!
n
Cx = (! is termed factorial)
x !(n − x)!

220 PAGE
© Department of Distance & Continuing Education, Campus of Open Learning,
School of Open Learning, University of Delhi
BUSINESS STATISTICS

The entire probability distribution of x = 0, 1, 2,....n can be written as Notes


follows:
Binomial Probability Distribution
Number of success Probability
x P(x)
0 n
C0 p 0 q n
1 n
C1 p1q n−1
2 n
C2 p 2 q n − 2
: :
x n
Cx p x q n− x
: :
n n
Cn p n q n − n

We should note that the variable x (number of successes) is discrete. It


can take integer values 0, 1, 2, ..., n. The probabilities specified in the
above table are in fact successes terms of the Binomial Expansion of (p
+ q)n, which is
(q + p ) n = nC0 q n p 0 + nC1q n−1 p1 + nC2 q n−2 p 2 + nC3q n−3 p 3 + ... nCn q n−n p n

2.3.1 Properties of Binomial Distribution


(i) The shape and location of binomial distribution changes as p changes
for a given n or n changes for given p. If n increases for a fixed
p, the binomial distribution moves to the right, flattens and spreads
out. The mean of the distribution (np) increases as n increases for
constant value of p.
(ii) The mode of the binomial distribution is equal to the value of x
which has the largest probability.
(iii) If n is large and p and q are not close to zero, the binomial distribution
can be approximated by a normal distribution with standardised
variable.
X − np
Z = npq
PAGE 221
© Department of Distance & Continuing Education, Campus of Open Learning,
School of Open Learning, University of Delhi
B.COM. (PROGRAMME)

Notes (iv) The mean and the standard deviation of the Binomial distribution
is np and npq respectively.
(v) The other constants of the distribution can be calculated.
μ2 = npq
μ3 = npq (q – p)
μ4 = 3n2p2q2 + npq (1 – 6pq)
We can calculate the value of E1 and E2 to measure nature of the
distribution.
μ32 n 2 p 2 q 2 (q − p)2 (q − p 2 )
E1 = = =
μ32 n3 p 3 q 3 npq

μ 4 3n 2 p 2 q 2 + npq (1 − 6 pq ) 1 − 6 pq
E2 = = = 3+
μ22 2 2 2
n p q npq

The binomial distribution is useful in describing variety of real life


events. Binomial distribution is useful to answer questions such as : If
we conduct an experiment n times under the stated conditions, what is
the probability of obtaining exactly x successes? For example, if 10 coins
are tossed simultaneously what is the probability of getting 4 heads ? We
shall explain the usefulness of binomial distribution by certain examples.
Example 1 : A coin is tossed eight times. What is probability of obtaining
0, 1, 2, 3, 4, 5, 6, 7 and all heads?
Solution : Let us denote the occurrence of head as success by p.
1
So that p =
2
1
? q = 1 – p = and n = 8 (given)
2
We can calculate various probabilities by expanding the binomial
theorem.
(q + p )8 = 8C0 q8 p 0 + 8C1q 7 p1 + 8C2 q 6 p 2 + 8C3q 5 p 3 + 8C4 q 4 p 4 + 8C5 q 3 p 5 + 8C6 q 2 p 6
+ 8C7 q1 p 7 + 8Cn q 0 p8

222 PAGE
© Department of Distance & Continuing Education, Campus of Open Learning,
School of Open Learning, University of Delhi
BUSINESS STATISTICS

Therefore the probability of obtaining 0 heads = Notes


8 0 8 8
8! ⎛ 1 ⎞ ⎛ 1 ⎞ 8! ⎛ 1 ⎞ ⎛ 1 ⎞ 1
8
C0 q8 p 0 = ×⎜ ⎟ ⎜ ⎟ = ×⎜ ⎟ = ⎜ ⎟ = Ans.
8!× 0! ⎝ 2 ⎠ ⎝ 2 ⎠ 8! ⎝ 2 ⎠ ⎝ 2 ⎠ 256
The probability of obtaining 1 head = 8C1q7p1
7 7
8! ⎛ 1 ⎞ 1 ⎛1⎞ 1 8
× ⎜ ⎟ × = 8× ⎜ ⎟ × = Ans.
= 7!×1! ⎝ 2 ⎠ 2 ⎝ 2 ⎠ 2 256
The probability of getting 2 heads = 8C2q6p2
6 2 8 8
8! ⎛ 1 ⎞ ⎛ 1 ⎞ 8 × 7 × 6! ⎛ 1 ⎞ 8 × 7 ⎛ 1 ⎞ 28
= ⎜ ⎟ ×⎜ ⎟ = ×⎜ ⎟ = ×⎜ ⎟ = Ans.
2!× 6! ⎝ 2 ⎠ ⎝ 2 ⎠ 2!× 6! ⎝ 2 ⎠ 2 ⎝2⎠ 256
The probability of getting 3 heads = 8C3q5p3
5 3 8
8! ⎛ 1 ⎞ ⎛ 1 ⎞ 8 × 7 × 6 × 5! ⎛ 1 ⎞ 56
= ×⎜ ⎟ ×⎜ ⎟ = ×⎜ ⎟ = Ans.
3!× 5! ⎝ 2 ⎠ ⎝ 2 ⎠ 3 × 2 ×1× 5! ⎝ 2 ⎠ 256
The probability of getting 4 heads = 8C4q4p4
4 4 8
8! ⎛ 1 ⎞ ⎛ 1 ⎞ 8 × 7 × 6 × 5 × 4! ⎛ 1 ⎞ 70
= ×⎜ ⎟ ×⎜ ⎟ = ×⎜ ⎟ = Ans.
4!× 4! ⎝ 2 ⎠ ⎝ 2 ⎠ 4 × 3 × 2 × 1× 4! ⎝ 2 ⎠ 256
The probability of getting 5 heads = 8C5q3p5
3 5 8
8! ⎛ 1 ⎞ ⎛ 1 ⎞ 8 × 7 × 6 × 5! ⎛ 1 ⎞ 56
= ×⎜ ⎟ ×⎜ ⎟ = ×⎜ ⎟ = Ans.
3!× 5! ⎝ 2 ⎠ ⎝ 2 ⎠ 3 × 2 ×1× 5! ⎝ 2 ⎠ 256
The probability of getting 6 heads = 8C6q2p6
2 6 8
8! ⎛ 1 ⎞ ⎛ 1 ⎞ 8 × 7 × 6! ⎛ 1 ⎞ 28
= ×⎜ ⎟ ×⎜ ⎟ = ×⎜ ⎟ = Ans.
2!× 6! ⎝ 2 ⎠ ⎝ 2 ⎠ 2 ×1× 6! ⎝ 2 ⎠ 256
The probability of getting 7 heads = 8C7q1p7
1 7 8
8! ⎛ 1 ⎞ ⎛ 1 ⎞ 8 × 7! ⎛ 1 ⎞ 8
= ×⎜ ⎟ ×⎜ ⎟ = ×⎜ ⎟ = Ans.
7!×1! ⎝ 2 ⎠ ⎝ 2 ⎠ 7! ⎝ 2 ⎠ 256
The probability of getting 8 heads = 8C8q0p8
8 8
8! ⎛ 1 ⎞ ⎛ 1 ⎞ 8! ⎛ 1 ⎞ 1
= ×⎜ ⎟×⎜ ⎟ = ×⎜ ⎟ = Ans.
8!× 0! ⎝ 2 ⎠ ⎝ 2 ⎠ 8! ⎝ 2 ⎠ 256

PAGE 223
© Department of Distance & Continuing Education, Campus of Open Learning,
School of Open Learning, University of Delhi
B.COM. (PROGRAMME)

Notes We can also calculate the probability of 4 or more heads or maximum 6


heads. Probability of 4 or more heads =
70 56 28 8 1 163
8
C4 q 4 p 4 + 8C5 q 3 p 5 + 8C6 q 2 p 6 + 8C7 q1 p 7 + 8Cn q 0 p8 = + + + + =
256 256 256 256 256 256
Ans.

8 1 9
Probability of getting more than 6 heads =
8
C7 q1 p 7 + 8Cn q 0 p8 = + =
256 256 256
Ans.

Example 2 : A box contains 100 transistors, 20 of which are defective, 10


are selected at dom. Calculate the probability that (i) all 10 are defective,
(ii) all 10 are good.
Solution : Let x represent the number of defective transistors selected.
Then the value of x would be, x = 0, 1, 2,.... 10.
Let us put p as the probability of a defective transistor.
20 1 4
? p = = and p   – p = , n = 10
100 5 5
Using the formula for binomial expansion, the probability of x defective
transistors is P(x) = nCxpxqn–x
10
(i) Probability that all 10 are defective = C10 p10q10 – 10
10 0 10
10! ⎛1⎞ ⎛4⎞ ⎛1⎞ 1
= × ⎜ ⎟ × ⎜ ⎟ = ⎜ ⎟ = 10 Ans.
10!×10! ⎝ 5 ⎠ ⎝5⎠ ⎝5⎠ 5
(ii) Probability that all 10 are good = 10
C0 p0q10
0 10 10
10! ⎛ 1 ⎞ ⎛ 4 ⎞ ⎛4⎞
×⎜ ⎟ ×⎜ ⎟ = ⎜ ⎟ Ans.
= 10! ⎝ 5 ⎠ ⎝ 5 ⎠ ⎝5⎠
Application of Binomial Distribution
1. Quality control measures and sampling process in industries to classify
items as defectives or non-defective.
2. Medical applications such as success or failure, cure or no cure.
3. Sports application such as win or lose.

224 PAGE
© Department of Distance & Continuing Education, Campus of Open Learning,
School of Open Learning, University of Delhi
BUSINESS STATISTICS

2.4 Poisson Distribution Notes

The binomial distribution is used in studies where there are only two
alternative outcomes which can be counted. However, there are many
situations in which number of successes can be counted but it becomes
impossible to count the number of failures such as number of vehicles
arriving at a toll gate during a day, here arrival of vehicle at toll gate is
considered as success which can be counted and non-arrival of vehicle at
toll gate is failure which can not be counted. Therefore, in such scenarios,
binomial distribution fails to calculate the probability of success. The
application of Poisson distribution is appropriate for such situation without
knowing the total possible outcomes.
The Poisson distribution is a discrete probability distribution that models
the number of events occurring within a fixed interval of time or space,
given a known average rate of occurrence and assuming events to be
independent. This distribution is characterized by a single parameter,
denoted as Ȝ representing the average rate of occurrence of an event in
a given interval.
It was originated by a French mathematician Simeon Denis Poisson in
1837. This distribution is used to calculate the likelihood of an independent
event occurring at a fixed interval of time or space with a constant mean
rate such as number of calls at call center in an hour, number of errors
on a page, number of deaths in a district in a year etc. In statistics, the
Poisson distribution is a probability distribution used to demonstrate how
frequently an event is likely to happen during a specific time frame. In
other terms, it is a distribution of counts. Thus, the Poisson distribution
is used to predict how many times an event will occur in a given time
period or space. Here, an interval of time may be of any length such,
minute, hour, month, year etc. and interval of space may be a piece of
paper, a district, country etc.
Let X be the Poisson random variable, which may take on any whole
number (X = 0,1,2,…n), representing number of event occurring in a
given interval of time or space, the probability of exactly x number of
events occur in a given interval is
P (X = x) = eȜ Ȝx / x!

PAGE 225
© Department of Distance & Continuing Education, Campus of Open Learning,
School of Open Learning, University of Delhi
B.COM. (PROGRAMME)

Notes Where, Ȝ is the parameter of the distribution and it is the mean number
of occurrences of event (or successes)
e = 2.71828
and, x is the Poisson random variable
Properties of Poisson Distribution
The Poisson distribution is useful in events with a large number of rare
and independent possible outcomes. The following are the properties of
the Poisson Distribution:
1. The events are assumed to be independent of each other.
2. The Poisson distribution can be defined only for non-negative integer
values of random variable.
3. The Poisson distribution is characterized by only one parameter Ȝ
4. The average number of successes (or occurrence of events) i.e., Ȝ
is constant from trial to trial.
5. The average number of successes in a given interval of time or
space is possible. Thus, two events cannot happen at the same time.
6. The mean and variance of Poisson distribution are same i.e., Ȝ
7. The Poisson distribution has the property of memorylessness, which
means the probability of an event occurring in future is not affected
by events in the past.
8. The Poisson distribution is positively skewed.
9. The Poisson distribution can approximate a binomial distribution,
if n is large and p is small.
10. The Poisson distribution is approximated by the normal distribution
when Ȝ is large.
11. The distribution is commonly used to model real world scenarios
involving rare events such as number of number of accidents at
intersection, number of bacteria in a given culture, number of
customers at checkout counter at departmental store etc.
Example 3: The average number of customers arriving at the checkout
counter of a store is 3 customers per hour. Find the probability that
during a given hour:
(i) No customer appears

226 PAGE
© Department of Distance & Continuing Education, Campus of Open Learning,
School of Open Learning, University of Delhi
BUSINESS STATISTICS

(ii) Exactly 5 customers appear Notes


(iii) 3 or more customers appear
(iv) Less than 3 customers appear
Note: e-3 = 0.04979
Solution: Let X be the number of customers arriving at the checkout
counter per hour.
And, the average number of customers arriving at checkout counter per
hour i.e., Ȝ 3. Thus, by using Poisson distribution, the probability that:
(i) x = 0 customer arriving is
P (X = x) = eȜ Ȝx / x!
Or P (X = 0) = e-3 30 / 0!
Or P (X = 0) = 0.04979 × 1/1 = 0.04979
(ii) x = 5
P (X = x) = eȜ Ȝx / x!
Or P (X = 5) = e-3 35 / 5! = 0.04979 × 243/120 = 0.1008
(iii) x •  FXVWRPHUV DUULYLQJ LV
3 ; • 3) = e-3 33 / 3! + e-3 43 / 4! + ………. + e-3 ’3  ’
2U 3 ; • 3) = 1 – [P (X <3)]
= 1 – [P (X = 0) + P (X = 1) + P (X = 2)]
= 1 – [e-3 30 / 0! + e-3 31 / 1! + e-3 32 / 2!]
= 1 – [0.04979 × 1/1 + 0.04979 × 3/1 + 0.04979 × 9/2]
= 1 – [0.04979 + 0.14936 + 0.22404]
= 1 – 0.42319
= 0.5768
(iv) x< 3 customers arriving is
P (X <3) = P (X = 0) + P (X = 1) + P (X = 2)
= e-3 30 / 0! + e-3 31 / 1! + e-3 32 / 2!
= 0.04979 × 1/1 + 0.04979 × 3/1 + 0.04979 × 9/2
= 0.04979 + 0.14936 + 0.22404
= 0.42319

PAGE 227
© Department of Distance & Continuing Education, Campus of Open Learning,
School of Open Learning, University of Delhi
B.COM. (PROGRAMME)

Notes Poisson Distribution as a Limiting form of Binomial Distribution


The Poisson distribution is the limiting form of binomial distribution
as n becomes infinitely large (n > 20) and p approaches zero (p <0.05)
such that np = Ȝ remains fixed. The Poisson distribution is useful for
rare events. Suppose in the binomial distribution:
(a) p is very small
(b) n is so large that np = Ȝ is constant
Then, we would get the following distribution
x 0 1 2 3 ............ x Total
λ1 λ2 λ3 λx
Probability e±Ȝ e- λ e- λ e-λ ...... e-λ 1
1! 2! 3! x!

It is a Poisson distribution. Under these conditions the probability


of getting x successes is
-λ λx
P(X = x) = e
x!
Sum of the probabilities of 0, 1, 2, 3 successes is
⎛ λ λ 2 λ3 λx ⎞
e ⎜1+ + + +....+ ⎟ = eλ .eλ = 1

⎝ 1! 2! 3! x! ⎠

where e LV D FRQVWDQW ZKRVH YDOXH LV  DQG Ȝ LV WKH SDUDPHWHU RI
the distribution i.e. the average number of occurrences of an event.
A classical example of the Poisson distribution is given by road accidents.
As we know the number of people travelling on the road is very large i.e.
n is large. Probability that any specific individual runs into an accident
is very small. However.
np = average number of road accidents is a finite constant on any
particular day.
Therefore, x (number of road accidents on a particular day) follows
Poisson distribution.
The various parameters of Poisson Distribution are:
0HDQ Ȝ = np
(variance) = np = Ȝ

228 PAGE
© Department of Distance & Continuing Education, Campus of Open Learning,
School of Open Learning, University of Delhi
BUSINESS STATISTICS

V = np Notes

μ2 =np = Ȝ
μ3 = Ȝ
μ4 = Ȝ + 3Ȝ2
μ 32 λ 2 1
? E1 = 3 = 3 =
μ2 λ λ

μ 4 λ + 3λ 2 1
E2 = 2 = = 3+
μ2 λ 2
λ

Example 4: If one house in 1000 has a fire in a district per year. What
is the probability that exactly 5 houses will have fire during the year if
there are 2000 houses?
Solution: We shall apply Poisson distribution
1
Ȝ = np where n = 2000, p =
1000
1
? Ȝ = np = 2000 × = 2.
1000

P(x) = eȜ× λ when x = 5 and e = 2.7183


x

x!

25 2× 2× 2× 2× 2 4
P(5) = 2.7183– 2 × = (2.7183)– 2 × = 2.7183– 2 ×
5! 5 × 4 × 3 × 2 ×1 15
4 ⎛ 4⎞
= Reciprocal (AL(2 log 2.7183)) = Reciprocal (7.389) ⎜ ⎟ = 0.1352
15 ⎝ 15 ⎠
4
× = 0.036 Ans.
15
Example 5: If 3% of the bulbs manufactured are defective, calculate the
probability that a sample of 100 bulbs-will contain no defective and one
defective bulb using Poisson distribution.
Solution: Given number of defective bulbs are 3% (3/100).

PAGE 229
© Department of Distance & Continuing Education, Campus of Open Learning,
School of Open Learning, University of Delhi
B.COM. (PROGRAMME)

Notes 3
? Ȝ = np = 100 × = 3.
100
Probability of no defective bulb in a sample of 100 is

P(x) = e±Ȝ × where m = 3, and e = 2.7183


P(o) = 2.7183–3 = 0.05 Ans.
Probability of one defective bulb in a sample of 100 is
Ȝ1
P(1) = e±Ȝ× 1! = 2.7183–3 × 3 = 0.15 Ans.
Application of Poisson Distribution
There are various applications of the Poisson distribution. The random
variables that follow a Poisson distribution are as follows:
1. To count the number of defective items in a batch of finished
product.
2. It is used in quality control statistics to count the number of defects
of an item.
3. In medical science, to count the number of bacteria in an experiment.
4. Number of errors in a page or book.
5. To predict the number of customers to visit website or place order.
6. Used by banks to model the number of expected customer bankruptcies
in a given interval of time or place.
7. The Poisson distribution is used by technology companies to model
the number of expected network failures per week or area specific.
8. To predict how many calls they will receive per hour, allowing
them to determine how many call centre representatives to keep
on staff.

2.5 Normal Distribution


The most important continuous probability distribution used in the entire
field of statistics is normal distribution. The normal curve is bell-shaped
that extends infinitely in both directions coming closer and closer to the
horizontal axis without touching it. The mathematical equation of normal
curve was developed by De Moivre in 1733. A continuous random variable

230 PAGE
© Department of Distance & Continuing Education, Campus of Open Learning,
School of Open Learning, University of Delhi
BUSINESS STATISTICS

x is said to be normally distributed if it has the probability density Notes


function represented by the equation of normal curve.
− ( x −μ )2
1 2 σ2
y = e , –∞≤ x≤+∞
σ 2π
Where μ and V are mean and standard deviation which are two parameters
and e = 2.7183, p = 3.1416 are constants.
It may be understood that the normal distributions can have different
shapes depending upon values of P and V but there is one and only one
normal distribution for any given pair values of P and V.

2.5.1 Properties of Normal Distribution


1. If the parameters P and V of the normal curve are specified, the
normal curve is fully determined and we can draw it by obtaining
the value of y corresponding to different values of x (the abscissa).
2. The normal curve tends to touch the x-axis only at infinity i.e.
the x-axis is an asymptotic to the normal curve. It is a continuous
curve stretching from – f to + f
3. The mean, median and mode of the normal distribution are equal.
4. The height of the normal curve is maximum at x = P. Hence the
mode of the normal curve is x = P
5. The two quartiles Q1 and Q3 are equidistant from the median.
Q1 = P – 0.6745 V
Q3 = P + 0.6745 V

Q3 − Q1
Hence Quartile Deviation = = 0.6745 V
2
4
6. Mean deviation about mean is σ or MD = 0.7979 V
5
7. The points of inflexion of the normal curve occur at x =P + V and
x = P – V

PAGE 231
© Department of Distance & Continuing Education, Campus of Open Learning,
School of Open Learning, University of Delhi
B.COM. (PROGRAMME)

Notes 8. The tails of curve extend to infinity on both sides of the mean. The
1
maximum ordinate at X = P is given by y =
σ 2π
9. Approximately 100% of the area under the curve is covered by μ+
3V.
Distance from the mean % of total area under the
ordinate in terms of ± V normal curve
Mean ± 3V 68.27
Mean ± 2V 95.45
Mean ± 3V 99.73
10. All odd moments are equal to zero.
μ1 = μ3 = 0
E1 = 0 and E2 = 3. Thus the curve is mesokurtic.
11. The normal distribution is formed with a continuous variable.
12. The fourth moment is equal to 3V4 for a normal distribution.
The equation of the normal curves gives the ordinate of the curve
corresponding to any given value of x. But we are interested in finding
out the area under the normal curve rather than its ordinate (y). A normal
curve with 0 mean and unit standard deviation is known as the standard
normal curve. With the help of a statistical table which gives the area
and ordinates of the normal curve are given corresponding to standard
normal variate.
x −μ
z = and not corresponding to x.
σ
Let us see the normal curve area under x-scale and z-scale.

232 PAGE
© Department of Distance & Continuing Education, Campus of Open Learning,
School of Open Learning, University of Delhi
BUSINESS STATISTICS

Notes

Fig. 1

2.5.2 Calculation of Probabilities


Now we discuss the method of calculating probabilities where the
distribution follows the normal pattern. In fact, the probability for the
variable to assume a value within a given range, say X1 and X2, is equal
to the ratio of the area under the curve in that range to the total area
under the curve. To obtain the relevant areas, we first transform a given
value of the variable X into standardized variate Z as follows :
X −μ
Z =
σ
Then we consult the normal area table. This table is constructed in a
manner such that the areas between mean (μ) and the particular values
of Z are given. The first column of this table contains values of Z from
0.0 to 3.0, while the top row of the table gives values 0.00; 0.01; 0.02;
....0.09. To find the area (from mean) to a specific value of Z, we look
up in the first column for Z-value upto its first decimal place while its
second decimal place is read from the top row. To illustrate, if we want
to find the area between mean and Z = 1.42, then we look for 1.4 in the
first column and 0.02 in the top row. Corresponding to these, the value
in the table reads 0.4222. Similarly, it can be verified that area upto Z
= 0.10 is equal to 0.0398 while for Z = 2.59, it is 0.4952.
Let us understand few more things :

PAGE 233
© Department of Distance & Continuing Education, Campus of Open Learning,
School of Open Learning, University of Delhi
B.COM. (PROGRAMME)

Notes (i) The area under the curve from Z = 0 (when X = μ) to a particular
value of Z gives the proportion of the area under this part of the
curve to the total area under the curve. Thus,
Z = 0 to Z = 1.42 the value 0.4222. Naturally, this is taken as the
probability that the variable in question will assume a value within
these limits.
(ii) Since the normal curve is symmetrical with respect to mean, the
area between μ(Z = 0) and particular value of Z to its right will be
same as the value of Z to its left. Thus, area between Z = 0 and
Z = 1.5 is equal to area between Z = 0 and Z = –1.5. Remember
that for values of X greater than μ, the Z value will be positive
while for X < μ, the value of Z would be negative.
(iii) The general procedure for calculating probabilities is like this :
(a) specify clearly the relevant area under the curve which is of
interest.
(b) determine the Z value (s).
(c) obtain the required area (s) with reference to the normal area
table.
Example 6 : Find the area under the normal curve :
(i) between Z = 0 and Z = 1.20
(ii) between Z = 1.0 and Z = 2.43
(iii) to the right of Z = 1.37
(iv) between Z = –1.3 and Z = 1.49
(v) to the right of Z = –1.78
Solution : For each of these, the relevant portions under the normal
curve are shown shaded and the areas determined with reference to the
normal area table.

Figure 2

234 PAGE
© Department of Distance & Continuing Education, Campus of Open Learning,
School of Open Learning, University of Delhi
BUSINESS STATISTICS

(i) Area between Z = 0 and Z = 1.20 is 0.3849. Notes

Figure 3
(ii) Area between Z = 0 and Z = 1.0 is 0.3413
Area between Z = 0 and Z = 2.43 is 0.4925
? Area between Z = 1.0 and Z = 2.43 is 0.4925 – 0.3413 = 0.1512.

Figure 4
(iii) Area between z = 0 and Z = 1.37 is 0.4147.
Total area under the curve being equal to 1, the area to the right
of Z = 0 is 0.5, as is the area to the left of it
? Area beyond Z = 1.37 is 0.5000 – 0.4147 = 0.0853.

Figure 5
(iv) Area between Z = 0 and Z = 1.3 is 0.4032.
Area between Z = 0 and Z = 1.49 is 0.4319.
? Area between Z = 1.3 and Z = 1.49 is 0.4032 + 0.4319 = 0.8351.

PAGE 235
© Department of Distance & Continuing Education, Campus of Open Learning,
School of Open Learning, University of Delhi
B.COM. (PROGRAMME)

Notes

Figure 6
(v) Area between Z = 0 and Z = – 1.78 is 0.4625.
Area between Z = 0 is 0.5.
? Area to the right of Z = – 1.78 is 0.4625 + 0.5 = 0.9625.
Example 7 : Balls are tested by dropping from a certain height of bounce.
A ball is said to be fast if it rises above 36 inches. The height of the
bounce may be taken to be normally distributed with mean 33 inches
and standard deviation of 1.2 inches. If a ball is drawn at random, what
is the chance that it would be fast?
Solution : The given information is depicted in figure 7. Here we have
to calculate the probability that the height of the bounce, X, would be
greater that 36. This is shown shaded in figure 7.

Figure 7
We have, X = 36, μ = 33 and V = 1.2
X − μ 36 − 33
Z = = = 2.5
σ 1.2
From the normal area table, area between Z = 0 and Z = 2.5 is equal
to 0.4938. So area beyond Z = 2.5 is 0.5 – 0.4938 = 0.0062. Therefore,
P(X > 36) = 0.0062, the chance of getting a fast ball.
Example 8 : The life (x) of electric bulbs in hours is supposed to be
normally distributed as

236 PAGE
© Department of Distance & Continuing Education, Campus of Open Learning,
School of Open Learning, University of Delhi
BUSINESS STATISTICS

( x −155)2
Notes
1
e 722
19. 2π
What is the probability that the life of a bulb will be :
(i) Less than 117 hours (ii) more than 193 hours (iii) between 117 and
193 hours.
Solution : Given μ = 155 and V = 19
Therefore, corresponding to x = 117 the standard normal variate is z =
117 − 155
= −2
19

Figure 8
We have to obtain the area to the left of Z = –2 [Pr(Z< – 2)].
From the table we see the area z = 0 and z = –2 and subtract it from 0.5.
? 0.5 – .4772 = 0.0228
Hence the probability of life of bulbs more than 193 hours is 0.0228.
To obtain the probability that the life of the bulb is more than 193 hours,
we obtain the corresponding standard normal variate
193 − 155
z = = +2
19

Figure 9

PAGE 237
© Department of Distance & Continuing Education, Campus of Open Learning,
School of Open Learning, University of Delhi
B.COM. (PROGRAMME)

Notes And the area between 117 hours and 193 hours shall be

Figure 10
where Z = + 2.
Hence Pr (–2 < Z < +2) = Pr (117 < x < 193)
= .4772 + .4772 = .9544 Ans.
Example 9 : The results of a particular examination are given below in
a summary form :
Result Percentage of Candidates
Total Passed 80
Passed with distinction 10
Failed 20
It is known that a candidate fails if he obtains less than 40 marks (out
of 100), while he must obtain at least 75 marks in order to pass with
distinction. Determine the mean and the standard deviation of marks
assuming distribution of marks to be normal.
Solution : According to the given information,
Percentage of students getting marks less than 40 = 20,
Percentage of students getting marks between 40 and 75 = 70, and
Percentage of students getting marks above 75 = 10.
The relevant area is shown in figure 15.

Figure 11
Here P(X < 40) = 0.20, P (40 < X < 75) = 0.70 and P(X > 75) = 0.10

238 PAGE
© Department of Distance & Continuing Education, Campus of Open Learning,
School of Open Learning, University of Delhi
BUSINESS STATISTICS

Let μ and V represent the mean and standard deviation of the distribution. Notes
We have, area between μ and X =40 equal to 0.30, and area between μ
and X = 75 as equal to 0.40.
Now we have,
40 − μ
For X = 40, Z = , and
σ
75 − μ
For X = 75, Z = ,
σ
Corresponding to the area 0.30 in the normal area table, Z = 0.84. Thus,
for X = 40, we have Z = – 0.84 (Since the value of 40 lies to the left
of μ). Similarly, for the area equal to 0.40, we have Z = 1.28.

40 − μ
We have, then = – 0.84 and ....(i)
σ
75 − μ
= – 1.28 ....(ii)
σ
Rearranging the above equations, we get
μ– 0.84 V = 40 and ....(iii)
μ +1.28 V = 75 ....(iv)
Subtracting equation (iii) from equation (iv), we get
2.12 V = 35
or V = 35/2.12 = 16.51
Substituting the value of V in equation (iii) and solving for μ, we
get
P – (0.84) (16.51) = 40
or P = 40 + 13.87 = 53.87
Thus, Mean = 53.87 marks and standard deviation = 16.51 marks.
Example 10 : There are 900 students in B.Com (Hons.) course of a
college and the probability of a student needing a particular book on a
day is 0.10. How many copies of the book should be kept in the library
that there should be at least 0.90 chance that a student needing that book
will not go disappointed ? Assume normal approximation to the binomial
distribution.

PAGE 239
© Department of Distance & Continuing Education, Campus of Open Learning,
School of Open Learning, University of Delhi
B.COM. (PROGRAMME)

Notes Solution : According to the given information, n = 900, p = 0.10, and q


= 1 – p = 1 – 0.10 = 0.90. Therefore, mean = np = 900 × 0.10 = 90,
and V= npq = 900 × 0.10 × 0.90 = 9. Here we are required to determine
X, to the right of which 10 per cent of the area under the curve lies.
Area between μ= 90 and X would be equal to 0.50 – 0.10 = 0.40. Now,
Z value corresponding to the area 0.40 equals 1.28.
Thus,
X − 90
Z = = 1.28
9
Solving for X, we get X = 1.28 × 9 + 90 = 11.52 + 90 = 101.52.
Therefore 102 books should be kept in the library to meet the demand
of students.
Table 1
Table of values of e–μ (0 d μ d 1)
μ 0 1 2 3 4 5 6 7 8 9
0.0 1.0000 0.9900 0.9802 0.9704 0.9608 0.9512 0.9418 0.9324 0.9231 0.9139
0.1 0.9048 0.8958 0.8869 0.8781 0.8694 0.8607 0.8521 0.8437 0.8353 0.8207
0.2 0.8187 0.8106 0.8025 0.7945 0.7866 0.7788 0.7711 0.7634 0.7558 0.7443
0.3 0.7408 0.7334 0.7261 0.7189 0.7118 0.7047 0.6977 0.6907 0.6839 0.6771
0.4 0.6703 0.6636 0.6570 0.6505 0.6440 0.6376 0.6313 0.6250 0.6188 0.6126
0.5 0.6065 0.6005 0.5945 0.5886 0.5827 0.5770 0.5712 0.5655 0.5599 0.5543
0.6 0.5488 0.5434 0.5379 0.5326 0.5273 0.5220 0.5169 0.5117 0.5066, 0.5016
0.7 0.4966 0.4916 0.4868 0.4819 0.4771 0.4724 0.4677 0.4630 0.4584 0.4538
0.8 0.4493 0.4449 0.4404 0.4360 0.4317 0.4274 0.4332 0.4190 0.4148 0.4107
0.9 0.4066 0.4025 0.3985 0.3946 0.3906 0.3867 0.3829 0.3791 0.3703 0.3716

Note: e–0.4 = 0.6703, e–0.37 = 0.6907, e–0.99 = 0.3716


Table of values of e–μ (0 d μ d 10)
μ 1 2 3 4 5 6 7 8 9 10
0.36788 0.13534 0.04979 0.01832 0.006738 0.00091 0.00012
0.00248 0.00033 0.00040

Note: e–1.5 = e–1 × e–0.5 = 0.36788 × 0.6065 = 0.2231


e–4.34 = e–4 × e–0.34 = 0.01832 × 0.7118 = 0.01304

240 PAGE
© Department of Distance & Continuing Education, Campus of Open Learning,
School of Open Learning, University of Delhi
BUSINESS STATISTICS

Table 2 Notes
Normal Curve Z-score
An entry in the table is the area under the curve between Z = 0 and
a positive value of Z. Areas for negative values of Z arc obtained by
symmetry.
Z 0.00 0.01 0.02 0.03 0.04 0.05 0.06 0.07 0.08 0.09
0.0 0.0000 0.0040 0.0080 0.0120 0.0160 0.0199 0.0239 0.0279 0.0319 0.0359
0.1 0.0398 0.0438 0.0478 0.0517 0.0557 0.0596 0.0636 0.0675 0.0714 0.0753
0.2 0.0793 0.0832 0.0871 0.0910 0.0948 0.0987 0.1026 0.1064 0.1103 0.1141
0.3 0.1179 0.1217 0.1255 0.1293 0.1331 0.1368 0.1406 0.1443 0.1480 0.1517
0.4 0.1554 0.1591 0.1628 0.1664 0.1700 0.1736 0.1772 0.1808 0.1844 0.1879
0.5 0.1915 0.1950 0.1985 0.2019 0.2054 0.2088 0.2123 0.2157 0.2190 0.2224
0.6 0.2257 0.2291 0.2324 0.2357 0.2389 0.2422 0.2454 0.2486 0.2517 0.2549
0.7 0.2580 0.2611 0.2642 0.2673 0.2703 0.2734 0.2764 0.2794 0.2823 0.2852
0.8 0.2881 0.2910 0.2939 0.2967 0.2995 0.3023 0.3051 0.3078 0.3106 0.3133
0.9 0.3159 0.3186 0.3212 0.3238 0.3264 0.3289 0.3315 0.3340 0.3365 0.3389
1.0 0.3413 0.3438 0.3461 0.3485 0.3508 0.3531 0.3554 0.3577 0.3599 0.3621
1.1 0.3642 0.3665 0.3686 0.3708 0.3729 0.3749 0.3770 0.3790 0.3810 0.3830
1.2 0.3849 0.3869 0.3888 0.3907 0.3925 0.3944 0.3962 0.3980 0.3997 0.4015
1.3 0.4032 0.4049 0.4066 0.4082 0.4099 0.4115 0.4131 0.4147 0.4162 0.4177
1.4 0.4192 0.4207 0.4222 0.4236 0.4251 0.4265 0.4279 0.4292 0.4306 0.4319
1.5 0.4332 0.4345 0.4357 0.4370 0.4382 0.4394 0.4406 0.4418 0.4429 0.4441
1.6 0.4452 0.4463 0.4474 0.4484 0.4495 0.4505 0.4515 0.4525 0.4535 0.4545
1.7 0.4554 0.4564 0.4573 0.4582 0.4591 0.4599 0.4608 0.4616 0.4625 0.4633
1.8 0.4641 0.4649 0.4656 0.4664 0.4671 0.4678 0.4686 0.4693 0.4699 0.4706
1.9 0.4713 0.4719 0.4726 0.4732 0.4738 0.4744 0.4750 0.4756 0.4761 0.4767
2.0 0.4772 0.4778 0.4783 0.4788 0.4793 0.4798 0.4803 0.4808 0.4812 0.4817
2.1 0.4821 0.4826 0.4830 0.4834 0.4838 0.4842 0.4846 0.4850 0.4854 0.4857
2.2 0.4861 0.4864 0.4868 0.4871 0.4875 0.4878 0.4881 0.4884 0.4887 0.4890
2.3 0.4893 0.4896 0.4898 0.4901 0.4904 0.4906 0.4909 0.4911 0.4913 0.4916
2.4 0.4918 0.4920 0.4922 0.4925 0.4927 0.4929 0.4931 0.4932 0.4934 0.4936
2.5 0.4938 0.4940 0.4941 0.4943 0.4945 0.4946 0.4948 0.4949 0.4951 0.4952
2.6 0.4953 0.4955 0.4956 0.4957 0.4959 0.4960 0.4961 0.4962 0.4963 0.4964
2.7 0.4965 0.4966 0.4967 0.4968 0.4669 0.4970 0.4971 0.4972 0.4973 0.4974
2.8 0.4974 0.4975 0.4976 0.4977 0.4977 0.4978 0.4779 0.4979 0.4980 0.4981
2.9 0.4981 0.4982 0.4982 0.4983 0.4984 0.4984 0.4985 0.4985 0.4986 0.4986
3.0 0.4987 0.4987 0.4987 0.4988 0.4988 0.4989 0.4989 0.4989 0.4990 0.4990

PAGE 241
© Department of Distance & Continuing Education, Campus of Open Learning,
School of Open Learning, University of Delhi
B.COM. (PROGRAMME)

Notes 2.6 Summary


Probability distributions are obtained primarily on theoretical considerations
and describe how the outcomes of an experiment are expected to vary. A
probability distribution may involve a discrete or a continuous random
variable. The two basic measures to describe a probability distribution
are expected value and standard deviation. For a discrete probability
distribution, expected value, P= Σpx and standard deviation, σ = Σp ( x − x ) 2 .
The binomial, and Poisson distributions involve discrete random variables
while normal distribution involve continuous random variables. A binomial
distribution involves n independent trials, each of which can result in
only two possible outcomes, called success and failure. The probabilities
of various number of successes are given by the binomial expansion (q
+ p)n. The binomial formula is used to calculate the probability of x
successes in n trials. The mean and standard deviation of a binomial
distribution are given by np and npq , respectively. The distribution is
symmetrical if p = q and skewed if p z q. For a given number of trials,
the greater the difference between p and q, more the skewness.
The Poisson distribution is normally used to analyse phenomena that
produce rare occurrences. So, it is called the distribution of rare events.
It is defined by a single parameter m, which is its mean value. Its mean
and variance are equal. A Poisson distribution is positively skewed and
skewness decreases as m increases. The Poisson distribution can also
be used as approximation to binomial distribution when the number of
trials is large and the probability of success in a trial is very small. The
normal distribution is an all important probability distribution. It involves
a continuous variable, has a curve that is unimodal. symmetrical and
asymptotic to the x-axis.
The normal distribution has two parameters – mean and standard deviation.
For every pair of values, there is a distinct normal distribution. The
distribution with mean μ = 0 and standard deviation V = 1 is called
standardised normal distribution. The proportion of area lying in a given
interval to the total area under the normal curve gives the probability
that the variable in question will take a value within that interval. To
obtain probabilities, the given values are transformed into standard

242 PAGE
© Department of Distance & Continuing Education, Campus of Open Learning,
School of Open Learning, University of Delhi
BUSINESS STATISTICS

normal distribution. This is called z-transformation. It is defined as z = Notes


(x – μ)/V. The normal area table is consulted to determine the relevant
area. Normal distribution can be used as an approximation to different
discrete probability distributions like binomial, and Poisson distributions
under appropriate conditions.

2.7 Self-Assessment Exercise


Exercise 1 : True and False Statements
(i) A variable is said to be a random variable if it takes different
values as a result of the outcomes of a random experiment.
(ii) A probability distribution can also be obtained using historical data.
(iii) A discrete random variable can assume any value in a given range
whereas a continuous random variable can assume only isolated
values.
(iv) The expected value, E(x), of a probability distribution is obtained by
summation of the products of the values of x and their corresponding
probabilities.
(v) The two parameters of a binomial distribution are n and p.
(vi) A binomial distribution involves infinite number of trials.
(vii) A binomial distribution with n trials involves a total of n number
of successes.
(viii) A binomial distribution can be completely identified by n and p.
(ix) If mean and standard deviation values are given, we can fit a binomial
distribution.
(x) The variance of a binomial distribution can be smaller than, equal
to or greater than its mean value.
(xi) Regardless of the value of n, a binomial distribution is symmetrical
if p = q.
(xii) A binomial distribution is positively skewed when p > q and
negatively skewed when p < q.
(xiii) A Poisson distribution is positively skewed and the skewness
decreases when m increases.
(xiv) The mean and standard deviation of a Poisson distribution are
always equal.

PAGE 243
© Department of Distance & Continuing Education, Campus of Open Learning,
School of Open Learning, University of Delhi
B.COM. (PROGRAMME)

Notes (xv) A binomial distribution with p = 0.5 results in a uniform distribution


involving discrete variable.
(xvi) Binomial approximation to Poisson distribution is used when n is
very large and p is very small.
(xvii) A normal distribution is defined by mean and standard deviation.
(xviii) The standard normal distribution has μ = 1 and V= 0.
(xix) The smaller the standard deviation, the greater the height of the
normal curve.
(xx) For a normal distribution with μ= 110 and V = 10, the area included
between x = 120 and x = 130 is equal to the area between x = 90
and x = 100.
(xxi) The curve of normal distribution is symmetrical and mesokurtic.
(xxii) μ ± 3V covers 99.27 percent area under the normal curve.
(xxiii) In using normal approximation to the binomial distribution, the
mean and standard deviation are taken to be equal to np and npq,
respectively.
Ans.
(i) T (ii) T (iii) F (iv) T (v) T
(vi) F (vii) F (viii) T (ix) T (x) F
(xi) T (xii) F (xiii) T (xiv) F (xv) F
(xvi) F (xvii) T (xviii) F (xix) F (xx) T
(xxi) T (xxii) F (xxiii) F
Exercise 2 : Questions and Answers
(i) Explain the concept of probability distributions. Give two examples
of how a probability binomial distribution can be obtained.
(ii) Distinguish between discrete and continuous probability distributions.
(iii) What are the conditions under which a binomial distribution is
used? How are the probabilities calculated in case~of a binomial
distribution? Discuss the conditions under which a binomial
distribution can be approximated as (i) a normal distribution and
(ii) a Poisson distribution.
(iv) Write a note on the binomial distribution. In particular, mention
its assumptions, its expected value and variance, and the shape.

244 PAGE
© Department of Distance & Continuing Education, Campus of Open Learning,
School of Open Learning, University of Delhi
BUSINESS STATISTICS

(v) Write a note on Poisson distribution. Under what conditions is it Notes


used as an approximation to the binomial distribution?
(vi) State and explain the properties of a normal curve. Show that the
height of a normal curve at mean is the highest.
(vii) What are the parameters of (i) binomial distribution, (ii) Poisson
distribution, (iii) hypergeometric distribution, and (iv) uniform
distribution?
(viii) Determine the probability of getting three heads in 6 tosses of a
fair coin.
(ix) The probability that a student will graduate next year is 0.4.
Determine the probability that out of five students, each of which
has the same chance of graduating, (i) none, (ii) one, (iii) at least
one, (iv) no more than one, and (v) all will graduate.
(x) The incidence of occupational disease in an industry is such that
the workmen have a 30 per cent chance of suffering from it. What
is the probability that out of 8 workmen, 6 or more will contact
the disease?
(xi) A factory finds that, on an average, 20 percent bolts are defective
from one machine. If 10 bolts are selected at random, find the
probability that:
(a) Exactly two bolts are defective,
(b) Less than two bolts are defective.
(c) More than two bolts are defective.
(d) More than eight bolts are defective.
(xii) A firm produces a product and finds that 10 per cent of its output
is defective. A small sample of 5 items is taken from the production
line. Find the probability of getting each of the following number
of defective items in the sample: 0, 1, 2, 3, 4, and 5.
(xiii) Find the probability of getting a 5 or 6 thrice in five tosses of an
unbiased die.
(xiv) A sign on the gas pumps of a chain of gasoline stations encourages
customers to have their oil checked, claiming that one out of five
cars needs to have oil added. If this is true, what is the probability
of the following events?

PAGE 245
© Department of Distance & Continuing Education, Campus of Open Learning,
School of Open Learning, University of Delhi
B.COM. (PROGRAMME)

Notes (a) One of the next four cars needs oil.


(b) Two out of the next eight cars need oil.
(xv) The mean of a binomial distribution is 20 and its standard deviation
is 4. Calculate the values of n. p and q.
(xvi) Assuming the binomial distribution applies, find the expected value,
variance, and standard deviation of the distribution determined by
n = 80 and p = 0.6.
(xvii) Calculate the probabilities of 0, 1. 2, 3, 4 and 5 heads on the toss
of a set of five balanced coins. Also, obtain the mean and standard
deviation of the distribution.
(xviii) Bring out the fallacy, if any, in the following statements:
(a) The mean of a binomial distribution is 6 and its standard
deviation is 3.
(b) The mean of a binomial distribution is 3 and its variance is
4.
(xix) For a binomial variable x, it is given that n = 8, and P(x – 2) = 16
P(x = 6). Determine the values of p and q.
(xx) If the probability of a defective bolt is 0.2, find the mean and
standard deviation of the number of defective bolts in a total of
900 such bolts.
(xxi) (a) In a binomial distribution involving 5 independent trials, probabilities
of 1 and 2 successes are 0.4096 and 0.2048, respectively. Find the
parameter p of the distribution.
(b) In a binomial distribution with 6 independent trials, the probabilities
of 3 and 4 successes are found to be 0.2457 and 0.0819, respectively.
Find mean and variance of this distribution.
(xxii) The administrative officer of a nursing home reports that the number
of patients admitted to the ICU on any day follows a Poisson
probability law, with a mean of 7. What is the probability that on
a given day:
(a) No patient will be admitted?
(b) Exactly seven patients will be admitted?
(c) No more than three patients will be admitted?
(xxiii) Assume that the number of network errors experienced in a day
on a local area network (LAN) is distributed as Poisson random

246 PAGE
© Department of Distance & Continuing Education, Campus of Open Learning,
School of Open Learning, University of Delhi
BUSINESS STATISTICS

variable. The average number of network errors experienced in a Notes


day is 2.4. What is the probability that in any given day
(a) Exactly one network error will result?
(b) Two or more network errors will result?
(xxiv) Assume the mean height of soldiers to be 68.22 inches with variance
of 10.8 inches, how many soldiers in a regiment of 1400 would
you expect to be taller than 72 inches?
(xxv) In an examination 10% of students passed with distinction, 60%
passed and 30% failed. If it is given that a candidate needs 40%
marks to pass and 75% marks to pass with distinction, determine
mean and standard deviation of the distribution of marks assuming
the marks are distributed normally.
(xxvi) A project yields an average case flow of Rs. 550 lakh and standard
deviation of Rs. 110 lakh.
Compute the following probabilities :
(a) Cash flow will be more than Rs. 675 lakh,
(b) Cash flow will be less than Rs. 450 lakh and
(c) Cash flow will be between Rs. 425 and Rs. 750 lakh.
Ans.
(viii) 0.3125 (ix) 0.0778, 0.2592, 0.92224,
0.3370, 0.01024
(x) 0.1129 (xi) 0.302, 0.3758, 0.3222,
0.0000042
(xii) 0.5905, 0.3281, 0.0729, (xiii) 0.165
0.0881, 0.00045, 0.00001
(xiv) 0.4096, 0.1536 (xv) 100, 0.2, 0.8
(xvi) 48, 19.2, 4.38 (xvii) 1/32, 5/32, 10/32, 10/32,
5/32, 1/32, 2.5, 1.12
(xviii) q > 1 in each case (xix) 1/3 and 2/3
(xx) 180, 12 (xxi) 0.25, 24/13, 216/169
(xxii) 0.0009, 0.149, 0.0818 (xxiii) 0.2177, 0.6916
(xxiv) 175 (xxv) 50.17, 19.4
(xxvi) 0.1271, 0.1814, 0.8385

PAGE 247
© Department of Distance & Continuing Education, Campus of Open Learning,
School of Open Learning, University of Delhi
L E S S O N

3
Statistical Decision Theory
STRUCTURE
3.1 Learning Objectives
3.2 Probability in Decision Making
3.3 Decision Making Process
3.4 Decision Under Uncertainty
3.5 Decision Under Risk
3.6 Expected Value of Perfect Information (EVPI)
3.7 Decision Tree
3.8 Summary
3.9 Self-Assessment Questions

3.1 Learning Objectives


After reading this lesson, you would be able to:
‹ Understand the steps of decision-making process.
‹ Comprehend the concepts of states of nature and courses of action and compute payoff
and regret tables.
‹ Learn about various decision-making rules.
‹ Compute EVPI and take decisions with the help of Decision tree analysis.

3.2 Probability in Decision Making


Statistical techniques are being widely used to solve business problems. These techniques
are being used to solve problems for which information is incomplete, uncertain or in
some cases almost completely lacking. This new area of statistics is known as statistical
decision theory. In decision theory we must decide among alternatives by taking into
account the monetary considerations of our actions. A manager who wants to select from
among a number of available investment alternatives should consider the profit or loss that

248 PAGE
© Department of Distance & Continuing Education, Campus of Open Learning,
School of Open Learning, University of Delhi
BUSINESS STATISTICS

might result from each alternative. Decision theory involves selecting an Notes
alternative and having a reasonable idea of the economic consequences
of choosing that action.
Decision theory may be applied to problems whether the time span is one
day or five years, whether it involves financial management or a plant
assembly line. Most of these problems have common characteristics. The
elements common to most decision theory problems are: (i) An objective,
(ii) Several courses of actions, (iii) A calculable measure of the benefit
or worth of various alternatives, (iv) Events beyond the control of the
decision maker, and (v) Uncertainty about which outcome or state of
nature will actually happen.
Most complex managerial decisions are made with some uncertainty.
Managers authorize substantial capital investments with incomplete knowledge
about product demand. When decisions are made under uncertain future
conditions, use of probabilities provides us with a rational technique for
making choices.
Example 1 : A bakery provides cakes at a cost of Rs. 6 and sells them
for Rs. 10 each. A cake not sold on a particular day is worthless. The
baker’s problem is to determine the optimum number of cakes to be made
each day. On days when his stock is more than his sales his profits are
reduced by the cost of unsold cakes. On days of demand exceeding his
stock he loses sales and makes smaller profits than he could have. He
has kept a record of his sales for past 100 days to tell him about the
historical pattern of sales.
Daily sales of cakes No. of days sold
300 15
400 20
500 45
600 15
700 5
100
Solution : On the basis of above information we can assign probabilities
of sale of cakes in different quantities. For example a probability of
0.45 is assigned to the sale figure of 500 cakes. The table assigning
probabilities to various quantities can be prepared.

PAGE 249
© Department of Distance & Continuing Education, Campus of Open Learning,
School of Open Learning, University of Delhi
B.COM. (PROGRAMME)

Notes Daily sales (events) No. of days sold Probabilities of each


number being sold
E1 300 15 0.15
E2 400 20 0.20
E3 500 45 0.45
E4 600 15 0.15
E5 700 5 0.05
100 1.00
We can make a table of net benefits that accrue to the decision maker
from a given combination of Act and Event. This is known as pay off
matrix. For the given example we can make the pay off table or matrix.
When the baker sells one cake he get a profit of Rs. (10 – 6) = Rs. 4
and when a cake remains unsold he loses Rs. 6. The pay off matrix is
given below :
Production
(Decision maker’s alternative)
A1 A2 A3 A4 A5
Units 300 400 500 600 700
E1 300 Rs. 1200 Rs. 600 0 (600) (1200)£
E2 400 1200 1600* 1000 400 (200)£
E3 500 1200 1600 2000* 1400 800£
E4 600 1200£ 1600 2000 2400* 1800
E5 700 1200£ 1600 2000 2400 2800*
We have calculated the profits or losses for various situations, when
demand and production both match, there is maximum profit but when
production exceeds demand profit reduces because the unsold quantity
is thrown and even the cost not recovered e.g., when both production
and demand is 300 units, there is a payoff (profit) of 300 × 4 = Rs.
1200. When demand is 400 but production is 300 units then also there
is profit of 1200 because he can sell only 300 units and earn (300 × 4 =
Rs. 1200). But when production is 400 units and demand 300 units, he
earns 300 × 4 = Rs. 1200 but looses 100 × 6 = Rs. 600 on account of
unsold quantity of 100 cakes and net pay off is just Rs. 600 i.e. (1200
– 600). When production is 500 units and sales 300 units he neither

250 PAGE
© Department of Distance & Continuing Education, Campus of Open Learning,
School of Open Learning, University of Delhi
BUSINESS STATISTICS

earns nor loses because by selling 300 units he makes a profit of 300 × Notes
4 = Rs. 1200. He loses Rs. 1200 cost of 200 units that remained unsold
and the net result is zero profit. But when he produces 600 cakes and
sells only 300 he incurred a loss of Rs. 600 [(300 × 4) – (300 × 6) =
1200 – 1800 = – 600].

3.3 Decision Making Process

As indicated, the decision theory is used to determine optimal strategy


where a decision-maker is faced with several decision alternatives and
an uncertain pattern of future events. All decision-making situations are
characterised by the fact that two or more alternative courses of action are
available to the decision-maker to choose from. Further, a decision may
be defined as the selection by the decision-maker of an act, considered
to be best according to some pre-designated standard, from among the
available options. The decision-making process uses the following steps:
(a) Identification of the various possible outcomes, called states of nature
or events. The events are beyond the control of the decision-maker.
(b) Identification of all the courses of action, Aj, or the strategies which
are available to the decision-maker. The decision-maker has control
over choice of these.
(c) Determination of the pay-off function which describes the consequences
resulting from different combinations of the acts and events.
(d) Choosing from among the various alternatives on the basis of some
criterion, which may involve the information given in step (c) only
or some additional information.
Example 2 : A toy making company is bringing out a new type of toy. It
is considering whether to bring out a full, partial or minimal product line.
The company has three levels of product acceptance. The management
will make its decision on the basis of anticipated profit from the first
year of production. The relevant data are given in the table below:

PAGE 251
© Department of Distance & Continuing Education, Campus of Open Learning,
School of Open Learning, University of Delhi
B.COM. (PROGRAMME)

Notes Anticipated Profit


Product Product Line
Acceptance Full Partial Minimal
Good Rs. 80,000 Rs. 70,000 Rs. 50,000
Fair Rs. 50,000 Rs. 45,000 Rs. 40,000
Poor Rs.–25,000 Rs.–10,000 Rs. 0
Solution : Take optimal decision using various decision criteria.
Let us first analyse the given information.
States of nature: The states of nature or events here are in terms of the
product acceptance which may be good, fair or poor. The management
has no control over this aspect.
Courses of actions: The management has to accept one of the product
lines: full, partial or minimal. These are the choices available out of which
one has to be adopted. The courses of action are also called simply acts
or strategies. The decision-maker has a control over these.
Pay-off table: A pay-off table explains the economics of the given problem.
A pay-off is a conditional value or profit/loss, or a conditional cost. It is
conditional in the sense that a certain profit/loss is associated with each
course of action. Thus, the profit or loss resulting from the adoption of
a certain strategy is dependent upon particular event that may occur. To
illustrate, if the management goes in for full product line and the product
acceptance is good, the company would make a profit of Rs. 80,000.
The given pay-off values are shown in Table 1 with additional information.
Regret or Opportunity Loss Table: The outcomes of various combinations
of acts and events can also be expressed in terms of regret or opportunity
loss values. The regret values are also conditional as each one of them
results from a certain combination of act and event. Regret is defined
as the amount of pay-off foregone by not adopting the optimal course
of action – that which would yield the highest pay-off, for each given
event. To illustrate, in the context of our example, in the event of good
product acceptance, the full product line would yield a profit of Rs.
80,000; a partial product line would result in a profit of Rs. 70,000; while
a minimal product line would give a profit of Rs. 50,000. The optimal
course of action in this case would be full product line. Thus, at the end
of the year, if the product acceptance was good, the management would

252 PAGE
© Department of Distance & Continuing Education, Campus of Open Learning,
School of Open Learning, University of Delhi
BUSINESS STATISTICS

not regret if it had gone for full product line. In case it had decided Notes
for partial product line, it would regret somewhat while a decision of
minimal product line would cause a greater regret. The regret would be
Rs. 10,000 (= 80,000 – 70,000) and Rs. 30,000 (= 80,000 – 50,000),
respectively, with these two policies.
Table 1 : Decision-making Using Different Criteria
Anticipated Profit (in Rs.)
Act
Event Full Product Partial Minimal
Line Product Line Product Line
Good Product Acceptance 80,000 70,000 50,000
Fair Product Acceptance 50,000 45,000 40,000
Poor Product Acceptance –25,000 –10,000 0
Maximum 80,000 70,000 50,000
Minimum –25,000 –10,000 0
Average 35,000 35,000 30,000
Table 2 : Conditional Regret Table
Act
Event Full Product Partial Minimal
Line Product Line Product Line
Good Product Acceptance 0 10,000 30,000
Fair Product Acceptance 0 5,000 10,000
Poor Product Acceptance 25,000 10,000 0
The elements of a decision process are:
A decision-maker.
A set of possible outcomes, or events, in the decision situation.
A set of courses of action available to the decision-maker.
A set of conditional pay-offs corresponding to various possible combinations
of events and actions.
Selection of a particular course of action based on some criterion.
After setting up the pay-off table and the regret table we proceed to take
a decision. There are several rules, or criteria, on the basis of which
decision may be taken. The selection of an appropriate criterion depends
on factors such as the nature of decision situation, attitude of the decision-

PAGE 253
© Department of Distance & Continuing Education, Campus of Open Learning,
School of Open Learning, University of Delhi
B.COM. (PROGRAMME)

Notes maker etc. We shall first discuss the decision rules for taking decisions
in conditions of uncertainty and then for conditions of risk.

3.4 Decisions Under Uncertainty


The decision situations where there is no way in which the decision-maker
can assess the probabilities of the various states of nature are called
decisions under uncertainty. In such situations, the decision-maker has no
idea as to which of the possible states of nature would occur nor has he
a reason to believe why a given state is more, or less, likely to occur as
another. With probabilities of the various outcomes not known, the actual
decisions are based on specific criteria. The several principles which may
be employed for taking decisions in such conditions are discussed below.

3.4.1 Maximum Criterion


An optimist believes that whatever course of action he chooses from all
possible outcomes would be the best. This rule suggests, that for each
strategy, the maximum pay-off should be considered. Further, the maximum
of these pay-offs should be chosen for decision. For our example, the
maximum profit associated with different courses of action is as follows:
Full product line: Rs. 80,000; Partial product line: Rs. 70,000; and
Minimal product line: Rs. 50,000.
The highest of these corresponds to the full product line. Therefore,
optimal decision is to go for full product line.

3.4.2 Minimum Criterion


This principle is adopted by pessimistic decision-makers who are conservative
in their approach. A pessimist is one who believes that whatever course
of action he chooses would be the worst out of all the possible outcomes.
Using this approach, the minimum pay-offs from various strategies are
considered and the maximum one is selected. It therefore chooses the
best (the maximum) profit from the set of worst (the minimum) profits.
The minimum pay-off associated with the full, partial and minimal product
line is Rs. (–) 25,000, Rs. (–) 10,000 and Rs. 0, respectively. According
to this criterion the management would go for minimal product line.

254 PAGE
© Department of Distance & Continuing Education, Campus of Open Learning,
School of Open Learning, University of Delhi
BUSINESS STATISTICS

3.4.3 Laplace Criterion Notes

The Laplace principle is based on the simple rule that if we are uncertain
about various events, then we may treat them as equally probable.
Therefore the expected value of pay-off for each strategy is calculated
and the strategy with the highest mean value is adopted. The expected
pay-offs for various courses of action are calculated as:
Full product line : (80,000 + 50,000 – 25,000)/3 = Rs. 35,000
Partial product line : (70,000 + 45,000 – 10,000)/3 = Rs. 35,000
Minimal product line : (50,000 + 40,000 + 0)/3 = Rs. 30,000
Since the highest expected pay-off is shared by the strategies of full product
line and partial product line, both could be adopted by the management.

3.4.4 Maximum Regret Principle


The Minimax Regret principle is based on the concept of regret. According
to the principle the course of action that minimises the maximum regret
will be selected. It is known as savage principle.
First the regret matrix is derived from the pay-off matrix. Then the maximum
regret value corresponding to each of the strategies is determined and the
strategy which minimises the maximum regret is chosen. The principle of
choice is also conservative in approach and is very close to the minimax
principle applied to the original matrix containing pay-off values.
From the regret matrix given in Table 2 we get the following
maximum regret values associated with the various couses of action:
Full product line : Rs. 25,000
Partial product line : Rs. 10,000
Minimal product line : Rs. 30,000
The maximum regret value for the strategy of partial product line is
minimum and will be the optimal choice.
When probabilities of various events are not given, the decision criteria
could be:
1. Considering best of the best pay-off for every act.
2. Considering best of the worst pay-off for every act.

PAGE 255
© Department of Distance & Continuing Education, Campus of Open Learning,
School of Open Learning, University of Delhi
B.COM. (PROGRAMME)

Notes 3. Considering best of the weighted average of the best and worst pay-
offs under every act.
4. Considering best of the simple average of all pay-offs under every
act.
5. Considering act with least of maximum regret values associated with
all acts.

3.5 Decision Under Risk


The decision situations wherein the decision-maker chooses to consider
several possible outcomes and the probabilities of their occurrence can
be stated are called decisions under risk. The probabilities of various
outcomes may be given or they may be determined from the past records.
Under conditions of risk, there are generally two criteria to choose from.
(a) Expectation Criterion (b) Expected opportunity loss or expected regret

3.5.1 Expectation Crietrion


Decision making in situations of risk is on the basis of the expectation
principle with the event probabilities assigned. The expected pay off
for each strategy is calculated by multiplying the pay off values with
their respective probabilities and then adding up these products. The
strategy with the highest expected payoff represents the optimal choice.
Symbolically, where aijŸ the pay off resulting from the combinations
of ith event and jth act. pi represents the probability of ith event.
In the example, we are given probabilities of various events and we can
determine the expected pay offs.
Events Probabilities A1 A2 A3 A4 A5
(Pi) 300 400 500 600 700
E1 300 0.15 1200 600 0 –600 –1200
E2 400 0.20 1200 1600 1000 400 200
E3 500 0.45 1200 1600 2000 1400 800
E4 600 0.15 1200 1600 2000 2400 1800
E5 700 0.05 1200 1600 2000 2400 2800

256 PAGE
© Department of Distance & Continuing Education, Campus of Open Learning,
School of Open Learning, University of Delhi
BUSINESS STATISTICS

The calculation of expected pay off values EP for various acts is shown Notes
below:—
for A1, EP1 = 0.15 × 1200 + 0.20 × 1200 + 0.45 × 1200 + 0.15 × 1200
+ 0.05 × 1200 = Rs. 1200
for A2, EP2 = 0.15 × 600 + 0.20 × 1600 + 0.45 × 1600 + 0.15 × 1600
+ 0.05 × 1600 = Rs. 1450
for A3, EP3 = 0.15 × 0 + 0.20 × 1000 + 0.45 × 2000 + 0.15 × 2000 +
0.05 × 2000 = Rs. 1500
for A4, EP4 = 0.15 × –600 + 0.20 × 400 + 0.45 × 1400 + 0.15 × 2400
+ 0.05 × 2400 = Rs. 1100
for A5, EP5 = 0.15 × –1200 + 0.20 × –200 + 0.45 × 800 + 0.15 × 1800
+ 0.05 × 2800 = Rs. 550
Since maximum expected pay off is associated with strategy A3, the best
course of action is to produce 500 cakes.

3.5.2 Expected Opportunity Loss


The expected opportunity loss or expected regret criterion is another
basis on which a decision may be taken. For this purpose opportunity
loss matrix alongwith probability distribution can be reproduced as given
below:
Calculation of expected Regret:
Acts
Event Probability A1 A2 A3 A4 A5
(Pi) 300 400 500 600 700
E1 300 0.15 0 600 1200 1800 2400
E2 400 0.15 400 0 600 1200 1800
E3 500 0.15 800 400 0 600 1200
E4 600 0.15 1200 800 400 0 600
E5 700 0.15 1600 1200 800 400 0
Now we can determine expected regret (ER) for various strategies
for A1, ER1 = 0.15 × 0 + 0.20 × 400 + 0.45 × 800 + 0.15 × 1200 + 0.05
× 1600 = Rs. 4700

PAGE 257
© Department of Distance & Continuing Education, Campus of Open Learning,
School of Open Learning, University of Delhi
B.COM. (PROGRAMME)

Notes for A2, ER2 = 0.15 × 600 + 0.20 × 0 + 0.45 × 400 + 0.15 × 800 + 0.05
× 1200 = Rs. 450
for A3, ER3 = 0.15 × 1200 + 0.20 × 600 + 0.45 × 0 + 0.15 × 400 + 0.05
× 800 = Rs. 400
for A4, ER4 = 0.15 × 1800 + 0.20 × 1200 + 0.45 × 600 + 0.15 × 0 +
0.05 × 400 = Rs. 800
for A5, ER5 = 0.15 × 2400 + 0.20 × 1800 + 0.45 × 1200 + 0.15 × 600
+ 0.05 × 0 = Rs. 1350
Under this criterion, the optimal strategy is the one which minimizes
the expected regret. Since the minimum value occurs at A3 it represents
the optimal decision. This is same as under expected pay off criterion.

3.6 Expected Value of Perfect Information (EVPI)


Assuming that we can obtain a perfect prediction about the future and also
the cost of this information, we can compare the cost of that information
with the additional profit we would realize as a result of having the
information. The difference between expected cost with optimal policy
and expected cost with perfect information is known as EPVI.
In our example if the bakery shop knows that next year’s demand is
going to be 300 units with probability 0.15 and a profit of Rs. 1200. We
calculate expected profit 1200 × 0.15 = Rs. 180. When the shop manager
knows that demand will be 400 units he will earn Rs. 1600 and since the
probability of this event is 0.20 his expected profit will be 0.20 × 1600
= Rs. 320. Similarly the expected pay off for each level of demand can
be obtained and aggregated. These expected profits we get are EVPI.
EVPI = 0.15 × 1200 + 0.20 × 1600 + 0.45 × 2000 + 0.15 × 2400 + 0.05
× 2800 = Rs. 1900
Alternatively, EVPI = EPj + ERj
For A1, EP1 = 1200 and ER1 = 700 and EVPI = 1200 + 700 = Rs. 1900
For A2, EP2 = 1450 and ER2 = 450 and EVPI = 1450 + 450 = Rs. 1900
For A3, EP3 = 1500 and ER3 = 400 and EVPI = 1500 + 400 = 1900 and
similarly for A4 and A5.

258 PAGE
© Department of Distance & Continuing Education, Campus of Open Learning,
School of Open Learning, University of Delhi
BUSINESS STATISTICS

Example 3 : A grocery shop is faced with the problem of how many Notes
cakes to buy in order to meet the day’s demand. The left over cakes are
a total loss. If the customer’s demand is not satisfied, the sales will be
lost. The shopkeeper has got the information regarding past sales for
past 200 days:
Sales per day No. of days Probability
25 20 0.10
26 60 0.30
27 100 0.50
28 20 0.10
(i) Prepare the payoff matrix and opportunity loss (regret) matrix.
(ii) Find the optimal number of cakes that should be bought each day.
(iii) Find EVPI.
The cost of a cake is Rs. 8 and it is sold for Rs. 10 each.
Solution :
(i) Profit = (Cakes sold × selling price) – (Cakes unsold × cost price)
Opportunity loss (regret) = Maximum profit in a row – Profit under
each column in that row.
Pay off Table (Rs.)
25 26 27 28 Probability
25 50 42 34 26 0.10
26 50 52 44 36 0.30
27 50 52 54 46 0.50
28 50 52 54 56 0.10
EMV 50 51 49 42
Regret Table (Rs.)
25 26 27 28 Probability
25 0 8 16 24 0.10
26 2 0 8 16 0.30
27 4 2 0 8 0.50
28 6 4 2 0 0.10
EOL 3.20 2.20 4.20 11.20
(ii) Now we can calculate expected monetary value and expected
opportunity loss

PAGE 259
© Department of Distance & Continuing Education, Campus of Open Learning,
School of Open Learning, University of Delhi
B.COM. (PROGRAMME)

Notes EMV = 6 (Profit Column × Probability column)


EOL = 6 (Regret Column × Probability column)
Max EMV is 52 corresponding to 26 cakes and min. EOL is 2.20
corresponding to 26 cakes
? optimum number of cakes to be purchased is 26
(iii) EVPI = (Max. Pay off in each row × Corresponding Probability) –
Max. EMV
= (50 × .10 + 52 ×.30 + 54 × .50 + 56 × .10) – 51
= (5 + 15.60 + 27 + 5.60) – 51 = 53.20 – 51 = Rs. 2.20
Example 4 : The payoff (in Rs.) of three acts A1, A2 and A3 and the
possible states of nature S1, S2 and S3 are given below:
Acts
States of Nature A1 A2 A3
S1 –20 –50 200
S2 200 –100 –50
S3 400 600 300
The probabilities of the states of nature are 0.3, 0.4 and 0.3 respectively.
Determine the optimal act using the expectation principle.
Solution :
Acts
States of Nature A1 A2 A3 Probabilities
S1 –20 –50 200 0.3
S2 200 –100 –50 0.4
S3 400 600 300 0.3
Expected profit for Act A1 = – 20 × 0.3 + 200 × 0.4 + 400 × 0.3
= – 6 + 80 + 120 = Rs. 194
Expected profit for Act A2 = – 50 × 0.3 – 100 × 0.4 + 600 × 0.3
= – 15 – 40 + 180 = Rs. 125
Expected profit for Act A3 = 200 × 0.3 – 50 × 0.4 + 300 × 0.3
= 60 – 20 + 90 = Rs. 130
? Act A1 is the Optimal Act.

260 PAGE
© Department of Distance & Continuing Education, Campus of Open Learning,
School of Open Learning, University of Delhi
BUSINESS STATISTICS

Example 5 : Each unit of a product produced and sold yields a profit of Notes
Rs. 50 but a unit produced but not sold results in a loss of Rs. 30. The
probability distribution of the number of units demanded is as follows:
No. of Units Demanded Probability
0 0.20
1 0.20
2 0.25
3 0.30
4 0.05
How many units be produced to maximise the expected profits? Also
calculate EVPI.
Solution :
Given : Profit for units produced and sold = Rs. 50
Loss for units produced and not sold = Rs. 30
Pay Off Table
(Production)
De- Proba- 0 EMV 1 EMV 2 EMV 3 EMV 4 EMV
mand bility
0 0.2 0 0 (30) (6) (60) (12) (90) (18) (120) (24)
1 0.2 0 0 50 10 20 4 (10) (2) (40) (8)
2 0.25 0 0 50 12.50 100 17.50 70 17.50 40 10
3 0.2 0 0 50 15 100 45 150 45 120 36
4 0.05 0 0 50 2.50 100 7.50 150 7.50 120 10
Total 0 34 52 50 42
Note: Values in brackets are negative.
We should produce 2 units because EMV = Rs. 52 (maximum).
Further, EVPI = EPPI – EMV
EPPI Table
Demand Probability Max. pay off EPPI
0 0.2 0 0 × 0.2 = 0
1 0.2 50 50 × 0.2 = 10
2 0.25 100 100 × 0.25 = 25
3 0.30 150 150 × 0.30 = 45
4 0.05 200 200 × 0.05 = 10
Total 90
? EVPI = 90 – 52 = Rs. 38

PAGE 261
© Department of Distance & Continuing Education, Campus of Open Learning,
School of Open Learning, University of Delhi
B.COM. (PROGRAMME)

Notes Example 6 : A physician purchases a particular vaccine on Monday


of each week. The vaccine must be used within the week following,
otherwise it becomes worthless. The vaccine costs Rs. 2 per dose and
the physician charges Rs. 4 per dose. In the post 50 weeks, the physician
has administered the vaccine in the following quantities:
Doses per Week Number of Weeks
20 5
25 15
50 25
60 5
On the basis of EMV, find how many doses the physician must purchase
each week to maximise his profit?
Solution : Given : Cost = Rs. 2.00, Price = Rs. 4.00
Profit = Rs. 4 – Rs. 2 = Rs. 2.00 per dose
We shall calculate pay off for different actions:
P11 (Demand/Dose = 20) = 20 × 2 = Rs. 40
P12 = 40 – 10 = 30
P13 = 40 – 60 = –20
P14 = 40 – 80 = – 40
P21 = 40 – 0 = 40
P22 = 50 – 0 = 50
P23 = 50 – 50 = 0
P24 = 50 – 70 = –20
P31 = 40 – 0 = 40
P32 = 50 = 50
P33 = 50 × 2 = 100
P34 = 100 – 20 = 80
P41 = 20 × 2 = 40
P42 = 25 × 2 = 50
P43 = 50 × 2 = 100
P44 = 60 × 2 = 120

262 PAGE
© Department of Distance & Continuing Education, Campus of Open Learning,
School of Open Learning, University of Delhi
BUSINESS STATISTICS

Pay Off Table Notes


Probability Doses per week A1 A2 A3 A4
Demand 20 25 50 60
(5/50) = 0.1 20 40 30 –20 –40
(15/50) = 0.3 25 40 50 0 –20
(25/50) = 0.5 50 40 50 100 80
(5/50) = 0.1 60 40 50 100 120
n
? EMV for A =
1
∑ pi xi
x =1

= (0.1 × 40) + (0.3 × 40) + (0.5 × 40) + (0.1 × 40)


= 4 + 12 + 20 + 4 = Rs. 40
EMV for A2 = (0.1 × 30) + (0.3 × 50) + (0.5 × 50) + (0.1 × 50)
= 3 + 15 + 25 + 5 = Rs. 48
EMV for A3 = (0.1 × –20) + (0.3 × 0) + (0.5 × 100) + (0.1 × 100)
= – 2 + 0 + 50 + 10 = Rs. 58
EMV for A4 = (0.1 × –40) + (0.3 × –20) + (0.5 × 80) + (0.1 × 120)
= – 4 – 6 + 40 + 12 = Rs. 42
The physician should purchase 50 doses each week because EMV for A3
is Rs. 58 (Maximum).

3.7 Decision Tree


It is quite useful to represent the structure of a decision problem under
uncertainty by a ‘decision tree diagram’ or by “decision tree.”
A decision tree is a graphic representation of decision process. We can
introduce probabilities into the analysis of complex decisions involving (i)
many alternatives, and (ii) future conditions that are not known but can
be specified in terms of a set of probabilities. The decision tree analysis
helps in making decision concerning a wide variety of problems such as
project management, personnel, new product strategies, acquisition or
disposal of physical properties, investments, etc.

PAGE 263
© Department of Distance & Continuing Education, Campus of Open Learning,
School of Open Learning, University of Delhi
B.COM. (PROGRAMME)

Notes Decision trees have standard symbols, squares symbolize decision points
and circles represent chance events. From each square and circle branches
are drawn. These represent each possible outcome or state of nature.
Steps in Decision Tree Analysis
In a decision tree analysis, the decision-maker follows the following six
steps:
1. Define the Problem in Structured Terms: First of all, the factors
relevant to the solution should be determined. Then probability
distributions that are appropriate to describe future behaviour of
those factors are estimated.
2. Model the Decision Process: A decision tree that illustrates all the
alternatives in a problem is constructed. The entire decision process
is presented in an organised step-by-step procedure.
3. Apply the Appropriate Probability Values and Financial Data:
To each of the branches and sub-branches of the decision tree
the appropriate probability values and financial data are applied.
This will help us to distinguish between the probability value and
conditional monetary value associated with each outcome.
4. “Solve” the Decision Tree: Using the method explained above locate
that particular branch of the tree that has the largest expected value
or that maximises the decision criteria.
5. Perform Sensitivity Analysis: Determine how the solution reacts
to changes in inputs. Changing probability value and conditional
financial values, enables the decision maker to test the magnitude
and the direction of the reaction.
6. List the Underlying Assumptions: The accounting, cost finding and
other assumptions used to arrive at a function should be explained.
This will help others to know what risks they are taking when they
use the results of decision tree analysis.
Advantages of Decision Tree Approach
The decision tree analysis is important because of the following:
1. It structures the decision process enabling decisions to be made in
an orderly manner.

264 PAGE
© Department of Distance & Continuing Education, Campus of Open Learning,
School of Open Learning, University of Delhi
BUSINESS STATISTICS

2. It requires the decision-maker to examine all possible outcomes – Notes


desirable and undesirable.
3. It communicates the decision-making process to others.
4. It allows a group to discuss alternatives by focusing on each financial
figure, probability value and underlying assumptions – one at a time
to arrive at a consensus decision instead of debating that decision
entirely.
5. It can be used with a computer so that many different sets of
assumptions can be simulated and their effects on the final outcomes
can be analysed.
Example 7 : There is 40% chance that a patient admitted to hospital is
suffering from cancer. A doctor has to decide whether an operation should
be performed or not. If the patient is suffering from cancer and the serious
operation is performed, the chance that he will recover is 70%, otherwise
it is 35%. On the other hand, if the patient is not suffering from cancer
and the operation is performed, the chance that he will recover is 20%
otherwise it is 100%. Assume that recovery and death are the only two
possible outcomes. What decision should the doctor take?
Solution :

The chance of recovery after operation = 0.28 + 0.12 = 0.40


The chance of recovery without operation = 0.14 + 0.60 = 0.74
Since, the chances of recovery without operation > chances of recovery
with operation.
Therefore, the doctors should not undertake operation.

PAGE 265
© Department of Distance & Continuing Education, Campus of Open Learning,
School of Open Learning, University of Delhi
B.COM. (PROGRAMME)

Notes Example 8 : A businessman has two independent investments A and


B available to him but he lacks the capital to undertake both of them
simultaneously. Investment A requires capital of Rs. 30,000 and investment
B Rs. 50,000. Market survey shows: High, medium and low demands with
corresponding probabilities of 0.4, 0.4 and 0.2 respectively in case of
investment A and 0.3, 0.4 and 0.3 for investment B. Returns from investment
A are Rs. 75,000, Rs. 55,000 and Rs. 35,000 and corresponding figures
for investment B are likely to be Rs. 100,000, Rs. 80,000 and Rs. 70,000
for high, medium and low demand respectively. What decision should
the company take? Decide by constructing an appropriate decision-tree.
Solution :

Expected net profit for investment A = 0.4 × 75,000 + 0.4 × 55,000 + 0.2
× 35,000 – 30,000 = Rs. 29,000
Expected net profit for investment B = 0.3 × 100,000 + 0.4 × 80,000 + 0.3
× 70,000 – 50,000 = Rs. 33,000
Since the expected net profit for investment B is more than A, the
businessman should invest in B.
Example 9 : A person has two independent investments A and B available
to him, but he can undertake only one at a time due to certain constraints.
He can choose A first and then stop, or if A is successful, then take B
or vice versa. The probability of success of A is 0.6 while for B it is
0.4. Both investments require an initial capital outlay of Rs. 10,000 and
both return nothing if the venture is unsuccessful. Successful completion
of A will return Rs. 20,000 (over cost) and successful completion of B

266 PAGE
© Department of Distance & Continuing Education, Campus of Open Learning,
School of Open Learning, University of Delhi
BUSINESS STATISTICS

will return Rs. 24,000 (over cost). Draw a decision-tree and determine Notes
the best strategy.

Evaluation of Decision Points


Decision Outcome Probability Conditional Expected
point values values
D3 (i) Accept A Success 0.6 20,000 12,000
Failure 0.4 –10,000 –4,000
8,000
(ii) Stop — — — —
D2 (i) Accept B Success 0.4 24,000 9,600
Failure 0.6 –10,000 –6,000
3,600
(ii) Stop — — — —
D1 (i) Accept A, Success 0.6 20,000 + 14,160
3,600
then B Failure 0.4 –10,000 –4,000
10,160
(ii) Accept B, Success 0.4 24,000 + 12,800
8,000
then A Failure 0.6 –10,000 –6,000
6,800
EMV is highest when he accepts investment A and on success invests
money in B.

PAGE 267
© Department of Distance & Continuing Education, Campus of Open Learning,
School of Open Learning, University of Delhi
B.COM. (PROGRAMME)

Notes 3.8 Summary


Decision theory is concerned with decision-making under conditions of
uncertainty. The decision process involves the steps of identification of
states of nature, courses of action and pay-off table depicting consequences
resulting from their interaction, and then choosing the appropriate action
on the basis of given criterion, pay-offs, the outcome of event-action
combinations can also be expressed in terms of regret. Among the rules
for taking decisions are (a) maximax/minimin: Where we select best of
the best; (b) maximin/minimax: Selecting best of the worst; (c) Hurwicz
criterion: Choosing the best using weighted average of the best and the
worst; (d) Laplace principle: Selecting the act with best average; and (e)
Savage principle: Involving selection of act with least of the maximum
regret values. Where probabilities are used, the decision rules include (a)
maximum likelihood principle where the best act of the most probable
event is selected; (b) expectation principle: In which the act with the best
expected value is chosen; and (c) expected regret criterion, where the
act with least expected value is selected, leading to identical decision as
under expectation principle.
In addition to the optimal course of action, expected value of perfect
information, EVPI, is also calculated. It equals the expected regret value of
the optimal act and represents the maximum amount that a decision-maker
would be willing to pay in case he is provided with perfect information.
The decision analysis using expectation principle can be extended to
cover situations where given probabilities of various states of nature are
revised on the basis of some additional information about them (the states).
The prior probabilities are converted into posterior probabilities using
the conditional, joint and total probabilities. Such an analysis is called
posterior analysis. The decision-tree approach to decision-making is used
in situations where multi-stage decisions are needed. The sequences of
action-event combinations available to the decision-maker are presented
graphically in the form of a decision tree. In analysing such situations,
alternatives are evaluated by proceeding in a backward manner – by
evaluating the best course at later stages to decide the best action at the
earlier stages. The decision criterion is expected monetary value, EMV.

268 PAGE
© Department of Distance & Continuing Education, Campus of Open Learning,
School of Open Learning, University of Delhi
BUSINESS STATISTICS

3.9 Self-Assessment Questions Notes

Exercise 1 : True and False Statements


(i) Decision theory is concerned with determining optimal strategies
where a decision-maker is faced with a number of alternatives and
a risky pattern of future events.
(ii) Regret is the amount of money paid for not adopting the optimal
course of action.
(iii) Laplace principle is based on the premise of equal chances of
occurrence of possible events.
(iv) Minimax is an optimist’s choice while Minimum is a pessimist’s
criterion.
(v) In Hurwicz criterion, the maximum pay-off is multiplied by D, and
the minimum by 1– D, where D may be any number between zero
and 100.
(vi) In situations of decisions under uncertainty, the Laplace criterion
is the least conservative while the minimax criterion is the most
conservative.
(vii) In case of pay-offs represented as profits, the Savage criterion for
selecting optimal course of action will be based on the maximin
principle.
(viii) Expected pay-off and expected regret criteria would both lead to
identical decisions in a given case.
(ix) All criteria for decisions under risk inherently assume that the
particular (optimal) decision reached will be repeated a large number
of times.
(x) EPPI is the expected pay-off under certainty.
(xi) EVPI is the expected regret value of any strategy.
(xii) In Bayesian approach to decision making, the optimal strategy is
determined using posterior probabilities.
(xiii) Posterior probabilities are obtained by modifying prior probabilities
taking into consideration the information from a sample.
(xiv) The decision tree approach to decision-making is appropriate in
those situations where a sequence of decisions is involved.

PAGE 269
© Department of Distance & Continuing Education, Campus of Open Learning,
School of Open Learning, University of Delhi
B.COM. (PROGRAMME)

Notes (xv) In the decision trees, no more than two alternative courses of action
can emanate from a decision node.
(xvi) In decision trees, the probabilities of all events at chance nodes and
the monetary evaluations of different alternatives must all be known
in advance.
(xvii) The probabilities of various outcomes at each chance node should
always add up to one.
(xviii) A decision taken on the basis of expected monetary value would
always prove to be the right decision.
Ans.
(i) True (ii) False (iii) True (iv) False (v) False (vi) True
(vii) False (viii) True (ix) True (x) True (xi) False (xii) True
(xiii) True (xiv) True (xv) False (xvi) True (xvii) False (viii) False
Exercise 2 : Questions and Answers
(i) Describe the steps involved in the process of decision-making.
(ii) What are pay-off and regret functions? How can entries in a regret
table be derived from a pay-off table?
(iii) Explain and illustrate the following principles of decision-making:
(a) Laplace,
(b) Maximax,
(c) Maximin,
(d) Hurwicz, and (e) Savage.
(iv) How are maximum likelihood and expectation principles of choice
differentiated? Do they always lead to same decisions?
(v) Define the term EPPI. How is it calculated? What does it signify?
(vi) What do you understand by EVPI? How is it calculated?
(vii) Explain the procedure of analysing a decision tree.
(viii) The research department of Hindustan Lever has recommended the
marketing department to launch a shampoo of three different types.
The marketing manager has to decide one of the types of shampoo
to be launched under the following estimated pay-offs (in millions
of Rs.) for various levels of sales:

270 PAGE
© Department of Distance & Continuing Education, Campus of Open Learning,
School of Open Learning, University of Delhi
BUSINESS STATISTICS

Type of Shampoo Estimated level Notes


of sale (Units)
15,000 10,000 5,000
Egg shampoo 30 10 10
Clinic shampoo 40 15 5
Deluxe shampoo 55 20 3
What will be the marketing manager’s decision if (a) Maximin, (b)
Maximax, (c) Laplace, and (d) Minimax regret criterion is applied?
(ix) A food products company is contemplating the introduction of
a revolutionary new product with new packaging to replace the
existing product at much higher price (S1) or a moderate change in
the composition of the existing product with a new packaging at a
small increase in price (S2) or a small change in the composition
of the existing product except the word ‘new’ with negligible
increase in price (S3). The three possible states of nature are: (i)
high increase in sales (N1), (ii) No change in sales (N2) and (iii)
decrease in sales (N3). The marketing department of the company
worked out pay-offs in terms of yearly net profits for each of the
three strategies of these events.
Pay-offs in Rs.
Strategies States of Nature
N1 N2 N3
S1 7,00,000 3,00,000 1,50,000
S2 5,00,000 4,50,000 0
S3 3,00,000 3,00,000 3,00,000
Which strategy should the executive concerned choose on the basis
of:
(a) Maximin criterion;
(b) Maximax criterion;
(c) Minimax criterion; and
(d) Laplace criterion?
(x) The oil company of India is interested in acquiring a piece of land
which is considered likely to contain oil deposits. The company has
the option of (a) buying the land outright, (b) obtaining an option

PAGE 271
© Department of Distance & Continuing Education, Campus of Open Learning,
School of Open Learning, University of Delhi
B.COM. (PROGRAMME)

Notes to buy, drill for oil and if found exercise the option, and (c) not
buying or obtaining option. There are three possibilities on such
land: Large oil reserves may be found; minor reserves may be found,
or there may be no oil. The pay-offs (in lacs of Rs. resulting from
various combinations of acts and events are tabulated below:
Acts
Buy land Obtain option No action
Large Reserves 40 28 0
Minor Reserves 10 1 0
No Oil –25 –2 0
What action should be taken by the company when the decision
criterion is:
(a) Laplace, (b) Maximin, (c) Maximax, (d) Minimax Regret, and
(e) Expected pay-off (when the probabilities of obtaining large,
minor, and no reserves are estimated to be 0.2, 0.5 and 0.3,
respectively)?
(xi) A firm is considering the purchase of some complex equipment
from either of the two suppliers S1 and S2. Supplier S1 is capable of
supplying the equipment on time to meet a certain desired deadline.
The price chargeable by S1 is, however, considerably higher than
that of S2. It is felt by the management of the firm that S2 may
deliver the equipment or may not be able to deliver on time. It is
even suspected that supplier S2 may never be able to deliver the
equipment to the specifications. However, the management believes
that if it waits for some months, it may get better information on
S2’s capabilities of supplying the equipment.
The management is considering three alternative courses of action.
A1 : Order from S1. If later on it is clear that S2 can supply, order
from S1 can be cancelled. Of course, delay would be caused when
the order is given to S2.
A2: Order from supplier S2. If it is known later on that S2 cannot
supply the equipment, the order may be switched to S1.
A3: Wait till the time information on S2’s capabilities is known.
This would obviously cause delay. The outcomes (profits) in the
various possible situations are:

272 PAGE
© Department of Distance & Continuing Education, Campus of Open Learning,
School of Open Learning, University of Delhi
BUSINESS STATISTICS

Event Course of action Notes


A1 A2 A3
E1 250 100 200
E2 250 125 300
E3 250 625 450
E1 : S2 fails to deliver
E2 : S2 delivers late
E3 : S2 delivers on time
What would be the management’s decision according to each of
the following criteria:
(a) Laplace, (b) Maximin, (c) Hurwicz (with D= 0.5), and (d) Minimax
Regret.
(xii) An investor is given the following investment alternatives and~percentage
rates of return:
States of Nature (Market Conditions)
Low Medium High
Regular shares 2% 5% 8%
Risky shares –5% 7% 15%
Property –10% 10% 20%
Over the past 300 days, 150 days have been medium market conditions
and 60 days have had high market increases.
On the basis of these data, state the optimal investment strategy for
the investor.
(xiii) The manager of a small departmental store must place order every
week for an item which costs Rs. 15 and sells for Rs. 25. Units
not sold during the week are disposed T of for Rs. 10 each. The
demand for this item is estimated to be as follows:
Demand (Units): 25 26 27 28 29 30
Probability: 0.10 0.15 0.30 0.20 0.15 0.10
(a) Determine the optimal number of units to be produced, using
the expected monetary value criterion.
(b) Determine the expected value of perfect information.

PAGE 273
© Department of Distance & Continuing Education, Campus of Open Learning,
School of Open Learning, University of Delhi
B.COM. (PROGRAMME)

Notes (xiv) Three types of souvenirs can be sold outside a stadium. From the
following conditional pay-off table, construct the opportunity loss
table. (Sales are dependent on the winning team.)
Types of Souvenir
I (Rs.) II (Rs.) III (Rs.)
Team A wins 1,200 800 300
Team B wins 250 700 1,100
Point out which type of souvenir should be bought if probability of
Team A’s winning is 0.6.
(xv) Chemical Products Ltd. produces a compound which must be sold
within the month it is produced, if the normal price of Rs. 100 per
drum is to be obtained. Anything unsold in that month is sold in a
different market for Rs. 20 per drum. The variable cost is Rs. 55
per drum.
During the last three years, monthly demand was recorded and showed
the following frequencies:
Monthly demand (No. of drums): 2,000 3,000 6,000
Frequency (No. of months): 8 16 12
(a) Prepare an appropriate pay-off table.
(b) Advise the production management on the number of drums
that should be produced next month.
(xvi) A stockist of a particular commodity makes a profit of Rs. 30 on
each sale made within the same week of purchase; otherwise he
incurs a loss of Rs. 30 on each item.
No. of items sold within : 5 6 7 8 9 10 11
the same week
Frequency : 0 9 12 24 9 6 0
(a) Find out the optimum number of items the stockist should buy
every week in order to maximize the profit.
(b) Calculate the expected value of perfect information.
(xvii) A physician purchases a particular vaccine on Monday of each week.
The vaccine must be used within the week following, otherwise
it becomes worthless. The vaccine costs Rs. 20 per dose and

274 PAGE
© Department of Distance & Continuing Education, Campus of Open Learning,
School of Open Learning, University of Delhi
BUSINESS STATISTICS

the physician charges Rs. 60 per dose. In the past 50 weeks, the Notes
physician has administered the vaccine in the following quantities:
Doses per week : 20 25 40 60
No. of weeks : 5 15 25 5
(a) Draw up a pay-off matrix.
(b) Obtain a regret matrix.
(c) Determine the optimum number of doses the physician should
buy.
(d) The maximum amount the physician would be willing to pay
per week for perfect information about the number of doses
expected to be demanded in a week.
Ans.
(viii) Egg, Deluxe, Deluxe, (ix) S3, S1, S1, S1 Obtain option, No
(x)
Deluxe action, Buy Land,
obtain option, buy
land or obtain option
(xi) A3 A1 A2 A2 A2 or A3 (xii) Property, Exp. (xiii) (i) (ii)
Return = 6%
(xiv) Type I ER = 340 (xv) 3000 drums (xvi) 8 units, EP = 210,
EVPI = 25.50
(xvii) 40, EMV = 1210,
EVPI = 1210.

PAGE 275
© Department of Distance & Continuing Education, Campus of Open Learning,
School of Open Learning, University of Delhi
UNIT-3

PAGE 277
© Department of Distance & Continuing Education, Campus of Open Learning,
School of Open Learning, University of Delhi
L E S S O N

1
Simple Correlation
STRUCTURE
1.1 Learning Objectives
1.2 Introduction
1.3 Utility of Correlation
1.4 Difference between Correlation and Causation
1.5 Types of Correlation
1.6 Methods of Studying Correlation
1.7 Summary
1.8 Self-Assessment Questions

1.1 Learning Objectives


After studying this lesson, you should be able to:
‹ Understand the concept of correlation.
‹ Identify different types of correlation.
‹ Understand the notion and interpretation of coefficient of correlation.
‹ Compute the value of correlation by different methods.
‹ Compute correlation coefficient for bivariate frequency distribution.

1.2 Introduction
In the earlier chapters we have discussed univariate distributions to highlight the important
characteristics by different statistical techniques. Univariate distribution means the study
related to one variable only. We may however come across certain series where each
item of the series may assume the values of two or more variables. The distributions in
which each unit of series assumes two values is called bivariate distribution. In a bivariate
distribution, we are interested to find out whether there is any relationship between two
variables. The correlation is a statistical technique which studies the relationship between
two or more variables and correlation analysis involves various methods and techniques

PAGE 279
© Department of Distance & Continuing Education, Campus of Open Learning,
School of Open Learning, University of Delhi
B.COM. (PROGRAMME)

Notes used for studying and measuring the extent of relationship between the
two variables. When two variables are related in such a way that a change
in the value of one is accompanied either by a direct change or by an
inverse change in the values of the other, the two variables are said to
be correlated. In the correlated variables an increase in one variable is
accompanied by an increase or decrease in the other variable. For instance,
relationship exists between the price and demand of a commodity because
keeping other things equal, an increase in the price of a commodity shall
cause a decrease in the demand for that commodity. Relationship might
exist between the heights and weights of the students and between amount
of rainfall in a city and the sales of raincoats in that city.
These are some of the important definitions about correlation.
Croxton and Cowden says, “When the relationship is of a quantitative
nature, the appropriate statistical tool for discovering and measuring the
relationship and expressing it in a brief formula is known as correlation”.
A.M. Tuttle says, “Correlation is an analysis of the covariation between
two or more variables.”
W.A. Neiswanger says, “Correlation analysis contributes to the understanding
of economic behaviour, aids in locating the critically important variables
on which others depend, may reveal to the economist the connections by
which disturbances spread and suggest to him the paths through which
stabilizing forces may become effective.
L.R. Conner says, “If two or more quantities vary in sympathy so that the
movements in one tends to be accompanied by corresponding movements
in others then they are said to be correlated.

1.3 Utility of Correlation


The study of correlation is very useful in practical life as revealed by
these points:
(1) With the help of correlation analysis, we can measure in one figure,
the degree of relationship existing between variables like price,
demand, supply, income, expenditure etc. Once we know that two
variables are correlated then we can easily estimate the value of
one variable, given the value of other.

280 PAGE
© Department of Distance & Continuing Education, Campus of Open Learning,
School of Open Learning, University of Delhi
BUSINESS STATISTICS

(2) Correlation analysis is of great use to economists and businessmen, Notes


it reveals to the economists the disturbing factors and suggest to
him the stabilizing forces. In business, it enables the executive to
estimate costs, sales etc. and plan accordingly.
(3) Correlation analysis is helpful to scientists. Nature has been found
to be a multiplicity of inter-related forces.

1.4 Difference Between Correlation and Causation


The term correlation should not be misunderstood as causation. If correlation
exists between two variables, it must not be assumed that a change in
one variable is the cause of a change in other variable. In simple words,
a change in one variable may be associated with a change in another
variable but this change need not necessarily be the cause of a change in
the other variable. When there is no cause and effect relationship between
two variables but a correlation is found between the two variables such
correlation is known as “spurious correlation” or “nonsense correlation”.
Correlation may exist due to the following:
(1) Pure change correlation : This happens in a small sample. Correlation
may exist between incomes and weights of four persons although
there may be no cause and effect relationship between incomes and
weights of people. This type of correlation may arise due to pure
random sampling variation or because of the bias of investigator
in selecting the sample.
(2) When the correlated variables are influenced by one or more
variables: A high degree of correlation between the variables may
exist, where the same cause is affecting each variable or different
cause affecting each with the same effect. For instance, a degree
of correlation may be found between yield per acre of rice and tea
due to the fact that both are related to the amount of rainfall but
none of the two variables is the cause of other.
(3) When the variable mutually influence each other so that neither
can be called the cause of other: At times it may be difficult to
say that which of the two variables is the cause and which is the
effect because both may be reacting on each other.

PAGE 281
© Department of Distance & Continuing Education, Campus of Open Learning,
School of Open Learning, University of Delhi
B.COM. (PROGRAMME)

Notes 1.5 Types of Correlation


Correlation can be categorised as one of the following :
(i) Positive and Negative Correlation
(ii) Simple and Multiple Correlation
(iii) Partial and Total Correlation
(iv) Linear and Non-Linear (Curvilinear) Correlation
Positive and Negative Correlation
Positive or direct Correlation refers to the movement of variables in the
same direction. The correlation is said to be positive when the increase
(decrease) in the value of one variable is accompanied by an increase
(decrease) in the value of other variable also. Negative or inverse correlation
refers to the movement of the variables in opposite direction. Correlation is
said to be negative, if an increase (decrease) in the value of one variable
is accompanied by a decrease (increase) in the value of other.
Simple and Multiple Correlation
Under simple correlation, we study the relationship between two variables
only i.e., between the yield of wheat and the amount of ramfall or between
demand and supply of a commodity. In case of multiple correlation,
the relationship is studied among three or more variables. For example,
the relationship of yield of wheat may be studied with both chemical
fertilizers and the pesticides.
Partial and Total Correlation
There are two categories of multiple correlation analysis. Under partial
correlation, the relationship of two or more variables is studied in such
a way that only one dependent variable and one independent variable
is considered and all others are kept constant. For example, coefficient
of correlation between yield of wheat and chemical fertilizers excluding
the effects of pesticides and manures is called partial correlation. Total
correlation is based upon all the variables.
Linear and Non-Linear Correlation
When the amount of change in one variable tends to keep a constant ratio
to the amount of change in the other variable, then the correlation is said
to be linear. But if the amount of change in one variable does not bear

282 PAGE
© Department of Distance & Continuing Education, Campus of Open Learning,
School of Open Learning, University of Delhi
BUSINESS STATISTICS

a constant ratio to the amount of change in the other variable then the Notes
correlation is said to be non-linear. The distinction between linear and
non-linear is based upon the consistency of the ratio of change between
the variables.

1.6 Methods of Studying Correlation


There are different methods which helps us to find out whether the
variables are related or not:
(1) Scatter Diagram Method
(2) Graphic Method
(3) Karl Pearson’s Coefficient of Correlation
(4) Properties of Coefficient of Correlation
(5) Probable Error of Coefficient of Correlation
(6) Rank Method
(7) Concurrent Deviation Method
Let us understand these methods one by one.

1.6.1 Scatter Diagram


Scatter diagram is drawn to visualise the relationship between two variables.
The values of more important variable is plotted on the X-axis while the
values of the other variable are plotted on the Y-axis. On the graph, dots
are plotted to represent different pairs of data. When dots are plotted to
represent all the pairs, we get a scatter diagram. The way the dots scatter
gives an indication of the kind of relationship which exists between the
two variables. While drawing scatter diagram, it is not necessary to take at
the point of sign the zero values of X and Y variables, but the minimum
values of the variables considered may be taken.
When there is a positive correlation between the variables, the dots on
the scatter diagram run from left hand bottom to the right hand upper
corner. In case of perfect positive correlation all the dots will lie on a
straight line.

PAGE 283
© Department of Distance & Continuing Education, Campus of Open Learning,
School of Open Learning, University of Delhi
B.COM. (PROGRAMME)

Notes

When a negative correlation exists between the variables, dots on the


scatter diagram run from the upper left hand corner to the bottom right
hand corner. In case of perfect negative correlation, all the dots lie on
a straight line.

If a scatter diagram is drawn and no path is formed, there is no correlation.


Students are advised to prepare two scatter diagrams on the basis of the
following data:

284 PAGE
© Department of Distance & Continuing Education, Campus of Open Learning,
School of Open Learning, University of Delhi
BUSINESS STATISTICS

(i) Data for the first Scatter Diagram: Notes


Demand Schedule
Price (Rs.) Commodity Demand (units)
6 180
7 150
8 130
9 120
10 125
(ii) Data for the second Scatter Diagram:
Supply Schedule
Price (Rs.) Commodity Supply
50 2,000
51 2,100
52 2,200
53 2,500
54 3,000
55 3,800
56 4,700
Students will find that the first diagram indicate a negative correlation
where the second diagram shall reveal a positive correlation.

1.6.2 Graphic Method


In this method the individual values of the two variables are plotted on
the graph paper. Therefore two curves are obtained – one for X variable
and another for Y variable.
The graph is interpreted as follows:
(i) If both the curves run parallel or nearly parallel or more in the same
direction, there is positive correlation.
(ii) On the other hand, if both the curves move in the opposite direction,
there is a negative correlation.

PAGE 285
© Department of Distance & Continuing Education, Campus of Open Learning,
School of Open Learning, University of Delhi
B.COM. (PROGRAMME)

Notes Example 1 : Show correlation from the following data by graphic method:
Year 2005 2006 2007 2008 2009 2010 2011 2012 2013 2014
Average Income 100 110 125 140 150 180 200 220 250 360
(Rs.)
Average 90 95 100 120 120 140 150 170 200 260
Expenditure
(Rs.)
Solution:

0 2005 06 07 08 09 10 11 12 13 2014

Y
YEARS

The graph prepared shows that income and expenditure have a close
positive correlation. As income increases, the expenditure also increases.
Covariance
The measurement of relationship between two random variables (assume
variables are X and Y) is called Covariance. It examines the directions of
relationship of two variable whether the variables are positively covariate
or negatively covariate. If both X and Y variables tend to be higher or
lower at the same time then it shows the positive covariance between X
and Y. On the other side, if X and Y indicate inverse relationship which
means one is showing higher value than the average and other is showing
the lower value than the average, it considered as negative covariance
between the variables. It is denoted by as follows:

286 PAGE
© Department of Distance & Continuing Education, Campus of Open Learning,
School of Open Learning, University of Delhi
BUSINESS STATISTICS

Notes

Covariance (X, Y) =
∑ ( X - X ) (Y - Y) = ∑xy
n n

Here X = ∑X , similarly = ∑Y
n n
Example: Calculate Covariance (X, Y) if
X: 1 2 3 4 5
Y: 6 7 8 9 10
Covariance (X, Y)
X Y x = X-X y = Y-Y xy = (X- X)(Y- Y)
1 6 -2 -2 4
2 7 -1 -1 1
3 8 0 0 0
4 9 1 1 1
5 10 2 2 4
6X = 15 6Y = 40 6xy = 10

X = ∑X = 15
= 3 Y = ∑Y = 40
= 8
n 5 n 5

Covariance (X, Y) = ∑
xy 10
= = 2
n 2

1.6.3 Karl Pearson’s Coefficient of Correlation


Karl Pearson’s method, popularly known as Pearsonian co-efficient of
correlation, is most widely applied in practice to measure correlation.
The Pearsonian co-efficient of correlation is represented by the symbol r.
According to Karl Pearson’s method, coefficient of correlation between
the variables is obtained by dividing the sum of the products of the
corresponding deviations of the various items of two series from their
respective means by the product of their standard deviations and the
number of pairs of observations. Symbolically,
Σxy
r =
N σxσ y

PAGE 287
© Department of Distance & Continuing Education, Campus of Open Learning,
School of Open Learning, University of Delhi
B.COM. (PROGRAMME)

Notes where r stands for coefficient of correlation ...(i)

where x1 , x2 , x3 , x4 .....xn are the deviations of various items of the first


variable from the mean,

y1 , y2 , y3 .... yn are the deviations of all items of the second variable from
mean,
6xy is the sum of products of these corresponding deviations. N stands
for the number of pairs, Vx stands for the standard deviation of X variable
and Vystands for the standard deviation of Y variable.
Σx 2 Σy 2
V x= and Vy=
N N
If we substitute the value of Vx and Vy in the above written formula of
computing r, we get
Σxy Σxy
r = or r =
⎛ Σx 2 Σy 2 ⎞ Σx 2 Σy 2
N⎜ × ⎟
⎜ N N ⎟
⎝ ⎠
Degree of correlation varies between +1 and –1; the result will be +1
in case of perfect positive correlation and –1 in case of perfect negative
correlation.
Computation of correlation coefficient can be simplified by dividing the
given data by a common factor. In such a case, the final result is not
multiplied by the common factor because coefficient of correlation is
independent of change of scale and origin.
Example 2 : Calculate Coefficient of Correlation from the following data:
X 50 100 150 200 250 300 350
Y 10 20 30 40 50 60 70
Solution:
X−X Y −Y
50 10
X (X − X ) x x2 Y Y −Y y y2 xy

50 – 150 – 3 9 10 – 30 – 3 9 9

288 PAGE
© Department of Distance & Continuing Education, Campus of Open Learning,
School of Open Learning, University of Delhi
BUSINESS STATISTICS

Notes
X−X Y −Y
50 10
X (X − X ) x x2 Y Y −Y y y2 xy

100 – 100 – 2 4 20 –20 – 2 4 4


150 – 50 – 1 1 30 – 10 – 1 1 1
200 0 0 0 40 0 0 0 0
250 + 50 + 1 1 50 + 10 + 1 1 1
300 + 100 + 2 4 60 + 20 + 2 4 4
350 + 150 + 3 9 70 + 30 + 3 9 9
6x = 0 6x 2 = 28 6y = 0 6y2 = 28 6xy= 28
Σxy
r =
Σx 2 Σy 2

28 28
By substituting the values we get r = = =1
28 × 28 28
Hence there is perfect positive correlation.
Example 3 : A sample of five items is taken from the production of a
firm, length and weight of the five items are given below:
Length (inches) 3 4 6 7 10
Weight (ounces) 9 11 14 15 16
Calculate Karl Pearson’s correlation coefficient between length and weight
and interpret the value of correlation coefficient.
ΣX 30 ΣY 65
Solution: X = = = 6 and Y = = = 13
N 5 N 5

(X − X ) (Y − Y )
X x x2 Y y y2 xy
3 – 3 9 9 – 4 16 12
4 – 2 4 11 – 2 4 4
6 0 0 14 +1 1 0
7 + 1 1 15 + 2 4 2
10 + 4 16 16 + 3 9 12
6X = 30 0 30 6Y = 65 0 34 30

PAGE 289
© Department of Distance & Continuing Education, Campus of Open Learning,
School of Open Learning, University of Delhi
B.COM. (PROGRAMME)

Notes Σxy
r = where Σxy = 30, Σx 2 = 30, and Σy 2 = 34
Σx Σy 2 2

30 30
r = = = + 0.939 Ans.
30 × 34 1020
The value of r indicates that there exists a high degree positive correlation
between lengths and weights.
Example 4 : From the following data, compute the coefficient of correlation
between X and Y :
X Series Y Series
Number of items 15 15
Arithmetic Mean 25 18
Square of deviation from Mean 136 138
Summation of product deviations of X and Y from their Arithmetic Means
= 122.
Solution: Denoting deviations of X and Y from their arithmetic means
by x and y respectively, the given data are : 6x2 = 136, 6xy = 122, and
6y2 = 138
Σxy 122 122
r = = = = 0.89 Ans.
Σx Σy
2 2
136 ×138 137

Short-cut Method: To avoid difficult calculations due to mean being


in fraction, deviations are taken from assumed means while calculating
coefficient of correlation. The formula is also modified for standard
deviations because deviations are taken from assumed means. Karl
Pearson’s formula for short-cut method is given below:
Σdx.Σdy
Σdxdy −
r = N or
⎧ ( Σ dx ) 2
⎫ ⎧ ( Σ dy ) 2

⎨Σdx − ⎬ ⎨Σdy −
2 2

⎩ N ⎭⎩ N ⎭

N Σdxdy − Σdx × Σdy


r =
{ N Σdx 2
− (Σdx) 2 }{ N Σdy 2 − (Σdy ) 2 }

290 PAGE
© Department of Distance & Continuing Education, Campus of Open Learning,
School of Open Learning, University of Delhi
BUSINESS STATISTICS

Example 5 : Compute the coefficient of correlation from the following Notes


data :
Marks in 20 30 28 17 19 23 35 13 16 38
Statistics
Marks in 18 35 20 18 25 28 33 18 20 40
Mathematics
Solution:
Marks in (X – 30) dx2 Marks in Y – 30
Statistics, X dx Maths, Y dy dy2 dxdy
20 – 10 100 18 – 12 144 + 120
30 0 0 35 + 5 25 0
28 – 2 4 20 – 10 100 + 20
17 – 13 169 18 – 12 144 + 156
19 – 11 121 25 – 5 25 + 55
23 – 7 49 28 – 2 4 + 14
35 + 5 25 33 + 3 9 + 15
13 – 17 289 18 – 12 144 + 204
16 – 14 196 20 – 10 100 + 140
38 + 8 64 40 + 10 100 + 80
N = 10 – 61 1017 – 45 795 804
N Σdxdy − Σdx.Σdy
r =
{N Σdx − (Σdx) 2 }{N Σdy 2 − (Σdy ) 2 }
2

where dx o deviations of X series from an assumed mean 30.


dy o deviations of Y series from an assumed mean 30.
dx2 o squares of the deviations of X series from assumed mean.
dy1 o squares of the deviations of Y series from assumed mean.
dxdy o the product of deviations of X and Y series from their
assumed means.
10 × 804 − (−61)(−45)
 ? r =
10 ×1017 − (−61) 2 10 × 795 − (−45) 2

8040 − 2745 5295


or r = = = 0.856
(10170 − 3721)(7950 − 2025) 6449 × 5925

PAGE 291
© Department of Distance & Continuing Education, Campus of Open Learning,
School of Open Learning, University of Delhi
B.COM. (PROGRAMME)

Notes Direct Method of Computing Correlation Coefficient


Correlation coefficient can also be computed from given X and Y values
by using the formula given below:
N ΣXY − (ΣX )(ΣY )
r =
N ΣX 2 − ( ΣX ) 2 N ΣY 2 − ( ΣY ) 2

The above given formula gives us the same answer as we are getting by
taking durations from actual mean or arbitrary mean.
Example 6 : Compute the coefficient of correlations from the following
data :
Marks in 20 30 28 17 19 23 35 13 16 38
Statistics
Marks in 18 35 20 18 25 28 33 18 20 40
Mathematics
Solution:
Marks in Marks in
Statistics, X Mathematics, Y X2 Y2 XY
20 18 400 324 360
30 35 900 1225 1050
28 20 784 400 560
17 18 289 324 306
19 25 361 625 475
23 28 529 784 644
35 33 1225 1089 1155
13 18 169 324 234
16 20 256 400 320
38 40 1444 1600 1520
6X = 239 6Y = 255 6X = 6357
2
6Y = 7095
2
6XY = 6624
Substitute the computed values in the formula given below,
N ΣXY − (ΣX )(ΣY )
r =
N ΣX 2 − ( ΣX ) 2 N ΣY 2 − ( ΣY ) 2

(10 × 6624) − (239)(255)


=
10 × 6357 − (239) 2 10 × 7095 − (255) 2

292 PAGE
© Department of Distance & Continuing Education, Campus of Open Learning,
School of Open Learning, University of Delhi
BUSINESS STATISTICS

66240 − 60945 5295 5295 Notes


= = = = 0.856
63570 − 57121 70950 − 65095 6449 5925 6181.45
Coefficient of Correlation in a Continuous Series
In the case of a continuous series, we assume that every item which falls
within a given class interval falls exactly at the middle of that class. The
formula, because of the presence of frequencies is modified as follows:
Σfdx.Σfdy
Σfdxdy −
Σf
r =
⎧ (Σfdx) 2 ⎫ ⎧ (Σfdy ) 2 ⎫
⎨ Σ fdx 2
− ⎬⎨ Σ dy 2
− ⎬
⎩ Σf ⎭ ⎩ Σf ⎭

Various values shall be calculated as follows:


(i) Take the step deviations of variable X and denote it as dx.
(ii) Take the step deviations of variable Y and denote it as dy.
(iii) Multiply dx dy and the respective frequency of each cell and write
the figure obtained in the right-hand upper corner of each cell.
(iv) Add all the cornered values calculated in step (iii) to get 6fdxdy.
(v) Multiply the frequencies of the variable X by the deviations of X to
get 6fdx.
(vi) Take the squares of the deviations of the variable X and multiply
them by the respective frequencies to get 6fdx2.
(vii) Multiply the frequencies of the variable Y by the deviations of Y to
get 6fdy.
(viii) Take the squares of the deviations of the variable Y and multiply
them by the respective frequencies to get 6fdy2.
(ix) Now substitute the values of 6fdxdy, 6fdx, 6fdx2, 6fdy, 6fdy2 in the
formula to get the value of r.
Example 7 : The following table gives the ages of husbands and wives at
the time of their marriages. Calculate the correlation coefficient between
the ages of husbands and wives.

PAGE 293
© Department of Distance & Continuing Education, Campus of Open Learning,
School of Open Learning, University of Delhi
B.COM. (PROGRAMME)

Notes Ages of Husbands


Age of 20–30 30–40 40–50 50–60 60–70 Total
Wives
15–25 5 9 3 – – 17
25–35 – 10 25 2 – 37
35–45 – 1 12 2 – 15
45–55 – – 4 16 5 25
55–65 – – – 4 2 6
Total 5 20 44 24 7 100
Solution:
Age of Husbands (X)

Σfdx.Σfdy
Σfdxdy −
Σf
r =
⎧ (Σfdx) 2 ⎫ ⎧ (Σfdy ) 2 ⎫
⎨ Σ fdx 2
− ⎬⎨ Σ dy 2
− ⎬
⎩ Σf ⎭ ⎩ Σf ⎭

294 PAGE
© Department of Distance & Continuing Education, Campus of Open Learning,
School of Open Learning, University of Delhi
BUSINESS STATISTICS

(+8)(−34) 90.72 Notes


88 −
100 100 90.72
= = = = + 0.79
⎧ (8) ⎫ ⎧
2
(−34) 2 ⎫ 91.36 142.44 91.36 + 14.244
⎨92 − ⎬ ⎨154 − ⎬ +
⎩ 100 ⎭ ⎩ 100 ⎭ 100 100

1.6.4 Properties of Coefficient of Correlation


Following are some of the important properties of r:
(1) The coefficient of correlation lies between –1 and +1 (–1 d r d +1)
(2) The coefficient of correlation is independent of change of scale and
origin of the variable X and Y.
(3) The coefficient of correlation is the geometric mean of two regression
coefficients.

r = bxy × dyx
Merits of Pearson’s coefficient of correlation : The correlation of coefficient
summarizes in one figure the degree and direction of correlation but also
the direction. Value varies between +1 and –1.
Demerits of Pearson’s coefficient of correlation: It always assumes
linear relationship between the variables; in fact the assumption may be
wrong. Secondly, it is not easy to interpret the significance of correlation
coefficient. The method is time consuming and affected by the extreme
items.

1.6.5 Probable Error of Coefficient of Correlation


It is calculated to find out how far the Pearson’s coefficient of correlation
is reliable in a particular case.
1− r2
P.E. of coefficient of correlation = −0.6745 ×
N

where r = coefficient of correlation and N= number of pairs of items.


If the probable error calculated is added to and subtracted from the
coefficient of correlation, it would give us such limits within which we
can expect the value of the coefficient of correlation to vary.

PAGE 295
© Department of Distance & Continuing Education, Campus of Open Learning,
School of Open Learning, University of Delhi
B.COM. (PROGRAMME)

Notes If r is less than probable error, then there is no real evidence of correlation.
If r is more than 6 times the probable error, the coefficient of correlation
is considered highly significant.
If r is more than 3 times the probable error but less than 6 times,
correlation is considered significant but not highly significant.
If the probable error is not much and the given r is more than the probable
error but less than 3 times of it, nothing definite can be concluded.

1.6.6 Rank Correlation


There are many problems of business and industry when it is not possible
to measure the variable under consideration quantitatively or the statistical
series is composed of items which can not be exactly measured. For
instance, it may be possible for the two judges to rank six different
brands of cigarettes in terms of taste, whereas it may be difficult to give
them a numerical grade in terms of taste. In such problems, Spearman’s
coefficient of rank correlation is used. The formula for rank correlation is :
6ΣD 2
or 1 − 6ΣD
2
U = 1−
N ( N − 1)
2
N3 − N
where U stands for rank coefficient of correlation
D refers to the difference of ranks between paired items
N refers to the number of paired observations.
The value of rank correlation coefficient varies between +1 and –1. When
the value of U = +1, there is complete agreement in the order of ranks
and the ranks will be in the same order. When U = –1, the ranks will
be in opposite direction showing complete disagreement in the order of
ranks. Let us understand with the help of an example.
Example 8 : Ranks of 10 individuals at the start and at the finish of a
course of training are given:
Individual : A B C D E F G H I J
Rank before : 1 6 3 9 5 2 7 10 8 4
Rank after : 6 8 3 7 2 1 5 9 4 10
Calculate coefficient of correlation.

296 PAGE
© Department of Distance & Continuing Education, Campus of Open Learning,
School of Open Learning, University of Delhi
BUSINESS STATISTICS

Solution: Notes
Individual Rank before Rank after (R1 – R2)
R1 R2 D D2
A 1 6 – 5 25
B 6 8 – 2 4
C 3 3 0 0
D 9 7 2 4
E 5 2 3 9
F 2 1 1 1
G 7 5 2 4
H 10 9 1 1
I 8 4 4 16
J 4 10 – 6 36
N = 10 2
6D = 100
By applying the formula,
6ΣD 2 6 × 100
U = 1− = 1− 3 = 1 − 0.609 = 0.394
N −N
3
10 − 10
When we are given the actual data and not the ranks, it becomes necessary
for us to assign the ranks. Ranks can be assigned by taking either the
highest value as one or the lowest value as one. But if we start by taking
the highest value or the lowest value we must follow the same order for
both the variables to assign ranks.
Example 9 : Calculate rank correlation from the following data:
X : 17 13 15 16 6 11 14 9 7 12
Y : 36 46 35 24 12 18 27 22 2 8
Solution:
Calculation of Rank Correlation
X (Ranks) Y (Ranks) D D2
R1 R2 (R1 – R2)
17 1 36 2 – 1 1
13 5 46 1 + 4 16
15 3 35 3 0 0

PAGE 297
© Department of Distance & Continuing Education, Campus of Open Learning,
School of Open Learning, University of Delhi
B.COM. (PROGRAMME)

Notes X (Ranks) Y (Ranks) D D2


R1 R2 (R1 – R2)
16 2 24 5 – 3 9
6 10 12 8 + 2 4
11 7 18 7 0 0
14 4 27 4 0 0
9 8 22 6 + 2 4
7 9 2 10 – 1 1
12 6 8 9 – 3 9
N = 10 6D = 44
2

Rank correlation coefficient is calculated as follows :


6ΣD 2
U = 1−
N3 − N
6 × 44 264
U = 1− = 1− = 1 − 0.266 = 0.734
10 − 10
3
990
In some cases it becomes necessary to rank two or more items an identical
rank. In such cases, it is customary to give each item an average rank.
Therefore, if two items are equal for 4th and 5th rank, each item shall
4+5
be ranked 4.5 i.e., . It means, where two or more items are to be
2
ranked equal, the rank assigned for purposes of calculating coefficient of
correlation is the average of ranks which these items would have got had
they differed slightly from each other. When equal ranks are assigned to
some items, the rank correlation formula is also adjusted. The adjustment
1
consists of adding (m 2 − m) to the value of 6D2 where m stands for
12
number of items whose ranks are identical.
⎧ 1 1 ⎫
6 ⎨ΣD 2 + (m3 − m) + (m3 − m) + ....⎬
U = 1− ⎩ ⎭
12 12
N −N
3

Let us take an example to understand this.

298 PAGE
© Department of Distance & Continuing Education, Campus of Open Learning,
School of Open Learning, University of Delhi
BUSINESS STATISTICS

Example 10 : Compute the rank correlation coefficient from the following Notes
data:
Section A : 115 109 112 87 98 98 120 100 98 118
Section B : 75 73 85 70 76 65 82 73 68 80
Solution :
Computation of Rank correlation coefficient.
Series Ranks Series Ranks D D2
A R1 B R2 (R1 – R2)
115 8 75 6 – 2 4
109 6 73 4.5 1.5 2.25
112 7 85 10 – 3 9
87 1 70 3 – 2 4
98 3 76 7 – 4 16
98 3 65 1 2 4
120 10 82 9 1 1
100 5 73 4.5 0.5 0.25
98 3 68 2 1 1
118 9 80 8 1 1
N = 10 6D = 42.50
2

Apply formula to calculate Rank Correlation


⎧ 1 1 ⎫
6 ⎨ΣD 2 + (m3 − m) + (m3 − m) + ....⎬
U = 1− ⎩ ⎭
12 12
N −N
3

Item 98 is repeated three times in series A. Hence m = 3. In series B


the item 73 is repeated two times and so m = 2.
⎧ 1 1 ⎫
6 ⎨42.50 + (33 − 3) + (23 − 2) ⎬
U = 1− ⎩ ⎭
12 12
10 − 10
3

6(42.50 + 2 + 0.50) 270


U = 1− = 1− = 0.727
1000 − 10 990

PAGE 299
© Department of Distance & Continuing Education, Campus of Open Learning,
School of Open Learning, University of Delhi
B.COM. (PROGRAMME)

Notes 1.6.7 Concurrent Deviation Method


This is the simplest method of studying correlation. The only thing to
be computed under this method is the direction of change of both the
variables. The formula is
⎛ 2C − N ⎞
rc = ± ± ⎜ ⎟
⎝ N ⎠

where rc= Coefficient of concurrent deviations.


C = Number of concurrent deviations.
N = Number of pairs of deviations compared.
The procedure of calculating coefficient of correlation under this method
is quite simple as explained below:
(i) Compute the direction of change for both the variables comparing
with the preceding values and assign + sign for increase and – sign
for decrease and 0 for no change.
(ii) Denote these two columns by Dx and Dy.
(iii) Multiply Dx with Dy and determine the value of C which means
positive.
(iv) Apply the formula.
We can understand by taking an example.
Example 11 : Calculate the coefficient of correlation by concurrent
deviation from the following data:
X : 100 120 135 135 115 110 120
Y : 50 40 60 80 80 55 65
Solution :
X Dx Y Dy D xD y
100 50
120 + 40 – –
135 + 60 + +
135 0 80 + 0
115 – 80 0 0
110 – 55 – +

300 PAGE
© Department of Distance & Continuing Education, Campus of Open Learning,
School of Open Learning, University of Delhi
BUSINESS STATISTICS

X Dx Y Dy D xD y Notes
120 + 65 + +
N = 6 C = 3
⎛ 2C − N ⎞
rc= ± ± ⎜ ⎟
⎝ N ⎠

⎛ 2×3 − 6 ⎞ 0
= ± ±⎜ ⎟ =± = 0.
⎝ 6 ⎠ 6

Therefore the correlation does not exist between the variables.

1.7 Summary
‹ Correlation analysis deals with bivariate and multivariate data.
Correlation is a study of the co-variation of the variables involved.
When changes in the variables occur in the same direction, they are
positively correlated and when the movements are in the opposite
directions, the correlation is negative. Correlation between two
variables would result either when one of them is the cause while
the other is the effect or when both of them are affected by some
common factors. It may also be spurious correlation, resulting from
chance when factors affecting each one have nothing in common.
‹ Correlation between variables may be of varying degrees ranging from
perfect to high, moderate, low and no correlation. Correlation may
be linear or non-linear. Only linear correlation is considered here.
Graphically, correlation is studied by means of a scatter diagram. If
dots representing pairs of data values are seen to fall on a straight
line, the correlation is perfect. The degree of correlation decreases as
the points lay more and more away from the line. Widely scattered
dots with no clear direction and dots in a line that is parallel to
either of the axes means absence of correlation. Numerically, the
correlation is measured and expressed in terms of Karl Pearson’s
coefficient of correlation which is defined as the ratio of covariance
to the product of standard deviations of the two series involved. Its
calculation can be done by measuring deviations of the observations
from their respective means or assumed mean values, and even

PAGE 301
© Department of Distance & Continuing Education, Campus of Open Learning,
School of Open Learning, University of Delhi
B.COM. (PROGRAMME)

Notes by not measuring deviations. The coefficient of correlation varies


between ±1 and is independent of the change of origin and scale.

1.8 Self-Assessment Questions


Exercise 1: True or False statements
(i) The correlation analysis is related to the examination or the nature
of relationship between variables.
(ii) Covariation implies that two variables would vary in the same
direction.
(iii) Negative correlation in two series means that as the value of one of
the variables decreases, the other would also decrease.
(iv) Graphic representation of correlation is done by means of a scatter
diagram.
(v) A straight line on a scatter diagram having a zero slope implies
that there is perfect positive correlation between the variables.
(vi) The coefficient of correlation contains in a single number the extent
and the direction of relationship between the variables.
(vii) The absolute value of the covariance between X and Y can be at
most equal to the product of the standard deviations of X and Y
series.
(viii) The coefficient of correlation always lies between 0 and 1.
(ix) The coefficient of correlation can be calculated without measuring
deviations from actual or assumed means.
(x) The probable error aims at establishing the dependability of the
coefficient of correlation.
(xi) For one set of data, r = 0.8 and for another set r = 0.4. It means
that correlation is twice as strong in the first set than in the second.
(xii) A correlation between two variables may exist because both of them
may be influenced by some common factors.
(xiii) For a given set of X and Y values, the coefficient of correlation is
found to be equal to 0.6. If the variables are interchanged so that
X becomes Y and Y becomes X, then the coefficient for the new
set of data may or may not be equal to 0.6.

302 PAGE
© Department of Distance & Continuing Education, Campus of Open Learning,
School of Open Learning, University of Delhi
BUSINESS STATISTICS

(xiv) If all the X values in a given set of paired data are subtracted from Notes
a constant K, it will have no effect on the value of the correlation
coefficient.
(xv) The coefficient of correlation is independent of the change of origin
and scale.
(xvi) If the coefficient of correlation between X and Y is 0.7, then the
coefficient of correlation between –X and –Y would be equal to
–0.7.
(xvii) The correlation is said to be significant only when | r|> 6PE.
(xviii) For the rank correlation to be calculated, it is necessary that the
given variables should not be quantifiable.
(xix) The coefficient of rank correlation has the same limits as the Karl
Pearson’s coefficient of correlation has.
(xx) Rank correlation can be used even when the variables under consideration
are quantifiable and not normally distributed.
Ans.
(i) F (ii) F (iii) F (iv) T (v) F (vi) T (vii) T
(viii) F (ix) T (x) T (xi) F (xii) T (xiii) F (xiv) F
(xv) T (xvi) F (xvii) T (xviii) F (xix) T (xx) T
Exercise 2 : Questions and Answers
(i) What is correlation? Distinguish between positive and negative
correlation. How is ‘scatter diagram’ method helpful in the study
of correlation?
(ii) What is a scatter diagram? How does it help in studying the degree
and direction of correlation between two variables? Illustrate with
some sketches.
(iii) Define Karl Pearson’s coefficient of correlation. Explain the general
rules for interpreting the coefficient. In this connection, also state
the meaning and significance of the concept of probable error.
(iv) State and explain the properties of the coefficient of correlation.
Also, state the assumptions underlying.
(v) What do you understand by the statement that coefficient of
correlation is independent of the change of origin and scale?

PAGE 303
© Department of Distance & Continuing Education, Campus of Open Learning,
School of Open Learning, University of Delhi
B.COM. (PROGRAMME)

Notes (vi) Does correlation imply the existence of cause and effect relationship
between the variables involved? Does cause and effect relationship
between variables result in correlation between them? Explain with
the help of suitable examples.
(vii) Define rank correlation. Write Spearman’s formula for rank correlation
coefficient when some ranks are tied and when ranks are not tied.
What are the limits of this coefficient? Interpret the case where
this coefficient assumes the minimum value.
(viii) For a given series of paired data, the following information is
available:
Covariance between X and Y series = –32.6
Standard deviation of X series = 8.6
Standard deviation of Y series = 4.8
No. of pairs of observations = 15
Calculate the coefficient of correlation.
(ix) Given the following information:
Number of pairs of observations of X and Y series = 15
X series arithmetic mean = 25
Y series arithmetic mean = 18
X series standard deviation = 3.0
Y series standard deviation = 3.03
Summation of the products of corresponding deviations of X and Y
series = 122
Calculate the coefficient of correlation between X and Y series.
(x) Given:
Total of multiplication of deviations of X and Y = 3,476
No. of pairs of observations = 12
Total of deviations of X = – 176
Total of deviations of Y = – 26
Total of squares of deviations of X = 8,288
Total of squares of deviations of Y = 2,556
Using this information, calculate the coefficient of correlation when
the arbitrary mean values of X and Y are 85 and 22, respectively.

304 PAGE
© Department of Distance & Continuing Education, Campus of Open Learning,
School of Open Learning, University of Delhi
BUSINESS STATISTICS

(xi) For a set of bivariate data, you are given the following information: Notes
6(X – 58) = 46, 6(Y – 58) = 19, 6(X – 58) (Y – 58) = 1,095,
6(X – 58)2 = 1,483, and 6(Y – 58)2 = 3,086
Number of pairs of observations = 8
Calculate the coefficient of correlation between X and Y.
(xii) The co-efficient of correlation between two variables X and Y is –
0.4 and their covariance is equal to –16. If variance of Y series is
36, find the second moment about mean of X series.
(xiii) Given below is the information relating to marks in Statistics (X)
and marks in Accountancy (Y) obtained by the students of a class:
Co-variance between X and Y = 144
Second moment of X about 20 = 244
First moment of X about 20 = 10
Arithmetic mean of Y = 45
Coefficient of correlation between X and Y = 0.75
Calculate coefficient of variation for marks in Statistics and that
for marks in Accountancy. In which subject is the performance of
students is more consistent?
(xiv) The coefficient of correlation between X and Y for 20 items is
0.3. The mean of X is 15 and that of Y is 20 while the respective
standard deviations are 4 and 5. At the time of calculation, one
item 27 has wrongly been taken as 17 in the case of X series and
35 instead of 30 in the case of Y series. Find the correct coefficient
of correlation.
(xv) While making calculations about coefficient of correlation, a student
obtained the following results:
n = 25, 6X = 125, 6X2 = 650, 6Y = 100, 6Y2 = 460, and 6XY = 508
It was discovered later, however, that two pairs of values were
wrongly recorded as:

PAGE 305
© Department of Distance & Continuing Education, Campus of Open Learning,
School of Open Learning, University of Delhi
B.COM. (PROGRAMME)

Notes X Y X Y
6 14 while the correct 8 12
values were:
8 6 6 8
Obtain the correct value of the coefficient of correlation.
(xvi) Find the coefficient of correlation between age and playing habits
of the following students:
Age of Players: 16 17 18 19 20 21
No. of Students: 2,500 2,000 1,500 1,200 1,000 800
Regular Players: 2,250 1,200 1,050 480 250 120
(xvii) A panel of judges A and B graded seven dramatic performances by
independently awarding marks as given here.
Performance : 1 2 3 4 5 6 7
Marks by A: 46 42 44 40 43 41 45
Marks by B: 40 38 36 35 39 37 41
Show by means of coefficient of correlation whether the marks given
by them are correlated.
(xviii) Calculate the coefficient of correlation between height and weight
of the students using the following data:
Height Weight (lbs)
(inches) 90–100 100–110 110–120 120–130
50–55 4 7
55–60 6 10 7
60–65 10 12 7
65–70 8 6 13
Ans.
(viii) –0.790 (ix) 0.895 (x) 0.819 (xi) 0.512
(xii) 44.44 (xiii) CV : Stats = 40% (xiv) 0.515 (xv) 0.667
Accs = 35.56%
(xvi) – 0.958 (xvii) r = 0.750 (xviii) 0.583

306 PAGE
© Department of Distance & Continuing Education, Campus of Open Learning,
School of Open Learning, University of Delhi
L E S S O N

2
Regression Analysis
STRUCTURE
2.1 Learning Objectives
2.2 Introduction
2.3 Difference between Correlation and Regression
2.4 Principle of Least Squares
2.5 Methods of Regression Analysis
2.6 3URSHUWLHV RI 5HJUHVVLRQ &RHI¿FLHQWV
2.7 Standard Error of an Estimate
2.8 Summary
2.9 Self-Assessment Questions

2.1 Learning Objectives


After studying this lesson, you should be able to :
‹ Understand the concept of regression analysis.
‹ Differentiate between correlation and regression analysis.
‹ Compute regression coefficients by different methods and draw regression lines.
‹ Comprehend properties of Regression Coefficients.
‹ Apply regression analysis to predict a dependent variable given the independent
variable.

2.2 Introduction
The statistical technique correlation establishes the degree and direction of relationship
between two or more variables. But we may be interested in estimating the value of an
unknown variable on the basis of a known variable. If we know the index of money
supply and price-level, we can find out the degree and direction of relationship between
these indices with the help of correlation technique. But the regression technique helps us
in determining what the general price-level would be assuming a fixed supply of money.

PAGE 307
© Department of Distance & Continuing Education, Campus of Open Learning,
School of Open Learning, University of Delhi
B.COM. (PROGRAMME)

Notes Similarly if we know that the price and demand of a commodity are
correlated we can find out the demand for that commodity for a fixed
price. Hence, the statistical tool with the help of which we can estimate
or predict the unknown variable from known variable is called regression.
The meaning of the term “Regression” is the act of returning or going
back. This term was first used by Sir Francis Galton in 1877 when he
studied the relationship between the height of fathers and sons. His study
revealed a very interesting relationship. All tall fathers tend to have tall
sons and all short fathers short sons but the average height of the sons of
a group of tall fathers was less than that of the fathers and the average
height of the sons of a group of short fathers was greater than that of
the fathers. The line describing this tendency of going back is called
“Regression Line”. Modern writers have started to use the term estimating
line instead of regression line because the expression estimating line is
more clear in character. According to Morris Myers Blair, regression is
the measure of the average relationship between two or more variables
in terms of the original units of the data.
Regression analysis is a branch of statistical theory which is widely used
in all the scientific disciplines. It is a basic technique for measuring or
estimating the relationship among economic variables that constitute the
essence of economic theory and economic life. The uses of regression
analysis are not confined to economics and business activities. Its
applications are extended to almost all the natural, physical and social
sciences. The regression technique can be extended to three or more
variables but we shall limit ourselves to problems having two variables
in this lesson.
Regression analysis is of great practical use even more than the correlation
analysis. Some of the uses of the regression analysis are given below :
(i) Regression Analysis helps in establishing a functional relationship
between two or more variables. Once this is established it can be
used for various analytic purposes.
(ii) With the use of electronic machines and computers, the medium of
calculation of regression equation particularly expressing multiple
and non-linear relations has been reduced considerably.

308 PAGE
© Department of Distance & Continuing Education, Campus of Open Learning,
School of Open Learning, University of Delhi
BUSINESS STATISTICS

(iii) Since most of the problems of economic analysis are based on cause Notes
and effect relationship, the regression analysis is a highly valuable
tool in economic and business research.
(iv) The regression analysis is very useful for prediction purposes. Once
a functional relationship is established the value of the dependent
variable can be estimated from the given value of the independent
variables.

2.3 Difference between Correlation and Regression


Both the techniques are directed towards a common purpose of establishing
the degree and direction of relationship between two or more variables
but the methods of doing so are different. The choice of one or the
other will depend on the purpose. If the purpose is to know the degree
and direction of relationship, correlation is an appropriate tool but if
the purpose is to estimate a dependent variable with the substitution of
one or more independent variables, the regression analysis shall be more
helpful. The point of difference are discussed below:
(i) Degree and Nature of Relationship: The correlation coefficient is a
measure of degree of covariability between two variables whereas
regression analysis is used to study the nature of relationship
between the variables so that we can predict the value of one on
the basis of another. The reliance on the estimates or predictions
depend upon the closeness of relationship between the variables.
(ii) Cause and Effect Relationship: The cause and effect relationship
is explained by regression analysis. Correlation is only a tool to
ascertain the degree of relationship between two variables and we
can not say that one variable is the cause and other the effect. A high
degree of correlation between price and demand for a commodity
or at a particular point of time may not suggest which is the cause
and which is the effect. However, in regression analysis cause and
effect relationship is clearly expressed – one variable is taken as
dependent and the other an independent.
The variable which is the basis of prediction is called independent variable
and the variable that is to be predicted is called dependent variable. The
independent variable is represented by X and the dependent variable by Y.

PAGE 309
© Department of Distance & Continuing Education, Campus of Open Learning,
School of Open Learning, University of Delhi
B.COM. (PROGRAMME)

Notes 2.4 Principle of Least Squares


Regression refers to an average of relationship between a dependent variable
with one or more independent variables. Such relationship is generally
expressed by a line of regression drawn by the method of the “Least
Squares”. This line of regression can be drawn graphically or derived
algebraically with the help of regression equations. Before the equation
of the least line can be determined some criterion must be established
as to what conditions the best line should satisfy. The condition usually
stipulated in regression analysis is that the sum of the squares of the
deviations of the observed Y values from the fitted line shall be minimum.
This is known as the least squares or minimum squared error criterion.
A line fitted by the method of least squares is the line of best fit. The
line satisfies the following conditions :
(i) The algebraic sum of deviations above the line and below the line
are equal to zero.
6 (x – xc) = 0 and 6 (y – yc) = 0
where xc and yc are the values derived with the help of regression
technique.
(ii) The sum of the squares of all these deviations is less than the sum
of the squares of deviations from any other value, we can say
6 (x – xc)2 is smaller than 6 (x – A)2 and
6 (y – yc)2 is smaller than 6 (y – A)2
where A is some other value or any other straight line.
(iii) The line of regression (best fit) intersect at the mean value of the
variables i.e., x and y.
(iv) When the data represent a sample from a larger population, the
least square line is the best estimate of the population line.

2.5 Methods of Regression Analysis


We can study regression by the following methods :
1. Graphic method (regression lines)
2. Algebraic method (regression equations)

310 PAGE
© Department of Distance & Continuing Education, Campus of Open Learning,
School of Open Learning, University of Delhi
BUSINESS STATISTICS

We shall understand these methods. Notes

2.5.1 Graphic Method


When we apply this method different points are plotted on a graph paper
representing different pairs of variables. These points give a picture of
a scatter diagram with several points spread over. A regression line may
be drawn between these points either by free hand or by a scale in such
a way that the squares of the vertical or horizontal distances between
the points and the line of regression is minimum. It should be drawn in
such a manner that the line leaves equal number of points on both sides.
However, to ensure this is rather difficult and the method only renders
a rough estimate which cannot be completely free from subjectivity of
person drawing it. Such a line can be a straight line or a curved line
depending upon the scatter of points and relationship to be established.
A non-linear free hand curve will have more element of subjectivity and
a straight line is generally drawn. Let us understand it with the help of
an example :
Example 1 :
Height of fathers Height of sons
(Inches) (Inches)
65 68
63 66
67 68
64 65
68 69
62 66
70 68
66 65
68 71
67 67
69 68
71 70
Solution : The diagram given below shows the height of fathers on
x-axis and the height of sons on y-axis. The line of regression called the
regression of y on x is drawn between the scatter dots.

PAGE 311
© Department of Distance & Continuing Education, Campus of Open Learning,
School of Open Learning, University of Delhi
B.COM. (PROGRAMME)

Notes

Fig. 1
Another line of regression called the regression line of x on y is drawn
amongst the same set of scatter dots in such a way that the squares of
the horizontal distances between dots are minimised.

Fig. 2

Fig. 3

312 PAGE
© Department of Distance & Continuing Education, Campus of Open Learning,
School of Open Learning, University of Delhi
BUSINESS STATISTICS

It is clear that the position of the regression line of x on y is not exactly Notes
like that of the regression line of y on x. In the following figure both
the regression of y on x and x on y are exhibited.

Fig. 4
When there is either perfect positive or perfect negative correlation
between the two variables, the two regression lines will coincide and
we will have only one line. The farther the two regression lines from
each other, the lesser is the degree of correlation and vice-versa. If the
variables are independent, correlation is zero and the lines of regression
will be at right angles. It should be noted that the regression lines cut
each other at the point of average of x and y, i.e., if from the point where
both the regression lines cut each other a perpendicular is drawn on the
x-axis, we will get the mean value of x series and if from that point a
horizontal line is drawn on the y-axis we will get the mean of y series.

2.5.2 Algebraic Method


The algebraic method for simple linear regression can be understood by
two methods:
(i) Regression Equations
(ii) Regression Coefficients
Regression Equations : These equations are known as estimating equations.
Regression equations are algebraic expressions of the regression lines.
As there are two regression lines, there are two regression equations :
(i) X on Y is used to describe the variations in the values of X for
given changes in Y.

PAGE 313
© Department of Distance & Continuing Education, Campus of Open Learning,
School of Open Learning, University of Delhi
B.COM. (PROGRAMME)

Notes (ii) Y on X is used to describe the variations in the values of Y for


given changes in X. The regression equations of Y on X is expressed
as
Yc = a + bX
The regression equations of X on Y is expressed as
Xc = a + bX
In these equations a and b are constants which determine the position
of the line completely. These constants are called the parameters of the
line. If the value of any of these parameters is changed, another line is
determined.
Parameter a refers to the intercept of the line and b to the slope of the
line. The symbol Yc and Xc refers to the values of Y computed and
the value of X computed on the basis of independent variable in both
the cases. If the values of both the parameters are obtained, the line is
completely determined. The values of these two parameters a and b can
be obtained by the method of least squares. With a little algebra and
differential calculus it can be shown that the following two equations,
are solved simultaneously, will give values of the parameters a and b
such that the least squares requirement is fulfilled;
For regression equation Yc = a + bX
6y = Na + b6x
6xy = a6x + b6x2
For regression equation Xc= a + bY
 6x = Na + b6y
6xy = a6y + b6y2
These equations are usually called the normal equations. In the equations
6x, 6y, 6xy, 6x 2, 6y 2 indicate totals which are computed from the
observed pairs of values of two variables x and y to which the least
squares estimating line is to be fitted and N is the number of observed
pairs of values. Let us understand by an example.
Example 2 : From the following data obtain the two regression equations :
x : 6 2 10 4 8
y : 9 11 5 8 7

314 PAGE
© Department of Distance & Continuing Education, Campus of Open Learning,
School of Open Learning, University of Delhi
BUSINESS STATISTICS

Solution : Notes
Computation of Regression Equations
x y xy x2 y2
6 9 54 36 81
2 11 22 4 121
10 5 50 100 25
4 8 32 16 64
8 7 56 64 49
6x = 30 6y = 40 6xy = 214 2
6x = 220 2
6y = 340
Regression line of Y on X is expressed by the equation of the form
Yc = a + bX
To determine the values of a and b, the following two normal equations
are solved
6y = Na + b6x
6xy = a6x + b6x2
Substituting the values, we get
40 = 5a + 30b ...(i)
214 = 30a + 220b ...(ii)
Multiplying equation (i) by 6, we get
240 = 30a + 180b ...(iii)
214 = 30a + 220b ...(iv)
Deduct equation (iv) from (iii)
– 40b = + 26
? b = – 0.65
Substitute the value of b in equation (i)
40 = 5a + 30 (– 0.65)
5a = 40 + 19.5 or a = 11.9
Substitute the values of a and b in the equation
Regression line of Y on X is
Yc = 11.9 – 0.65X

PAGE 315
© Department of Distance & Continuing Education, Campus of Open Learning,
School of Open Learning, University of Delhi
B.COM. (PROGRAMME)

Notes Regression line of X on Y is


Xc = a + bY
The corresponding normal equations are
  6x = Na + b6y
2
  6xy = a6y + b6y
Substituting the values
30 = 5a + 40b ...(i)
214 = 40a + 340b ...(ii)
Multiply equation (i) by 8
240 = 40a + 320b ...(iii)
214 = 40a + 340b ...(iv)
Deduct equation (iv) from (iii)
–20b = 26 or b = – 1.3
Substitute the value of b in equation (i)
30 = 5a + 40(– 1.3)
5a = 30 + 52 or a = 16.4
Substitute the values of a and b in the equation. Regression line of X
on Y is
Xc = 16.4 – 1.3Y
Regression Coefficients : In the regression equation b is the regression
coefficient which indicates the degree and direction of change in the
dependent variable with respect to a change in the independent variable.
In the two regression equations:
Xc = a + bY
Yc = a + bX
where bxy (b) and byx (b) are known as the regression coefficients of the
two equations. These coefficients can be obtained independently without
using simultaneous normal equations with these formulae:
Regression coefficients of X on Y is
σx
bxy = r
σy

316 PAGE
© Department of Distance & Continuing Education, Campus of Open Learning,
School of Open Learning, University of Delhi
BUSINESS STATISTICS

Σxy σ Σxy Notes


bxy = × x =
N σ x σ y σ y N σ2y

Σxy
bxy = where x = X − X and y = Y − Y
Σy 2

Regression Coefficient of Y on X is
σy
byx = r
σx

Σxy σ Σxy
byx = × y =
N σ x σ y σ x N σ 2x

Σxy
byx = where x = X − X and y = Y − Y
Σx 2
Example 3 : Calculate the regression coefficients from data given below :
Series x Series y
Average 25 22
Standard deviation 4 5 r = 0.8
Solution : The coefficient of regression of x on y is
σx 4
bxy = r = 0.8 × = + 0.64
σy 5

The coefficient of regression of y on x is


σy 4
byx = r = 0.8 × = 1.00
σx 5

2.6 Properties of Regression Coefficients


(i) The coefficient of correlation is the geometric mean of the two
regression coefficients, r = ± bxy × byx .

σy σx
i.e byx× bxy = r ×r
σx σy
byx × bxy = (for coefficient of correlation i.e r, put square root
both sides)

PAGE 317
© Department of Distance & Continuing Education, Campus of Open Learning,
School of Open Learning, University of Delhi
B.COM. (PROGRAMME)

Notes hence byx × bxy = r

(ii) Both the regression coefficients are either positive or negative. It


means that they always have identical sign i.e., either both have
positive sign or negative sign.
(iii) The coefficient of correlation and the regression coefficients will
also have same sign.
Covariance (X,Y)
byx =
σ 2y

Covariance (X,Y)
bxy =
σ 2x

both regression coefficients have the same sign as covariance (X, Y).
The covariance (X, Y) may be positive or negative then bxy and
byx also positive or negative because and never be negative
which means that and are always positive.
(iv) If one of the regression coefficient is more than unity, the other must
be less than unity because the value of coefficient of correlation
cannot exceed one (r = ± 1). It means r varies from -1 to +1.
So if  ” 
byx×bxy = r 2 ≤ 1 (because byx × bxy = r2 )
byx × bxy = ” 
1
then byx ”
bxy

Hence it is proved that if one regression coefficient is greater than one,


the other regression must coefficient is less than one.
(v) Regression coefficients are independent of the change in the origin
but not of the scale.
(vi) The average of regression coefficients is always greater than
correlation coefficient.

byx + bxy
Therefore, > r
2

318 PAGE
© Department of Distance & Continuing Education, Campus of Open Learning,
School of Open Learning, University of Delhi
BUSINESS STATISTICS

We can compute the regression equations with the help of regression Notes
coefficients by the following equations:
1. Regression equation X on Y
σx
X −X = r (Y − Y )
σy

where X is the mean of X series


Y is the mean of Y series
σx
r is the regression coefficient of x and y
σy

2. Regression equation Y on X
We can explain this by taking an example :
σy
Y −Y = r (X − X )
σx

Example 4 : Calculate the following from the below given data :


(a) the two regression equations,
(b) the coefficient of correlation and
(c) the most likely marks in Statistics when the marks in Economics are
30.
Marks in : 25 28 35 32 31 36 29 38 34 32
Economics
Marks in : 43 46 49 41 36 32 31 30 33 39
Statistics
Solution :
Calculation of Regression Equations and Correlation Coefficient

Marks in (X – X ) Marks in (Y – Y )
Eco (X) Stats (Y)
x x2 y y2 xy
25 – 7 49 43 + 5 25 – 35
28 – 4 16 46 + 8 64 – 32
35 + 3 9 49 + 11 121 + 33
32 0 0 41 + 3 9 0

PAGE 319
© Department of Distance & Continuing Education, Campus of Open Learning,
School of Open Learning, University of Delhi
B.COM. (PROGRAMME)

Notes
Marks in (X – X ) Marks in (Y – Y )
Eco (X) Stats (Y)
x x2 y y2 xy
31 – 1 1 36 – 2 4 + 2
36 + 4 16 32 – 6 36 – 24
29 – 3 9 31 – 7 49 + 21
38 + 6 36 30 – 8 64 – 48
34 + 2 4 33 – 5 25 – 10
32 0 0 39 + 1 1 0
6X = 320 6x = 0 6x2 = 140 6Y = 380 6y = 0 6y2 = 398 6xy = – 93
(a) Regression equation X on Y
X − X = bxy (Y − Y )
Σxy −93
bxy = = = −0.234
Σy 2 398

X = ΣX = 320 = 32 and Y = ΣY = 380 = 38


N 10 N 10
Substituting the values
X – 32 = – 0.234 (Y – 38)
X – 32 = – 0.234Y + 8.892
or X = 40.892 – 0.234Y
Regression equation Y on X
(Y − Y ) = byx ( X − X )
Σxy −93
byx = = = −0.664
Σx 2 140

X = 32, Y = 38, b = – 0.664


? Y – 38 = – 0.664 (X – 32)
= – 0.664Y + 21.248
or Y = 59.248 – 0.664X

(b) Correlation Coefficient (r) = ± bxy × byx

= – −0.234 × −0.664 = −0.394

320 PAGE
© Department of Distance & Continuing Education, Campus of Open Learning,
School of Open Learning, University of Delhi
BUSINESS STATISTICS

Since both the regression coefficients are negative, value of r must Notes
also be negative.
(c) Likely marks in statistics when marks in Economics are 30.
Y = – 0.664 X + 59.248 where X = 30
Y = (– 0.664 × 30) + 59.248 = 39.328 or 39.
Example 5 : The following scores were worked out from a test in
Mathematics and English in an annual examination.
Scores in English (y)
Mathematics
(x)
Mean 39.5 47.5
Standard deviation 10.8 16.8 r = + 0.42
Find both the regression equations. Using these regression estimate find
the value of Y for X = 50 and the value of X for Y = 30.
Solution : Regression of X on Y
σx
X −X = r (Y − Y )
σy

where Y = 47.5, X = 39.5, r = 0.42, Vx = 10.8, and Vy = 16.8


By substituting values, we get
10.8
X – 39.5 = 0.42 (Y − 47.5)
16.8
= 0.27 (Y – 47.5) = 0.27 Y – 12.82
or X = 0.27Y – 12.82 + 39.5 = 0.27Y + 26.68
when Y = 30
Value of X = (0.27 × 30 + 26.68) = 34.78
Regression equation of Y on X
σy
Y −Y = r (X − X )
σx

where X = 39.5, Y = 47.5, r = 0.42, Vx= 10.8, and Vy = 16.8

PAGE 321
© Department of Distance & Continuing Education, Campus of Open Learning,
School of Open Learning, University of Delhi
B.COM. (PROGRAMME)

Notes 16.8
Y – 47.5 = 0.42 ( X − 39.5)
10.8
Y – 47.5 = 0.653 (X – 39.5) = 0.653 X – 25.79
or Y = 0.653 X – 25.79 + 47.5 = 0.653X + 21.71
When X = 50
Value of Y= (0.653 × 50 + 21.71) = 32.65 + 21.71 = 54.36
Thus the regression equations are :
Xc = 0.27y + 26.68
Yc = 0.653x + 21.71
Value of X when Y = 30 is 34.78
Value of Y when X = 50 is 54.36
When actual mean of both the variables X and Y come out to be in fractions,
the deviation from actual means create a problem and it is advisable to
take deviations from the assumed mean. Thus when deviations are taken
from assumed means, the value of bxy and byx is given by
(Σdx) × (Σdy )
Σdxdy −
bxy = N where dx = (X – A) and dy = (Y – A)
( Σ dy )2
Σdy −
2

N
The regression equation is :

( X − X ) = bxy (Y − Y )
Similarly the regression equation of Y on X is

(Y − Y ) = byx ( X − X )
(Σdx) × (Σdy )
Σdxdy −
byx = N
( Σ dx )2
Σdx 2 −
N
Let us try to understand with the help of an example :
Example 6 : You are given the data relating to purchases and sales.
Compute the two regression equations by method of least squares and
estimate the likely sales when the purchases are 100.

322 PAGE
© Department of Distance & Continuing Education, Campus of Open Learning,
School of Open Learning, University of Delhi
BUSINESS STATISTICS

Purchases : 62 72 98 76 81 56 76 92 88 49 Notes
Sales : 112 124 131 117 132 96 120 136 97 85
Solution :
Calculations of Regression Equations
Purchases (X–76) Sales (Y–120)
2
X dx dx Y dy dy2 dxdy
62 – 14 196 112 – 8 64 + 112
72 – 4 16 124 + 4 16 – 16
98 + 22 484 131 + 11 121 + 242
76 0 0 117 – 3 9 0
81 + 5 25 132 + 12 144 + 60
56 – 20 400 96 – 24 576 + 480
76 0 0 120 0 0 0
92 + 16 256 136 + 16 256 + 256
88 +12 144 97 –23 529 – 276
49 –27 729 85 –35 1225 + 945
6dx = – 10 6dx 2 = 6dy = – 6dy2 = 6dxdy =
2250 50 2940 1803
Σdx 10 Σdy 50
X = A+ = 76 − = 75 and Y = A + = 120 − = 115
N 10 N 10
Regression Coefficients : X on Y
(Σdx) × (Σdy ) (−10) × (−50)
Σdxdy − 1803 −
N 10 1753
bxy = = = = 0.652
(Σdy ) 2
(−50) 2
2690
Σdy −
2
2940 −
N 10
Y on X
(Σdx) × (Σdy ) (−10) × (−50)
Σdxdy − 1803 −
N 10 1753
byx = = = = 0.78
(Σdx) 2
(−10) 2
2240
Σdx −
2
2250 −
N 10
Regression equation : X on Y

X − X = bxy (Y − Y )

PAGE 323
© Department of Distance & Continuing Education, Campus of Open Learning,
School of Open Learning, University of Delhi
B.COM. (PROGRAMME)

Notes Substituting the values


X – 75 = 0.652 (Y – 115) = 0.652Y – 74.98
or X = 0.652Y + 0.02
Regression equation : Y on X

(Y − Y ) = bxy ( X − X )
Y – 115 = 0.78 (X – 75) = 0.78 X – 58.5
Y = 0.78 X + 56.5
when X = 100
Y = 0.78 × 100 + 56.5= 134.5

2.7 Standard Error of an Estimate


Standard error of an estimate is the measure of the spread of observed
values from estimated ones, expressed by regression line or equation.
The concept of standard error an estimate is analogous to the standard
deviation which measures the variation or scatter of individual items about
the arithmetic mean. Therefore, like the standard deviation which is the
average of square of deviations about the arithmetic mean, the standard
error of an estimate is the average of the square of deviations between
the actual or the observed values and the estimated values based on the
regression equation. It can also be expressed as the root of the measure
of unexplained variations divided by N – 2:
Unexplained variation Σ(Y − Yc ) 2
Syx = =
N–2 N −2

Σ( X − X c ) 2
and Sxy =
N −2
where Syx refers to standard error of estimate of Y values on X values.
Sxy refers to standard error of estimate of X values on Y values.
Yc and Xc are the estimated values of Y and X variables by means of their
regression equations respectively. N – 2 is used for getting an unbiased
estimate of standard error. The usual explanation given for this division
by N – 2 is that the two constants a and b were calculated on the basis

324 PAGE
© Department of Distance & Continuing Education, Campus of Open Learning,
School of Open Learning, University of Delhi
BUSINESS STATISTICS

of original data and we lose two degrees of freedom. Degrees of freedom Notes
means the number of classes to which values can be assigned at will
without violating any restrictions.
However a simpler method of computing Syx and Sxy is to use the
following formulae :
ΣY 2 − aΣY − bΣXY
Syx =
N −2

ΣX 2 − aΣX − bΣXY
and Sxy =
N −2
The standard error of estimate measures the accuracy of the estimated
figures. The smaller the values of standard error of estimate, the closer
will be the dots to the regression line and the better the estimates based
on the equation for this line. If standard error of estimate is zero, then
there is no variation about the line and the correlation will be perfect.
Thus with the help of standard error of estimate it is possible for us to
ascertain how good and representative the regression line is as a description
of the average relationship between two series.
Example 7 : Given the following data :
X : 6 2 10 4 8
Y : 9 11 5 8 7
And two regression equations Y = 11.09 – 0.65 X and X = 16.4 – 1.3
Y. Calculate the standard error of estimate i.e. Syx and Sxy.
Solution :
We can calculate Xc and Yc values from these regression equations.
X Y Yc Xc (Y – Yc)2 (X – Xc)2
6 9 8.0 4.7 1.00 1.69
2 11 10.6 2.1 0.16 0.01
10 5 5.4 9.9 0.16 0.01
4 8 9.3 6.0 1.69 4.00
8 7 6.7 7.3 0.09 0.49
6X = 30 6Y = 40 6Yc = 40 6Xc = 30 6(Y–Yc)2 = 6(X –Xc)2
3.1 = 6.20
Thus we can calculate Syx and Sxy from the above calculated values.

PAGE 325
© Department of Distance & Continuing Education, Campus of Open Learning,
School of Open Learning, University of Delhi
B.COM. (PROGRAMME)

Notes
Σ(Y − Yc ) 2 3.1
Syx = = = 1.03 = 1.01
N −2 5−2

Σ( X − X c ) 2 6.2
Sxy = = = 2.07 = 1.44
N −2 5−2

2.8 Summary
Regression analysis deals with estimating values of one variable based on
the values of one or more other variables. The variable being estimated
is called dependent variable while the variable/s used to make estimates
is/are called independent variable/s. The simple regression analysis
involves one independent variable and one dependent variable. It is based
on the assumption of linear relationship between the two variables. The
relationship between variables is presented by means of a regression
equation which is obtained using the principle of least squares. For a
given set of data involving two variables, X and Y, we can derive two
regression equations: one treating Y as the dependent variable and the
other treating X as the dependent variable.
When correlation between two variables is perfect, the two regression
equations are reversible because they both actually represent the same
line. The closer the two regression lines to each other, the higher is
the degree of correlation. The sign of the two regression coefficients is
always the same as the sign of the coefficient of correlation. Standard
error of estimate measures the variation around the regression line. A
small value of the standard error implies that the data cluster around
the regression line.

2.9 Self-Assessment Questions


Exercise 1 : True or False statements :
(i) Regression is a tool for making estimates of an independent variable
for a given value of a dependent variable.
(ii) In the regression equation Yc= a + bX, the variable Y is the independent
variable.

326 PAGE
© Department of Distance & Continuing Education, Campus of Open Learning,
School of Open Learning, University of Delhi
BUSINESS STATISTICS

(iii) In the regression equation Yc = a + bX, a and b are estimates of Notes


the population intercept D and slope E respectively.
(iv) The least squares principle ensures that sum of deviations from
regression line is equal to zero and Σ(Y − Y ) 2 is the minimum.
(v) In a regression equation, both a and b must bear the same sign.
(vi) In the case of negative correlation between the variables, one regression
line is positively sloped while the other is negatively sloped.
(vii) The sum of two regression coefficients is always equal to 1.
(viii) If the two regression equations are solved simultaneously, the X
and Y values are respectively the values of X and Y .
(ix) In the case of perfect correlation between two variables, both the
regression coefficients are equal.
(x) It is feasible to have byx= 5.4 and bxy= 0.15 for a given set of data.
(xi) The two regression coefficients byx and bxy cannot both be smaller
than 1.
(xii) The difference between Y and Yc is called error.
(xiii) The standard error of estimate can never be equal to zero.
(xiv) The coefficient of correlation is equal to geometric mean of the
two regression coefficients.
(xv) The regression coefficients are independent of the change of scale,
but they are not independent of the change of origin.
Ans.
(i) F (ii) F (iii) T (iv) F (v) F
(vi) F (vii) F (viii) T (ix) F (x) T
(xi) F (xii) T (xiii) F (xiv) T (xv) F
Exercise 2 : Questions and Answers
(i) What do you understand by regression? What role does it play in
business and economic analysis?
(ii) Explain in your own words as to why there are two regression lines
in the case of paired values of two variables. At what point do the

PAGE 327
© Department of Distance & Continuing Education, Campus of Open Learning,
School of Open Learning, University of Delhi
B.COM. (PROGRAMME)

Notes two regression lines intersect? If the two regression lines coincide,
what does it imply?
(iii) Explain the properties of the regression coefficients. Do you agree
that for a given set of data if each of the X values, is multiplied by
5, then the regression coefficient of Y on X would also be multiplied
by 5 while the regression coefficient of X on Y will be reduced to
l/5th of its original value? Explain.
(iv) Explain the properties of regression coefficients. What is the difference
between Regression and Correlation Analysis?
(v) Given the following data:
X : 7 9 7 12 12 11 14 16
Y : 6 12 12 14 14 16 18 20
(a) Fit the regression equation of Y on X.
(b) Estimate the value of Y for X = 15.
(vi) Given the following information: 6X = 56; 6Y = 40; 6X2 = 524;
6Y2: = 256; 6XY= 364: and n = 8. Obtain the regression equation
of X on Y.
(vii) In the estimation of the regression equation of two variables X and
Y, the following results were obtained:
6X = 90: 6Y = 70; 6X2 = 6,360; 6Y2 = 2,860; 6XY = 3,900: and n
= 10
Obtain the two regression equations.
(viii) The following data relate to 50 workers of a factory in respect of
their experience (X) in months and time needed (Y) in minutes to
fit an apparatus.
Mean of X = 50
Mean of Y = 60
Standard deviation of X = 20
Standard deviation of Y = 20
Covariance (XY) = –100

328 PAGE
© Department of Distance & Continuing Education, Campus of Open Learning,
School of Open Learning, University of Delhi
BUSINESS STATISTICS

Calculate the two regression coefficients and the coefficient of Notes


determination.
(ix) Using the following data:
(a) Obtain the two regression equations.
(b) Find the likely sales when advertising expenditure is Rs. 25
crores.
(c) Estimate the advertising budget to achieve the sales target of
Rs. 150 crores?
Advertising Sales
expenditure
(Rs. crores) (Rs. crores)
Mean 20 120
Standard deviation 5 25
Coefficient of correlation 0.8
(x) The following data about the sales and advertisement expenditure
of a firm are given :
Sales (X) Advertisement
expenditure
(in crores of Rs.) (in lakhs of Rs.)
Mean 40 60
Standard deviation 10 15
Coefficient of correlation 0.9
(a) Estimate the likely sales for a proposed advertisement expenditure
of Rs. 100 lakh.
(b) What should the advertisement expenditure be if the firm
proposes a sale target of Rs. 60 crores?
(xi) The HR manager of Anomaly International wants to study the
relationship between number of years, experience and performance
scores of the employees. An analysis of five employees shows the
following results :
No. of years’ 6 2 10 4 8
experience (X) :
Performance 19 11 25 18 17
score (Y) :

PAGE 329
© Department of Distance & Continuing Education, Campus of Open Learning,
School of Open Learning, University of Delhi
B.COM. (PROGRAMME)

Notes (a) Fit a regression equation of Y on X and interpret it.


(b) Calculate the likely performance score if experience is five
years.
(c) Calculate the standard error of estimate and comment on the
reliability of the estimating equation.
(xii) While making calculations about regression equations, a student
obtained the following results :
n = 25, 6X = 125, 6X2 = 650; 6Y = 100, 6Y2 = 460, and 6XY =
508
It was discovered later, however, that two pairs of values were
wrongly recorded as
X Y X Y
6 14 while the correct values 8 12
were :
8 6 6 8
Obtain the two regression equations.
(xiii) The equations of two lines of regression between variables X and
Y (not necessarily in that order) are 2X + 3Y – 8 = 0 and X + 2Y
– 5 = 0. The variance of X is 4. Find
(a) Variance of Y.
(b) Coefficient of Determination of X and Y
(c) Standard error of estimate of X on Y, and standard error of
estimate of Y on X.
(xiv) Given the following data :
Age Salary Total
(X)
(Years) 250–300 300–350 350–400 400–450 450–500
20–30 5 5
30–40 2 3 2 7
40–50 1 6 3 10
50–60 1 2 1 4 8
Total 7 5 10 4 4 30

330 PAGE
© Department of Distance & Continuing Education, Campus of Open Learning,
School of Open Learning, University of Delhi
BUSINESS STATISTICS

Obtain the two regression coefficients and the regression equations. Notes
Ans.
(v) Y = 0.861 + 01.194X, 18.78
(vi) X = 0.5 + 1.5Y
(vii) Y = 1.70 + 0.589X and X = – 0.66 + 1.38Y
(viii) byx = bxy = – 0.25, r2 = 0.0625
(ix) (a) Y = 4 + 4X, X = 0.8 + 0.16Y (b) 140 crores (c) 24.8 crores
(x) (i) 64 crores, (ii) 87 lakhs
(xi) (i) Y = 9.9 + 1.35X (ii) 16.65, (iii) 3.006
(xii) Y = + 0.8 X and X = 2.778
(xiii) (a) 1.333 (b) 0.75 (c) SEyx = 0.5774, SExy = 1
(xiv) byx = 5.143, bxy = 0.125, Y = 146.86 + 5.143X, X = – 3.07 +
0.125Y

PAGE 331
© Department of Distance & Continuing Education, Campus of Open Learning,
School of Open Learning, University of Delhi
UNIT-4

PAGE 333
© Department of Distance & Continuing Education, Campus of Open Learning,
School of Open Learning, University of Delhi
L E S S O N

1
Index Numbers
STRUCTURE
1.1 Learning Objectives
1.2 Introduction
1.3 Features of Index Numbers
1.4 Problems of Index Numbers
1.5 Methods of Constructing Index Numbers
1.6 Tests of Adequacy or Consistency
1.7 Chain Base Index
1.8 Splicing
1.9 Consumer Price Index
1.10 Index Number of Industrial Production
1.11 Limitations of Index Numbers
1.12 Construction of BSE Sensex and NSE Nifty
1.13 Summary
1.14 Self-Assessment Questions

1.1 Learning Objectives


After studying this lesson you should be able to:
‹ Understand the meaning and uses of index numbers.
‹ Identify various problems faced in the construction of index numbers.
‹ Learn different methods of constructing index numbers including BSE sensex and
NIFTY.
‹ Appreciated different tests of consistency of index numbers.
‹ Learn the consumer price index and its computation.
‹ Learn the process of base shifting, spacing and deflating of index numbers.

PAGE 335
© Department of Distance & Continuing Education, Campus of Open Learning,
School of Open Learning, University of Delhi
B.COM. (PROGRAMME)

Notes 1.2 Introduction


Economic activities have constant tendency to change. Prices of commodities
which are the total result of number of economic activities also have a
tendency to fluctuate. The problem of change in prices is very important.
But it is not very simple to study this problem and derive conclusions
because price of different commodities change by different degrees. Hence,
there is a great need for a device which can smoothen the irregularities in
the prices to obtain a conclusion. This need is satisfied by Index Numbers
which makes use of percentages and average for achieving the desired
objective. Index Number is a device for comparing the general level of
the magnitude of a group of distinct but related variables in two or more
situations. Index Numbers are used to feel the pulse of the economy and
they reveal the inflationary or deflationary tendencies. In reality, Index
Numbers are described as barometers of economic activity because if
one wants to have an idea as to what is happening in an economy, he
should check the important indices like the index numbers of industrial
production, agricultural production, business activity etc.
The various definitions of Index Numbers are discussed under three heads :
(i) Measure of change.
(ii) Device to measure change.
(iii) A series representing the process of change.
According to Maslow, it is a numerical value characterising the change
in complex economic phenomenon over a period of time.
Spiegal explains an index number is a statistical measure designed to
show changes in variable or a group of related variables with respect to
time, geographical location or other characteristics.
Gregory and Ward describes it as a measure over time designed to show
average change in the price, quantity or value of a group of items.
Croxton and Cowden says Index numbers are devices for measuring
differences in the magnitude of a group of related variables.
B.L. Bowley describes Index Numbers as a series which reflects in its trend
and fluctuations the movements of some quantity to which it is related.
Blair puts Index Numbers as a specialised kind of average.

336 PAGE
© Department of Distance & Continuing Education, Campus of Open Learning,
School of Open Learning, University of Delhi
BUSINESS STATISTICS

1.3 Features of Index Numbers Notes

Index Numbers have the following features :


(i) Index numbers are specialised averages which are capable of being
expressed in percentage.
(ii) Index numbers measure the changes in the level of a given phenomenon.
(iii) Index numbers measure the effect of changes over a period of time.
Index Numbers are indispensable tools of economic and business analysis.
Their significance can be appreciated by following points:
1. Index number helps in measuring relative changes in a set of items.
2. Index numbers provide a good basis of comparison because they are
expressed in abstract unit distinct from the unit of element.
3. Index numbers help in framing suitable policies for business and
economic activities.
4. Index numbers help in measuring the general trend of the phenomenon.
5. Index numbers are used in deflating. They are used to adjust the
original data for price changes or to adjust wages for cost of living
changes.
6. The utility of index numbers has increased a great deal because
of the method of splicing whereby the index prepared on any one
base can be adjusted with reference to any other base.
7. As a measure of average change in a group of elements the index
numbers can be used for forecasting future events. Whereas a trend
line gives an average rate of change in a single phenomenon, it
indicates the trend for a group of commodities.
8. It is helpful in a study of comparative purchasing power of money
in different countries of the world.
9. Index numbers of business activities throw light on the economic
progress made by various countries.
Uses of Index Numbers
Index numbers are widely used in various fields and industries to measure
changes, compare data, and make informed decisions. Their versatility

PAGE 337
© Department of Distance & Continuing Education, Campus of Open Learning,
School of Open Learning, University of Delhi
B.COM. (PROGRAMME)

Notes and ability to represent trends and relative changes make them valuable
tools. Here are some common uses of index numbers:
‹ Inflation Measurement: Consumer Price Index (CPI) and Producer
Price Index (PPI) are used to track inflation rates by measuring
changes in the prices of goods and services over time. These indexes
help policymakers and economists understand price trends and their
impact on the economy.
‹ Economic Indicators: Index numbers are used to calculate economic
indicators such as Gross Domestic Product (GDP) deflator and
real GDP growth. These indicators provide insights into economic
performance while adjusting for inflation.
‹ Cost-of-Living Adjustment: Index numbers help determine Cost-of-
Living Adjustments (COLA) for salaries, pensions, and benefits.
This ensures that payments keep pace with changes in the price
level, maintaining purchasing power.

1.4 Problems of Index Numbers


While constructing Index Number, the following problems arise:

1.4.1 Purpose of Index Numbers


Before constructing an Index Number, it is necessary to define precisely
the purpose for which they are to be constructed. A single Index can
not fulfil all the purposes. Index Numbers are specialised tools which
are more efficient and useful when properly used. If the purpose is not
clear, the data used may be unsuitable and the indices obtained may be
misleading. If it is desired to construct a Cost of Living Index Number of
Labour class, then only those item will be included, which are required
by the Labour class.

1.4.2 Selection of Items


The list of commodities included in the Index numbers is called the
‘Regimen’. Because it may not be possible to include all the items, it
becomes necessary to decide what items are to be included. Only those

338 PAGE
© Department of Distance & Continuing Education, Campus of Open Learning,
School of Open Learning, University of Delhi
BUSINESS STATISTICS

items should be selected which are representative of the data, e.g. in Notes
a consumer Price Index for working class, items like scooters, cars,
refrigerators, cosmetics, etc. find no place. There is no hard and fast rule
regarding the inclusion of number of commodities while constructing Index
Numbers. The number of commodities should be such as to permit the
influence of the inertia of large numbers. At the same time the numbers
should not be so large as to make the work of computation uneconomical
and even difficult. The number of commodities should therefore be
reasonable. The following points should be considered while selecting
the items to be included in the Index :
(i) The items should be representative.
(ii) The items should be of a standard quality.
(iii) Non-tangible items should be excluded.
(iv) The items should be reasonable in number.

1.4.3 Price Quotations


It is neither possible nor necessary to collect prices of the commodities
from all markets in the country where it is dealt with, we should take
a sample of the markets. Selection must be made of the representative
places and persons. These places should be well known for trading these
commodities.
It is necessary to select a reliable agency from where price quotations
are obtained.

1.4.4 Selection of Base Period


In the construction of Index Numbers, the selection of the base period
is very important step since the base period serves as a reference period
and the prices for a given period are expressed as percentages of those
for the base year, it is therefore necessary that:
(i) The base period should be normal and
(ii) It should not be too far in the past.
There are two methods by which base period can be selected (i) Fixed
base method and (ii) Chain base method.

PAGE 339
© Department of Distance & Continuing Education, Campus of Open Learning,
School of Open Learning, University of Delhi
B.COM. (PROGRAMME)

Notes Fixed Base Method : According to this any year is taken as a base.
Prices during the year are taken equal to 100 and the prices of other
years are shown as percentages of those prices of the base year. Thus if
indices for 1998, 1999, 2000, and 2001 are calculated with 1997 as base
year, such indices will be called as fixed base indices.
Chain Case Method : According to this method, relatives of each year
are calculated on the basis of the prices of the preceding year. The Chain
base Index Numbers are called as Link Relatives e.g., if index numbers
are constructed for 1997, 1998, 1999, 2000 and 2001 then for 1998,1997
will be the base and for 1999, 1998 will be the base and so on.

1.4.5 Choice of An Average


An Index number is a technique of ‘averaging’ all the changes in the
group of series over a period of time, the main problem is to select an
average which may be able to summarise the change in the component
series adequately. Median, Mode and Harmonic Mean are never used in
the construction of index numbers. A choice has to be made between
the Arithmetic Mean and the Geometric Mean. Merits and demerits of
the two are then to be compared. Theoretically G.M. is superior to the
A.M. in many respects but due to difficulty in its computation, it is not
widely used for this purpose.

1.4.6 Selection of Appropriate Weights


The term weight refers to the relative importance of the different items in
the construction of index numbers. All items are not of equal importance
and hence it is necessary to find out some suitable methods by which
the varying importance of the different items is taken into account. The
system of weighing depends upon the purpose of index numbers, but
they ought to reflect the relative importance of the commodities in the
regimen. The system may be either arbitrary or rational. The weightage
may be according to either:
(1) The value of quantity produced, or
(2) The value of quantity consumed, or
(3) The value or quantity sold or put on sale.

340 PAGE
© Department of Distance & Continuing Education, Campus of Open Learning,
School of Open Learning, University of Delhi
BUSINESS STATISTICS

There are two methods of assigning weights. Notes


(i) Implicit and (ii) Explicit.
Implicit : Under this method, the commodity to which greater importance
has to be given is repeated a number of times i.e., a number of varieties
of such commodities are included in the index numbers as separate items.
Explicit : In this case, the weights are explicitly assigned to commodities.
Only one kind of a commodity is included in the construction of Index
numbers but its price relative is multiplied by the figure of weights assigned
to it. There has to be some logic in assigning such type of weights.

1.5 Methods of Constructing Index Numbers


The index number for this purpose is divided into two heads:
(1) Unweighted Indices; and
(2) Weighted Indices.
Each one of these types is further sub-divided under two categories:
(i) Simple aggregative; and
(ii) Average of price relatives.

1.5.1 Unweighted Index Numbers


(i) Simple Aggregative Method : Under this method the total of the
current year prices for various commodities is divided by the total
of the base year and the quotient is multiplied by 100.
Symbolically,
Σp1
P01 = ×100
Σp0

where P01 represents the Price Index, P1 represents prices of current


year and P0 prices of base year.
Example 1 : From the following data construct the index for 2013 taking
2000 at base year.

PAGE 341
© Department of Distance & Continuing Education, Campus of Open Learning,
School of Open Learning, University of Delhi
B.COM. (PROGRAMME)

Notes Commodity Prices in 2000 Prices in 2013


(Rs.) (Rs.)
A 30 30
B 35 50
C 45 75
D 45 70
E 25 40
Solution : Construction of Price Index.
Commodity Prices in 2000 Prices in 2013
(Rs.) (Rs.)
A 30 30
B 35 50
C 45 75
D 45 70
E 25 40
= 180 = 265
Price Index for 2013 with 2000 as base
Symbolically,
ΣP1 265
P01 = × 100 = × 100 = 147.2
ΣP0 180

Hence there is an increase of 47.2% in prices of commodities during the


year 2013 as compared to 2000.
(ii) Average of Price Relative Method: Under this method, calculate first
the price relatives for the various items included in the index and
then average the price relatives by using any of the measures of
the central value, i.e. A.M.; the median; the mode; the Geometric
mean or the Harmonic mean.
⎧p ⎫
Σ ⎨ 1 ×100 ⎬
⎩ p0 ⎭
(a) When arithmetic mean is used P01 =
N

342 PAGE
© Department of Distance & Continuing Education, Campus of Open Learning,
School of Open Learning, University of Delhi
BUSINESS STATISTICS

⎧ ⎧⎪ Notes
⎛p ⎞ ⎫⎪ ⎫
⎪ ⎨Σ log ⎜ 1 ×100 ⎟ ⎬ ⎪
⎨ ⎪⎩ ⎝ p0 ⎠ ⎪⎭ ⎬
(b) When geometric mean is used P01 = AL ⎪ ⎪
⎩ N ⎭
where N refers to the number of items whose price relatives are averaged.
Example 2 : Calculate Index Numbers for 2011, 2012 and 2013 taking
2010 as base from the following data by average of relatives method.
Commodity 2010 2011 2012 2013
A 2 5 4 3
B 8 11 13 6
C 4 5 6 8
D 6 4 5 7
E 5 4 6 3
Solution :
Construction of Index Numbers based on Mean of Relatives
Commodity 2010 2011 2012 2013
p0 p1 p2 p3
A 2 100 5 250.0 4 200.0 3 150.0
B 8 100 11 137.5 13 162.5 6 75.0
C 4 100 5 125.0 6 150.0 8 200.0
D 6 100 4 66.7 5 83.3 7 116.7
E 5 100 4 80.0 6 120.0 3 60.0
500 659.2 715.8 601.7
P01 = Index with 2010 as base and 2011 as current year
⎛p ⎞
Σ ⎜ 1 ×100 ⎟
p
P01 = ⎝ 0 ⎠ = 659.2 = 131.84
N 5

P02= Index with 2010 as base and 2012 as current year


⎛p ⎞
Σ ⎜ 2 × 100 ⎟
p
P02 = ⎝ 0 ⎠ = 715.8 = 143.16
N 5
P03 = Price Index with 2010 as base and 2013 as current year
Advantage and Disadvantage of Simple average of Price relatives:

PAGE 343
© Department of Distance & Continuing Education, Campus of Open Learning,
School of Open Learning, University of Delhi
B.COM. (PROGRAMME)

Notes Advantage
‹ The index number based on the simple average of price relatives
method is not influenced by the units in which prices are quoted.
‹ The index number based on this method is not influenced by extreme
price quotations.
Disadvantage
‹ It gives equal importance to all the items and thus neglects their
relative importance in the group. This drawback can be removed
by taking the weighted average of the price relatives.
‹ A fair amount of difficulties is observed regarding the selection
of an appropriate average. G.M though difficult to compute, is
theoretically a better average than Arithmetic Mean. However, because
of the computational ease, arithmetic mean is used in practice.

1.5.2 Weighted Index Numbers


(I) Aggregative Method: These indices are of the simple aggregative type
with the only difference that the weights are assigned to the various
items included in the index. This method in fact can be described
as an extension of the simple aggregative method in the sense that
the weights are assigned to the different commodities included in
the index. There are various methods by which weights can be
assigned and hence a large number of formulae for constructing
Index Numbers have been devised. Some commonly used methods
suggested by different authorities are as follows :
(i) Laspeyre’s method.
(ii) Paasche’s method.
(iii) Fisher’s ideal method.
(iv) Marshall Edgeworth method.
(v) Kelly’s method.
(vi) Dorbish and Bowley’s method.

344 PAGE
© Department of Distance & Continuing Education, Campus of Open Learning,
School of Open Learning, University of Delhi
BUSINESS STATISTICS

(i) Laspeyre’s Method Notes


Laspeyre suggested that for calculating Price Indices, the quantities in the
base year should be used as weights. Hence the formula for computing
price Index number would be :
where P01 refers to Price Index,
p refers to price of each commodity,
q refers to quantity of each commodity,
0 base year,
1 current year, and
¦ refers to the summation of items.
The steps for calculating Index Numbers are :
(a) Multiply the price of each commodity for current year with its
respective Quantity for the base year (p1 × q0) and then find out
the total of this product ¦(P1q0).
(b) Multiply the price of each commodity for the base year with the
respective quantity for the base year (P0 × q0) and then find out
the total of these products for different commodities ¦(P1q0).
(c) Divide (p q ) with (p q ) and multiply the quotient by 100. On the
1 0 0 0
other hand, if Quantity Index is to be calculated, the prices of base
year will be used as weights. Symbolically,
∑ q1p 0
Q01 = ×100
∑ q 0 p0

Example 3 : Compute Price Index and Quantity Index from data given
below by Laspeyre’s method.
Items Base year Current year
Quantity Price Quantity Price
A 6 units 40 paise 7 units 30 paise
B 4 units 45 paise 5 units 50 paise
C 5 units 90 paise 1.5 units 40 paise

PAGE 345
© Department of Distance & Continuing Education, Campus of Open Learning,
School of Open Learning, University of Delhi
B.COM. (PROGRAMME)

Notes Solution : Computation of Price and Quantity Indices.


Base Current
year year
Items q0 p0 q1 p1 p 0q 0 p 1q 0 p 0q 1 p 1q 1
A 6 40 7 30 240 180 280 210
B 4 45 5 50 180 200 225 250
C 5 90 1.5 40 45 20 135 60
p0q0 = 465 6p1q0 = 400 6p0q1 = 640 6p1q1 = 520

Σp1q0 400
Price Index (P01) = ×100 = ×100 = 86.00
Σp0 q0 465

Σq1 p0 640
Quantity Index (Q01) = ×100 = ×100 = 137.63
Σq0 p0 665

(ii) Paasche’s Method: Under this method of calculating Price Index the
quantities of the current year are used as weights as compared to
base year quantities used by Lespeyre. Symbolically
Price Index or P01
Steps of construction Index according to Paasche’s method are :
(i) Calculate the product of the current year prices of different commodities
and their respective quantities for the current year (p1× q1)and find
out the total of the product of different commodities 6(p1× q1) .
(ii) Calculate the product of p0 and q1 of different commodities and
aggregate them 6(p0q1).
(iii) Divide 6(p1× q1) with 6(p0q1) and multiply the quotient by 100 to
obtain Price Index. Similarly, quantity index is calculated using the
current year price as weights. Symbolically,
Σq1 p1
Q01 = ×100
Σq0 p1
Example 4 : From the data of previous illustration, calculate (i) Price
Index (ii) Quantity Index by Paasche’s method.

346 PAGE
© Department of Distance & Continuing Education, Campus of Open Learning,
School of Open Learning, University of Delhi
BUSINESS STATISTICS

Base year Current year Notes


Items q0 p0 q1 p1 p 0q 0 p 1q 0 p 0q 1 p 1q 1
A 6 40 7 30 240 180 280 210
B 4 45 5 50 180 200 225 250
C 5 90 1.5 40 45 20 135 60
Total 465 400 640 520
Σp1q1 520
Price Index P01 = ×100 = ×100 = 81.5
Σp0 q1 640

Σq1 p1 520
Quantity Index Q01 = ×100 = ×100 = 130
Σq0 p1 400

(iii) Fisher’s Ideal Index : Laspeyre has used base year quantities as
weights whereas Paasche’s has used current year quantities as weights
for the computation of Index Number of prices. Fisher suggested
that both the current year quantities and the base year quantities
should be used but geometric mean of the two be calculated and
that figure should be the Index Number. Symbolically,
Fisher’s Price Index P01 =
Fisher’s Index =
On the other hand if quantity Indices by this method are to be
calculated the geometric mean of the Index Number of quantities
with base year prices as weights and Index Number of Quantities
with current year as weights be found out. Symbolically,
Fisher’s Quantity Index
Example 5 : Construct Index Number of Prices and Quantities from the
following data using Fisher’s method (2010 = 100).
2010 2014
Commodity Price Qty. Price Qty.
A 2 8 4 6
B 5 10 6 5
C 4 14 5 10
D 2 19 2 13

PAGE 347
© Department of Distance & Continuing Education, Campus of Open Learning,
School of Open Learning, University of Delhi
B.COM. (PROGRAMME)

Notes Solution : Calculation of Price and Production Indices.


2010 2014
Items price Qty. price(p1) Qty. p 0q 0 p 1q 1 p 1q 0 p 0q 1
(p0) (q0) (q1)
A 2 8 4 6 16 24 32 12
B 5 10 6 5 50 30 60 25
C 4 14 5 10 56 50 70 40
D 2 19 2 13 38 26 38 26
Total 160 130 200 103
(iv) Marshall & Edgeworth’s Method: In this method also both current
year as well as base year prices and quantities are considered. The
formula is as follows:
and Quantity Index is calculated by the formula
(v) Kelly’s Method: Truman Kelly has suggested the following formula
for constructing Index Number.
where q refers to the average quantity of two periods. This is also
known as fixed aggregative method.
(vi) Dorbish & Bowley’s Method: Dorbish & Bowley have suggested
the simple arithmetic mean of Lespeyre’s and Paasche’s formula.
Symbolically.
(II) Weighted Average of Price Relatives : This method is also known
as the Family Budget Method. Weights are values (p0q0) of the
base year in this method. The Index Number for the current year is
calculated by dividing the sum of the products of the current year’s
price relatives and base year values by the total of the weights,
i.e., the weighted arithmetic average of the price relatives gives
the required index numbers. Symbolically,
ΣIV
Weighted Index number of the current year =
ΣV
where I stands for Price Relatives of the current year and V stands
for the values of the base year.
Example 6 : From the data given below, calculate the Weighted Index
Number by using weighted average of Relatives.

348 PAGE
© Department of Distance & Continuing Education, Campus of Open Learning,
School of Open Learning, University of Delhi
BUSINESS STATISTICS

Commodities Units Base Yr. Base Year’s Current Yr. Notes


Qty. Price Price
A Quintal 7 16 19.6
B Kg. 6 2 3.2
C Dozen 16 5.6 7.0
D Metre 21 1.5 1.4
Solution :
Current Year's Price
The Price relative of the current year = ×100
Base Year's Price

Value of a weights = Quantity of base year × Price of the base year


Commodities Price Relatives Value of Weights ×
⎛p ⎞ Weights Price Relatives
I = ⎜ 1 ×100 ⎟ i.e. V = p0q0 V × I
⎝ p0 ⎠
A 122.5 112.0 13,720
B 160.0 12.0 1,920
C 125 89.6 11,200
D 93.3 31.5 2,939
6V = 245.1 6,V = 29,779

ΣIV 29779
Weighted Index Number of the Current year = = = 121.5
ΣV 245
In weighted average of relatives, the Geometric mean may be used
instead of arithmetic mean. The weighted geometric mean of relatives
is calculated by applying logarithms to the relatives. When this mean is
used, then formula is:

P01 = Antilog {
ΣV .log I
ΣV } p1
where I = p ×100 and V = p0q0
0

Example 7 : Find out price index by weighted average of price relatives


from the following commodities using geometric mean :
Commodities p0 q0 p1
X 3.0 20 4.0
Y 1.5 40 1.6
Z 1.0 10 1.5

PAGE 349
© Department of Distance & Continuing Education, Campus of Open Learning,
School of Open Learning, University of Delhi
B.COM. (PROGRAMME)

Notes Solution :
Calculation of Index Number

⎛p ⎞
(p0q0) ⎜ p1 ×100 ⎟
⎝ 0 ⎠
Com- p0 q0 p1 V I Log I V. log I
modities
X 3.0 20 4.0 60 133.33 2.1249 127.494
Y 1.5 40 1.6 60 106.7 2.0282 121.692
Z 1.0 10 1.5 10 150.0 2.1761 21.761
6V = 130 6V log I = 270.947
By applying the formula:

P01 = AL { ΣV .log I
ΣV } ⎛ 270.947 ⎞
= AL ⎜
⎝ 130 ⎠
⎟ = AL2.084 = 121.3

1.6 Tests of Adequacy or Consistency


Since several formulas have been suggested for the construction of index
numbers, then the question arises which method of index number is the
most suitable in a given situation. These are some tests to choose an
appropriate index:
(i) Unit Test: It requires that the method of constructing index should
be independent of the units of the problem. All the methods except
simple aggregative method satisfy this test.
(ii) Circular Test: It is based on the shiftability of the base. Accordingly,
the index should work in a circular fashion i.e., if an index number
is computed for the period 1 on the base period 0, another index
is computed for period 2 on the base period 1, and still another
index number is computed for period 3 on the base period 2. Then
the product should be equal to one.
P01 × P12 × P23 ..........× Pn0 = 1
Only simple aggregative and fixed weight aggregative method satisfy
the test.

350 PAGE
© Department of Distance & Continuing Education, Campus of Open Learning,
School of Open Learning, University of Delhi
BUSINESS STATISTICS

If the test is applied to simple aggregative method, we will get Notes


Σp1 Σp2 Σp3
× × =1
Σp0 Σp1 Σp2

The test is met by simple geometric mean of price relatives and


the weighted aggregative of fixed weights.
(iii) Time Reversal Test: According to Prof. Fisher the formula for
calculating an index number should be such that it gives the same
ratio between one point of time and the other, no matter which of
the two time is taken as the base. In other words, when the data
for any two years are treated by the same method, but with the
base reversed, the two index numbers should be reciprocals of each
other.
P01 × P10 = 1 (omitting the factor 100 from each index).
where P01 denotes the index for current year 1 based on the base
year 0 and P10 is for current year 0 on the base year 1.
It can be easily verified that simple geometric mean of price relatives
index, weighted aggregative formula, weighted geometric mean of
relatives and Marshall Edgeworth and Fisher’s ideal method satisfies
the test.
Let us see how Fisher’s ideal method satisfies the test.
Σp1q0 Σp1q1
P01 = ×
Σp0 q0 Σp0 q1

By changing time from 0 to 1 and 1 to 0


Σp0 q1 Σp0 q0
P01 = ×
Σp1q1 Σp1q0

Now P01 × P10 = 1


Substitute the value of P01 and P10
Σp1q0 Σp1q1 Σp0 q1 Σp0 q0
P01 × P10 = × × × =1
Σp0 q0 Σp0 q1 Σp1q1 Σp1q0

PAGE 351
© Department of Distance & Continuing Education, Campus of Open Learning,
School of Open Learning, University of Delhi
B.COM. (PROGRAMME)

Notes (iv) Factor Reversal Test: It says that the product of a price index and
the quantity index should be equal to value index. In the words of
Fisher, just as each formula should permit the interchange of the two
times without giving inconsistent results similarly it should permit
interchanging the prices and quantities without giving inconsistent
results which means two results multiplied together should give the
true value ratio. The test says that the change in price multiplied
by change in quantity should be equal to total change in value. If
P01 is a price index for the current year with reference to base year
and Q01 is the quantity index for the current year,
Σp1q1
then P01 × Q01 =
Σp0 q0

This test is satisfied only by Fisher’s ideal index method.


Σp1q0 Σp1q1
P01 = ×
Σp0 q0 Σp0 q1

Changing p to q and q to p.
Σq1 p0 Σq1 p1
Q01 = ×
Σq0 p0 Σq0 p1

Σp1q0 Σp1q1 Σq1 p0 Σq1 p1 (Σq1 p1 ) 2 (Σp1q1 )


? P01 × Q01 = × × × = =
Σp0 q0 Σp0 q1 Σq0 p0 Σq0 p1 (Σp0 q0 ) 2 (Σp0 q0 )

In other words, factor reversal test is based on the following analogy. If


the price per unit of a commodity increases from Rs. 10 in 1995 to Rs.
15 in 1998, and the quantity of consumption changes from 100 units to
140 units during the same period, then the price and quantity in 1998
are 15 and 140 respectively. The values of consumption (p × q) were Rs.
1000 in 1995 and Rs. 2100 in 1998 giving a value ratio.
Σp1q1 2100
= = 2.1
Σp0 q0 1000

Thus we find that the product of price ratio and quantity ratio equals
the value ratio:
1.5 × 1.4 = 2.1

352 PAGE
© Department of Distance & Continuing Education, Campus of Open Learning,
School of Open Learning, University of Delhi
BUSINESS STATISTICS

1.7 Chain Base Index Notes

The various formulas discussed so far assume that base period is some
fixed previous period. The index of a given year on a given fixed base is
not affected by changes in the prices or the quantities of any other year.
On the other hand, in the chain base method, the value of each period
is related with that of the immediately proceeding period and not with
any fixed period. To construct index numbers by chain base method, a
series of index numbers are computed for each year with preceding year
as the base. These index numbers are known as Link relatives. The link
relatives when multiplied successively known as the chaining process
give link to a common base. The products obtained are expressed as %
and give the required index number. The steps of chain base index are:
(i) Express the figures of each period as a % of the preceding period
to obtain Link Relatives (LR)
(ii) These link relatives are chained together by successive multiplication
to get chain indices by the formula:
Current year LR × Preceding year Chain Index
Chain Base Index (CBI) =
100
(iii) The chain index can be converted into a fixed base index by this
formula:
Current year CBI × Previous year FBI
Fixed Base Index (FBI) =
100
Chain relatives are computed from link relatives whereas fixed base
relatives are computed directly from the original data. The results obtained
by fixed base and chain base index invariably are the same.
We shall understand the process by taking some examples.
Example 8 : Construct Index Numbers by chain base method from the
following data of wholesale prices.
Year : 2005 2006 2007 2008 2009 2010 2011 2012 2013 2014
Prices : 75 50 65 60 72 70 69 75 84 80

PAGE 353
© Department of Distance & Continuing Education, Campus of Open Learning,
School of Open Learning, University of Delhi
B.COM. (PROGRAMME)

Notes Solution :
Computation of Chain Index
Year Price Link Relatives Chain Base Index Fixed Base Index
2005 75 100 100 100
50 66.67 ×100 50
2006 50 ×100 = 66.67 = 66.67 ×100 = 66.67
75 100 75

65 130 × 66.67 65
2007 65 ×100 = 130 = 86.67 ×100 = 86.67
50 100 75

60 92.31× 86.67 60
2008 60 ×100 = 92.31 = 80.00 ×100 = 80
65 100 75

72 120 × 80 72
2009 72 ×100 = 120 = 96.00 ×100 = 96
60 100 75

70 97.22 × 96 70
2010 70 ×100 = 97.22 = 93.33 ×100 = 93.33
72 100 75

69 98.57 × 93.33 69
2011 69 ×100 = 98.57 = 92.00 ×100 = 92
70 100 75

75 108.69 × 92 75
2012 75 ×100 = 108.69 = 100.00 ×100 = 100
69 100 75

84 112 ×100 84
2013 84 ×100 = 112 = 112.00 ×100 = 112
75 100 75

80 95.24 ×112 80
2014 80 ×100 = 95.24 = 106.67 ×100 = 106.67
84 100 75

It may be seen that index by chain base and fixed base method comes
to the same.
Example 9 : Construct chain index numbers from the link relatives given
below:
Year : 2011 2012 2013 2014 2015
Link Relatives : 100 105 95 115 102

354 PAGE
© Department of Distance & Continuing Education, Campus of Open Learning,
School of Open Learning, University of Delhi
BUSINESS STATISTICS

Solution : Notes
Calculations for Chain Base Index
Year Link Relatives Chain Index Number
2011 100 100

2012 105 105


×100 = 105.00
100

2013 95 95
×105 = 99.75
100

2014 115 115


× 99.75 = 114.7
100

2015 102 102


×114.75 = 117
100
Base Shifting: Sometimes it becomes necessary to change the base of index
number series from one period to another for the purpose of comparison.
In such circumstances it is necessary to recompute all index numbers
using new base period. Such computation of index numbers using new
base period is to divide index number in each period by the index number
corresponding to the new base period and then to express the result as
percentages. This process is known as the Base shifting.
Example 10 : Compute Index Numbers from the following taking 2012
as the base and shift the base to 2014.
Solution :
Year Price Index Number Shift of base from
Base 2012 2012 to 2014

100
2012 10 100 ×100 = 67
150

12 120
2013 12 ×100 = 120 ×100 = 80
10 150

PAGE 355
© Department of Distance & Continuing Education, Campus of Open Learning,
School of Open Learning, University of Delhi
B.COM. (PROGRAMME)

Notes Year Price Index Number Shift of base from


Base 2012 2012 to 2014
15
2014 15 ×100 = 150 100
10

21 210
2015 21 ×100 = 210 ×100 = 140
10 150

20 200
2016 20 ×100 = 200 ×100 = 133
10 150

1.8 Splicing
On several occasions the base year may give discontinuity in the
construction of index numbers. We would always like to compare figures
with a recent year and not with distant past. For example, the weights of
an index number may become out of data and we may construct another
index with new weights. Two indices would appear. It becomes necessary
to convert these two indices into a continuous series. The procedure
employed to do the conversion is known as splicing. The formulae are:
For Forward Splicing:
Old index of the New Base Year × Index to be adjusted
Spliced Index Number:
100
For Backward Splicing:
100
Spliced Index Number: × Index to be adjusted
Old index of the New Base Year
Example 11 : Splice the following two Index number series, A series
forward and B series backward:
Year : 2010 2011 2012 2013 2014 2015
Series A : 100 120 150 — — —
Series B : — — 100 110 120 150

356 PAGE
© Department of Distance & Continuing Education, Campus of Open Learning,
School of Open Learning, University of Delhi
BUSINESS STATISTICS

Solution : Notes
Splicing of two Index Number Series
Year Series Series Index Number Spliced Index Numbers Spliced
A B forward to Series A backward to Series B
100
2010 100 ×100 = 66.66
150

100
2011 120 ×120 = 80.00
150

150 100
2012 150 100 ×100 = 150 ×150 = 100.00
100 150

150
2013 110 ×110 = 165
100

150
2014 120 ×120 = 180
100

150
2015 150 ×150 = 225
100

Deflating: It means making allowance for the changes in the purchasing


power of money due to a change in general price level. It is the technique
of converting a series of value calculated at current prices in to a series
at constant prices of a given year. In other words the process of removing
the effects of price changes from the current money values is called
Deflation. By this process the real value of the phenomenon is calculated
which is free from the influence of price changes. Deflation is used in
computation of national income and other economic variables. The relevant
price index is called the deflator whether it is to be the wholesale price
index or consumer price index. Normally separate price deflators are
found out for deflating the national income data from different sectors
of the economy considering the changes in prices in those sectors. The
method is:
Current value
Deflated value = ×100
Deflator

PAGE 357
© Department of Distance & Continuing Education, Campus of Open Learning,
School of Open Learning, University of Delhi
B.COM. (PROGRAMME)

Notes 1.9 Consumer Price Index


The consumer price index known as cost of living index is calculated
to know the average change over time in the prices of commodities
consumed by the consumers. The need to construct consumer price indices
arises because the general index numbers fail to give an exact idea of
the effect of the change in the general price level on the cost of living
of different classes of people, because a given change in the level of
prices affect different classes of people in different manners. Different
people consume different commodities and if same commodities then in
different proportions. The consumer price index helps us in determining
the effect of rise and fall in prices on different classes of consumers
living in different area. The consumer price index is significant because
the demand for higher wages is based on the cost of living index and the
wages and salaries in most nations are adjusted according to this index.
We should understand that the cost of living index does not measure
the actual cost of living nor the fluctuations in the cost of living due to
causes other than the change in price level but its object is to find out
how much the consumers of a particular class have to pay more for a
certain basket of goods and services. That is why the term cost of living
index has been replaced by the term price of living index, cost of living
price index or consumer price index.
The significance of studying the consumer price index is that it helps in
wage negotiations and wage contracts. It also helps in preparing wage
policy, price policy, rent control, taxation and general economic policies.
This index is also used to find out the changing purchasing power of
different currencies.
Consumer Price Index can be prepared by two methods:
(i) Aggregative Method;
(ii) Weighted Relatives Method.
When, aggregative method is used to prepare consumer price index, the
aggregative expenditure for current year and base year are calculated and
the below given formula is applied.
Σp 1q0
Consumer Price Index = ×100
Σp0 q0

358 PAGE
© Department of Distance & Continuing Education, Campus of Open Learning,
School of Open Learning, University of Delhi
BUSINESS STATISTICS

When weighted relatives method is used then the family budgets of a Notes
large number of people for whom the index is meant are carefully studied
and the aggregative expenditure of an average family on various items
is estimated. These will be weights. In other words, the weights are
calculated by multiplying the base year quantities and prices (p0q0). The
price relatives for all the commodities are prepared and multiplied by the
weights. By applying the formula, we can calculate Consumer price index.
ΣIV p
Consumer Price Index = where I = 1 ×100 and V = p0q0
ΣV p0
Example 12 : Prepare the Consumer price index for 2013 on the basis
of 2010 from the following data by both methods.
Commodities Quantities Consumed Prices Prices
2010 2010 2013
A 6 5.75 6.00
B 6 5.00 8.00
C 1 6.00 9.00
D 6 8.00 10.00
E 4 2.00 1.50
F 1 20.00 15.00
Solution :
Consumer Price Index by Aggregative Method
Commodities q0 p0 p1 p 1q 0 p 0q 0
A 6 5.75 6.00 36.00 34.50
B 6 5.00 8.00 48.00 30.00
C 1 6.00 9.00 9.00 6.00
D 6 8.00 10.00 60.00 48.00
E 4 2.00 1.50 6.00 8.00
F 1 20.00 15.00 15.00 20.00
6p1q0 = 174 6p0q0 = 146.5

Σp1q0 174
Consumer Price Index = ×100 = ×100 = 118.77
Σp0 q0 146.5

PAGE 359
© Department of Distance & Continuing Education, Campus of Open Learning,
School of Open Learning, University of Delhi
B.COM. (PROGRAMME)

Notes Consumer Price Index by Weighted Relatives


Commodities q0 p0 p1 I V IV
A 6 5.75 6.00 104.34 34.50 3600
B 6 5.00 8.00 160.00 30.00 4800
C 1 6.00 9.00 150.00 6.00 900
D 6 8.00 10.00 125.00 48.00 6000
E 4 2.00 1.50 75.00 8.00 600
F 1 20.00 15.00 75.00 20.00 1500
6V = 146.5 6IV = 17400

ΣIV 17400
Consumer Price Index = = = 118.77
ΣV 146.5

1.10 Index Number of Industrial Production


The Index Number of industrial production is prepared to know the
increase or decrease in the level of industrial production in a given period
compared with some other period. This index measures the changes in
quantum of production. To prepare such an index it is necessary for us
to compute the production for two periods i.e. for the current year and
for the base year. Generally the data are collected under these heads:
(i) Textile industries – cotton, woollen, silk etc.
(ii) Mining industries – iron-ore, coal, copper, petroleum etc.
(iii) Metallurgical industries – iron-ore, coal, copper, petroleum etc.
(iv) Mechanical industries – locomotives, ships, aeroplane etc.
(v) Miscellaneous – glass, soap, chemical, cement etc.
The output for various industries are computed. Weights are assigned
to various industries on the basis of some criteria as capital invested,
turnover, net output, production etc. We apply this formula:

ΣIW
Index of Industrial Production =
ΣW

q1
where I = and W = Relative importance of different outputs
q0

360 PAGE
© Department of Distance & Continuing Education, Campus of Open Learning,
School of Open Learning, University of Delhi
BUSINESS STATISTICS

1.11 Limitations of Index Numbers Notes

1. They are only approximate indicators of the relatives level of a


phenomenon.
2. Index number are good for achieving one objective may be unsuitable
for the other.
3. Index numbers can be manipulated in a manner as to draw the desired
conclusion.

1.12 Construction of BSE Sensex and NSE Nifty


Bombay Stock Exchange
The first organised stock exchange was established in July 1875 as an
association of native brokers, named as Native Shares and Stock Brokers
Association. Its formal deed of association was executed in 1887. This
stock exchange is now popularly known as the Bombay Stock Exchange
(BSE). This stock exchange played a significant role during the phase of
recovery from several years of depression. It was the first to be recognised
by the Government of India.
The Exchange, while providing an efficient and transparent market for
trading in securities, debt and derivatives upholds the interests of the
investors and ensures redressal of their grievances whether against the
companies or its own member-brokers. It also strives to educate and
enlighten the investors by conducting investor education programmes and
making available to them necessary informative inputs.
SENSEX – The Barometer of Indian Capital Markets
Bombay Stock Exchange (BSE) Sensitive Index Number of equity prices
(SENSEX) is the most widely used and accepted equity price index in the
country. SENSEX, first compiled in 1986, was calculated on a “Market
Capitalization-Weighted” methodology of 30 component stocks representing
large, well-established and financially sound companies across key sectors.
The base year of SENSEX was taken as 1978-79. SENSEX today is
widely reported in both domestic and international markets through print
as well as electronic media. It is scientifically designed and is based on
globally accepted construction and review methodology. Since September
1, 2003, SENSEX is being calculated on a free-float market capitalization

PAGE 361
© Department of Distance & Continuing Education, Campus of Open Learning,
School of Open Learning, University of Delhi
B.COM. (PROGRAMME)

Notes methodology. The “free-float market capitalization-weighted” methodology


is a widely followed index construction methodology on which majority
of global equity indices are based; all major index providers like MSCI,
FTSE, STOXX, S&P and Dow Jones use the free-float methodology.
Index Specification
Base Year 1978-79
Base Index Value 100
Date of Launch 01-01-1986
Method of calculation Launched on full market capitalization
method and effective September 01, 2003,
calculation method shifted to free-float
market capitalization.
Number of scrips 30
Index calculation frequency Real Time
Historical Value of Index Index, Price Earnings, Price to Book Value
Ratio and Dividend Yield %
SENSEX Calculation Methodology
SENSEX is calculated using the “Free-float Market Capitalization”
methodology, wherein, the level of index at any point of time reflects the
free-float market value of 30 component stocks relative to a base period.
The market capitalization of a company is determined by multiplying
the price of its stock by the number of shares issued by the company.
This market capitalization is further multiplied by the free-float factor
to determine the free-float market capitalization.
The base period of SENSEX is 1978-79 and the base value is 100 index
points. This is often indicated by the notation 1978-79 = 100. The calculation
of SENSEX involves dividing the free-float market capitalization of 30
companies in the Index by a number called the Index Divisor. The Divisor
is the only link to the original base period value of the SENSEX. It keeps
the Index comparable over time and is the adjustment point for all Index
adjustment arising out of corporate actions, replacement of scrips, etc.
During market hours, prices of the index scrips, at which latest trades
are executed, are used by the trading system to calculate SENSEX on a
continuous basis.

362 PAGE
© Department of Distance & Continuing Education, Campus of Open Learning,
School of Open Learning, University of Delhi
BUSINESS STATISTICS

Calculation of BSE SENSEX Notes


Sensex is calculated using a “Market Capitalisation-Weighted” methodology.
As per this methodology, the level of index at any point of time reflects
the total market value of 30 component stocks relative to a base period.
(The market capitalisation of a company is determined by multiplying
the price of its stock by the number of shares issued by the company).
Statisticians call an index of a set of combined variables (such as price
and number of shares) a composite index. A single indexed number is
used to represent the results of this calculation in order to make the value
easier to work with and track over time. It is much easier to graph a
chart based on indexed values than one based on actual values.
The base period of Sensex is 1978-79. The actual total market value of
the stocks in the index during the base period has been set equal to an
indexed value of 100. This is often indicated by the notation 1978-79 =
100. The formula used to calculate the index is fairly straightforward.
However, the calculation of the adjustments to the index (commonly
called Index maintenance) is more complex.
The calculation of Sensex involves dividing the total market capitalisation
of 30 companies in the index by a number called the Index Divisor. The
Divisor is the only link to the original base period value of the Sensex.
It keeps the index comparable over time and is the adjustment point for
all index maintenance adjustments. During market hours, prices of the
index scrips, at which latest trades are executed, are used by the trading
system to calculate Sensex every 15 seconds and disseminated, all over
the country through BOLT terminals in real time.
Calculation of Closing SENSEX
The closing Sensex is computed taking the weighted average of all the
trades on Sensex constituents in the last 15 minutes of trading session. If
a Sensex constituent has not traded in the last 15 minutes, the last traded
price is taken for computation of the index closure. If a Sensex constituent
has not traded at all in a day, then its last day’s closing price is taken
for computation of index closure. The use of Index Closure Algorithm
prevents any intentional manipulation of the closing index value.

PAGE 363
© Department of Distance & Continuing Education, Campus of Open Learning,
School of Open Learning, University of Delhi
B.COM. (PROGRAMME)

Notes BSE Sensex as on January 7, 2012


Scrip Company Close No. of shares Full mkt. Free- Free-float Weight,
code price (normal) cap. (Rs. float mkt. cap. (Rs. in in-
crore) adj. crore) dex (%)
factor
500209 Infosys Ltd. 2,837.50 574,203,082 162,930.12 0.85 138,490.61 10.66
500325 Reliance 714.70 3,274,452,139 234,025.09 0.55 128,713.80 9.91
500875 ITC Ltd. 201.60 7,789,453,850 157,035.39 0.70 109,924.77 8.46
500010 Housing 668.05 1,470,391,801 98,229.52 0.95 93,318.05 7.18
Deve.
532174 ICICI Bank 745.45 1,152,540,454 85,916.13 1.00 85,916.13 6.61
Ltd.
500180 HDFC Bank 451.10 2,337,610,305 105,449.60 0.80 84,359.68 6.49
Ltd.
532540 TCS Ltd. 1,171.35 1,957,220,996 229,259.08 0.30 68,777.72 5.29
500510 Larsen & 1,081.45 611,844,627 66,167.94 0.90 59,551.14 4.58
Toubro
500112 State Bank 1,669.75 634,999,595 106,029.06 0.45 47,713.08 3.67
of India
532454 Bharti Airtel 330.90 3,797,530,096 125,660.27 0.35 43,981.09 3.49
500312 ONG Corp. 256.65 8,555,490,120 219,576.65 0.20 43,915.33 3.38
Ltd.
500696 Hind Uni 396.00 2,160,391,918 85,551.52 0.50 42,775.76 3.29
Ltd.
500570 Tata Motors 203.55 2,691,486,150 54,785.20 0.65 35,610.38 2.74
500520 Mahindra 654.25 613,974,839 40,169.30 0.75 30,126.98 2.32
& M
532555 NTPC Ltd. 157.15 8,245,464,400 129,577.47 0.20 25,915.49 2.00
507685 Wipro Ltd. 406.55 2,457,821,578 99,922.74 0.25 24,980.68 1.92
500470 Tata Steel 362.80 959,214,779 34,800.31 0.70 24,360.22 1.88
500103 BHEL 250.65 2,447,600,000 61,349.09 0.35 21,472.18 1.65
532977 Bajaj Auto 1,448.05 289,367,020 41,901.79 0.50 20,950.90 1.61
524715 Sun Phar- 500.40 1,035,550,385 51,818.94 0.40 20,727.58 1.60
mace
533278 Coal India 319.80 6,316,364,400 201,997.33 0.10 20,199.73 1.56
532286 Jindal Steel 466.05 934,509,595 43,552.82 0.45 19,598.77 1.51
500087 CIPLA Ltd. 335.80 802,921,357 26,962.10 0.65 17,525,36 1.35
500182 Hero Moto 1,729.80 199,687,500 34,541.94 0.50 17,270.97 1.33
Co.
500440 Hindalco In 117.95 1,918,596,448 22,629.85 0.70 15,840.89 1.22
500400 Tata Power 91.95 2,373,072,360 21,820.40 0.70 15,274.28 1.18
500900 Sterlite In 94.35 3,360,700,478 31,708.21 0.45 14,268.69 1.10

364 PAGE
© Department of Distance & Continuing Education, Campus of Open Learning,
School of Open Learning, University of Delhi
BUSINESS STATISTICS

Scrip Company Close No. of shares Full mkt. Free- Free-float Weight, Notes
code price (normal) cap. (Rs. float mkt. cap. (Rs. in in-
crore) adj. crore) dex (%)
factor
532500 Maruti Su- 954.75 288,910,060 27,583.69 0.50 13,791.84 1.06
zuki
532868 DLF Limited 176.60 1,698,157,659 29,989.46 0.25 7,497.37 0.58
532532 Jaiprak Asso 51.90 2,126,433,182 11,036.19 0.55 6,069,90 0.47
Total 2,641,977.20 1,298,919.37

Sector-wise Market Capitalisation of SENSEX as on January 7, 2012


Sl. No. SENSEX / Sectors Free float market capitalization
(Rs. crore) %
SENSEX 1,427,501.46 100.00
1. Finance 311,306.94 23.97
2. Information Technology 232,249.01 17.88
3. Oil & Gas 172,629.13 13.29
4. FMCG 152,700.53 11.76
5. Transport Equipments 117,751.07 9.07
6. Metal, Metal Product & 94,268.30 7.26
Mining
7. Capital Goods 81,023.32 6.24
8. Telecom 43,981.09 3.39
9. Power 41,189.77 3.17
10. Healthcare 38,252.94 2.94
11. Housing Related 13,567.27 1.04
Free-float Methodology for Calculating Sensex
From 1st September 2003, the country’s equity benchmark Sensex is being
calculated based on the Free-float methodology. Prior to 1-9-2003, the
Sensex was calculated based on the full market capitalisation methodology.
Globally, the Free-float methodology of index construction is considered
to be an industry best practice and all major index providers like MSCI,
FTSE, S&P and STOXX have adopted the same. The MSCI India Standard
Index is also based on the Free-float methodology.
In India, BSE pioneered the concept of Free-float with the launch of
the country’s first Free-float based index – BSE TECk in July 2001 and
BANKEX in June 2003. The shifting of Sensex to this methodology is a

PAGE 365
© Department of Distance & Continuing Education, Campus of Open Learning,
School of Open Learning, University of Delhi
B.COM. (PROGRAMME)

Notes culmination of successful experiences with these two indices and a series
of debates and discussions in the last few years.
The new methodology would align the Sensex with the best global practice
in index construction. A smooth transition from full market capitalisation
to Free-float market capitalisation methodology would ensure that the
basic characteristics of Sensex are retained. Importantly, the Free-float
methodology will further improve the benchmarking qualities of Sensex
while maintaining its historical continuity.
The following Free-float factors will be applied to the Sensex companies.
A Free-float factor of say 0.9 means that only 90% of the total market
capitalisation of that company would be taken into consideration for
index calculation.
Free-float Index
Currently all equity indices in India, except the BSE-TECk Index and
BANKEX, are calculated using the ‘full-market capitalisation’ methodology.
Under the ‘full-market capitalisation’ methodology, the total market
capitalisation of a company, irrespective of who is holding the shares,
is taken into consideration for computation of an index. However, if
instead of taking the total market capitalisation, only the Free-float market
capitalisation of a company is considered for index calculation, it is called
the Free-float methodology. Free-float market capitalisation is defined as
that proportion of total shares issued by the company, which are readily
available for trading in the market. It generally excludes promoters’ holding,
government holding, strategic holding and other locked-in shares, which
will not come to the market for trading in the normal course. Thus, the
market capitalisation of each company in a Free-float index is reduced
to the extent of its Free-float available in the market.
National Stock Exchange (NSE)
In order to provide a nationwide stock trading facility to investors and
to bring the Indian financial market in line with international market, the
National Stock Exchange (NSE) was set up and it started its operations
by the end of 1993. Further, it started trading in debt instruments in
May, 1994 and in equity shares by the end of November 1994. The NSE
uses the electronic trading system and computerised settlement system.
This system is so designed that it can be extended to every corner of

366 PAGE
© Department of Distance & Continuing Education, Campus of Open Learning,
School of Open Learning, University of Delhi
BUSINESS STATISTICS

the country through the medium of electronic network. It was recently Notes
accorded recognition as a stock exchange by the Department of Company
Affairs. The instruments traded are treasury bills, government security
and bonds issued by public sector companies.
The exchange has two separate segments, i.e., capital market segment and
money market segment. The former is concerned with trading in equity
shares, convertible debentures and debt instruments as non-convertible
debentures. In the money market segment, also known as wholesale debt
market segment, facilitates trading in debts, public sector bonds, mutual
fund units, treasury bills, government securities, call money instruments,
etc. The transactions in this segment are of high values. The main
participants, in this market are usually banks, financial institutions and
other financial agencies.
NSE-50, NIFTY
The NSE-50 index, NIFTY was launched by the National Stock Exchange
of India Limited (NSE) in April 1996, taking as base the closing prices
of November prices of November 3, 1995 when one year of operations
of its capital market segment was completed. According to the NSE, the
index was introduced with the objectives of:
1. reflecting market movement more accurately,
2. providing fund managers with a tool for measuring portfolio returns
vis-a-vis market returns, and
3. providing a basis for introducing index-based derivatives.
The index is based on the prices of shares of 50 companies (chosen
from among the companies traded on the NSE), each with a market
capitalisation of at least Rs. 500 crores and having a high degree of
liquidity. The methodology used for the computation of this index is
‘market capitalisation weightage’ as followed by the S&P-500. The base
value of the index has been set at 1000, and not the usual 100.
S&P CNX NIFTY
The S&P CNX Nifty is the headline index on the National Stock Exchange
of India Ltd. (NSE). It includes 50 of the approximately 1,300 companies
listed on the NSE, captures approximately 60% of its equity market
capitalization and is a true reflection of the Indian stock market.

PAGE 367
© Department of Distance & Continuing Education, Campus of Open Learning,
School of Open Learning, University of Delhi
B.COM. (PROGRAMME)

Notes S&P CNX Nifty tracks the behaviour of a portfolio of blue chip companies,
the largest and most liquid Indian securities. It covers 25 sectors of the
Indian economy and offers investment managers exposure to the Indian
market in one efficient portfolio. The index has been trading since April
of 1996 and is well suited for benchmarking, index funds, and index-
based derivatives.
The S&P CNX Nifty index is owned and managed by the Indian Index
Services and Products Ltd. (IISL), with which Standard and Poor’s has a
consulting and licensing agreement. IISL is a joint venture between NSE
and CRISIL (formerly Credit Rating Information Services of India Ltd.).
Index Methodology
S&P CNX Nifty is maintained by IISL’s Index Policy Committee, which
manages policy and guidelines for all CNX (CRISIL/NSE) indices. This
Index Policy Committee follows a clear published set of rules for index
revision and meets quarterly to consider their application. Additionally, the
IISL’s Index Maintenance Sub-Committee reviews decisions about additions
and deletions to the index on a quarterly basis. Complete details of these
rules are available on the website at www.indices.standardandpoors.com.
NIFTY COMPOSITION
Sl. Scrip Equity Free Weigh- Beta R2 Volatility Monthly Impact
No. Capital Float tage% Returns Cost
(1) (2) (3) (4) (5) (6) (7) (8) (9) (10)
1. ACC 1,877,452,660 9,629 0.59 0.72 0.48 1.23 6.43 0.07
2. Ambuja 3,063,349,822 10,652 0.65 0.97 0.48 1.61 –3.22 0.08
Cement
3. Axis 4,116,997,330 34,573 2.10 1.36 0.74 1.86 3.72 0.07
Bank
4. Bajaj 2,893,670,200 19,722 1.20 0.75 0.49 1.07 4.44 0.06
Auto
5. Bharti 18,987,650,480 52,650 3.20 0.76 0.46 2.05 10.72 0.07
Airtel
6. BHEL 4,895,200,000 29,067 1.77 0.86 0.58 1.99 –10.29 0.06
7. BPCL 3,615,421,240 8,500 0.52 0.79 0.45 0.92 1.17 0.07
8. CAIRN 19,022,340,290 11,439 0.70 0.59 0.40 1.79 –1.11 0.07
9. CIPLA 1,605,842,714 15,558 0.95 0.71 0.51 1.26 –7.39 0.07
10. DLF 3,395,150,248 8,380 0.51 1.42 0.66 2.59 9.76 0.07
11. DR 846,959,090 19,974 1.22 0.59 0.46 1.55 3.53 0.08
Reddy

368 PAGE
© Department of Distance & Continuing Education, Campus of Open Learning,
School of Open Learning, University of Delhi
BUSINESS STATISTICS

Sl. Scrip Equity Free Weigh- Beta R2 Volatility Monthly Impact Notes
No. Capital Float tage% Returns Cost
(1) (2) (3) (4) (5) (6) (7) (8) (9) (10)
12. GAIL 12,684,774,000 20,712 1.26 0.62 0.46 1.34 4.51 0.08
13. GRASIM 917,018,380 13,942 0.85 0.67 0.49 1.37 4.78 0.08
14. HCL Tech 1,376,020,536 11,839 0.72 0.95 0.60 1.21 –1.61 0.06
15. HDFC 2,936,038,650 89,641 5.45 1.19 0.71 1.26 –2.45 0.07
16. HDFC 4,667,711,360 87,080 5.30 1.06 0.74 1.06 –3.24 0.07
Bank
17. Hero 399,375,000 17,035 1.04 0.53 0.25 1.20 –4.91 0.06
Honda
18. Hindal 1,914,419,297 21,647 1.32 1.42 0.67 2.44 –6.70 0.06
Co.
19. Hind 2,160,683,560 33,208 2.02 0.58 0.42 0.84 –5.72 0.05
Unilever
20. ICICI 11,518,614,870 119,419 7.27 1.40 0.78 1.47 –5.29 0.06
Bank
21. IDFC 14,627,715,770 15,139 0.92 1.47 0.71 2.45 –3.93 0.07
22. INFY 2,870,938,460 133,825 8.14 0.83 0.62 1.38 –4.62 0.04
23. ITC 7,738,144,280 110,948 6.75 0.75 0.54 1.26 2.66 0.06
24. Jindal 934,509,595 22,847 1.39 0.99 0.67 1.21 –9.87 0.06
Steel
25. JP 4,252,866,364 7,527 0.46 1.78 0.70 1.69 –17.73 0.07
Associate
26. Kotak 3,689,051,845 15,763 0.96 1.20 0.67 2.02 –7.16 0.08
Bank
27. LT 1,220,046,436 92,399 5.62 1.18 0.73 1.68 –5.38 0.06
28. M&M 3,069,874,195 33,234 2.02 1.20 0.65 1.99 2.69 0.07
29. Maruti 1,444,550,300 15,963 0.97 0.82 0.56 1.14 4.03 0.06
30. NTPC 82,454,644,000 22,507 1.37 0.75 0.57 1.23 –5.78 0.07
31. ONGC 42,777,450,600 36,322 2.21 0.71 0.48 0.94 –1.82 0.07
32. PNB 3,168,121,570 14,955 0.91 0.97 0.64 1.73 3.18 0.08
33. Power 46,297,253,530 14,879 0.91 0.53 0.45 0.96 –4.02 0.06
Grid
34. Ranbaxy 2,106,816,150 8,231 0.50 0.89 0.50 1.29 –0.35 0.06
35. RCom 10,320,134,405 6,736 0.41 1.22 0.47 4.16 6.11 0.08
36. Rel 2,456,328,000 6,484 0.39 1.24 0.57 1.54 –0.50 0.06
Capital
37. Reliance 32,738,103,000 139,612 8.50 0.96 0.68 1.60 –7.85 0.06
38. Re Infra 2,653,702,620 7,502 0.46 1.13 0.46 2.08 1.07 0.07
39. R Power 28,051,264,660 6,149 0.37 0.98 0.52 1.19 –3.41 0.07

PAGE 369
© Department of Distance & Continuing Education, Campus of Open Learning,
School of Open Learning, University of Delhi
B.COM. (PROGRAMME)

Notes Sl. Scrip Equity Free Weigh- Beta R2 Volatility Monthly Impact
No. Capital Float tage% Returns Cost
(1) (2) (3) (4) (5) (6) (7) (8) (9) (10)
40. SAIL 41,304,005,450 7,400 0.45 1.06 0.61 1.64 –8.11 0.07
41. SBIN 6,349,989,910 60,447 3.68 1.25 0.69 1.35 –2.49 0.04
42. SESAGOA 869,101,423 10,727 0.65 1.05 0.51 1.95 –2.48 0.06
43. Siemens 680,589,800 7,881 0.48 0.52 0.33 1.17 3.86 0.07
44. Ster 3,361,568,684 22,704 1.38 1.21 0.61 1.58 –5.05 0.06
45. Sun 1,035,581,955 19,472 1.18 0.68 0.45 1.17 4.08 0.07
Pharma
46. Tata 5,382,725,080 33,259 2.02 1.27 0.61 1.93 –4.64 0.05
Motors
47. Tata 2,373,072,360 20,733 1.26 0.57 0.46 1.30 –2.07 0.07
Power
48. Tata Steel 9,592,144,500 37,547 2.28 1.15 0.70 1.14 –7.58 0.05
49. TCS 1,957,220,996 57,746 3.51 0.95 0.61 1.15 –3.99 0.06
50. WIPRO 4,911,272,378 19,741 1.20 0.83 0.56 1.37 –6.78 0.06

Limitation of Index Number


So far we have studied various types of index numbers. However, they
have certain limitations.
They are:
1. As the indices are constructed mostly from deliberate samples,
chances of errors creepingin cannot be always avoided.
2. Since index numbers are based on some selected items, they simply
depict the broad trend and not the real picture.
3. Since many methods are employed for constructing index numbers,
the result gives different values and this at times create confusion.

1.13 Summary
An index number measures relative changes in the value of some economic
variable/s over a period of time. It is always expressed in terms of a
base of usually 100. Index numbers showing changes in the values of one
variable over time are called univariate while those showing changes in a
group of variables are known as composite index numbers. The base of
an index may either be fixed or chained. In fixed base index numbers,

370 PAGE
© Department of Distance & Continuing Education, Campus of Open Learning,
School of Open Learning, University of Delhi
BUSINESS STATISTICS

the base period is common while in chain base indices, for every period Notes
its immediately preceding period is taken as the base.
In order to compare two time series of price relatives, it is necessary
that each series should have the same base period.The composite index
numbers may be simple or weighted, and aggregative or average-of-
relatives. Simple aggregative price index shows the aggregate of the
current year prices as a percentage of the aggregate of the base year
prices. For weighted aggregative price indices, the quantities are used
as weights whereas for aggregative quantity indices, the prices are used
as weights.The two basic aggregative price indices are those using base
year quantities as weights, known as Laspeyre’s index and current year
quantities as weights, known as Paasche’s index.
Fisher’s index is equal to the geometric mean of Laspeyre’s and Paasche’s
indexes.There are four tests of adequacy of index numbers: Units test,
time-reversal test, factor-reversal test and circular test. Fisher’s method is
the only one that satisfies the first three of these tests.Value index may
be calculated by taking the ratio of current year value to base year value
expressed as a percentage. Purchasing power of rupee varies inversely
with the price index. Splicing refers to joining two or more series of
index numbers for the reason of continuity.

1.14 Self-Assessment Questions

Exercise 1 : True or False Statements


(i) Index numbers are used to measure changes in the magnitude of one
variable or a group of distinct but related variables.
(ii) Univariate index numbers involve only one variable.
(iii) Index numbers are called specialized averages because they are used
only for special purposes.
(iv) For a given set of data, the simple average-of-price relatives index
using arithmetic mean would be smaller than simple average-of-price
relatives index using geometric mean if the current year prices are
lower than the corresponding base year prices.

PAGE 371
© Department of Distance & Continuing Education, Campus of Open Learning,
School of Open Learning, University of Delhi
B.COM. (PROGRAMME)

Notes (v) Laspeyre’s formula uses base year quantities as weights both for
price and quantity index numbers.
(vi) Laspeyre’s price index can never be smaller than 100 in value.
(vii) Since it is not practical to include all commodities in the construction
of an index number, the sample of commodities should be selected
by the method of sample random sampling for reasons of objectivity.
(viii) For a given set of data. Fisher’s price index number cannot exceed
both Laspeyre’s and Paasche’s price indices.
(ix) Fixed base index numbers are also called link relatives while price
relatives is another name for chain base indices.
(x) In weighted average-of-relatives index, the weights used are either
the base year quantities or the current year quantities.
(xi) Paasche’s price index is calculated by using current year values as
weights.
(xii) Laspeyre’s price index always has greater value than Paasche’s price
index.
(xiii) Dorbish-Bowley index is equal to the arithmetic mean of Laspeyre’s
and Paasche’s price indices.
(xiv) The value index is given by the product or price and quantity index
numbers.
(xv) If an index satisfies time-reversal test, it means that for that index.
P01 and P10 are reciprocal of each other.
(xvi) Simple aggregative index satisfies circular test.
(xvii) Fisher’s index is called ‘ideal’ because it satisfies all the tests of
adequacy of index numbers.
(xviii) Splicing refers to connecting two or more series of index numbers
for the purpose of continuity.
(xix) The purchasing power of rupee is inversely related to the price
index.

372 PAGE
© Department of Distance & Continuing Education, Campus of Open Learning,
School of Open Learning, University of Delhi
BUSINESS STATISTICS

Ans. Notes
(i) T (ii) T (iii) F (iv) F (v) F
(vi) F (vii) F (viii) T (ix) F (x) F
(xi) F (xii) F (xiii) T (xiv) F (xv) T
(xvi) T (xvii) F (xviii) T (xix) T

Exercise 2 : Questions and Answers


(i) What are index numbers? Why are they called specialized averages?
(ii) “Index numbers are said to be economic barometers.” Explain this
statement.
(iii) Distinguish between simple and weighted index numbers. Explain
the importance of weighting in the construction of index numbers.
Enumerate some of the important methods of weighting a price
index and discuss their relative merits and demerits.
(iv) Laspeyre’s price index generally shows an upward bias while Paasche’s
index shows a downward bias. Do you agree. Explain.
(v) Explain and illustrate the following:
(a) Base shifting
(b) Splicing
(c) Deflating
(vi) What is consumer price index number and what is its utility? What
methods are used to calculate these numbers?
(vii) State and explain the tests of adequacy of index numbers. Which of
these are satisfied by the following index numbers?
(a) Laspeyre
(b) Paasche
(c) Fisher
(d) Dorbish-Bowley
(e) Marshall-Edgeworth
(viii) Information on the price of a commodity over the last few years
is given below:

PAGE 373
© Department of Distance & Continuing Education, Campus of Open Learning,
School of Open Learning, University of Delhi
B.COM. (PROGRAMME)

Notes Year: 2004 2005 2006 2007 2008 2009 2010 2011 2012
Price in 62 68 63 69 75 78 82 98 100
Rs. per
kg.:
(a) Construct price index numbers taking (i) 2004 as base, and (ii)
2008 as base.
(b) Calculate chain base index numbers.
(ix) The price index for 2006 stood at 100. It increased by 8 per cent
in 2007, decreased by 6 per cent in 2008, decreased by 2 per cent
in 2009, increased by 14 per cent in 2010, remained unchanged
in 2011, and increased by 12 per cent in 2012. Calculate index
numbers for the years 2004 through 2012 taking 2006 = 100, and
then shift the origin to 2008.
(x) Using the following link relatives, calculate price relatives taking
2005 = 100.
Year: 2005 2006 2007 2008 2009 2010 2011 2012
Link relative: 100 114 120 120 114 136 105 110
(xi) Using following data, calculate the simple average-of-price relatives
index by taking
(a) Prices of 2005 as the base
(b) Average prices as the base
Year Commodity
A B C D E
2005 20 12 40 24 36
2008 30 20 20 30 15
2012 40 10 20 42 15
(xii) Calculate Laspeyre’s and Paasche’s price index numbers for the year
2012 using the following data about five commodities:
Year Commodity
A B C D E
Quantity: 2010 12 20 180 36 48
2012 9 32 244 25 60

374 PAGE
© Department of Distance & Continuing Education, Campus of Open Learning,
School of Open Learning, University of Delhi
BUSINESS STATISTICS

Year Commodity Notes


A B C D E
2010 24 18 24 48 24
Price: 2012 28 20 22 52 20
Further, test whether each of these satisfies the time-reversal test.
(xiii) From the following data, construct quantity index numbers using
(a) Fisher’s method, and (b) Marshall-Edgeworth method.
Commodity Base year Current year
Price Quantity Price Value
A 16 30 40 720
B 25 40 50 2,000
C 24 25 30 1,200
D 54 16 40 1,320
E 22 44 45 1,350
(xiv) Using the following data, show that Fisher’s formula satisfies the
time-reversal and factor-reversal tests.
Commodity Base year Current year
Price Value Price Value
A 6 300 10 440
B 3 300 8 1,200
C 12 720 15 1,050
D 15 600 20 900
E 4 60 5 80
(xv) An enquiry into the budgets of the middle class families of a certain
city revealed that on an average the percentage expenses on the
different groups were: Food 45, Clothing 12, Fuel & Light 8, and
Miscellaneous 20. The increases in group index numbers for the
current year as compared with a fixed base period were, respectively,
310, 50, 243, 148 and 185.
(a) Calculate the consumer price index number for the current year.
(b) A person was getting Rs. 14,400 pm in the base year and
Rs. 28,300 pm in the current year. State how much he ought
to have received as extra allowance to maintain his former
standard of living.

PAGE 375
© Department of Distance & Continuing Education, Campus of Open Learning,
School of Open Learning, University of Delhi
B.COM. (PROGRAMME)

Notes (xvi) From the information given below about the consumer price index
number for a certain group of families in a city, obtain the percentage
weights assigned to (a) clothing, and (b) housing. The consumer
price index number is known to be 152.3.
Group: Food Clothing Housing Fuel and Miscellaneous
electricity
Index: 140 185 205 120 156
Weight: 60 ? ? 8 10
(xvii) The monthly income of a person is Rs. 21,000. It is given that the
consumer price index number for a particular month is 136. Find
out the amount spent by him on (i) food, and (ii) clothing.
Group Expenditure Index
Food ? 180
Rent 2,940 100
Clothing ? 150
Fuel and power 3,360 110
Miscellaneous 3,780 80
(xviii) Owing to a sudden price disturbance, the consumer price index of a
working class in a certain area increased in a month by one-quarter
of what it was before, to 225. The index of food became 252 from
198, that of clothing from 185 to 205, that of fuel & lighting from
175 to 195, and that of miscellaneous from 138 to 212. The index
of rent, however, remained unchanged at 150. It was known that
the weights of clothing, rent and fuel & lighting were the same.
Find out the exact percentage weights of each of the groups.
Ans.
(xi) (a) 106.67, 110 (b) 108.20, (xii) L = 94.52, P = 94.16, No
95.95, 95.84,
(xiii) (a) = 109.66 (b) = 107.61 (xiv) (a) = 325 (b) = 18500
(xv) (a) = 10 (b) = 12 (xvi) (i) = 8400 (ii) = 2520
(xvii) F = 54, C = FL = R = 10,
Misc = 16

376 PAGE
© Department of Distance & Continuing Education, Campus of Open Learning,
School of Open Learning, University of Delhi
UNIT-5

PAGE 377
© Department of Distance & Continuing Education, Campus of Open Learning,
School of Open Learning, University of Delhi
L E S S O N

1
Time Series Analysis
STRUCTURE
1.1 Learning Objectives
1.2 Introduction
1.3 Components of Time Series
1.4 Models of Time Series
1.5 Methods of Measuring Trend
1.6 Second Degree Parabola
1.7 Exponential Trend
1.8 Shifting the Trend Origin
1.9 Conversion of Annual Trend to Monthly Trend
1.10 Measurement of Seasonal Variations
1.11 Summary
1.12 Self-Assessment Questions

1.1 Learning Objectives


After studying this lesson, you should be able to :
‹ Understand the meaning of time series analysis.
‹ Understand the importance and components of time series.
‹ Measure trend and seasonal variations by different methods.
‹ Measure trend by second degree parabola and exponential technique.
‹ Learn shifting the trend origin and conversion of annual trend to monthly basis and
vice versa.

1.2 Introduction
When quantitative data are arranged in the order of their occurrence, the resulting statistical
series is called a time series. The quantitative values are usually recorded over equal time
interval daily, weekly, monthly, quarterly, half yearly, yearly, or any other time measure.

PAGE 379
© Department of Distance & Continuing Education, Campus of Open Learning,
School of Open Learning, University of Delhi
B.COM. (PROGRAMME)

Notes Monthly statistics of industrial production in India, annual birth-rate


figures for the entire world, yield on ordinary shares, weekly wholesale
price of rice, daily records of tea sales or census data are some of the
examples of time series. Each has a common characteristic of recording
magnitudes that vary with passage of time.
Time series are influenced by a variety of forces. Some are continuously
effective other make themselves felt at recurring time intervals, and still
others are non-recurring or random in nature. Therefore, the first task is
to break down the data and study each of these influences in isolation.
This is known as decomposition of the time series. It enables us to
understand fully the nature of the forces at work. We can then analyse
their combined interactions. Such a study is known as time-series analysis.

1.3 Components of Time Series


A time series consists of the following four components or elements :
1. Basic or Secular or Long-time trend;
2. Seasonal variations;
3. Business cycles or cyclical movement; and
4. Erratic or Irregular fluctuations.

These components provide a basis for the explanation of the past behaviour.
They help us to predict the future behaviour. The major tendency of each
component or constituent is largely due to casual factors. Therefore a
brief description of the components and the causal factors associated with
each component should be given before proceeding further.

380 PAGE
© Department of Distance & Continuing Education, Campus of Open Learning,
School of Open Learning, University of Delhi
BUSINESS STATISTICS

1.3.1 Secular Trend Notes

Basic trend underlines the tendency to grow or decline over a period of


years. It is the movement that the series would have taken, had there been
no seasonal, cyclical or erratic factors. It is the effect of such factors
which are more or less constant for a long time or which change very
gradually and slowly. Such factors are gradual growth in population, tastes
and habits or the effect on industrial output due to improved methods.
Increase in production of automobiles and a gradual decrease in production
of foodgrains are examples of increasing and decreasing secular trend.
All basic trends are not of the same nature. Sometimes the predominating
tendency will be a constant amount of growth. This type of trend movement
takes the form of a straight line when the trend values are plotted on a
graph paper. Sometimes the trend will be constant percentage increase or
decrease. This type takes the form of a straight line when the trend values
are plotted on a semi-logarithmic chart. Other types of trend encountered
are “logistic”, “S-curves”, etc.
Properly recognising and accurately measuring basic trends is one of the
most important problems in time series analysis. Trend values are used as
the base from which other three movements are measured. Therefore, any
inaccuracy in its measurement may vitiate the entire work. Fortunately,
the causal elements controlling trend growth are relatively stable. Trends
do not commonly change their nature quickly and without warning. It
is therefore reasonable to assume that a representative trend, which has
characterized the data for a past period, is prevailing at present, and that
it may be projected into the future for a year or so.

1.3.2 Seasonal Variations


The two principal factors liable for seasonal changes are the climate or
weather and customs. Since, the growth of all vegetation depends upon
temperature and moisture, agricultural activity is confined largely to warm
weather in the temperate zones and to the rainy or post-rainy season in
the torrid zone (tropical countries or sub-tropical countries like India).
Winter and dry season make farming a highly seasonal business. This
high irregularity of month to month agricultural production determines
largely all harvesting, marketing, canning, preserving, storing, financing,

PAGE 381
© Department of Distance & Continuing Education, Campus of Open Learning,
School of Open Learning, University of Delhi
B.COM. (PROGRAMME)

Notes and pricing of farm products. Manufacturers, bankers and merchants who
deal with farmers find their business taking on the same seasonal pattern
which characterise the agriculture of their area.
The second cause of seasonal variation is custom, education or tradition.
Such traditional days as Diwali, Christmas. Id etc., product marked
variations in business activity, travel, sales, gifts, finance, accident, and
vacationing.
The successful operation of any business requires that its seasonal
variations be known, measured and exploited fully. Frequently, the
purchase of seasonal item is made from six months to a year in advance.
Departments with opposite seasonal changes are frequently combined in
the same firm to avoid dull seasons and to keep sales or production up
during the entire year.
Seasonal variations are measured as a percentage of the trend rather
than in absolute quantities. The seasonal index for any month (week,
quarter etc.) may be defined as the ratio of the normally expected value
(excluding the business cycle and erratic movements) to the corresponding
trend value. When cyclical movement and erratic fluctuations are absent
in a time series, such a series is called normal. Normal values thus are
consisting of trend and seasonal components. Thus when normal values are
divided by the corresponding trend values, we obtain seasonal component
of time series.

1.3.3 Business Cycle


Because of the persistent tendency for business to prosper, decline,
stagnate recover; and prosper again, the third characteristic movement
in economic time series is called the business cycle. The business cycle
does not recur regularly like seasonal movement, but moves in response
to causes which develop intermittently out of complex combinations of
economic and other considerations.
When the business of a country or a community is above or below
normal, the excess deficiency is usually attributed to the business cycle.
Its measurement becomes a process of contrast occurrences with a normal
estimate arrived at by combining the calculated trend and seasonal
movements. The measurement of the variations from normal may be

382 PAGE
© Department of Distance & Continuing Education, Campus of Open Learning,
School of Open Learning, University of Delhi
BUSINESS STATISTICS

made in terms of actual quantities or it may be made in such terms as Notes


percentage deviations, which is generally more satisfactory method as it
places the measure of cyclical tendencies on comparable base throughout
the entire period under analysis.

1.3.4 Irregular Variations


These movements are exceedingly difficult to dissociate quantitatively
from the business cycle. Their causes are such irregular and unpredictable
happenings such as wars, droughts, floods, fires, pestilence, fads and
fashions which operate as spurs or deterrents upon the progress of the
cycle. Examples of such movements are : high activity in middle forties
due to erratic effects of 2nd world war, depression of thirties throughout
the world, export boom associated with Korean War in 1950. The common
denominator of every random factor is that it does not come about as
a result of the ordinary operation of the business system and does not
recur in any meaningful manner.

1.4 Models of Times Series


A time series may not be affected by all type of variations. Some of
these type of variations may affect a few time series, while the other
series may be effected by all of them. Hence, in analyzing time series,
these effects are isolated.
(i) Multiplicative Model:
In classical time series analysis it is assumed that any given observation
is made up of trend, seasonal, cyclical and irregular movements and these
four components have multiplicative relationship.
Symbolically:
O = T × S × C × I
where O refers to original data,
T refers to trend,
S refers to seasonal variations,
C refers to cyclical variations and
I refers to irregular variations.

PAGE 383
© Department of Distance & Continuing Education, Campus of Open Learning,
School of Open Learning, University of Delhi
B.COM. (PROGRAMME)

Notes This is the most commonly used model in the decomposition of time series.
(ii) Additive Model
There is another model called Additive model in which a particular
observation in a time series is the sum of these four components.
O = T + S + C + I
To prevent confusion between the two models, it should be made clear
that in Multiplicative model S, C, and I are indices expressed as decimal
percents whereas in Additive model S, C and I are quantitative deviations
about trend that can be expressed as seasonal, cyclical and irregular in
nature.
If in a multiplicative model, T = 500, S = 1.4, C = 1.20 and I = 0.7 then
O = T × S × C × I
By substituting the values we get
O = 500 × 1.4 × 1.20 × 0.7 = 588
In additive model, T = 500, S = 100, C = 25, I = –50
O = 500 + 100 + 25 – 50 = 575
The assumption underlying the two schemes of analysis is that whereas
there is no interaction among the different constituents or components
under the additive scheme, such interaction is very much present in the
multiplicative scheme. Time series analysis, generally, proceed on the
assumption of multiplicative formulation.

1.5 Methods of Measuring Trend


Trend can be determined : (i) Free hand curve method; (ii) moving averages
method; (iii) semi-averages method; and (iv) least-squares method. Each
of these methods is described below.

1.5.1 Freehand Curve Method


The term freehand is used to any non-mathematical curve in statistical
analysis even if it is drawn with the aid of drafting instruments. This is
the simplest method of studying trend of a time series. The procedure
for drawing free hand curve is as follows:

384 PAGE
© Department of Distance & Continuing Education, Campus of Open Learning,
School of Open Learning, University of Delhi
BUSINESS STATISTICS

(i) The original data are first plotted on a graph paper. Notes
(ii) The direction of the plotted data is carefully observed.
(iii) A smooth line is drawn through the plotted points.
While fitting a trend line by the freehand method, an attempt should be
made that the fitted curve conforms to these conditions.
(i) The curve should be smooth either a straight line or a combination
of long gradual curves.
(ii) The trend line or curve should be drawn through the graph of the
data in such a way that the areas below and above the trend line
are equal to each other.
(iii) The vertical deviations of the data above the trend line must equal
to the deviations below the line.
(iv) Sum of the squares of the vertical deviations of the observations
from the trend should be minimum.
Example 1 : Draw a time series graph relating to the following data and
fit the trend by freehand method :
Year Production of Steel
(million tonnes)
TREND OF STEEL PRODUCTION
2007 20 Y
PRODUCTION OF STEEL

E
2008 22 40
D LIN
EN
TR ATA
2009 30 30 UA
LD
ACT
2010 28
20
2011 32
2012 25 10

2013 29 0 X
2007 2008 2009 2011 2012 2013 2014 2015 2016
2014 35 YEARS
2015 40
2016 32
The trend line drawn by the freehand method can be extended to project
future values. However, the free-hand curve fitting is too subjective and
should not be used as a basis for prediction.

PAGE 385
© Department of Distance & Continuing Education, Campus of Open Learning,
School of Open Learning, University of Delhi
B.COM. (PROGRAMME)

Notes 1.5.2 Moving Averages


The moving average is a simple and flexible process of trend measurement
which is quite accurate under certain conditions. This method establishes
a trend by means of a series of averages covering overlapping periods
of the data.
The process of successively averaging, say, three years data, and establishing
each average as the moving-average value of the central year in the
group, should be carried throughout the entire series. For a five-item,
seven-item or other moving averages, the same procedure is followed :
the average obtained each time being considered as representative of the
middle period of the group.
The choice of a 5-year, 7-year, 9-year, or other moving average is
determined by the length of period necessary to eliminate the effects of
the business cycle and erratic fluctuations. A good trend must be free from
such movements, and if there is any definite periodicity to the cycle, it
is well to have the moving average to cover one cycle period. Ordinarily,
the necessary periods will range between three and ten years for general
business series but even longer periods are required for certain industries.
In the preceding discussion, the moving averages of odd number of years
were representatives of the middle years. If the moving average covers
an even number of years, each average will still be representative of
the mid-point of the period covered, but this mid-point will fall half
way between the two middle years. In the case of a four-year moving
average, for instance each average represents a point half way between
the second and third years. In such a case, a second moving average may
be used to ‘recentre’ the averages. That is, if the first moving averages
gives averages centering half-way between the years, a further two-point
moving average will recentre the data exactly on the years.
This method, however, is valuable in approximating trends in a period of
transition when the mathematical lines or curves may be inadequate. This
method provides a basis for testing other types of trends, even though
the data are not such as to justify its use otherwise.
Example 2 : Calculate 5-yearly moving average trend for the time series
given below.

386 PAGE
© Department of Distance & Continuing Education, Campus of Open Learning,
School of Open Learning, University of Delhi
BUSINESS STATISTICS

Year : 1995 1996 1997 1998 1999 2000 2001 2002 2003 2004 2005 Notes
Quantity : 239 242 238 252 257 250 273 270 268 288 284
Year : 2006 2007 2008 2009 2010 2011 2012 2013 2014 2015
Quantity : 282 300 303 298 313 317 309 329 333 327
Solution :
Year Quantity 5-yearly moving total 5-yearly
moving
average
1995 239
1996 242
1997 238 1228 245.6
1998 252 1239 247.8
1999 257 1270 254.0
2000 250 1302 260.4
2001 273 1318 263.6
2002 270 1349 269.8
2003 268 1383 276.6
2004 288 1392 278.4
2005 284 1422 284.4
2006 282 1457 291.4
2007 300 1467 293.4
2008 303 1496 299.2
2009 298 1531 306.2
2010 313 1540 308.0
2011 317 1566 313.2
2012 309 1601 320.2
2013 329 1615 323.0
2014 333
2015 327
To simplify calculation work: Obtain the total of first five years data.
Find out the difference between the first and sixth term and add to the
total to obtain the total of second to sixth term. In this way the difference
between the term to be omitted and the term to be included is added to
the preceding total in order to obtain the next successive total.
Example 3 : Fit a trend line by the method of four-yearly moving average
to the following time series data.

PAGE 387
© Department of Distance & Continuing Education, Campus of Open Learning,
School of Open Learning, University of Delhi
B.COM. (PROGRAMME)

Notes Year : 2001 2002 2003 2004 2005 2006 2007 2008
Sugar production : 5 6 7 7 6 8 9 10
(lakh tons)
Year : 2009 2010 2011 2012
Sugar production : 9 10 11 11
(lakh tons)
Solution :
Year Sugar 4-yearly 4-yearly To recenter trend
Production moving moving values
(lakh tons)
2 yearly 2 yearly
centred moving
total average total average
1. 2. 3. 4. 5. 6.
2001 5
2002 6
2003 7 25 6.25 12.75 6.375
2004 7 26 6.50 13.50 6.75
2005 6 28 7.00 14.50 7.25
2006 8 30 7.50 15.75 7.875
2007 9 33 8.25 17.25 8.625
2008 10 36 9.00 18.50 9.25
2009 9 38 9.50 19.50 9.75
2010 10 40 10.00 20.25 10.125
2011 11 41 10.25
2012 11
Remark : Observe carefully the placement of totals, averages between
the lines.
Merits
1. This is a very simple method.
2. The element of flexibility is always present in this method as all
the calculations have not to be altered if same data is added. It
only provides additional trend values.
3. If there is a coincidence of the period of moving averages and
the period of cyclical fluctuations, the fluctuations automatically
disappear.

388 PAGE
© Department of Distance & Continuing Education, Campus of Open Learning,
School of Open Learning, University of Delhi
BUSINESS STATISTICS

4. The pattern of moving average is determined in the trend of data Notes


and remains unaffected by the choice of method to be employed.
5. It can be put to utmost use in case of series having strikingly irregular
trend.
Limitations
1. It is not possible to have a trend value for each and every year. As
the period of moving average increases, there is always an increase
in the number of years for which trend values cannot be calculated
and known. For example, in a five yearly moving average, trend
value cannot be obtained for the first two years and last two years,
in a seven yearly moving average for the first three years and last
three years and so on. But usually values of the extreme years are
of great interest.
2. There is no hard and fast rule for the selection of a period of
moving average.
3. Forecasting is one of the leading objectives of trend analysis. But
this objective remains unfulfilled because moving average is not
represented by a mathematical function.
4. Theoretically it is claimed that cyclical fluctuations are ironed out
if period of moving average coincide with period of cycle, but in
practice cycles are not perfectly periodic.

1.5.3 Semi-Average Method


This simple method can be used if a straight line trend is to be obtained.
Since the location of only two points is necessary to obtain a straight line
equation, it is obvious that we may select two representative points and
connect them by a straight line. Data are divided into two halves and an
average is obtained for each half. Each such average is shown against
the mid-point of the half period, we obtain two points on a graph paper.
By joining these points, a straight line trend is obtained.
The method is to be commended for its simplicity and used to some extent
in practical work. This method is also flexible, for it is permissible to
select representative periods to determine the two points. Unrepresentative
years may be ignored.

PAGE 389
© Department of Distance & Continuing Education, Campus of Open Learning,
School of Open Learning, University of Delhi
B.COM. (PROGRAMME)

Notes Example 4: Fit a trend line to the following data by the method of
Semi-averages:
Year 2000 2001 2002 2003 2004 2005 2006
Sales 100 105 115 110 120 105 115
(Lac
units)
Solution: Since the data consist of seven years, the middle year shall be
left out and an average of the first years and last three shall be obtained.
The average of first three year is (100 + 105 +115)/3 or 320/3 or 106.67
and the average of last three years (120 + 105 + 115)/3 or 340/3 or
133.33.

1.5.4 Method of Least Squares


If a straight line is fitted to the data it will serve as a satisfactory trend,
perhaps the most accurate method of fitting is that of least squares. This
method is designed to accomplish two results.
(i) The sum of the vertical deviations from the straight line must equal
zero.
(ii) The sum of the squares of all deviations must be less than the sum
of the squares for any other conceivable straight line.
There will be many straight lines which can meet the first condition.
Among all different lines, only one line will satisfy the second condition.

390 PAGE
© Department of Distance & Continuing Education, Campus of Open Learning,
School of Open Learning, University of Delhi
BUSINESS STATISTICS

It is because of this second condition that this method is known as the Notes
method of least squares. It may be mentioned that a line fitted to satisfy
the second condition, will automatically satisfy the first condition.
The formula for a straight-line trend can most simply be expressed as
Yc = a + bX
where X represents time variable, Yc is the dependent variable for which
trend values are to be calculated and a and b are the constants of the
straight line to be found by the method of least squares.
Constant a is the Y-intercept. This is the difference between the point of
the origin (O) and the point when the trend line and Y-axis intersect. It
shows the value of Y when X = 0, constant b indicates the slope which
is the change in Y for each unit change in X.
Let us assume that we are given observations of Y for n number of years.
If we wish to find the values of constants a and b in such a manner that
the two conditions laid down above are satisfied by the fitted equation.
Mathematical reasoning suggests that, to obtain the values of constants
a and b according to the Principle of Least Squares, we have to solve
simultaneously the following two equations.
6Y = na + b6X ...(i)
6XY = a6X + b6X2 ...(ii)
Solution of the two normal equations yield the following values for the
constants a and b:

nΣXY − ΣX ΣY
b =
nΣX 2 − (ΣX ) 2
Σ Y − bΣ X
and a =
n
Least Squares Long Method : It makes use of the abovementioned
two normal equations without attempting to shift the time variable to
convenient mid-year. This method is illustrated by the following example.
Example 5 :
Fit a linear trend curve by the least-squares method to the following data :

PAGE 391
© Department of Distance & Continuing Education, Campus of Open Learning,
School of Open Learning, University of Delhi
B.COM. (PROGRAMME)

Notes Year Production (Kg.)


2006 3
2007 5
2008 6
2009 6
2010 8
2011 10
2012 11
2013 12
2014 13
2015 15
Solution : The first year 2006 is assumed to be 0, 2007 would become
1, 2008 would be 2 and so on. The various steps are outlined in the
following table.
Year Production
Y X XY X2
1 2 3 4 5
2006 3 0 0 0
2007 5 1 5 1
2008 6 2 12 4
2009 6 3 18 9
2010 8 4 32 16
2011 10 5 50 25
2012 11 6 66 36
2013 12 7 84 49
2014 13 8 104 64
2015 15 9 135 81
Total 89 45 506 285
The above table yields the following values for various terms mentioned
below :
n = 10, 6X = 45, 6X2 = 285, 6Y = 89, and 6XY = 506
Substituting these values in the two normal equations, we obtain
89 = 10a + 45b ...(i)
506 = 45a + 285b ...(ii)

392 PAGE
© Department of Distance & Continuing Education, Campus of Open Learning,
School of Open Learning, University of Delhi
BUSINESS STATISTICS

Multiplying equation (i) by 9 and equation (ii) by 2. we obtain Notes


801 = 90a + 405b ...(iii)
1012 = 90a + 570b ....(iv)
Subtracting equation (iii) from equation (iv), we obtain
211 = 165b or b = 211/165 = 1.28
Substituting the value of b in equation (i), we obtain
89 = 10a + 45 × 1.28
89 = 10a + 57.60
10a = 89 – 57.6
10a = 31.4
a = 31.4/10 = 3.14
Substituting these values of a and b in the linear equation, we obtain the
following trend line
Yc = 3.14 + 1.28X
Inserting various values of X in this equation, we obtain the trend values
as below :
Year Observed Y b × X Yc (Col. 3
plus Col. 4)
1 2 3 4 5
2006 3 3.14 1.28 × 0 3.14
2007 5 3.14 1.28 × 1 4.42
2008 6 3.14 1.28 × 2 5.70
2009 6 3.14 1.28 × 3 6.98
2010 8 3.14 1.28 × 4 8.26
2011 10 3.14 1.28 × 5 9.54
2012 11 3.14 1.28 × 6 10.82
2013 12 3.14 1.28 × 7 12.10
2014 13 3.14 1.28 × 8 13.38
2015 15 3.14 1.28 × 9 14.66
Least Squares Method : We can take any other year as the origin, and
for that year X would be 0. Considerable saving of both time and effort
is possible if the origin is taken in the middle of the whole time span
covered by the entire series. The origin would then be located at the

PAGE 393
© Department of Distance & Continuing Education, Campus of Open Learning,
School of Open Learning, University of Delhi
B.COM. (PROGRAMME)

Notes mean of the X values. Sum of the X values would then equal 0. The
two normal equations would then be simplified to
6Y = Na ... (i)

ΣY
or a =
N
ΣXY
and 6XY = b6X2 or b =
.... (ii)
ΣX 2
Two cases of short cut method are given below. In the first case there are
odd number of years while in the second case the number of observations
are even.
Example 6 : Fit a straight line trend on the following data :
Year 2008 2009 2010 2011 2012 2013 2014 2015 2016
Y 4 7 7 8 9 11 13 14 17
Solution : Since we have 9 observations, the origin, is taken at 2012 for
which X is assumed to be 0.
Year Y X XY X2
2008 4 – 4 – 16 16
2009 7 – 3 – 21 9
2010 7 – 2 – 14 4
2011 8 – 1 – 8 1
2012 9 0 0 0
2013 11 1 11 1
2014 13 2 26 4
2015 14 3 42 9
2016 17 4 68 16
Total 90 0 88 60
Thus n = 9, 6Y = 90, 6X =0,6XY= 88, and 6X2 = 60
Substituting these values in the two normal equations, we get
90 = 9a or a = 90/9 or a = 10
88 = 60b or b = 88/60 or b = 1.47
? Trend equation is : Yc = 10 + 1.47 X
Inserting the various values of X, we obtain the trend values as below.

394 PAGE
© Department of Distance & Continuing Education, Campus of Open Learning,
School of Open Learning, University of Delhi
BUSINESS STATISTICS

Years Observed X a b × X Yc(Col. 4 plus Notes


Y Col. 5)
2008 4 – 4 10 1.47 × –4 = – 5.88 4.12
2009 7 – 3 10 1.47 × –3 = – 4.41 5.59
2010 7 – 2 10 1.47 × –2 = – 2.94 7.06
2011 8 – 1 10 1.47 × –1 = – 1.47 8.53
2012 9 0 10 1.47 × 0 = 0 10.00
2013 11 1 10 1.47 × 1 = 1.47 11.47
2014 13 2 10 1.47 × 2 = 2.94 12.94
2015 14 3 10 1.47 × 3 = 4.41 14.41
2016 17 4 10 1.47 × 4 = 5.88 15.88
Example 7 : Fit a straight line trend to the data which gives number of
passenger cars sold (millions)
Year 2009 2010 2011 2012 2013 2014 2015 2016
No. of 6.7 5.3 4.3 6.1 5.6 7.9 5.8 6.1
cars
(millions)
Solution : Here there are two mid-years viz; 2012 and 2013. The mid-
point of the two years is assumed to be 0 and the time of six months is
treated to be the unit. On this basis the calculations are as shown below :
Year Observed Y X XY X2
2009 6.7 – 7 – 46.9 49
2010 5.3 – 5 – 26.5 25
2011 4.3 – 3 –12.9 9
2012 6.1 – 1 – 6.1 1
2013 5.6 1 5.6 1
2014 7.9 3 23.7 9
2015 5.8 5 29.0 25
2016 6.1 7 42.7 49
Total 47.8 0 8.6 168
From the above computations, we get the following values.
n = 8, 6Y = 47.8, 6X = 0, 6XY = 8.6, 6X2= 168
Substituting these values in the two normal equations, we obtain
47.8 = 8a or a = 47.8/8 or a = 5.98
and 8.6b = 168b or b = 8.6/168 or b = 0.051

PAGE 395
© Department of Distance & Continuing Education, Campus of Open Learning,
School of Open Learning, University of Delhi
B.COM. (PROGRAMME)

Notes The equation for the trend line is : Yc = 5.98 + 0.051


Trend values generated by this equation are given below.
Years Observed X a b × X Yc(Col. 4 plus
Y Col. 5)
2009 6.7 – 7 5.98 .051 × –7 = –.357 5.623
2010 5.3 – 5 5.98 .051 × –5 = –.255 5.725
2011 4.3 – 3 5.98 .051 × –3 = –.153 5.827
2012 6.1 – 1 5.98 .051 × –1 = –.051 5.929
2013 5.6 1 5.98 .051 × 1 = .051 6.031
2014 7.9 3 5.98 .051 × 3 = .153 6.133
2015 5.8 5 5.98 .051 × 5 = .255 6.235
2016 5.1 7 5.98 .051 × 7 = .357 6.337

1.6 Second Degree Parabola


The simplest example of the non-linear trend is the second degree parabola,
the equation is written in the form:
Yc = a + bX+ cX2
When numerical values for a, b and c have been derived, the trend value
for any year may be computed substituting in the equation the value of
X for that year. The values of a, b and c can be determined by solving
the following three normal equations simultaneously:
(i) 6Y = Na + b6X + c6X2
(ii) 6XY = a6X + b6X2 + c6X3
(iii) 6X2Y = a6X2 + b6X3 + c6X4
Note that the first equation is merely the summation of the given function,
the second is the summation of X multiplied into the given function,
and the third is the summation of X2 multiplied into the given function.
When time origin is taken between two middle years 6X would be zero.
In that case the equations are reduced to:
(i) 6Y = Na + c6X2
(ii) 6XY = b6X2
(iii) 6X2Y = a6X2 + c6X4
The value of b can now directly be obtained from equation (ii) and value
of a and c by solving equations (i) and (iii) simultaneously. Thus,

396 PAGE
© Department of Distance & Continuing Education, Campus of Open Learning,
School of Open Learning, University of Delhi
BUSINESS STATISTICS

Notes
ΣY − cΣX 2 ΣXY N ΣX 2Y − ΣX 2 ΣY
a = ; b = ; c =
N ΣX 2 N ΣX 4 − ( ΣX 2 ) 2
Example 8 : The price of a commodity during 2010–2015 is given below.
Fit a parabola Y = a + bX + cX2 to this data. Estimate the price of the
commodity for the year 2016
Year Price Year Price
2010 100 2013 140
2011 107 2014 181
2012 128 2015 192
Also plot the actual and trend values on graph.
Solution : To determine the value a, b and c, we solve the following
normal equations:
6Y = Na + b6X + c6X2
6XY = a6X + b6X2 + c6X3
6X2Y = a6X2 + b6X3 + c6X4
Year Y X X2 X3 X4 XY X 2Y Yc
2010 100 –2 4 –8 16 – 200 400 97.744
2011 107 –1 1 –1 1 – 107 107 110.426
2012 128 0 0 0 0 0 0 126.680
2013 140 +1 1 +1 1 + 140 140 146.506
2014 181 +2 4 +8 16 + 362 724 169.904
2015 192 +3 9 +27 81 +576 1728 196.874
N = 6 6Y = 6X = 6X2 = 6X3 = 6X4 = 6XY = 6X2Y = 6Yc =
848 3 19 27 115 771 3099 848.134
848 = 6a + 3b + 19c ...(i)
771 = 3a + 19b + 27c ...(ii)
3,099 = 19a+27b + 115c ...(iii)
Solving Eqns. (i) and (ii), we get
35b +35c = 695 ...(iv)
Multiplying Eqn. (ii) by 19 and Eqn. (iii) by 3. Subtracting (iii) from
(ii), we get
5352 = 280b + 168c ...(v)

PAGE 397
© Department of Distance & Continuing Education, Campus of Open Learning,
School of Open Learning, University of Delhi
B.COM. (PROGRAMME)

Notes Solving Eqns. (iv) and (v), we get


c= 1.786
Substituting the value of c in Eqn. (iv), we get
b = 18.04 [35b + (35 × l.786) = 695]
Putting the value of b and c in Eqn. (i), we get
a = 126.68 [848 = 6a + (3 × 18.04) + (19 × 1.786)]
Thus a = 126.68, b = 18.04 and c = 1.786
Substituting the values in the equation
Yc = 126.68+ 18.04 X+ 1.786X2
When X = – 2, Y
= 126.68 – 36.08 + 7.144 = 97.7442
When X = – 1, Y = 126.68 + 18.04(–1) + 1.786 (–1)2
= 126.68 – 18.04 + 1.786 = 110.426
When X = 0, Y = 126.68
When X= 1, Y = 126.68 + 18.04 + 1.786 = 146.506
When X = 2, Y = 126.68 + 18.04 (2) + 1.786 (2)2
= 126.68 + 36.08 + 7.144 = 169.904
When X = 3, Y = 126.68 + 18.04(3) + 1.786(3)2
= 126.68 + 54.12 + 16.074 = 196.874
Price for 2016, Y = 126.68 + 18.04(4) + 1.786(4)2
When X = 4 = 126.68 + 72.16 + 28.576 = 227.416
Thus the likely price of the commodity for the year 2016 is Rs. 227.416.

1.7 Exponential Trends


The equation of the exponential curve is
Y = abx
Putting the equation in logarithmic form, we get
log Y = log a +X logb.
When plotted on a semi-logarithmic graph, the curve gives a straight
line. However, on an arithmetic chart the curve gives a non-linear trend.
To obtain the value of the constants a and b, the two normal equations
to be solved are :

398 PAGE
© Department of Distance & Continuing Education, Campus of Open Learning,
School of Open Learning, University of Delhi
BUSINESS STATISTICS

6log Y = N log a + log b6X Notes


6(X log Y) = log a6X + log b6X2
where ais the Y intercept and bthe slope of the curve.
When deviations are taken from middle year, 6X = 0, the above
equation becomes
Σ logY
6log Y = Nlog a ? log a =
N
ΣX log Y
and 6(X log Y) = log b6X2 ? log b =
ΣX 2
Steps. The steps in fitting a curve are :
(i) Find the time deviation of each year from the middle year and denote
these deviations by X.
(ii) Square these deviations and obtain 6X2.
(iii) Obtain logarithms of the variable Y.
(iv) Multiply log Y by the corresponding time deviation and obtain X log
Y.
(v) Divide 6 log Y by N. This would give the value of log a.
(vi) Divide 6(X log Y) by 6X2. This would give the value of log b, i.e.,
rate of growth or the slope of the line.
(vii) Put the value of log a before the middle year and add or subtract
the slope of the line, i.e., the value of log b to get trend ordinates
in logarithms.
(viii) Take the antilogs of these logs to arrive at the actual trend values.
Example 9: The sales of a company for the years to are given below :
Years : 2009 2010 2011 2012 2013 2014 2015
Sales (Rs. : 32 47 65 92 190 132 275
million)
Estimate sales figure for the year 2018 using the equation of the form Y
= abx where X = years and Y = Sales

PAGE 399
© Department of Distance & Continuing Education, Campus of Open Learning,
School of Open Learning, University of Delhi
B.COM. (PROGRAMME)

Notes Solution : Fitting Y = abx


Equation
Year Sales (Rs. X log Y X2 X log Y
million) Y
2009 32 – 3 1.5051 9 – 4.5153
2010 47 – 2 1.6721 4 – 3.3442
2011 65 – 1 1.8129 1 – 1.8129
2012 92 0 1.9638 0 0
2013 190 + 1 2.2788 1 + 2.2788
2014 132 + 2 2.1206 4 + 4.2412
2015 275 + 3 2.4393 9 + 7.3179
N = 7 6Y = 833 6X = 0 6log Y 6X = 28
2
6 Xlog Y
= 13.7926 = 4.1655
Σ log Y 13.7926
log a = = = 1.9704
N 7
ΣX log Y 4.1655
log b = = = 0.149
ΣX 2 28
log Y = 1.9704 + 0.149 X

1.8 Shifting the Trend Origin


Trends are usually fitted to annual data with the middle of the series as
origin. At times it may be necessary to change the origin of the trend
equation to some other point in the series. For example, annual trend
values may be changed to monthly or quarterly values if we wish to
study seasonal or cyclical patterns.
For an arithmetic straight line we have to find out new Y intercept, lies
the value of ‘a’. The value of ‘b’ remains unchanged, since the slope of
the trend line remains same irrespective of the origin. The procedure of
shifting the origin may be done by the expression :
Yt = a + b(X + k)
where k is the number of time units shifted. If the origin is shifted
forward in time, k is positive, if shifted backward in time, k is negative.
Example 10 : You are given the trend equation
Yc= 110 + 2X

400 PAGE
© Department of Distance & Continuing Education, Campus of Open Learning,
School of Open Learning, University of Delhi
BUSINESS STATISTICS

(origin 2008, time unit 1 year) Notes


Shift the origin to 2012.
Solution : We are required to shift the origin to 2012, 4 years forward.
Here k = 4. The required equation can be obtained as :
Yt = a + b (X + k)
= 110 + 2(X + 4) = 110 + 2X + 8 = 118 + 2X
(origin 2012, X unit = 1 year)
Example 11 : You are given the trend equation
Yc = 210 – 1.5X
(origin 2012, time unit 1 year)
Shift the origin to 2007.
Solution : Changing origin from 2012 to 2007 means going back by 5
years. Using the formula
Yt = a + b(X + k)
= 210 – 1.5(X – 5) = 210 = 1.5X + 7.5 = 21.75 – 1.5X
(origin 2007, time period one year)
Example 12 : You are given the following equation :
Y = 126.55 + 18.04X + 1.786X2
(origin 2011–12)
Solution : If we wish to shift the origin for this equation to 2012, we
may follow the procedure explained above
Yt = 126.55 + 18.04(X + 0.5) + 1.786(X + 0.5)2
= 126.55 + 18.04X + 9.02 + 1.786(X2 + X + 0.25)
= 126.55 + 18.04X + 9.02 +1.786X2 + 1.786X + 0.4465
= 136.0165 + 19.826X + 1.786X2

1.9 Conversion of Annual Trend to Monthly Trend


From annual trend equations we can obtain monthly trend equations
without any loss in accuracy. When the Y units are annual totals then
an annual trend equation can be converted into an equation of monthly

PAGE 401
© Department of Distance & Continuing Education, Campus of Open Learning,
School of Open Learning, University of Delhi
B.COM. (PROGRAMME)

Notes totals by dividing the computed constant ‘a’ by 12 and the value of ‘b’
by 144. Justification of dividing ‘a’ and ‘b’ by 12 and 144 is that the
data are sums of 12 months hence ‘a’ and ‘b’ must be divided by 12 and
‘b’ is again divided by 12 so that the time units (X’s will be in months
as well, i.e., ‘b’ would give monthly increments). Therefore the monthly
trend equation becomes:
a b
Y = + X
12 144
The annual trend equation can also be reduced to quarterly trend equation
which will be given by :
a b a b
Y = + X or + X
4 4× 4 4 16
Example 13 : The trend of the annual sales of ABC Co. Ltd. is given
by the following equation :
Yc = 30 + 3.6X (origin 2012, X unit = 1 year, Y unit = annual sales)
Convert the equation on monthly basis.
Solution : To convert an annual trend equation on monthly basis, the
value of ‘a’ is divided by 12 and the value of ‘b’ by 144. The equation
on monthly basis is
30 3.6
Yc = + X
12 144

Yc = 2.5 + 0.025X
If the annual trend equation is of second degree, the corresponding
monthly trend equation is obtained by dividing ‘a’ by 12, ‘b’by 144 and
‘c’ by 1728 (the last being identical to dividing ‘c’ by 12 three times).
Example 14 : Convert the following annual trend equation on a monthly
basis :
Yc = 10.6 + 0.8 X + 0.64X2
Solution : To convert annual trend equation of the second degree on
monthly basis, divide ‘a’ by 12, ‘b’ by 144 and ‘c’ by 1,728. Thus, the
required equation will be :

402 PAGE
© Department of Distance & Continuing Education, Campus of Open Learning,
School of Open Learning, University of Delhi
BUSINESS STATISTICS

10.6 0.8 0.64 2 Notes


Yc = + X+ X
12 144 1728
= 0.883 + 0.0056X + 0.00037X2
where data are given as monthly averages per year, the value of ‘a’ remains
unchanged and the ‘b’ is divided by 12 only once. The reason is that ‘a’
is already at the monthly level and ‘b’now represents the annual change
in monthly magnitudes. In case of a second-degree trend equation, the
value of ‘c’ is divided by 144.
Example 15 : You are given the following trend equation:
Yc = 280 – 1.8X (origin June 30, 2012,
Y unit = annual monthly average sales)
Convert this equation into monthly terms and shift the origin half a
month forward.
Solution : (i) The given annual trend equation reduced to monthly values
will be :
1.8
Yc = 280 − X
12
= 280 – 0.15X
(origin: June 30, 2012; X unit = 1 month; Y unit = average monthly sales)
(ii) Shifting the origin half a month forward :
Yt = 280 – 0.15 (X + 0.5)
= 280 – 0.15X – 0.075
= 279.925 – 0.15X

1.10 Measurement of Seasonal Variations


Seasonal variations are those rhythmic changes in the time series data
that occur regularly each year. They have their origin in climatic or
institutional factors that affect either supply or demand or both. It is
important that these variations be measured accurately for three reasons.
First, the investigator wants to eliminate seasonal variations from the data
he is studying. Second, a precise knowledge of the seasonal pattern aid
in planning future operations. Lastly, complete knowledge of seasonal

PAGE 403
© Department of Distance & Continuing Education, Campus of Open Learning,
School of Open Learning, University of Delhi
B.COM. (PROGRAMME)

Notes variations is of use to those who are trying to remove the cause of
seasonals or are attempting to mitigate the problem by diversification,
offsetting opposing seasonal patterns, or some other means.
Since the number of calender days and working days vary from month
to month, therefore, it is essential to adjust the monthly figures if the
same are based on daily quantities, otherwise, there is no need for such
adjustment when we deal with either volume of inventories or of bank
deposits because then the values are not influenced by the number of
calender days or working days.
Three Reason for Studying seasonal variation
‹ To determine the effects of seasonal variations on the value of a
given phenomenon
‹ Projection of past patterns into the future
‹ Elimination of seasonal component
Methods of Measuring Seasonal Variations
1. Method of Simple Averages (Weekly, Monthly or Quarterly).
2. Ratio-to-Moving Average Method.
3. Ratio-to-Trend Method.
4. Link Relatives Method.

1.10.1 Methods of Simple Averages


This is the simplest method of obtaining a seasonal index. The following
steps are necessary for calculating the index:
(i) Average the unadjusted date by years and months or quarters if
quarterly data are given.
(ii) Find totals of January, February etc.
(iii) Divide each total by the number of years for which data are given.
For example, if we are given monthly data for five years then we
shall first obtain total for each month for five years and divide each
total by 5 to obtain an average.
(iv) Obtain an average of monthly averages by dividing the total of
monthly averages by 12.

404 PAGE
© Department of Distance & Continuing Education, Campus of Open Learning,
School of Open Learning, University of Delhi
BUSINESS STATISTICS

(v) Taking the average of monthly average as 100, compute the percentage Notes
of various monthly averages as follows:
Monthly average for January
Seasonal Index for January = ×100
Average of monthly average
If instead of the average of each month, the total of each month are
obtained, we will get the same result. The following example shall
illustrate the method.
Example 16 : Consumption of monthly electric power in million of Kw
hours for street lighting in India during 2011 – 2015 is given below:
Year Jan. Feb. Mar. Apr. May June July Aug. Sept. Oct. Nov. Dec
2011 318 281 278 250 231 216 223 245 269 302 325 347
2012 342 309 299 268 249 236 242 262 288 321 342 364
2013 367 328 320 287 269 251 259 284 309 245 367 394
2014 392 349 342 311 290 273 282 305 328 364 389 417
2015 420 378 370 334 314 296 305 330 356 396 422 452
Find out seasonal variation by the method of monthly averages.
Solution : Computation of Seasonal Indices by Monthly Averages
Monthly Five Percentage
Consumption of monthly electric
Month total for yearly
power
5 years average
(1) (2) (3) (4) (5) (6) (7) (8) (9)
Jan. 318 342 367 392 420 1,839 367.8 116.1
Feb. 281 309 328 349 378 1,645 329.0 103.9
March 278 299 320 342 370 1,609 321.8 101.6
April 250 268 287 311 334 1,450 290.0 91.6
May 231 249 269 290 314 1,353 270.6 85.4
June 216 236 251 273 296 1,272 254.4 80.3
July 223 242 259 282 305 1,311 262.2 82.8
Aug. 245 262 284 305 330 1,426 285.2 90.1
Sept. 269 288 309 328 356 1,550 310.0 97.9
Oct. 302 321 245 364 396 1,728 345.6 109.1
Nov. 325 342 367 389 422 1,845 369.0 116.5
Dec. 347 364 394 417 452 1,974 394.8 124.7
Total 19,002 3,800.4 1,200
Average 1,583.5 316.7 100

PAGE 405
© Department of Distance & Continuing Education, Campus of Open Learning,
School of Open Learning, University of Delhi
B.COM. (PROGRAMME)

Notes The above calculations are explained below:


(i) Column No. 7 gives the total for each month for five years.
(ii) In column No. 8 each total of column No. 7 has been divided by 5
to obtain an average for each month.
(iii) The average of monthly averages is obtained by dividing the total
of monthly averages by 12.
(iv) In column No. 9 each monthly average has been expressed as
percentage of the average of monthly averages. Thus, the percentage
for January
367.8
= ×100 = 116.1
316.7
329.0
Percentage for February = ×100 = 103.9
316.7
If instead of monthly data, we are given weekly or quarterly data, we shall
compute weekly or quarterly averages by following the same procedure.
Advantages of the Method of Simple Averages:
‹ Simplicity: The method is straightforward and easy to calculate.
It involves adding up all the values and dividing by the number
of values, making it accessible for individuals without advanced
statistical knowledge.
‹ Quick Computation: Calculating the arithmetic mean using simple
averages is relatively quick, which can be advantageous when
dealing with large datasets or when time is limited.
‹ Useful for Quick Insights: Simple averages can provide a quick
overview of the central tendency of the data. They offer a general
sense of the “typical” value, making it easy to communicate the
data’s central value to a broad audience.
‹ Stability: In cases where the data does not exhibit extreme outliers
or significant variations, the simple average can provide a stable
estimate of the central value.

406 PAGE
© Department of Distance & Continuing Education, Campus of Open Learning,
School of Open Learning, University of Delhi
BUSINESS STATISTICS

Disadvantages of the Method of Simple Averages: Notes


‹ The method attempts to eliminate the effects of cyclical or irregular
variations by averaging the monthly figures over a different year
but this is not sufficient.
‹ When the data is not symmetrically distributed and is skewed, the
mean may not accurately represent the “typical” value. In skewed
distributions, the mean is dragged towards the tail of the distribution.

1.10.2 Ratio-to-Moving Average Method


The method of monthly totals or monthly averages does not give any
consideration to the trend which may be present in the data. The ratio-
to-moving average method is one of the simplest of the commonly used
devices for measuring seasonal variation which takes the trend into
consideration: The steps to compute seasonal variation are as follows:-
(i) Arrange the unadjusted data by years and months.
(ii) Compute the trend values by the method of moving averages. For this
purpose take 12 month moving average followed by a two-month
moving average to recentre the trend values.
(iii) Express the data for each month as a percentage ratio of the
corresponding moving-average trend value.
(iv) Arrange these ratios by months and years.
(v) Aggregate the ratios for January, February etc.
(vi) Find the average ratio for each month.
(vii) Adjust the average monthly ratios found in step (vi) so that they
will themselves average 100 per cent. These adjusted ratios will be
the seasonal indices for various months.
A seasonal index computed by the ratios-to-moving-average method
ordinarily does not fluctuate so much as the index based on straight-line
trends. This is because the 12-month moving average follows the cyclical
course of the actual data quite closely. Therefore the index ratios obtained
by this method are often more representative of the data from which they
are obtained than is the case in the ratio-to-trend method which will be
discussed later on.

PAGE 407
© Department of Distance & Continuing Education, Campus of Open Learning,
School of Open Learning, University of Delhi
B.COM. (PROGRAMME)

Notes Example 17 : Prepare a monthly seasonal index from the following data,
using moving averages method :
Monthly Sales of XYZ Products Co,. Ltd. (Rs.)
Year
Month 2010 2011 2012
January 3,639 3,913 4,393
February 3,591 3,856 4,530
March 3,326 3,714 4,287
April 3,469 3,820 4,405
May 3,321 3,647 4,024
June 3,320 3,498 3,992
July 3,205 3,476 3,795
August 3,205 3,354 3,492
September 3,255 3,594 3,571
October 3,550 3,830 3,923
November 3,771 4,183 3,984
December 3,772 4,482 3,880
Solution :
Computations of Ratios to 12-month centered moving averages for sales
(Rs.)
Year & Sales (Rs.) 12-month 12-month Centred Ratio to
month moving moving 12-months moving
total average moving average
average
1 2 3 4 5 6
2010
Jan. 3,639
Feb. 3,591
March 3,326
April 3,469
May 3,321
June 3,320
41,424 3,452
July 3,205 3,463 92.55
41,698 3,475
Aug. 3,205 3,486 91.94

408 PAGE
© Department of Distance & Continuing Education, Campus of Open Learning,
School of Open Learning, University of Delhi
BUSINESS STATISTICS

Year & Sales (Rs.) 12-month 12-month Centred Ratio to Notes


month moving moving 12-months moving
total average moving average
average
1 2 3 4 5 6
41,963 3,497
Sept. 3,255 3,513 92.66
42,351 3,529
Oct. 3,550 3,543 100.20
42,702 3,558
Nov. 3,771 3,572 105.57
43,028 3,586
Dec. 3,772 3,593 104.98
2011 43,206 3,601
Jan. 3,913 3,612 108.33
43,477 3,623
Feb. 3,856 3,630 106.23
43,626 3,636
March 3,714 3,650 101.75
43,965 3,664
April 3,820 3,675 103.95
44,245 3,687
May 3,647 3,704 98.46
44,657 3,721
June 3,498 3,751 93.26
45,367 3,781
July 3,476 3,801 91.45
45,847 3,821
Aug. 3,354 3,849 87.14
46,521 3,877
Sept. 3,594 3,901 92.13
47,094 3,925
Oct. 3,830 3,949 96.99
47,679 3,973
Nov. 4,183 3,989 104.86
48,056 4,005
Dec. 4,482 4,025 111.35

PAGE 409
© Department of Distance & Continuing Education, Campus of Open Learning,
School of Open Learning, University of Delhi
B.COM. (PROGRAMME)

Notes Year & Sales (Rs.) 12-month 12-month Centred Ratio to


month moving moving 12-months moving
total average moving average
average
1 2 3 4 5 6
2012 48,550 4,046
Jan. 4,393 4,059 108.23
48,869 4,072
Feb. 4,530 4,078 111.08
49,007 4,084
March 4,287 4,083 105.00
48,984 4,082
April 4,405 4,086 107.81
49,077 4,090
May 4,024 4,081 98.60
48,878 4,073
June 3,992 4,048 98.62
48,276 4,023
July 3,795
Aug. 3,492
Sept. 3,571
Oct. 3,923
Nov. 3,984
Dec. 3,880
Arranging the ratios-to-moving average by months and years we obtain
the following table from which the seasonal index for each month is
also obtained.
Computation of Seasonal Index by Ratios-to-Moving Averages of XYZ
Products Co. Ltd.
Year Seasonal
Month 2010 2011 2012 Total Average Index
January — 108.33 108.23 216.56 108.28 107.6
February — 106.23 111.08 217.31 108.65 108.1
March — 101.75 105.00 206.75 103.37 102.8
April — 103.95 107.81 211.76 105.88 105.3
May — 98.46 98.60 197.06 98.53 98.0
June — 93.26 98.62 191.88 95.54 95.4

410 PAGE
© Department of Distance & Continuing Education, Campus of Open Learning,
School of Open Learning, University of Delhi
BUSINESS STATISTICS

Year Seasonal Notes


Month 2010 2011 2012 Total Average Index
July 92.55 91.45 — 184.00 92.00 91.5
August 91.94 87.14 — 179.08 89.54 89.0
September 92.66 92.13 — 184.79 92.40 91.9
October 100.20 96.99 — 197.19 98.60 98.1
November 105.57 104.86 — 210.43 105.21 104.06
December 104.98 111.35 — 216.33 108.16 107.6
Total of Monthly Averages 1206.56
Average of Monthly Averages 100.55
Putting average of monthly averages as 100, monthly averages have been
admitted to obtain seasonal index for each month.
108.28
For example, Seasonal Index for January = ×100 = 107.6
100.55
108.65
for February = ×100 = 108.1
100.55
Merits
This method is more widely used in practice than other methods. The
index calculated by the ratio-to-moving average method does not fluctuate
very much. The 12-month moving average follows the cyclical course of
the actual data closely. So index ratios are the true representative of the
data from which they have been obtained.
Limitations
All seasonal index numbers cannot be calculated for each month for
which data is available. When a four month average is taken, 2 months,
in the beginning and 2 months in the end are left out for which we cannot
calculate seasonal index numbers.

1.10.3 Ratio-to-Trend-Method
The ratio-to-trend method is similar to ratio-to-moving-average method.
The only difference is the way of obtaining the trend values. Whereas in
the ratio-to-moving-average method, the trend values are obtained by the
method of moving averages, in the ratio-to-trend method, the corresponding
trend is obtained by the method of least squares.

PAGE 411
© Department of Distance & Continuing Education, Campus of Open Learning,
School of Open Learning, University of Delhi
B.COM. (PROGRAMME)

Notes The steps in the calculation of seasonal variation are as follows :


(i) Arrange the unadjusted data by years and months.
(ii) Compute the trend values for each month with the help of least
squares equation.
(iii) Express the data for each month as a percentage ratio of the
corresponding trend value.
(iv) Aggregate the January’s ratios, February’s ratios, etc., computed
previously
(v) Find the average ratio for each month.
(vi) Adjust the average ratios found in step (v) so that they will themselves
average 100 per cent.
The last step gives us the seasonal index for each month.
Sometimes the median is used in place of the arithmetic average of the
ratios-to-trend. The choice depends upon circumstances but there is a
preference for the median if several erratic ratios are found. In fact, if a
fairly large number of years, say, 20 or 15, are used in the computation,
it is not uncommon to omit extremely erratic ratios from the computation
of average of monthly ratios. Only the arithmetic average should be used
for small number of years.
This method has the advantage of simplicity and case of interpretation.
Although it makes allowance for the trend, it may be influenced by errors
in the calculation of the trend. The method may also be influenced by
cyclical and erratic influences. This source of possible error is eliminated
by the selection of a period of time in which depression is offset by
prosperity.
Example 18 : Find seasonal variations by the ratio-to-trend method from
the following data:
Year 1st Quarter 2nd Quarter 3rd Quarter 4th Quarter
2010 30 40 36 34
2011 34 52 40 44
2012 40 58 54 48
2013 54 76 68 62
2014 80 92 86 82

412 PAGE
© Department of Distance & Continuing Education, Campus of Open Learning,
School of Open Learning, University of Delhi
BUSINESS STATISTICS

Solution : For finding out seasonal variations by ratio-to-trend method, Notes


first the trend for yearly data will be obtained and convert them into
quarterly data.
Year Yearly Average of X XY X2 Trend Y = 56+ 12X
totals quarterly values
values of Y
2010 140 35 – 2 – 70 4 32 56 + 12 (–2)
2011 180 45 – 1 – 45 1 44 56+ 12 (–1)
2012 200 50 0 0 0 56 56+12(0)
2013 260 65 1 65 1 68 56+12(1)
2014 340 85 2 170 4 80 56+12(2)
Total 1120 280 + 120 10 280
Y = a + bX
ΣY 280
a = = = 56
N 5
ΣXY 120
b = = = 12
ΣX 2 10
The trend value for the middle quarter 2010, i.e., which should come
between 2nd and 3rd quarter is 32.
Yearly increment 12
Quarterly increment is : = =3
4 4
3
Therefore, the trend value for 2nd quarter will be 32 − = 30.5
2
3
The trend value for 3rd quarter is 32 + = 33.5
2
Similarly other values will be calculated.
Quarterly Trend Values
Year 1st 2nd 3rd 4th Total
2010 27.5 30.5 33.5 36.5 128
2011 39.5 42.5 45.5 48.5 176
2012 51.5 54.5 57.5 60.5 224
2013 63.5 66.5 69.5 72.5 272
2014 75.5 78.5 81.5 84.5 320

PAGE 413
© Department of Distance & Continuing Education, Campus of Open Learning,
School of Open Learning, University of Delhi
B.COM. (PROGRAMME)

Notes Now we calculate percentage of trend values on the basis of quarterly


⎛O ⎞
trend values ⎜ ×100 ⎟
⎝ T ⎠

Year 1st 2nd 3rd 4th


2010 109.1 131.1 107.5 93.1
2011 86.1 122.4 109.9 90.7
2012 77.7 106.4 93.9 79.3
2013 85.0 114.3 97.8 85.5
2014 106.0 117.2 105.5 97.0
Total 463.9 591.4 514.6 445.6
Average 92.78 118.28 102.92 89.12
The average of quarterly average of trend figures :
92.78 + 118.28 + 102.92 + 89.12 403.10
= = 100.775
4 4
92.78
Quarterly seasonal Index for 1st Quarter : ×100 = 92.1
100.775
118.28
Quarterly seasonal Index for 2nd Quarter : ×100 = 117.4
100.775
102.92
Quarterly seasonal Index for 3rd Quarter : ×100 = 102.1
100.775
89.12
Quarterly seasonal Index for 4th Quarter : ×100 = 88.4
100.775
The total of seasonal indices should be equal to 400 and that for monthly
indices should be 1200.
Merits
(i) This method is based on a logical procedure for measuring seasonal
variations. This procedure has advantage over the moving average
method for it has a ratio-to-trend value for each month for which
data is available. So this method avoids loss of data which is inherent
in the case of moving averages. If the period of time series is very
short then the advantage becomes more prominent.
(ii) It is a simple method.
(iii) It is easy to understand.

414 PAGE
© Department of Distance & Continuing Education, Campus of Open Learning,
School of Open Learning, University of Delhi
BUSINESS STATISTICS

Limitations: Notes
If the cyclical changes are very wide in the time series, the trend can
never follow the actual data, as closely as a 12-month moving average
will follow, under the ratio-to-trend method. There will be more bias in
a seasonal index computed by ratio-to-trend method.

1.10.4 Link Relative Method


Among all the methods of measuring seasonal variation, link relatives
method is the most difficult one. When this method is adopted the following
steps are taken to calculate the seasonal variation indices:
(i) Calculate the link relatives of the seasonal figures. Link relatives are
calculated by dividing the figure of each season × by the figure of
immediately preceding season and multiplying it by 100.
Current season’s figure
×100
Previous season’s figure
These percentages are called link relatives since they link each
month (or quarter or other time period) to the preceding one.
(ii) Calculating the average of the link relatives for each season. While
calculating average we might take arithmetic average but median is
probably better. The arithmetic average would give undue weight to
extreme cases which were not primarily due to seasonal influences.
(iii) Convert these averages into chain relatives on the base of the first
season.
(iv) Calculate the chain relatives of the first season on the basis of
the last season. There will be some difference between the chain
relative of the first season and the chain relative calculated by the
previous method. This difference will be due to long-term changes.
It is therefore necessary to correct these chain relatives.
(v) For correction, the chain relative of the first season calculated by
first method is deducted from the chain relative (of the first season)
calculated by the second method. The difference is divided by the
number of seasons. The resulting figure multiplied by 1, 2, 3 (and
so on) is deducted respectively from the chain relatives of the 2nd,
3rd, 4rd (and so on) seasons. These are correct chain relatives.

PAGE 415
© Department of Distance & Continuing Education, Campus of Open Learning,
School of Open Learning, University of Delhi
B.COM. (PROGRAMME)

Notes (vi) Express the corrected chain relatives as percentage of their averages.
These provide the required seasonal indices by the method of link
relatives.
The following example will illustrate the process.
Example 19 : Apply method of link relatives to the following data and
calculate seasonal indices.
Quarterly Figures
Quarter 2011 2012 2013 2014 2015
I 6.0 5.4 6.8 7.2 6.6
II 6.5 7.9 6.6 5.8 7.4
III 7.8 8.4 9.3 7.5 8.0
IV 8.7 7.3 6.4 8.5 7.1
Solution : Calculation of Seasonal Indices by Method of Link Relatives
Quarter
Year I II III IV
2011 — 108.3 120.0 111.5
2012 62.1 146.3 106.3 86.9
2013 93.2 95.6 143.1 68.8
2014 112.5 80.6 129.3 113.3
2015 77.6 110.6 109.6 88.8

Arithmetic 345.4 541.4 608.3 469.3


average = 86.35 = 108.28 = 121.66 = 93.86
4 5 5 5
Chain relative 100 × 108.28 121.66 × 108.28 93.86 × 131.73
100 100 100 100
= 108.28 = 131.73 = 123.64
Corrected 108.28 – 1.675 131.73 – 3.35 123.64 – 5.025
Chain relative = 106.605 = 128.38 = 118.615
100
Seasonal 106.605 128.38 118.615
Indices 100 × 100 × 100 × 100
113.4 113.4 113.4
= 94.0 = 113.21 = 104.60

416 PAGE
© Department of Distance & Continuing Education, Campus of Open Learning,
School of Open Learning, University of Delhi
BUSINESS STATISTICS

The correction factor is calculated as follows: Notes


Chain relative of the first quarter (on the basis of first quarter) = 100
Chain relative of the first quarter (on the basis of the last quarter) =
86.35 × 123.6
= 106.7
100
Difference between these chain relatives = 106.7 – 100 = 6.7
6.7
Difference per quarter = = 1.675
4
Adjusted chain relatives are obtained by subtracting 1 × l.675, 2 × 1.675,
3 × 1.675 from the chain relatives of 2nd, 3rd and 4th quarters,
respectively.
Seasonal variation indices are calculated as below:
100 + 106.605 + 128.38 + 118.615 453.6
= = 113.4
4 4
Correct chain relatives × 100
Seasonal variation index =
113.4
Advantage and disadvantage of Link Relative Method
Though the method is not easy to understand, the computation involved
in this method are less extensive than Ratio-to-Trend or Ratio-to-moving
average method. In this method only one link relative, that of first
period is lost whereas in the ratio-to-moving average, a number of values
at the beginning and at the end are lost. The average of link relatives
includes the effect of cyclical and trend components. Though trend is
subsequently eliminated by applying correction, the method would give
good estimates of seasonal indices provided the trend present in the time
series is linear. However, this is not the case with most of the economic
and business time series.
Meaning of “Normal” in Business Statistics
Business is often said to be “above normal” or “below normal”. When
so used the term “normal” is generally recognized to mean a level
of activity which is characterized by the presence of basic trend and
seasonal variation. This implies that the influence of business cycles and
erratic fluctuations on the level of activity is assumed to be insignificant.

PAGE 417
© Department of Distance & Continuing Education, Campus of Open Learning,
School of Open Learning, University of Delhi
B.COM. (PROGRAMME)

Notes Therefore, the product of trend value for any period when adjusted by
the seasonal index for that period gives us an estimate of the normal
activity during that period.

1.11 Summary
A time series refers to the observations of a random variable like sales,
employment, etc. placed in a chronological order. The twin reasons for
studying the time series include a historical understanding of the past
data and to make forecast for the future. There are four components of
a time series: (i) Secular trend, (ii) Cyclical variations, (iii) Seasonal
variations, and (iv) Irregular variations.
Secular trend refers to the general pattern of the values in a time series
– it is the long-term tendency of the movement of the variable. The
cyclical variations are caused by business cycles. A business cycle has
four phases: (i) peak time or prosperity (ii) recession (iii) trough or
depression, and (iv) recovery. Seasonal variations, which are caused by
weather, customs, festivals, etc show themselves in a period of one year.
They repeat year after year. Irregular variations or random fluctuations
are those which result from unpredictable events like strikes, natural or
other calamities etc.
The two models used for the purpose of decomposing are (i) additive
model, and (ii) multiplicative model. The additive model is based on the
assumption that the four components add up to make time series. They
are assumed to be independent. The multiplicative model is based on the
assumption that a time series is the product of the four components. The
linear trend is obtained by fitting a straight line to the given data. It is
fitted on the principle of least squares. It is possible to shift the origin
of an equation as Yt= a + b(X ± k).The annual trend equations can be
changed on a monthly or quarterly basis, and reverse is also possible. The
parabolic trend involves fitting a second-degree parabola to the given data.
It is of the form Yt = a + bX + cX2.The exponential trend is appropriate
where the variable in consideration grows or declines exponentially. It
takes the form Yt= abx.
The method of moving averages is another way of obtaining trend.
Beginning with a certain number of time periods, average is calculated

418 PAGE
© Department of Distance & Continuing Education, Campus of Open Learning,
School of Open Learning, University of Delhi
BUSINESS STATISTICS

and then successive averages are calculated by dropping the first of the Notes
values and including the next one. Seasonal variations are measured and
expressed as seasonal indices. The methods of simple averages, ratio-to-
moving averages and ratio-to-trend are primarily used for the purpose.

1.12 Self-Assessment Questions


Exercise 1: Mark the following statements as True or False:
(i) A time series refers to a sequence of observations of a random
variable over time and placed in chronological order.
(ii) The components of a time series are: secular trend, cyclical variations,
seasonal variations and chance variations.
(iii) Secular trend and cyclical variations are related to long-term movements
while seasonal variations and random variations refer to the short-
term changes.
(iv) Seasonal variations are highly predictable.
(v) Factors like population change, tastes, consumer incomes, etc. explain
cyclical variations.
(vi) Technological innovations cause cyclical variations.
(vii) Business cycles relate to the economy as a whole and never to a
particular industry.
(viii) The seasonal variations are caused by the changing seasons in a
country.
(ix) The seasonal variations component is most important to analyse for
purposes of forecasting and planning in the short-term.
(x) Irregular variations are erratic in nature.
(xi) In additive model, S, C and I are expressed as deviations from their
respective mean values.
(xii) Using the multiplicative model of analysis, all the components of a
time series are expressed as percentages.
(xiii) In obtaining trend equation to a given set of data, the origin should
be taken in such a manner that 6X must work out to be equal to
zero.

PAGE 419
© Department of Distance & Continuing Education, Campus of Open Learning,
School of Open Learning, University of Delhi
B.COM. (PROGRAMME)

Notes (xiv) The value of ‘a’, the intercept of a trend equation, is related to its
origin.
(xv) The trend values for various years given in the data and the projected
values do not change with a shift in the origin.
(xvi) The monthly trend equations can also be converted into annual trend
equations, and for this we first need to shift the origin of the trend
equation to July 1 of the year of origin.
(xvii) In exponential trend, a straight line trend is fitted to the log values
of the Y variable.
(xviii) The exponential trend is an example of non-linear trend.
(xix) Moving averages require centering whenever the underlying period,
n, used in their calculation is even.
(xx) A monthly sales budget can be drawn up by multiplying monthly
seasonal indices by average monthly sales and dividing each by
100.
Ans.
(i) T (ii) T (iii) F (iv) T (v) F
(vi) F (vii) F (viii) F (ix) T (x) T
(xi) F (xii) F (xiii) F (xiv) T (xv) T
(xvi) T (xvii) T (xviii) T (xix) T (xx) T
Exercise 2: Questions and Answers
(i) What is a time series? What are its components? With which
component of a time series would you mainly associate each of
the following?
(a) Wild cat strike in a factory, interrupting production for 15
days
(b) Increase in sales in a departmental store on Diwali
(c) An era of prosperity
(d) Fall in death rate due to advances in medical science.
(ii) What is meant by decomposition of a time series? Explain the
difference between additive and multiplicative models of analysing
time series.
(iii) Explain the rules of converting annual trend equation (i) on a
monthly basis, and (ii) on a quarterly basis. How can a quarterly
trend equation be converted on an annual basis?

420 PAGE
© Department of Distance & Continuing Education, Campus of Open Learning,
School of Open Learning, University of Delhi
BUSINESS STATISTICS

(iv) How are seasonal variations measured under the multiplicative model Notes
of analysing time series? How are the seasonal indices interpreted?
(v) Explain the following methods of calculating seasonal indices:
(a) Method of Simple Averages
(b) Ratio-to-trend Method
(c) Ratio-to-moving Averages Method
(vi) The following data relates to gross ex-factory value (in Rs. crores)
of output of a factory over the last few years:
Year : 2006 2007 2008 2009 2010 2011 2012
Value : 320 360 368 332 376 396 368
(a) Fit a straight line trend by the method of least squares, taking
the year of origin as 2006.
(b) What is the average annual change in the value of output?
(c) Obtain trend equation using the year 2009 as the origin. How
does it compare with equation obtained in (a) above?
(vii) Demand (in ‘000 metric tonnes) for sugar of Sweet India is given
here:
Year : 2006 2007 2008 2009 2010 2011 2012
Demand : 77 88 94 85 91 98 90
(a) Fit a straight line trend by the method of least squares
(b) Calculate trend values and plot observed values and trend values
on a graph
(c) Eliminate trend component using the multiplicative model
(d) Obtain the forecast of demand for the year 2014.
(viii) Below are given figures of production of a sugar factory:
Year : 2005 2006 2007 2008 2009 2010 2011 2012
Production : 88 98 100 91 102 107 100 118
(‘000 tons)
(a) Fit straight line trend to the above data by the method of least
squares
(b) What is the average annual change in the sugar production?
(c) Obtain trend values for various years. Show that the sum of
difference between actual and trend values is equal to zero

PAGE 421
© Department of Distance & Continuing Education, Campus of Open Learning,
School of Open Learning, University of Delhi
B.COM. (PROGRAMME)

Notes (d) Eliminate the trend using multiplicative model. What components
are thus left over?
(e) Convert the trend equation on a month-to-month basis and shift
the origin to January 2006
(ix) For each of the following derive the monthly trend equation:
(a) Yt = 960 + 72X Origin: 2008, X Unit = 1 Year, Y unit = Annual
sales of coffee in Rs.
(b) Yt = 169.58 + 78X Origin: 2009, X Unit = 1 Year, Y unit =
Average monthly production
(c) Yt = 2,760 + 212X Origin: 2007, X Unit = 1/2 Year, Y unit =
Annual earnings in Rs.
(d) Yt= 72 + 12X Origin: 2010, X Unit = 1/2 Year, Y unit = Average
monthly production
(x) Given the trend equation:
Yt = 204 + 24X
(2008 = 0, X unit = 1 Year, Y unit = Average monthly values)
(a) Convert this equation on a monthly basis
(b) Shift the origin of the monthly trend equation to January, 2007
(c) Estimate the value for January 2010
(xi) Given the following trend equation:
Yt= 1,880+ 6X
[2009 = 0, X unit = 1 Year, Y unit = Average monthly sales (in
‘000 Rs.)]
(a) Convert this equation on a yearly basis
(b) Estimate sales for the year 2013
(c) Obtain a quarterly trend equation from (a) above
(d) Obtain quarterly trend equation with origin at I Quarter, 2010.
(xii) Given below is a trend equation:
Yt= 372 + 288X
(Origin 2006, X unit = 1 Year, Y unit = Annual sales) Convert the
above equation:
(a) To monthly trend equation with January 2007 as origin and
estimate sales for March, 2007.

422 PAGE
© Department of Distance & Continuing Education, Campus of Open Learning,
School of Open Learning, University of Delhi
BUSINESS STATISTICS

(b) To quarterly trend equation with first quarter, 2007 as the Notes
origin and estimate sales for third quarter of 2007.
(xiii) Convert the following trend equation on a monthly basis and obtain
the trend value for November 2012:
Yt = 432 + 144X – 60X2
2010 = 0; X unit = 1 Year; Y unit = Yearly production in‘000 units
(xiv) The sales made by a company in the years 2006 through 2012 are
given here:
Year : 2006 2007 2008 2009 2010 2011 2012
Sales (in millions : 30 38 75 90 88 140 188
of Rs.)
(a) Fit an exponential trend Yt = abx to the data and obtain the
trend equation.
(b) Plot and data on a graph and also plot the trend line.
(c) Find the projected sales for the year 2014.
(d) What is the average rate of growth of sales?
(xv) From the following data, estimate the trend values by taking 4-yearly
moving averages:
Year Sales (Rs. lakh) Year Sales (Rs. Lakh)
1993 200 1999 360
1994 120 2000 400
1995 280 2001 320
1996 240 2002 360
1997 160 2003 360
1998 320
(xvi) The trend equation for quarterly sales of a firm is estimated to
be as: Y = 20 + 2X, where Y is sales per quarter in millions of
rupees, unit of X is one quarter and the origin is the middle of the
first quarter (Jan.-Mar.) of 2005. The seasonal indices of sales for
the four quarters are given below:
Quarter : I II III IV
Seasonal Index: 120 105 85 90
Estimate the sales for each quarter of 2010.
(xvii) Calculate seasonal indices from the following data by ratio-to-moving
averages method:

PAGE 423
© Department of Distance & Continuing Education, Campus of Open Learning,
School of Open Learning, University of Delhi
B.COM. (PROGRAMME)

Notes Year Quarter


I II III IV
2007 48 52 44 60
2008 60 72 64 80
2009 92 84 88 88
2010 96 100 96 104
2011 102 108 96 112
2012 108 116 120 116
(xviii) The ratios of observed values to moving averages in percentages
are given in the following table:
Year Quarter
I II III IV
2009 — — 112.2 96.8
2010 110.8 118.6 98.3 92.2
2011 102.4 116.2 96.3 88.0
2012 89.8 96.3 — —
Calculate the quarterly seasonal indices.
Ans.
(vi) Yt = 336 + 8X, 8 crores, 360 + 8X
(vii) Yt = 89 + 2X, 99000 mt
(viii) Yt = 99 + 3X(2008 = 0), 3000 tons, 90, 93, 96 etc. C & I, Yt
= 7.635 + 0.0208X (Jan 2006 = 0)
(ix) Yt = 80 + 0.5 X (July 2008 = 0) Yt = 169.58 + 6.5 X (July 1,
2009 = 0), Yt = 230 + 2.944 X (July 1, 2007 = 0), Yt = 72 +
2X (July 1, 2010 = 0)
(x) Yt = 204 + 2X (July 1, 2008 = 0), Yt = 169 + 2X (Jan 2007
= 0), 241
(xi) Yt = 22.56 + 72X (2009 = 0), Y 2013 = 22.848, Yt = 5651.25
+ 4.5X (Q1, 2010 = 0)
(xii) Yt = 44 + 2X (Jan 2007 = 0), 48, Yt = 138 + 18X (Q1, 2007
= 0), 174
(xiii) Yt = 36 + X – 0.0347 X2(2005 = 0), Y Nov. 2012 = 36.315
(xiv) (a) = 78.16 (1.3438)x (c) = 342.5, (d) = 34.38%
(xv) 205, 225, 260, 290, 330, 355, 360
(xvi) 75.6, 66.15, 53.55, 56.7
(xvii) 101, 92, 103.17, 92.08, 102.84
(xviii) 99.52, 108.74, 100.76, 90.98

424 PAGE
© Department of Distance & Continuing Education, Campus of Open Learning,
School of Open Learning, University of Delhi
BUSINESS STATISTICS

This question paper contains 16 printed pages Notes


Your Roll No. ............
5565
B.Com. (Hons.)/l
Paper Code : A-105
Paper IV—BUSINESS STATISTICS
(New Course : Admissions of 2004 and onwards)
Time : 3 Hours Maximum Marks: 55
(Write your Roll No. on the top immediately on receipt of this question
paper.)
Note :— The maximum marks printed on the question paper are
applicable for the students of the regular colleges (Cat. ‘A’).
These marks will, however, be scaled up proportionately in
respect of the students of SOL at the time of posting of awards
for compilation of result.
Note : Answers may be written either in English or in Hindi; but
the same medium should be used throughout the paper.
Attempt All questions.
All questions carry equal marks.
1.(a) Distinguish between Mean Deviation and Standard Deviation. Why
Standard Deviation is considered a better method of variation as
compared to Mean Deviation? 6
(b) In 2000 and 2010 the population of a country was 151.3 million and
179.3 million respectively:
(i) What was the average percentage increase per year?
(ii) Calculate the population for the year 2004.
(iii) Calculate population for the year 2020. 5
Or
(a) The median and mode of the following wage distribution are known
to be Rs. 33.5 and Rs. 34 respectively. Three frequency values from
the table are, however, missing. Find these missing values : 6

PAGE 425
© Department of Distance & Continuing Education, Campus of Open Learning,
School of Open Learning, University of Delhi
B.COM. (PROGRAMME)

Notes Daily wages Frequencies


(in Rs.)
0 – 10 10
10 – 20 10
20 – 30 ?
30 – 40 ?
40 – 50 ?
50 – 60 6
60 – 70 4
230
(b) The following table gives heights of boys and girls studying in a
college. Find:
(i) Standard deviation of the heights of boys and girls taken
together.
(ii) Whose heights are more variable? 5
Boys Girls
Number 400 100
Average height 68 inches 65 inches
Variance 9 4
2.(a) If the first four moments of distribution about the value 5 are
equal to –4, 22, –117 and 560, determine the corresponding
moments: 6
(i) About the mean, and
(ii) About zero.
(b) Given the bivariate data:
X Y
1 6
5 1
3 0
2 0
1 1
1 2
7 1
3 5

426 PAGE
© Department of Distance & Continuing Education, Campus of Open Learning,
School of Open Learning, University of Delhi
BUSINESS STATISTICS

(i) Fit a regression line of Y and X and hence predict Y if X = 5 Notes


(ii) Fit a regression line of X on Y and hence predict X if Y = 2.5
(iii) Calculate Karl Pearson’s coefficient of correlation. 5
Or
(a) What is Kurtosis ? Explain the significance of studying Kurtosis.5
(b) Coefficient of correlation between X and Y for 20 items is 0.3, mean
of X is 15 and that of Y is 20, standard deviations are 4 and 5
respectively. At the time of calculation one item 27 has wrongly
been taken as 17 in case of X series and 35 instead of 20 in case
of Y series. Find the correct coefficient of correlation. 6
3.(a) Two sets of Indices one with 1998 as base and the other with 2006
are given below: 6
Year Index A Year Index B
1998 100 2006 100
1999 110 2007 105
2000 120 2008 90
2001 190 2009 95
2002 300 2010 102
2003 330 2011 110
2004 360 2012 96
2005 390
2006 400
You are required to splice the Index B to Index A. Then also shift
the base to 2008.
(b) Calculate 5 yearly moving average of the number of students studying
in a college shown below: 5
Year No. of Students
2003 332
2004 317
2005 357
2006 392
2007 402
2008 405
2009 410
2010 427

PAGE 427
© Department of Distance & Continuing Education, Campus of Open Learning,
School of Open Learning, University of Delhi
B.COM. (PROGRAMME)

Notes Year No. of Students


2011 405
2012 431
Or
(a) From the data given below, calculate price index number for 2012
with 2011 as base year :
Commodities 2011 2012
Price Quantity Price Quantity
A 20 8 40 6
B 50 10 60 5
C 40 15 50 15
D 20 20 20 25
Calculate the following:
(i) Laspeyre’s Index
(ii) Paasche Index
(iii) Bowley’s Index and
(iv) Fisher’s Ideal Index 6
(b) Identify the component of a time series with which each of the
following be associated and also give reasons why:
(i) A fire in factory delaying production for three weeks
(ii) An era of prosperity
(iii) Sale of sweets during Deepawali
(iv) A need for increased wheat production due to constant increase
in population
(v) The increase in day temperature from winter to summer 5
4.(a) Define probability. Discuss the importance of probability in decision-
making. 5
(b) A bag contains 2 white balls and 3 black balls. Four persons A, B,
C, D in the order named each draws one ball and does not replace
it. The first to draw a white ball receives Rs. 20. Determine their
expectations. 6
Or

428 PAGE
© Department of Distance & Continuing Education, Campus of Open Learning,
School of Open Learning, University of Delhi
BUSINESS STATISTICS

(a) A man has five coins, one of which has two heads. He randomly Notes
takes out a coin and tosses it three times:
(i) What is the probability that it will fall head upward all the
times?
(ii) If it always falls head upward, what is the probability that it
is the coin with two heads?
(b) In a binomial distribution consisting of 5 independent trials, probabilities
of 1 and 2 successes are 0.4096 and 0.2048. Find the parameter ‘p’
of the distribution.
5. A food products company is contemplating the introduction of
a revolutionary new product with new packaging to replace the
existing product at much higher price (S1) or a moderate change in
the composition of the existing product with a new packaging at a
small increase in price (S2) or a small change in the composition
of the existing except the word “New’ with a negligible increase
in price (S3). The three possible states of nature are:
(i) high increase in sales (N1),
(ii) No change in sales (N2) and
(iii) decrease in sales (N3).
The marketing department of the company worked out the payoffs
in terms of yearly net profits for each of the strategies of these
events. This is represented in the following table 11
State of Nature
Payoffs (in Rs.)
Strategies N1 N2 N3
S1 7,00,000 3,00,000 1,50,000
S2 5,00,000 4,50,000 0
S3 3,00,000 3,00,000 3,00,000
Which strategy should the executive choose on the basis of:
(i) Maximin Criterion
(ii) Maximax Criterion
(iii) Minimax Regret Criterion, and
(iv) Laplace Criterion.

PAGE 429
© Department of Distance & Continuing Education, Campus of Open Learning,
School of Open Learning, University of Delhi
B.COM. (PROGRAMME)

Notes Or
(a) What is Standard Error of Estimate ? Why is it calculated? 4
(b) The arithmetic mean of a set of a statistical observations is 20 while
its geometric mean is 19 and Harmonic Mean is 25. Comment on
the statement. 3
(c) The Mean and Standard Deviation of two brands and interpret the
result:
Brand-I Brand-II
Mean 800 hours 770 hours
Standard Deviation 100 hours 60 hours
Calculate a measure of relative dispersion for the two brands and
interpret the result. 4

430 PAGE
© Department of Distance & Continuing Education, Campus of Open Learning,
School of Open Learning, University of Delhi
Glossary
Arithmetic Mean: Arithmetic mean is a mathematical average which is most commonly
used and understood and also very extensively used in statistical work.
Average: An average is a typical value which is used to represent the entire set of values
and is used as a benchmark to make comparisons.
Average Deviation: It is defined as a value, which is obtained by taking the average of
the deviations of various items, from a measure of central tendency, Mean or Median or
Mode, after ignoring negative signs.
Binomial Distribution: Refer to a set of mathematical models of the relative frequencies of
a finite number of observations of a variable. It is systematic arrangement of probabilities
of mutually exclusive and collectively exhaustive elementary events of an experiment.
Bombay Stock Exchange: The first organised stock exchange was established in July 1875
as an association of native brokers, named as native shares and stock brokers association.
its formal deed of association was executed in 1887. This stock exchange is now popularly
known as the Bombay Stock Exchange (BSE).
Bowley’s Method of Skewness: Bowley’s method of skewness is based on the values of
median, lower and upper quartiles.
Central Moments: Moments calculated about mean are called central moments.
Circular Test: It is based on the shiftability of the base accordingly, the index should
work in a circular fashion i.e., if an index number is computed for the period 1 on the
base period 0, another index is computed for period 2 on the base period 1, and still
another index number is computed for period 3 on the base period 2 then the product
should be equal to one.
Class Boundaries: The lower and upper class limits of new exclusive type classes are
called class boundaries.
Classical Approach: Ratio of number of favourable outcomes to total possible outcome.
Coefficients of Moments: 7KHUH DUH WKUHH FRHIILFLHQWV DQG WKH\ DUH Į $OSKD  ȕ %HWD 
Ȗ (Gamma) coefficients. These coefficients are calculated on individual relationships of
various Moments.
Complementary Events: Opposite to an event which has already occurred.

PAGE 431
© Department of Distance & Continuing Education, Campus of Open Learning,
School of Open Learning, University of Delhi
B.COM. (PROGRAMME)

Notes Compound Event: Subset of sample space consisting of multiple event.


Concurrent Deviation: This is the simplest method of studying correlation.
The only thing to be computed under this method is the direction of
change of both the variables.
Correlation: Correlation analysis contributes to the understanding of
economic behaviour, aids in locating the critically important variables
on which others depend, may reveal to the economist the connections by
which disturbances spread and suggest to him the paths through which
stabilizing forces may become effective.
Decision Theory: Decision Theory is concerned with decision-making
under conditions of uncertainty.
Decision Tree: It is a graphic representation of decision process. The
decision tree analysis helps in making decision concerning a wide variety
of problems such as project management, personnel, new product strategies,
acquisition or disposal of physical properties, investments, etc.
Deflating: It means making allowance for the changes in the purchasing
power of money due to a change in general price level. It is the technique
of converting a series of value calculated at current prices in to a series
at constant prices of a given year.
Dependent Event: Occurrence of an event influence the succeeding event.
Dependent Variable: Regression analysis deals with estimating values
of one variable based on the values of one or more other variables. The
variable being estimated is called dependent variable
Event: It refers to any set of possible outcomes in a sample space. If
the sample space for an event has the elements S1, S2, S3,...Sn, an event
in the sample space S would be any one, or collection.
Expected Value: Weighted average of discrete random variable corresponding
to its probability of occurrence.
Experiment: An experiment is any operation that can be repeated
indefinitely and has a well-defined collection of possible outcomes.
Frequency Distribution: Collected and classified data are presented
in a form of frequency distribution frequency distribution is simply a
table in which the data are grouped into classes on the basis of common

432 PAGE
© Department of Distance & Continuing Education, Campus of Open Learning,
School of Open Learning, University of Delhi
GLOSSARY

characteristics and the number of cases which fall in each class are Notes
recorded.
Frequency Polygon: This is a graph of frequency distribution which
has more than four sides. It is particularly effective in comparing two
or more frequency distributions.
Harmonic Mean: Harmonic mean is equal to the reciprocal of the
arithmetic mean of reciprocals. It is used to average rates.
Histogram: Histogram is the best way of presenting graphically a simple
frequency distribution. Histogram is that it is a graph that represents the
class frequencies in a frequency distribution by vertical adjacent rectangles.
Independent Events: Two events are said to be independent if the
occurrence of one event in no way influences the occurrence of the other
event.
Independent Variable: Regression analysis deals with estimating values
of one variable which is not based on the values of one or more other
variables. The variable being estimates is/are called independent variable/s.
Index Numbers: That are specialised averages which are capable of
being expressed in percentage & index numbers measure the changes in
the level of a given phenomenon. Index numbers measure the effect of
changes over a period of time.
Kurtosis: Kurtosis refers to relative height of the frequency curve, when
two or more symmetrical distributions are compared, the difference in
them are studied with ‘kurtosis’.
Laplace Principle: The Laplace principle is based on the simple rule
that if we are uncertain about various events, then we may treat them
as equally probable.
Linear and Non-Linear Correlation: When the amount of change in
one variable tends to keep a constant ratio to the amount of change in
the other variable, then the correlation is said to be linear. But if the
amount of change in one variable does not bear a constant ratio to the
amount of change in the other variable then the correlation is said to be
non-linear. The distinction between linear and non-linear is based upon
the consistency of the ratio of change between the variables.

PAGE 433
© Department of Distance & Continuing Education, Campus of Open Learning,
School of Open Learning, University of Delhi
B.COM. (PROGRAMME)

Notes Lorenz Curve: Lorenz Curve is used to describe inequalities of income.


Mean Deviation: A measure based on each observation in the data is
the mean deviation which is equal to the sum of absolute deviations of
the various observations from their mean or median.
Median: Median is a positional averages and it refers to the central value
in a set of arrayed values.
Mode: It is also a positional average and refer to that value in a series
which appears the maximum number of times.
Moments: Moments provide a useful method to study various characteristics
of a set of data.
Moments About Origin: There can also be moments about any given
value a when a = 0, moments calculated are called moments about origin.
Moving Averages: The moving average is a simple and flexible process
of trend measurement which is quite accurate under certain conditions.
This method establishes a trend by means of a series of averages covering
overlapping periods of the data.
Mutually Exclusive Events: The occurrence of an event precludes the
occurrence of others.
Normal Distribution: The most important continuous probability distribution
used in the entire field of statistics is normal distribution. The normal
curve is bell-shaped.
Overlapping Events: Events that have outcomes in common.
Partial and Total Correlation: There are two categories of multiple
correlation analysis. Under partial correlation, the relationship of two
or more variables is studied in such a way that only one dependent
variable and one independent variable is considered and all others are
kept constant. For example, coefficient of correlation between yield of
wheat and chemical fertilizers excluding the effects of pesticides and
manures is called partial correlation. Total correlation is based upon all
the variables.
Positive and Negative Correlation: Positive or direct Correlation refers
to the movement of variables in the same direction. The correlation is

434 PAGE
© Department of Distance & Continuing Education, Campus of Open Learning,
School of Open Learning, University of Delhi
GLOSSARY

said to be positive when the increase (decrease) in the value of one Notes
variable is accompanied by an increase (decrease) in the value of other
variable also. Negative or inverse correlation refers to the movement of
the variables in opposite direction. Correlation is said to be negative, if
an increase (decrease) in the value of one variable is accompanied by a
decrease (increase) in the value of other.
Possion Distribution: This is also a discrete distribution. It was originated
by a French mathematician Simeon Denis Poisson in 1837. The Poisson
distribution is the limiting form of binomial distribution as n becomes
infinitely large (n > 20) and p approaches zero (p <0.05)
Probability: It is the likelihood that something will happen. When we
calculate the probability of an event, we assign it a number between zero
and one, depicting how likely it is to happen.
Quartile deviation: It is a quick, inspectional measure of variability and
used when there are scattered or extreme values included in the data.
Random Experiment: A random experiment is defined as an experiment
whose outcome cannot be predicted with certainty.
Ratio-To-Trend Method: It is similar to ratio-to-moving-average method.
The only difference is the way of obtaining the trend values. Whereas in
the ratio-to-moving-average method, the trend values are obtained by the
method of moving averages, in the ratio-to-trend method, the corresponding
trend is obtained by the method of least squares.
Regression Analysis: Regression Analysis is a branch of statistical theory
which is widely used in all the scientific disciplines. It is a basic technique
for measuring or estimating the relationship among economic variables
that constitute the essence of economic theory and economic life.
Relative Frequency Approach: It is based on the actual observation.
Sample Point: Each element of a sample space is termed as sample point.
Sample Space: Set of all possible outcome of a trial.
Seasonal Variations: Seasonal variations are those rhythmic changes in
the time series data that occur regularly each year. They have their origin
in climatic or institutional factors that affect either supply or demand

PAGE 435
© Department of Distance & Continuing Education, Campus of Open Learning,
School of Open Learning, University of Delhi
B.COM. (PROGRAMME)

Notes or both. It is important that these variations be measured accurately for


three reasons.
Secular Trends: Secular trend refers to the general pattern of the values
in a time series– it is the long-term tendency of the movement of the
variable.
Simple and Multiple Correlation: Under simple correlation, we study
the relationship between two variables only i.e., between the yield of
wheat and the amount of ramfall or between demand and supply of a
commodity. In case of multiple correlation, the relationship is studied
among three or more variables. For example, the relationship of yield of
wheat may be studied with both chemical fertilizers and the pesticides.
Skewness: Skewness means lack of symmetry, which implies that the
mean, median and mode are unequal in such a case.
Standard Deviation: Standard deviation is calculated as the square root
of average of squared deviations taken from actual mean. It is also called
root mean square deviation. The square of standard deviation i.e. ı2 is
called ‘variance’ in statistics.
Standard Error: Standard error of an estimate is the measure of the
spread of observed values from estimated ones, expressed by regression
line or equation.
Standardised Normal Distribution: The normal distribution has two
parameters – mean and standard deviation. For every pair of values,
there is a distinct normal distribution. The distribution with mean μ = 0
DQG VWDQGDUG GHYLDWLRQ ı = 1 is called standardised normal distribution.
Statistical Decision Theory: Statistical techniques are being widely used
to solve business problems. These techniques are being used to solve
problems for which information is incomplete, uncertain or in some
cases almost completely lacking. This new area of statistics is known as
statistical decision theory.
Subjective Approach: Assigning probability based on available evidence.
Time Series: A time series refers to the observations of a random variable
like sales, employment, etc. placed in a chronological order.

436 PAGE
© Department of Distance & Continuing Education, Campus of Open Learning,
School of Open Learning, University of Delhi
GLOSSARY

Trial: Each performance of an experiment. Notes


Unit Test: It requires that the method of constructing index should be
independent of the units of the problem. all the methods except simple
aggregative method satisfy this test.
Variance of Expected Value: The degree to which actual observation
dispersed from expected value.
Weight: The term weight refers to the relative importance of the different
items in the construction of index numbers.

PAGE 437
© Department of Distance & Continuing Education, Campus of Open Learning,
School of Open Learning, University of Delhi

You might also like